Download as pdf or txt
Download as pdf or txt
You are on page 1of 308

440

TextVolt
Question Bank
May 2022

t in India
F i rs

Question Bank
with Audio PoD
ADVANCED AUDITING &
PROFESSIONAL ETHICS
AUDIT POWER QUESTIONS BANK

Chapter Topic Name Page No. Questions

1 Auditing Standards, Statements and Guidance Notes - An Overview 1 112

2 Audit Planning, Strategy and Execution 80 7

3 Risk Assessment and Internal Control 86 16

4 Special Aspects of Auditing in an Automated Environment 97 8

5 Company Audit 103 52

6 Audit Reports 141 10

7 Audit Committee and Corporate Governance 148 15

8 Audit of Consolidated Financial Statements 158 11

9 Audit of Banks 168 18

10 Audit of Insurance Company 180 10

11 Audit of Non-Banking Financial Companies 187 7

12 Audit under Fiscal Laws 193 35

13 Audit of Public Sector Undertakings 216 8

14 Liabilities of Auditor 221 9

15 Internal Audit, Management and Operational Audit 228 16

16 Due Diligence, Investigation and Forensic Audit 238 17

17 Peer Review and Quality Review 253 9

18 Professional Ethics 259 79

CA Final Audit Dec-2021 New Course Paper 301 19

CA Final Audit Dec-2021 Old Course Paper 304 19


Auditing Standards, Statements and Guidance Notes - An Overview

Chapter AUDITING STANDARDS, STATEMENTS AND


1 GUIDANCE NOTES - AN OVERVIEW

Past Exams Weightage


35

30

25

20

15

10

0
MAY-19 NOV-19 NOV-20 JAN-21 JUL-21 DEC-21

May-19 Nov-19 Nov-20 Jan-21 Jul-21 Dec-21


Marks 19 18 34 24 25 15

1.
FAST Engagement of Quality Control Standards : Introduction
Compliance with Standards on Auditing by a Non-Profit Organisation

CA.Sarthak Niraj Jain


Comment on M/s XYZ & Co., auditors of Goodwill Education Foundation, a recognised nonprofit organisation
feels that the standards on auditing need not to be applied as Goodwill Education Foundation is a non-profit
making concern.
Ans. (i) Preface to Standards on Auditing
 As per the Preface:- The SAs will apply whenever an independent audit is carried out; that is,
in the independent examination of financial statements/information of any entity;
 Whether profit oriented or not and irrespective of its size, or legal form (unless specified
otherwise) when such an examination is conducted with a view to expressing an opinion thereon.
(ii) Section 143 of the Companies Act, 2013
 As per section 143(9): - Every auditor shall comply with the auditing standards.
 As per section 143(10): - The Central Government may prescribe the standards of auditing or
any addendum thereto, as recommended by the ICAI, constituted under section 3 of the CA Act,
1949, in consultation with and after examination of the recommendations made by the NFRA.
 It is provided that until any auditing standards are notified, any standard, or standards of
auditing specified by the ICAI shall be deemed to be the auditing standards.
(iii) ICAI / Code of Ethics requirements
 It is the duty of the Chartered Accountant to ensure that SAs are followed in the audit of
financial information covered by their audit reports.
(iv) Conclusion: - In the given case, even though the client is a non-profit oriented entity the SAs shall
apply and the auditor shall be guilty of professional misconduct for failing to discharge his duty in
case of non-compliance with SAs.

CA Final Audit - By CA. Sarthak Niraj Jain 1


Auditing Standards, Statements and Guidance Notes - An Overview

SA – 200
Overall Objectives of the Independent Auditor and the Conduct of an Audit in Accordance
with Standards on Auditing
2. Certain Assertions or Subject Matters where it is Difficult to detect MM due to Potential Effects of
Inherent Limitations
M/s SG & Co. Chartered Accountants were appointed as Statutory Auditors of XYZ Limited for the F.Y 2021-
2022. The Company implemented internal controls for prevention and early detection of any fraudulent
activity. Auditors carried out test of controls and found out no major observations. After the completion of
audit, audit report was submitted by the auditors and audited results were issued. Fraud pertaining to the
area of inventory came to light subsequently for the period covered by audit and auditors were asked to make
submission as to why audit failed to identify such fraud. Auditors submitted that because of inherent
limitations of audit, it is not possible to get persuasive evidence of certain matters like fraud. Do you think
auditor made correct statement? Also discuss certain subject matters or assertions where it is difficult to
detect material misstatements due to potential effects of inherent limitations. (July-2021-New)
Ans. (i) As per SA 200- “Overall Objectives of the Independent Auditor and the Conduct of an Audit in
Accordance with Standards on Auditing” and as per SQC 1 because of the inherent limitations of an
audit, there is an unavoidable risk that some material misstatements of the financial statements may
not be detected, even though the audit is properly planned and performed in accordance with SAs.

Accordingly, the subsequent discovery of a material misstatement of the financial statements


resulting from fraud or error does not by itself indicate a failure to conduct an audit in accordance
with SAs.

FAST
However, the inherent limitations of an audit are not a justification for the auditor to be satisfied
with less-than-persuasive audit evidence.
Whether the auditor has performed an audit in accordance with SAs is determined by the audit

CA.Sarthak Niraj Jain


procedures performed in the circumstances, the sufficiency and appropriateness of the audit
evidence obtained as a result thereof and the suitability of the auditor’s report based on an evaluation
of that evidence in the light of the overall objectives of the auditor.

(ii) Conclusion:- In view of above, it can be concluded that auditors did not give correct statement.
In the case of certain assertions or subject matters, the potential effects of the inherent
limitations on the auditor’s ability to detect material misstatements are particularly significant.
Such assertions or subject matters include:
(i) Fraud, particularly fraud involving senior management or collusion.
(ii) The existence and completeness of related party relationships and transactions.
(iii) The occurrence of non-compliance with laws and regulations.
(iv) Future events or conditions that may cause an entity to cease to continue as a going concern.
3. Limitations Imposed by Management
Director (Finance) of KK Ltd. informed their newly appointed statutory auditor that they have sound Internal
control system implemented by a renowned professional firm and he is satisfied with its effectiveness and
functioning and therefore, the statutory auditor should concentrate on verifying only the routine books and
financial statements. What further action should auditor take?
Ans. (i) As per SA 200 “Overall Objectives of the Independent Auditor and the Conduct of an Audit in
Accordance with Standards on Auditing”,
 The purpose of an audit is to enhance the degree of confidence of intended users in the financial
statements. This is achieved by the expression of an opinion by the auditor on whether the
financial statements are prepared, in all material respects, in accordance with an applicable
financial reporting framework.

2 Ab Audit Hoga Sabse Scoring - By CA. Sarthak Niraj Jain


Auditing Standards, Statements and Guidance Notes - An Overview

 In the case of most general purpose frameworks: - That opinion is on whether the financial
statements are presented fairly, in all material respects, or give a true and fair view in
accordance with the framework.
(ii) Terms of Engagement & Limitation on Terms
 The scope of an audit of financial statements will be determined by the auditor for having regard
to the terms of the engagement, the requirement of relevant legislation and the pronouncements of
the Institute.
 The terms of engagement cannot, however, restrict the scope of an audit in relation to matters
which are prescribed by legislation or by the pronouncements of the Institute.
(iii) Limitation imposed by Management
 In the instant case -management has no right to guide and place any restriction on the work of
the auditor as it would amount to restriction on the scope of the audit.
 The auditor should ask the management - Not to impose such restriction on his scope of the
audit that impairs his ability to examine and express an opinion and
 If the management does not agrees, he should issue a qualified opinion or disclaimer, as
appropriate.
SA-210
Agreeing the Terms of Audit Engagements
4. Preconditions for an Audit Engagement
Mr. Ram Kapoor, Chartered Accountant, has been appointed as the statutory auditor by XYZ Private Limited for
the audit of their financial statements for the year 2018-19. The company has mentioned in the audit terms that
they will not be able to provide internal audit reports to Mr. Ram during the course of audit. Further, company

FAST
also imposed some limitation on scope of Mr. Ram. What are the preconditions Mr. Ram should ensure before
accepting/ refusing the proposal? Also advise, whether Mr. Ram should accept the proposed audit engagement?
(RTP-Nov-2019)
Ans. (i)
CA.Sarthak Niraj Jain
Provision: -SA 210 “Agreeing the Terms of Engagement” deals with the auditor’s responsibilities in
agreeing the terms of the audit engagement with management. Before accepting/ refusing an audit
engagement, to establish whether the preconditions for an audit are present, the auditor shall:
(a) Determine whether the financial reporting framework to be applied in the preparation of the
financial statements is acceptable; and
(b) Obtain the agreement of management that it acknowledges and understands its
responsibilities for followings:
(1) the preparation of the F.S. in accordance with the applicable FRF.
(2) exercising necessary internal control to enable the preparation of F.S. that are free from
material misstatement, whether due to fraud or error.
(3) to provide the auditor with:
 Access to all relevant information such as records, documentation and other matters;
 Additional information that the auditor may request from management for the purpose
of the audit; and
 Unrestricted access to persons within the entity from whom the auditor determines it
necessary to obtain audit evidence.
(ii) Acceptance of limitation:
 if management or those charged with governance impose a limitation on the scope of the auditor's
work in the terms of a proposed audit engagement such that the auditor believes the limitation will
result in the auditor disclaiming an opinion on the financial statements, the auditor shall not
accept such a limited engagement as an audit engagement, unless required by law or regulation
to do so.

CA Final Audit - By CA. Sarthak Niraj Jain 3


Auditing Standards, Statements and Guidance Notes - An Overview

 In addition if the preconditions for an audit are not present, the auditor shall discuss the matter
with management. Unless required by law or regulation to do so, the auditor shall not accept the
proposed audit engagement.
(iii) Conclusion: -In the instant case, Mr. Ram should not accept the appointment as statutory auditor of
XYZ Private Limited due to limitation imposed on his scope of work.
5. Audit Engagement Terms : Missing Content
MEA Limited is a listed company having its operation across India. MEA Limited appointed Mr. X, Mr. Y and Mr.
Z, as its joint auditors for the year 2021-22. After making sure that all of them are qualified to be appointed as
statutory auditor, MEA Limited issued engagement letter to all of them. But Mr. X was not clear on some points,
so he requested MEA Limited to slightly change the terms of his engagement. This change will not impact the
ultimate opinion on the financial statement. The engagement letter contains the details on objective and scope
of audit, responsibilities of auditor and identification of framework applicable. It also contains the reference to
expected form and content of report from all three joint auditors. In your opinion what was the discrepancy in
the Audit engagement letter issued by MEA Limited? (RTP-Nov-2020)(MTP-Dec-2021)
Ans. (i) As per SA 210, “Agreeing the Terms of Audit Engagements”:
 The auditor shall agree the terms of the audit engagement with management or those charged
with governance, as appropriate.
 Subject to prescribed details under Law or Regulations, the agreed terms of the audit engagement
shall be recorded in an audit engagement letter or other suitable form of written agreement and
shall include:
(a) The objective and scope of the audit of the financial statements;
(b) The responsibilities of the auditor;
(c)
FAST
The responsibilities of management;
(d) Identification of the applicable financial reporting framework for the preparation of the
financial statements; and

CA.Sarthak Niraj Jain


(e) Reference to the expected form and content of any reports to be issued by the auditor and a
statement that there may be circumstances in which a report may differ from its expected
form and content..
(ii) In the given scenario: - MEA Limited appointed Mr. X, Mr. Y and Mr. Z, as its joint auditors for the
year 2019-20 and issued engagement letter to all of them. The engagement letter contains the details
on objective and scope of audit, responsibilities of auditor, identification of framework applicable and
reference to expected form and content of report from all three joint auditors.
(iii) Conclusion: -In the given case engagement letter issued by MEA Ltd. does not specify the
responsibilities of management, whereas as per SA 210, it should also specify responsibilities of
management.
6. Acceptance of a Change in Engagement
As an auditor, how would you consider the acceptance of a change in audit engagement? (May-2018-Old)
OR
X, a Chartered Accountant was engaged by PQR & Co. Ltd. for auditing their accounts. He sent his letter of
engagement to the Board of Directors, which was accepted by the Company. In the course of audit of the
company, the auditor was unable to obtain appropriate sufficient audit evidence regarding receivables. The
client requested for a change in the terms of engagement.
OR
“An auditor who before the completion of the engagement is requested to change the engagement to one which
provides a lower level of assurance should consider the appropriateness of doing so.” Discuss.
Ans. As per SA 210 “Agreeing the Terms of Engagement”
 If prior to completing the audit engagement, the auditor is requested to change the audit engagement
to an engagement that conveys a lower level of assurance, The auditor shall determine whether there is
reasonable justification for doing so.

4 Ab Audit Hoga Sabse Scoring - By CA. Sarthak Niraj Jain


Auditing Standards, Statements and Guidance Notes - An Overview

 The auditor shall not agree to a change in the terms of the audit engagement where there is no
reasonable justification for doing so.
 If the terms of the audit engagement are changed, the auditor and management shall agree on and
record the new terms of the engagement in an engagement letter or other suitable form of written
agreement.
 If the auditor is unable to agree to a change of the terms of the audit engagement and is not permitted
by management to continue the original audit engagement, the auditor shall:
 Withdraw from the audit engagement where possible under applicable law or regulation; and
 Determine whether there is any obligation, either contractual or otherwise, to report the
circumstances to other parties, such as TCWG, owners or regulators.
SA -220 Quality Control for an Audit of Financial Statements
&
SQC-1 Quality Control for Firms that Perform Audits and Reviews of Historical
Financial Information, and Other Assurance and Related Services Engagements
7. Considerations as to Integrity of Clients
MB & Associates is a partnership firm of the Chartered Accountants which was established seven years back.
The firm is getting new clients and has also been offered new engagement services with existing clients. The
firm is concerned about obtaining such information as It considers necessary in the circumstances before
accepting an engagement with a new client and acceptance of a new engagement with an existing client. The
firm is looking to work with only select clients to adhere to the Quality Control Standards. Guide MB &
Associates about the matters to be considered with regard to the integrity of a client, as per the requirements of
SQC 1. (Study Material)(Nov-2019-New)

FAST OR
BSS & Associates is a partnership firm of Chartered Accountants which was established five years back. The
firm was offering only advisory services at the beginning, however, after audit rotation and advent of GST, firm

CA.Sarthak Niraj Jain


sees lot of potential in these areas also and started looking for opportunities in these areas also. These services
being assurance in nature, the firm required some internal restructuring and set up some policies and
procedures for compliance year on year.
The firm started getting new clients for these new services and is now looking to obtain such information as it
considers necessary in the circumstances before accepting an engagement with a new client, when deciding
whether to continue an existing engagement, and when considering acceptance of a new engagement with an
existing client. Where issues have been identified and the firm decides to accept or continue the client
relationship or a specific engagement, it has been setting up a process to document how the issues were
resolved.
The firm is now looking to work with only select clients which are in line with the policies of the firm. The firm
understands that the extent of knowledge it will have regarding the integrity of a client will grow within the
context of an ongoing relationship with that client. With regard to the integrity of a client, you are required to
give some examples of the matters to be considered by the firm as per the requirements of SQC1.
(RTP-May-2019)
Ans. As per SQC 1, the firm should obtain such information as it considers necessary in the circumstances before
accepting an engagement with a new client, when deciding whether to continue an existing engagement,
and when considering acceptance of a new engagement with an existing client.
Where issues have been identified, and the firm decides to accept or continue the client relationship or a
specific engagement, it should document how the issues were resolved.
With regard to the integrity of a client, matters that the firm considers include, for example:
a) The identity and business reputation of the client’s principal owners, key management, related
parties and those charged with its governance.

CA Final Audit - By CA. Sarthak Niraj Jain 5


Auditing Standards, Statements and Guidance Notes - An Overview

b) The nature of the client’s operations, including its business practices.


c) Information concerning the attitude of the client’s principal owners, key management and those
charged with its governance towards such matters as aggressive interpretation of accounting
standards and the internal control environment.
d) Whether the client is aggressively concerned with maintaining the firm’s fees as low as possible.
e) Indications of an inappropriate limitation in the scope of work.
f) Indications that the client might be involved in money laundering or other criminal activities.
g) The reasons for the proposed appointment of the firm and non-reappointment of the previous
firm.
The extent of knowledge a firm will have regarding the integrity of a client will generally grow within the
context of an ongoing relationship with that client.
8. Determination of Review Responsibility of EP
J.A.C.K. & Co., a Chartered Accountant firm was appointed as the statutory auditor of Falcon Ltd. after ensuring
the compliance with relevant provisions of the Companies Act, 2013. Mr. Jay was the engagement partner for
the aforesaid audit and prior to commencement of the audit, Mr. Jay had called for a meeting of the engagement
team in order to direct them and assign them their responsibilities. At the end of meeting, Mr. Jay assigned
review responsibilities to two of the engagement team members who were the most experienced amongst all,
for reviewing the work performed by the less experienced team members. While reviewing the work
performed by the less experienced members of the engagement team, what shall be the considerations of the
reviewers? (MTP-May-2021)
Ans. (i) As per SQC 1, “Quality Control for Firms that Perform Audits and Reviews of Historical Financial


FAST
Information, and Other Assurance and Related Services Engagements”:
Review responsibilities are determined on the basis that more experienced team members,

(ii) CA.Sarthak Niraj Jain


including the engagement partner, review work performed by less experienced team members.
In the given situation: - Mr. Jay, engagement partner assigned review responsibilities to two of the
engagement team members who were the most experienced team members.
(iii) While reviewing the work performed by less experienced members of the engagement team, both
the more experienced Reviewers should consider whether:
1. The work has been performed in accordance with professional standards and regulatory and
legal requirements.
2. Significant matters have been raised for further consideration.
3. Appropriate consultations have taken place and the resulting conclusions have been documented
and implemented.
4. There is a need to revise the nature, timing and extent of work performed.
5. The work performed supports the conclusions reached and is appropriately documented.
6. The evidence obtained is sufficient and appropriate to support the report; and
7. The objectives of the engagement procedures have been achieved.
9. Relying on Work Performed by Another Partner
M/s Suresh Chandra & Co. has been appointed as an auditor of SC Ltd. for the financial year 2021-22. CA.
Suresh, one of the partners of M/s Suresh Chandra & Co., completed entire routine audit work by 29 th May,
2022. Unfortunately, on the very next morning, while roving towards office of SC Ltd. to sign final audit report,
he met with a road accident and died. CA. Chandra, another partner of M/s Suresh Chandra & Co., therefore,
signed the accounts of SC Ltd., without reviewing the work performed by CA. Suresh. State with reasons
whether CA. Chandra is right in expressing an opinion on financial statements the audit of which is performed
by another auditor. (MTP-May-2018)

6 Ab Audit Hoga Sabse Scoring - By CA. Sarthak Niraj Jain


Auditing Standards, Statements and Guidance Notes - An Overview

Ans. (i) As per SA 220, “Quality Control for an Audit of Financial Statements”:
The engagement partner shall take responsibility for reviews being performed in accordance with the
firm’s review policies and procedures. Review procedures consists of the considerations, whether,
1. The work has been performed in accordance with professional standards and regulatory and
legal requirements;
2. Significant matters have been raised for further consideration;
3. Appropriate consultations have taken place and the resulting conclusions have been documented
and implemented;
4. The work performed supports the conclusions reached and is appropriately documented;
5. The evidence obtained is sufficient and appropriate to support the auditor’s report; and
6. The objectives of the engagement procedures have been achieved.
(ii) Using work performed by other
 When the auditor delegates work to assistants or uses work performed by other
auditors/experts he will continue to be responsible for forming and expressing his opinion on
the financial statements.
 However, he will be entitled to rely on the work performed by others, provided he exercises
adequate skill and care and is not aware of any reason to believe that he should not have so relied.
 The auditor should carefully direct, supervise and review work delegated to assistants.
 He should obtain reasonable assurance that work performed by other auditors/experts and
assistants is adequate for his purpose.
(iii) In the instant case: - Mr. Suresh, a partner of the firm had completed routine audit work and died

FAST
before signing audit report. Mr. Chandra another partner of the firm has signed the accounts of SC
Ltd, relying on the work performed by Mr. Suresh.
(iv) Conclusion:- CA. Chandra is allowed to sign the audit report, though, will be responsible for

CA.Sarthak Niraj Jain


expressing the opinion. He may rely on the work performed by CA. Suresh provided he further
exercises adequate skill and due care and review the work performed by him.
10. Difference of Opinion/Limitation on Auditor:
You are an audit senior working for the firm Bohra & Company. You are currently carrying out the audit of
Wisdom Ltd., a manufacturer of waste paper bins. You are unhappy with Wisdom Ltd.’s inventory valuation
policy and have raised the issue several times with the audit manager. He has dealt with the client for a number
of years and does not see what you are making an objection about. He has refused to meet you on site to discuss
those issues. As the audit manager had dealt with Wisdom Ltd. for so many years, the other partners have
decided to leave the audit of Wisdom Ltd. in his capable hands. Comment on the situation outlines above.
Ans. (i) Provision: - SQC – 1 “Quality Control for Firms that perform Audits and Reviews of Historical
Financial Information and Other Assurance and Related Services Engagements” requires:
 A firm to establish the policies & procedures for dealing/resolving differences of opinion with in
engagement team.
 An engagement partner is usually appointed to each audit engagement undertaken by the firm,
to take responsibility for the engagement on behalf of the firm. Assigning the audit to an
experienced audit manager is not sufficient.
(ii) SA 220 “Quality Control for an Audit of Financial Statement”: - Requires that the audit engagement
partner takes responsibility for settling disputes in accordance with the firm’s policy in respect of
resolution of difference of opinion required by SQC 1.
(iii) In the present case:
 Partners of the firm have decided to leave the audit in the hands of Audit manager and no
engagement partner has been assigned.

CA Final Audit - By CA. Sarthak Niraj Jain 7


Auditing Standards, Statements and Guidance Notes - An Overview

 The lack of an audit engagement partner also means that several of the requirements of SA 220,
about ensuring that engagements in relation to independence and directing, supervising and
reviewing the audit are not in place.
 Further, the audit manager and senior have conflicting views about the valuation of inventory.
This does not appear to have been handled well, with the manager refusing to discuss the issue with
the senior.
(iv) Conclusion: Failure to resolve the difference of opinion is a breach of the firm’s policy under SQC 1.
It indicates that the firm does not have a suitable policy concerning such disputes required by SQC1.
11. Date of Signing of Audit Report
OP & Associates are the statutory auditors of BB Ltd. BB Ltd is a listed company and started its operations 5
years back. The field work during the audit of the financial statements of the company for the year ended
March 31, 2022 got completed on May 1, 2022. The auditor’s report was dated May 12, 2022. During the
documentation review of the engagement, it was observed that the engagement quality control review was
completed on May 15, 2022. Engagement partner had completed his reviews in entirety by May 10, 2022 and
signed the report on May 12, 2022. Comment. (MTP-Nov-2018)
Ans. (i) As per SA 220, “Quality Control for an Audit of Financial Statements”
 The engagement partner shall take responsibility for reviews being performed in accordance
with the firm’s review policies and procedures.
 For audits of financial statements of listed entities, the engagement partner shall:
(a) Determine that an engagement quality control reviewer has been appointed;
(b) Discuss significant matters arising during the audit engagement, including those identified

(c)
FAST
during the engagement quality control review, with the engagement quality control reviewer;
and
Not date the auditor’s report until the completion of the engagement quality control review.
(ii)
CA.Sarthak Niraj Jain
SA 700, “Forming an Opinion and Reporting on Financial Statements”, requires:
 The auditor’s report to be dated no earlier than the date on which the auditor has obtained
sufficient appropriate evidence on which to base the auditor’s opinion on the financial statements
(iii) In the present case: - OP & Associates are the statutory auditors of a listed company which started its
operations 5 years back. The field work during the audit of the financial statements of the company for
the year ended March 31, 2022 got completed on May 1, 2022. The auditor’s report was dated May 12,
2022. During the documentation review of the engagement, it was observed that the engagement quality
control review was completed on May 15, 2022.
(iv) Conclusion: - Signing of auditor’s report i.e. on May 12, 2022 which is before the completion of
review engagement quality control review i.e. May 15, 2022, is not in order.
12. Responsibilities of EP and EQCR in relation to Assessment of Independence
During the audit of FMP Ltd, a listed company, Engagement Partner (EP) completed his reviews and also
ensured compliance with independence requirements that apply to the audit engagement. The engagement
files were also reviewed by the Engagement Quality Control Reviewer (EQCR) except the independence
assessment documentation. Engagement Partner was of the view that matters related to independence
assessment are the responsibility of the Engagement Partner and not Engagement Quality Control Reviewer.
Engagement Quality Control Reviewer objected to this and refused to sign off the documentation. Please advise
as per SA 220. (RTP-May-2022)(Study Material)(MTP-Nov-2019)(RTP-May-2019)
Ans. (i) As per SA 220 - Quality control for an Audit of Financial Statements
The engagement partner shall form a conclusion on compliance with independence requirements
that apply to the audit engagement. In doing so, the engagement partner shall:
(a) Obtain relevant information from the firm and, where applicable, network firms, to identify
and evaluate circumstances and relationships that create threats to independence;

8 Ab Audit Hoga Sabse Scoring - By CA. Sarthak Niraj Jain


Auditing Standards, Statements and Guidance Notes - An Overview

(b) Evaluate information on identified breaches, if any, of the firm’s independence policies and
procedures to determine whether they create a threat to independence for the audit
engagement; and
(c) Take appropriate action to eliminate such threats or reduce them to an acceptable level by
applying safeguards, or, if considered appropriate, to withdraw from the audit engagement, where
withdrawal is permitted by law or regulation. The engagement partner shall promptly report to
the firm any inability to resolve the matter for appropriate action.
(ii) For audits of financial statements of listed entities: - The engagement quality control reviewer, on
performing an engagement quality control review, shall also consider among other things, the
engagement team’s evaluation of the firm’s independence in relation to the audit engagement.
(iii) Conclusion: - In the given case, the Engagement Partner is not right. The independence assessment
documentation should also be given to Engagement Quality Control Reviewer for his review.
13. SQC-1 : Complaints & Allegations
M/s NK & Co., Chartered Accountants were appointed as Statutory Auditors of Fresh Juice Limited for the F.Y
2021-2022. The previous year's audit was conducted by M/s. LP & Associates. After the audit was completed
and report submitted, it was found that closing balances of last financial year i.e., 2020-21 were incorrectly
brought forward. It was found that M/s NK & Co. did not apply any audit procedures to ensure that correct
opening balances have been brought forward to the current period. Accordingly, a complaint was filed against
NK & Co. in relation to this matter. You are required to inform what policies are required to be implemented by
NK & Co. for dealing with such complaints and allegations as required by Standard on Quality Control (SQC).
(Jan-2021-New)
Ans. (i) In the given question: - NK & Co. did not apply audit procedures to ensure that opening balances
had been correctly brought forward. A complaint was filed against the auditors in this context.
(ii) FAST
As per Standard on Quality Control (SQC) 1 “Quality Control for Firms that Perform Audits and
Reviews of Historical Financial Information, and Other Assurance and Related Services Engagements”,

CA.Sarthak Niraj Jain


 The firm should establish policies and procedures designed to provide it with reasonable
assurance that it deals appropriately with:
(a) Complaints and allegations that the work performed by the firm fails to comply with
professional standards and regulatory and legal requirements; and
(b) Allegations of non-compliance with the firm’s system of quality control.
 Complaints and allegations (which do not include those that are clearly frivolous) may originate
from within or outside the firm.
 They may be made by firm personnel, clients or other third parties. They may be received by
engagement team members or other firm personnel.
 As part of this process, the firm establishes clearly defined channels for firm personnel to raise
any concerns in a manner that enables them to come forward without fear of reprisals.
 The firm investigates such complaints and allegations in accordance with established policies
and procedures. The investigation is supervised by a partner with sufficient and appropriate
experience and authority within the firm but who is not otherwise involved in the engagement, and
includes involving legal counsel as necessary.
 Small firms and sole practitioners may use the services of a suitably qualified external person
or another firm to carry out the investigation. Complaints, allegations and the responses to them are
documented.
 Where the results of the investigations indicate deficiencies in the design or operation of the
firm’s quality control policies and procedures, or non-compliance with the firm’s system of
quality control by an individual or individuals, the firm takes appropriate action

CA Final Audit - By CA. Sarthak Niraj Jain 9


Auditing Standards, Statements and Guidance Notes - An Overview

14. Compliance with Standard on Quality Control


HK & Co. Chartered Accountants have been auditors of SAT Ltd (a listed entity) for the last 8 financial years.
CA. H, partner of the firm, has been handling the audit assignment very well since the appointment. The audit
work of CA. H and her team is reviewed by a senior partner CA. K to assure that audit is performed in
accordance with professional standards and regulatory and legal requirements. CA. K was out of India for
some personal reasons, so this year CA. G has been asked to review the audit work. In your opinion, what
areas CA. G should consider at the time of review. List any four areas and also comment whether firm is
complying with Standard on Quality Control or not? (July-2021-New)
Ans. (a) As per SQC 1, an engagement quality control review for audits of financial statements of listed entities
includes considering the following:
(i) The work has been performed in accordance with professional standards and regulatory and
legal requirements;
(ii) Significant matters have been raised for further consideration;
(iii) Appropriate consultations have taken place and the resulting conclusions have been documented
and implemented;
(iv) There is a need to revise the nature, timing and extent of work performed;
(v) The work performed supports the conclusions reached and is appropriately documented;
(vi) The evidence obtained is sufficient and appropriate to support the report; and
(vii) The objectives of the engagement procedures have been achieved.
(b) The firm should establish policies and procedures:
(i) Setting out criteria for determining the need for safeguards to reduce the familiarity threat to
an acceptable level when using the same senior personnel on an assurance engagement over a

FAST
long period of time; and
(ii) For all audits of financial statements of listed entities
 Requiring the rotation of the engagement partner after a specified period in compliance with

CA.Sarthak Niraj Jain



the Code.
The familiarity threat is particularly relevant in the context of financial statement audits of
listed entities.
 For these audits, the engagement partner should be rotated after a predefined period,
normally not more than seven years.
(c) Conclusion:- From the facts given in the question and from the above stated paras of SQC 1, it can be
concluded that firm is not complying with SQC 1 as Engagement Partner H is continuing for more than
7 years.
SA-230
Audit Documentation
15. Access to WPs to Regulators & Third Parties
Discuss the Auditor’s responsibility to provide access to his audit working papers to Regulators and third
parties.
Ans. (i) Code Of Ethics: Clause (1) of Part I of Second Schedule to the CAs Act, 1949 states that:
 A CA in practice shall be deemed to be guilty of professional misconduct if he discloses
information acquired in the course of his professional engagement to any person other than his
client, without the consent of his client or otherwise than as required by law for the time being in
force.
(ii) SA 200 on “Overall Objectives of the Independent Auditor and the conduct of an audit in
accordance with Standards on Auditing” also reiterates that,

10 Ab Audit Hoga Sabse Scoring - By CA. Sarthak Niraj Jain


Auditing Standards, Statements and Guidance Notes - An Overview

 The auditor should respect the confidentiality of the information obtained and should not
disclose any such information to any third party without specific authority or unless there is a
legal or professional duty to disclose”.
 If there is a request to provide access by the regulator based on the legal requirement, the same
has to be complied with after informing the client about the same.
(iii) SQC–1, “Quality Control for Firms that Perform Audits and Reviews of Historical Financial
Information, and Other Assurance and Related Services Engagements”, provides that,
 Unless otherwise specified by law or regulation, audit documentation is the property of the
auditor.
 He may at his discretion, make portions of, or extracts from, audit documentation available to
clients, provided such disclosure does not undermine the validity of the work performed, or, in the
case of assurance engagements, the independence of the auditor or of his personnel.
(iv) As per SA 230, Audit documentation serves a number of additional purposes, including the enabling
the conduct of external inspections in accordance with applicable legal, regulatory or other
requirements.
(v) Conclusion: - It is auditor’s responsibility to provide access to his audit working papers to Regulators
when required by law whereas auditor is under no obligation to provide access to working papers to
third parties.
SA-240
The Auditor’s Responsibilities relating to Fraud in an Audit of Financial Statements
16. Possible Sources of Fraudulent Financial Reporting
In the course of audit of Quick Ltd, you suspect that the management has made misstatements in the financial
FAST
statements intentionally to deceive the users and to succumb to pressures to meet market expectations.
Elucidate how the fraudulent financial reporting may be accomplished and also discuss the techniques of
committing fraud by management overriding controls. (Nov-2020-New) (MTP-Dec-2021)
Ans. (i) CA.Sarthak Niraj Jain
As per SA 240, “The Auditor’s responsibilities relating to Fraud in an Audit of Financial
Statements” fraudulent financial reporting involves intentional misstatements or omissions of amounts
or disclosures in financial statements to deceive financial statement users. It may be accomplished by,
(a) Manipulation, falsification, or alteration of accounting records or supporting documents from
which the financial statements are prepared or
(b) Misrepresentation in, or intentional omission from, the financial statements of events,
transactions or other significant information or
(c) Intentional misstatements involve intentional misapplication of accounting principles
relating to measurement, recognition, classification, presentation, or disclosure etc.

(ii) Fraud can be committed by management overriding controls using such techniques as:
(i) Recording fictitious journal entries, particularly close to the end of an accounting period, to
manipulate operating results or achieve other objectives.
(ii) Inappropriately adjusting assumptions and changing judgments used to estimate account
balances.
(iii) Omitting, advancing or delaying recognition in the financial statements of events and transactions
that have occurred during the reporting period.
(iv) Concealing, or not disclosing, facts that could affect the amounts recorded in the financial
statements.
(v) Engaging in complex transactions that are structured to misrepresent the financial position or
financial performance of the entity.

CA Final Audit - By CA. Sarthak Niraj Jain 11


Auditing Standards, Statements and Guidance Notes - An Overview

(vi) Altering records and terms related to significant and unusual transactions.
(vii) Embezzling receipts (for example, misappropriating collections on accounts receivable or
diverting receipts in respect of written-off accounts to personal bank accounts).
(viii) Stealing physical assets or intellectual property (for example, stealing inventory for personal
use or for sale, stealing scrap for resale, colluding with a competitor by disclosing technological
data in return for payment).
(ix) Causing an entity to pay for goods and services not received (for example, payments to fictitious
vendors, kickbacks paid by vendors to the entity’s purchasing agents in return for inflating prices,
payments to fictitious employees).
(x) Using an entity’s assets for personal use (for example, using the entity’s assets as collateral for a
personal loan or a loan to a related party).
(iii) As per section 143(12) of the Companies Act, 2013:
 If an auditor of a company, in the course of the performance of his duties as auditor, has reason
to believe that an offence involving fraud is being or has been committed against the company
by officers or employees of the company, he shall immediately report the matter to the Central
Government within 60 days of his knowledge and after following the prescribed procedure.
17. Possibility of Fraud due to Problematic or Unusual Relationships
Write the circumstances that indicate the possibility of fraud due to problematic or unusual relationships
between the auditor and management. (May-2018-Old)
Ans. Problematic or unusual relationships between the auditor and management, including:
1. Denial of access to records, facilities, certain employees, customers, vendors, or others from whom

2.
FAST
audit evidence might be sought.
Undue time pressures imposed by management to resolve complex or contentious issues.
3.
CA.Sarthak Niraj Jain
Complaints by management about the conduct of the audit or management intimidation of engagement
team members, particularly in connection with the auditor’s critical assessment of audit evidence or in
the resolution of potential disagreements with management.
4. Unusual delays by the entity in providing requested information.
5. Unwillingness to facilitate auditor access to key electronic files for testing through the use of
computer-assisted audit techniques.
6. Denial of access to key IT operations staff and facilities, including security, operations, and systems
development personnel.
7. An unwillingness to add or revise disclosures in the financial statements to make them more
complete and understandable.
8. An unwillingness to address identified deficiencies in internal control on a timely basis.
9. Unwillingness by management to permit the auditor to meet privately with TCWG
10. Accounting Policy that appears to be variance with industry norms
11. Frequent changes in accounting estimates that do not appear to result from changed circumstances
12. Tolerance of variations in the entity’s code of conduct
18. Auditor’s Duties in case of MM resulting from Management Fraud
Cloud Ltd. appointed an auditor for the financial year 2021-22. While going through the audit procedure, the
auditor observed that the management has entered into certain transactions which are irregular in nature and
the management is personally benefited from such transactions. Explain briefly the duties and responsibilities
of an auditor in case of material misstatement resulting from Management Fraud.

12 Ab Audit Hoga Sabse Scoring - By CA. Sarthak Niraj Jain


Auditing Standards, Statements and Guidance Notes - An Overview

OR
Fraud can be committed by management overriding controls using such techniques as engaging in complex
transactions that are structured to misrepresent the financial position or financial performance of the entity. In
view of the above-mentioned circumstances of management fraud, explain briefly duties and responsibilities of
an auditor in case of material misstatement resulting from such Management Fraud.
OR
Explain briefly duties and responsibilities of an auditor in case of material misstatement resulting from
Management Fraud.
Ans. (i) Management & Auditor’s Responsibility
 As per SA 240 “The Auditor’s Responsibilities Relating to Fraud in an Audit of Financial
Statements”, the primary responsibility for the prevention and detection of fraud rests with both
TCWG of the entity and management.
 The auditor, conducting an audit, is responsible for obtaining reasonable assurance that the
financial statements taken as a whole are free from material misstatement, whether caused by
fraud or error.
(ii) Inherent Limitation of Auditor
 Owing to the inherent limitations of an audit, there is an unavoidable risk that some material
misstatements of the financial statements may not be detected, even though the audit is properly
planned and performed in accordance with the SAs.
 As described in SA 200, the potential effects of inherent limitations are particularly significant in the
case of misstatement resulting from fraud.
(iii) Detection Risk Higher in Frauds than Errors
 The risk of not detecting a material misstatement resulting from fraud is higher than the risk of
FAST
not detecting one resulting from error.
 This is because fraud may involve sophisticated and carefully organized schemes designed to
conceal it, such as forgery, deliberate failure to record transactions, or intentional
CA.Sarthak Niraj Jain
misrepresentations being made to the auditor.
 Such attempts at concealment may be even more difficult to detect when accompanied by collusion.
Conclusion may cause the auditor to believe that audit evidence is persuasive when it is, in fact,
false.
 While the auditor may be able to identify potential opportunities for fraud to be perpetrated, it is
difficult for the auditor to determine whether misstatements in judgment areas such as accounting
estimates are caused by fraud or error.
(iv) Detection Risk even Higher for Management Frauds
 Furthermore, the risk of the auditor not detecting a material misstatement resulting from
management fraud is greater than for employee fraud, because management is frequently in a
position to directly or indirectly manipulate accounting records, present fraudulent financial
information or override control procedures designed to prevent similar frauds by other
employees.
(v) Professional Skepticism
 When obtaining reasonable assurance, the auditor is responsible for maintaining professional
skepticism throughout the audit, considering the potential for management override of controls
and recognizing the fact that audit procedures that are effective for detecting error may not be
effective in detecting fraud.
(vi) Companies Act – Reporting of Frauds
 As per section 143(12) of the Companies Act, 2013: - if an auditor of a company, in the course of
the performance of his duties as auditor, has reason to believe that an offence involving fraud is
being or has been committed against the company by officers or employees of the company, he
shall immediately report the matter to the Central Government within 60 days of his knowledge
and after following the prescribed procedure.

CA Final Audit - By CA. Sarthak Niraj Jain 13


Auditing Standards, Statements and Guidance Notes - An Overview

19. Auditor’s Duties in case of Suspected Fraud


M/s Honest Limited has entered into a transaction on 5th March, 2022, near year-end, whereby it has agreed to
pay ` 5 lakhs per month to Mr. Y as annual retainer-ship fee for "engineering consultation". No amount was
actually paid, but ` 60 lakhs are provided in books of account as on March 31, 2022. Your inquiry elicits a
response that need-based consultation was obtained round the year, but there is no documentary or other
evidence of receipt of the service. As the auditor of M/s Honest Limited, what would be your approach?
(RTP-Nov-2018)
Ans. (i) As per SA 240 on “The Auditor’s Responsibilities Relating to Fraud in an Audit of Financial
Statements” fraud can be committed by management by various means including therein recording
of fictitious journal entries, particularly close to the end of an accounting period, to manipulate
operating results or achieve other objectives.
(ii) In the given case: - Honest Ltd. has entered into an agreement with Mr. Y, at year-end, for engineering
consultation. It also provides ` 60 lakhs in the books of accounts, however, no documentary or other
evidence of receipt of such service is available. It appears that company has passed fictitious journal
entries, near year-end, to manipulate the operating results.
(ii) SA 240 further provides that If, as a result of a misstatement resulting from fraud or suspected
fraud, the auditor encounters exceptional circumstances that bring into question the auditor’s ability to
continue performing the audit, the auditor shall:
(1) Determine the professional and legal responsibilities applicable in the circumstances, including
whether there is a requirement for the auditor to report to the person or persons who made the
audit appointment or, in some cases, to regulatory authorities;
(2) Consider whether it is appropriate to withdraw from the engagement, where withdrawal from

FAST
the engagement is legally permitted.
(iii) Sec. 143(12) of the Companies Act, 2013 read with Rule 13 of Companies (Audit & Auditor’s) Rules,

CA.Sarthak Niraj Jain


2016 requires that if an auditor of a company, in the course of the performance of his duties as
auditor, has reason to believe that an offence involving fraud is being or has been committed against
the company by officers or employees of the company, he shall immediately report the matter to the
audit committee within 2 days of his knowledge (as amount involved is less than ` 1 Cr.) mentioning the
following:
(a) Nature of Fraud with description;
(b) Approximate amount involved; and
(c) Parties involved etc.
(iv) Para 3(xi) of CARO, 2020 also requires the company auditor to report:
 Whether any fraud by the company or any fraud on the company has been noticed or reported
during the year;
 If yes, the nature and the amount involved is to be indicated.
20. Reporting of Fraud committed by Management/Directors of the Company
On 15th March, 2022, the directors of Phony Ltd. instructed their accountant to enter purchases amounting
` 1.02 crores from a company incorporated dated 11th March, 2022. However, no amount was actually paid
and ` 1.02 crore was provided in the books of account as purchases for the year ending on 31st March, 2022.
On inspection, no documentary or other evidence of such purchases was found. As the auditor of Phony Ltd.,
what would be your approach regarding reporting of such bogus purchases? (MTP-May-2020)
Ans. (i) As per SA 240 on “The Auditor’s Responsibilities Relating to Fraud in an Audit of Financial
Statements”, fraud can be committed by management overriding controls using such techniques as
recording fictitious journal entries, particularly close to the end of an accounting period, to
manipulate operating results or achieve other objectives.

14 Ab Audit Hoga Sabse Scoring - By CA. Sarthak Niraj Jain


Auditing Standards, Statements and Guidance Notes - An Overview

(ii) In the given case: - Phony Ltd. has made purchases amounting ` 1.02 crores, at year-end. It also
debited the sum in the books of account, however, no documentary or other evidence of such
purchases was found, on investigation. It is clear that company has passed fictitious journal entries,
near year-end, to manipulate the operating results.
(iii) Auditor’s Approach in the above case:
 The auditor would adopt the approach which will be based on the result of misstatement on the
basis of such fictitious journal entry, i.e. if, as a result of a misstatement resulting from fraud or
suspected fraud,
 The auditor encounters exceptional circumstances that bring into question the auditor’s ability to
continue performing the audit, the auditor shall determine the professional and legal
responsibilities applicable in the circumstances, including
 Whether there is a requirement for the auditor to report to the person or persons who made the
audit appointment or, in some cases, to regulatory authorities; or
 The auditor may consider for appropriateness of withdrawal from such engagement, where
withdrawal from the engagement is legally permitted.
(iv) Reporting requirement: -
 As per section 143(12), if an auditor of a company in the course of the performance of his duties
as auditor, has reason to believe that an offence of fraud, which involves or is expected to involve
individually an amount of ` 1 crore or above is being or has been committed in the company by its
officers or employees, he shall report the matter to the Central Government in prescribed
manner.
 The auditor is also required to report under clause (xi) of paragraph 3 of Companies (Auditor’s
Report) Order, 2020, whether any fraud by the company or any fraud on the Company has been
noticed or reported during the year. If yes, the nature and the amount involved is to be indicated.
21.
FAST
Impossibility to Continue the Performance of Audit
You notice a misstatement resulting from fraud or suspected fraud during the audit and conclude that it is not
possible to continue the performance of audit. As a Statutory Auditor, how would you deal?
Ans. (i) CA.Sarthak Niraj Jain
Report and Withdraw, unless Legally Prohibited – SA 240
According to SA 240 “The Auditor’s Responsibilities Relating to Fraud in an Audit of Financial
Statements”, if, as a result of a misstatement resulting from fraud or suspected fraud, the auditor
encounters exceptional circumstances that bring into question the auditor’s ability to continue
performing the audit, the auditor shall:
(1) Determine the professional and legal responsibilities applicable in the circumstances, including
whether there is a requirement for the auditor to report to the person or persons who made the
audit appointment or, in some cases, to regulatory authorities;
(2) Consider whether it is appropriate to withdraw from the engagement, where withdrawal
from the engagement is legally permitted and
(3) If the auditor withdraws:
(a) Discuss with the appropriate level of management and those charged with governance,
the auditor’s withdrawal from the engagement and the reasons for the withdrawal; and
(b) Determine whether there is a professional or legal requirement to report to the person
or persons who made the audit appointment or, in some cases, to regulatory authorities,
the auditor’s withdrawal from the engagement and the reasons for the withdrawal.
(ii) Companies Act – Reporting of Frauds
As per section 143(12) of the Companies Act, 2013: - if an auditor of a company, in the course of the
performance of his duties as auditor, has reason to believe that an offence involving fraud is being or
has been committed against the company by officers or employees of the company, he shall
immediately report the matter to the Central Government within 60 days of his knowledge and after
following the prescribed procedure.

CA Final Audit - By CA. Sarthak Niraj Jain 15


Auditing Standards, Statements and Guidance Notes - An Overview

SA-250
Consideration of Laws and Regulations in an Audit of Financial Statements
22. Management’s Responsibility for Compliance with Laws and Regulations
PQ Limited, a listed entity, is in the business of manufacturing of specialty chemicals. The company has
appointed CA Jazz as CFO of the company. CA Jazz is concerned about compliance with the provisions of laws
and regulations that determine the reported amounts and disclosure in financial statements of PQ Limited.
Accordingly, CA Jazz wants to implement such policies and procedures that can assist him in the prevention
and detection of non-compliance with laws and regulations. Help CA Jazz by citing examples of such policies
and procedures. (Nov-2020-New)
OR
“Management is responsible for compliance with laws and regulations.”
Ans. (i) According to SA 250 on “Consideration of Laws and Regulations in an Audit of Financial
Statements”
 It is management’s responsibility, to ensure that the entity’s operations are conducted in
accordance with the provisions of laws and regulations.
 Laws and regulations may affect an entity’s financial statements in different ways for example,
most directly; they may affect specific disclosures required of the entity in the financial
statements.
(ii) The following are the procedures an entity may implement to assist in the prevention and
detection of non-compliance with laws and regulations-
a) Monitoring legal requirements and ensuring that operating procedures are designed to meet

FAST
these requirements.
b) Instituting and operating appropriate systems of internal control.
c) Developing, publicizing and following a code of conduct.

CA.Sarthak Niraj Jain


d) Monitoring compliance with the code of conduct and acting appropriately to discipline employees
who fail to comply with it.
e) Engaging legal advisors to assist in monitoring legal requirements.
f) Maintaining a register of significant laws and regulations with which the entity has to comply
within its particular industry and a record of complaints.
g) Ensuring employees are properly trained and understand the code of conduct.
23. Compliance with other Laws & Regulations
While verifying the employee records in a company, it was found that a major portion of the labour employed
was child labour. On questioning the management, the auditor was told that it was outside his scope of the
financial audit to look into the compliance with other laws. (MTP-May-2021)(RTP-May-2018)
OR
CA. Young has been appointed as an auditor of Rama Ltd., a textile entity. While going through the employee
records of the company, CA. Young identified that most of the labourers employed are of the age between 11-12
years. On enquiring the same, the management argues that there is no such boundation with regard to
employment of such lower age children and contends that it is out of the scope of audit as well to check such
compliance. Comment in the context of relevant standard on auditing whether the contention of management is
tenable. (MTP-May-2020)
Ans. (i) As per SA 250, “Consideration of Laws and Regulations in an Audit of Financial Statements”
 The auditor shall obtain sufficient appropriate audit evidence regarding compliance with the
provisions of those laws and regulations generally recognised to have a direct effect on the
determination of material amounts and disclosures in the financial statements including tax and
labour laws.

16 Ab Audit Hoga Sabse Scoring - By CA. Sarthak Niraj Jain


Auditing Standards, Statements and Guidance Notes - An Overview

(ii) In case of other laws:


 On-compliance with other laws and regulations may result in found that a major portion for the
entity, the costs of which may need to be provided for in the financial statements, but are not
considered to have a direct effect on the financial statements.
(iii) In the instant case: - Major portion of the labour employed in the company was child labour. While
questioning by auditor, reply of the management that it was outside his scope of financial audit to look
into the compliance with other laws is not acceptable as it may have a material effect on financial
statements.
(iv) Conclusion: -Thus, auditor should ensure the disclosure of above fact and provision for the cost of
fines, litigation or other consequences for the entity. In case if the auditor concludes that non-
compliance has a material effect on the financial statements and has not been adequately reflected
in the financial statements, the auditor shall express a qualified or adverse opinion on the financial
statement.
24. Non-compliance is Identified or Suspected – Audit Procedure
CA Abhinanadan is an auditor of KM Private Limited. During the course of audit, CA Abhinanadan becomes
aware of information concerning an instance of noncompliance or suspected non-compliance with laws and
regulations. Being a senior partner of CA. Abhinanadan, guide him regarding audit procedures to be followed
when non-compliance is identified or suspected. (RTP-May-2022)
Ans.  As per SA 250, “Consideration of Laws and Regulations in an Audit of Financial Statements”, if the
auditor becomes aware of information concerning an instance of non-compliance or suspected non-
compliance with laws and regulations, the auditor shall obtain:
(i) An understanding of the nature of the act and the circumstances in which it has occurred; and


(ii)
FASTFurther information to evaluate the possible effect on the financial statements.
If the auditor suspects there may be non-compliance, the auditor shall discuss the matter with

 CA.Sarthak Niraj Jain


management and, where appropriate, those charged with governance.
If management or, as appropriate, those charged with governance do not provide sufficient
information that supports that the entity is in compliance with laws and regulations and, in the auditor’s
judgment, the effect of the suspected non-compliance may be material to the financial statements, the
auditor shall consider the need to obtain legal advice.
 If sufficient information about suspected non-compliance cannot be obtained, the auditor shall
evaluate the effect of the lack of sufficient appropriate audit evidence on the auditor’s opinion.
 The auditor shall evaluate the implications of non-compliance in relation to other aspects of the audit,
including the auditor’s risk assessment and the reliability of written representations, and take
appropriate action.
SA-260
Communication with Those Charged with Governance
25. Significant Difficulties in the Audit
M/s Manidhari & Associates have been appointed as an auditor of JIN Limited, a multinational company dealing
in spare parts. During the course of audit, CA Manidhari is facing many problems including the problem of not
getting the desired information from the management. Accordingly, he decided to communicate with those
charged with the governance about significant difficulties encountered during the audit. CA Manidhari seeks
your guidance on matters which can be considered as significant difficulties as per SA 260. (RTP-May-2022)
OR
State the significant difficulties encountered during audit with reference to SA-260. (May-2015)
Ans. As per SA 260 “Communication with Those Charged with Governance”, significant difficulties encountered
during the audit may include such matters as:

CA Final Audit - By CA. Sarthak Niraj Jain 17


Auditing Standards, Statements and Guidance Notes - An Overview

• Significant delays by management, the unavailability of entity personnel, or an unwillingness by


management to provide information necessary for the auditor to perform the auditor’s procedures.
• An unnecessarily brief time within which to complete the audit.
• Extensive unexpected effort required to obtain sufficient appropriate audit evidence.
• The unavailability of expected information.
• Restrictions imposed on the auditor by management.
• Management’s unwillingness to make or extend its assessment of the entity’s ability to continue as a
going concern when requested.
In some circumstances, such difficulties may constitute a scope limitation that leads to a modification of
the auditor’s opinion.as per SA 705 (Revised), Modifications to the Opinion in the Independent Auditor’s
Report.
26. Communication to TCWG & Mandatory Review Areas
Mr. BK, Partner in M/s. BK and Associates, as part of their audit presentation to the Audit Committee of M/s.
XYZ Limited, a listed company, highlighted the following:
• Difficulties faced during the audit
• Disagreements with the management
• Management Letter Points
• Draft Management Representation letter to be provided by the Company in connection with the audit.
Some of the Audit Committee members were not happy with the above presentation and asked Mr. BK to take it
back and submit directly to the Board. They believe that Audit Committee is not the forum for discussing such
problems and this has to be sorted out between auditors and the management. Please comment on the above.

Ans. (i)
FAST
As per SA 260 – “Communication with Those Charged with Governance”:
(Nov-2020-New)

CA.Sarthak Niraj Jain


The statutory auditor of the Company is having an obligation to bring certain matters to the attention
of those in charge of governance, which inter alia includes aspects such as –
 Difficulties faced by them during the audit
 Disagreements with the management
 Management Letter points
 Draft Management Representation letter to be provided by the Company in connection with the
audit.
(ii) AS per LODR Regulations:
 The Audit Committee is also having an obligation to mandatorily review certain areas before
providing their recommendations/inputs to the board. Given below are the areas required to be
mandatorily reviewed by the ACM in the case of listed companies
 The Audit Committee shall mandatorily review among other points the following information
as per LODR Regulations:
 Management discussion and analysis of financial condition and results of operations;
 Statement of significant related party transactions (as defined by the Audit Committee),
submitted by management;
 Management letters / letters of internal control weaknesses issued by the statutory auditors;
 Internal audit reports relating to internal control weaknesses;
 The auditor should further ascertain whether the Management Discussion and Analysis report
includes discussion on the matters stipulated. Where certain deficiencies or adverse findings
are noted by the Audit Committee, the auditor will be required to see that these have been suitably
dealt with by the management in the report on corporate governance.

18 Ab Audit Hoga Sabse Scoring - By CA. Sarthak Niraj Jain


Auditing Standards, Statements and Guidance Notes - An Overview

(iii) In the instant case: - Mr. BK, Partner in M/s BK and Associates highlighted the facts such as:
 difficulties faced during the audit,
 disagreements with the management,
 managements letters points and
 draft management letters to be provided by the Company in connection with the audit.
However, some of the audit committee members were not happy and as according to them audit
committee is not the forum for discussing such problems.
(iv) Conclusion: -Contention of those audit committee members regarding problems to be sorted out
between auditors and the management is not in order as Audit Committee is required to
mandatorily review the same in accordance with Schedule II of SEBI (LODR) Regulations, 2015
27. Modes of Communication – Factors
Factors governing modes of communication of auditor with those charged with governance.
Ans. Factors governing modes of communication of auditor with those charges with governance
As per SA 260, “Communication with Those Charge with Governance” the auditor may decide whether to
communicate orally or in writing, the extent of detail or summarisation in the communication, and
whether to communicate in a structured or unstructured manner) may be affected by such factors as:
(i) The size, operating structure, control environment, and legal structure of the entity.
(ii) In the case of an audit of special purpose financial statements, whether the auditor also audits the
entity’s general purpose financial statements.
(iii) Legal requirements. In some jurisdictions, a written communication with TCWG is required in a
prescribed form by local law.
(iv)
FAST
The expectations of TCWG, including arrangements made for periodic meetings or communications
with the auditor.
(v) The amount of ongoing contact and dialogue the auditor has with those charged with governance.
(vi) CA.Sarthak Niraj Jain
Whether there have been significant changes in the membership of a governing body
SA-265
Communicating Deficiencies in Internal Control to Those Charged with Governance
and Management
28. Significant Deficiencies in Internal Control
CA. N has been appointed as an auditor of TRP Ltd. While conducting the audit he has identified some
deficiencies in the Internal control. He needs to determine whether a deficiency or combination of deficiencies
in internal control constitutes a "significant deficiency" and has to communicate them in writing to those
charged with Governance and management on a timely basis. Guide CA. N with some examples of matters to be
considered while determining 'significant deficiency' in internal control with reference to relevant SA
(Nov-2020-New)
Ans. (i) Significant deficiencies: - As per SA 265 “Communicating Deficiencies in Internal Control to Those
Charged with Governance and Management”, significant deficiency in internal control means:
 A deficiency or combination of deficiencies in internal control that, in the auditor’s professional
judgement, is of sufficient importance to merit the attention of those charged with governance.
(ii) Examples of matters that CA N, auditor of TRP Ltd may consider in determining whether a
deficiency or combination of deficiencies in internal control constitutes a significant deficiency
include:
 The likelihood of the deficiencies leading to material misstatements in the financial statements in
the future.
 The susceptibility to loss or fraud of the related asset or liability.

CA Final Audit - By CA. Sarthak Niraj Jain 19


Auditing Standards, Statements and Guidance Notes - An Overview

 The subjectivity and complexity of determining estimated amounts, such as fair value
accounting estimates.
 The financial statement amounts exposed to the deficiencies.
 The volume of activity that has occurred or could occur in the account balance or class of
transactions exposed to the deficiency or deficiencies.
 The importance of the controls to the financial reporting process; for example:
 General monitoring controls (such as oversight of management).
 Controls over the prevention and detection of fraud.
 Controls over the selection and application of significant accounting policies.
 Controls over significant transactions with related parties.
 Controls over significant transactions outside the entity’s normal course of business.
 Controls over the period-end financial reporting process (such as controls over non-recurring
journal entries).
 The cause and frequency of the exceptions detected as a result of the deficiencies in the controls.
 The interaction of the deficiency with other deficiencies in internal control.
29. Indicators of Significant Deficiencies in Internal Control
Auditors are required to obtain an understanding of internal control relevant to the audit when identifying and
assessing its effectiveness and risk of material misstatement. During the course of audit of ABC Ltd., you
observed that significant deficiency exists in the internal control system and you want to ascertain the same.
Elucidate the various indicators of significant deficiencies which will help you in assessing the efficiency of
internal control system of the organization. (Jan-2021-New)
Ans. (i)
FAST
In the given case of ABC Ltd: - Auditors, while conducting audit has come across significant deficiency
existing in the internal control system and also auditors wanted to ascertain that deficiency.
(ii)
CA.Sarthak Niraj Jain
As per SA 265, “Communicating Deficiencies in Internal Control to Those Charged with
Governance and Management “: - Indicators of significant deficiencies in internal control include, for
example:
a) Evidence of ineffective aspects of the control environment, such as:
1. Indications that significant transactions in which management is financially interested are
not being appropriately scrutinised by those charged with governance.
2. Identification of management fraud, whether or not material, that was not prevented by the
entity’s internal control.
3. Management’s failure to implement appropriate remedial action on significant deficiencies
previously communicated.
b) Absence of a risk assessment process within the entity where such a process would ordinarily be
expected to have been established.
c) Evidence of an ineffective entity risk assessment process, such as management’s failure to
identify a risk of material misstatement that the auditor would expect the entity’s risk assessment
process to have identified.
d) Evidence of an ineffective response to identified significant risks (e.g., absence of controls over
such a risk).
e) Misstatements detected by the auditor’s procedures that were not prevented, or detected and
corrected, by the entity’s internal control.
f) Disclosure of a material misstatement due to error or fraud as prior period items in the current
year’s Statement of Profit and Loss.
g) Evidence of management’s inability to oversee the preparation of the financial statements.

20 Ab Audit Hoga Sabse Scoring - By CA. Sarthak Niraj Jain


Auditing Standards, Statements and Guidance Notes - An Overview

SA-299
Joint Audit of Financial Statements
30. Responsibility and Co-ordination among Joint Auditors
Dice Ltd. appointed two CA firms MN & Associates and PQ & Co. as joint auditors for conducting audit for the
year ended 31st March, 2022. In the course of audit, it has been observed that there is a major understatement
in the value of inventory. The inventory valuation work was looked after by MN & Associates but there was no
documentation for the division of the work between the joint auditors. Comment on the above situation with
regard to responsibilities among joint auditors. (May-2019-New)
OR
ABC & Co. and DEF & Co. Chartered Accountant firms were appointed as joint auditors of Good Health Care Ltd.
for 2020-21. An investigation was conducted under Companies Act 2013 during March 2022 and observed
gross understatement of Revenue. The revenue aspects were looked after by DEF & Co, but there was no
documentation for the division of work between the joint auditors. Comment.
Ans. (i) As per SA 299, “Joint Audit of Financial Statements”, where joint auditors are appointed, they should,
by mutual discussion, divide the audit work among themselves.
 The division of the work would usually be in terms of audit identifiable units or specified area.
 In some cases due to the nature of the business entity under audit, such a division of the work may
not be possible.
 In such situations: - The division of the work may be with reference to items of assets or
liabilities or income or expenditure or with reference to period of time.
 The division of the work among joint auditors as well as the areas of work to be covered by all of
them should be adequately documented and preferably communicated to the entity.

(ii)
FAST
In respect of the work divided amongst the joint auditors, each joint auditor is responsible only for

CA.Sarthak Niraj Jain


the work allocated to him, whether or not he has made a separate report on the work performed by
him. On the other hand the joint auditors are jointly and severally responsible in respect of the audit
conducted by them as under:
i. in respect of the audit work which is not divided among the joint auditors and is carried out by
all of them;
ii. in respect of decisions taken by all the joint auditors under audit planning in respect of common
audit areas concerning the nature, timing and extent of the audit procedures to be performed by
each of the joint auditors;
iii. in respect of matters which are brought to the notice of the joint auditors by any one of them and
on which there is an agreement among the joint auditors;
iv. for examining that the financial statements of the entity comply with the requirements of the
relevant statute;
v. for ensuring presentation and disclosure of the financial statements as required by the
applicable financial reporting framework;
vi. for ensuring that the audit report complies with the requirements of the relevant statutes, the
applicable Standards on Auditing and the other relevant pronouncements issued by ICAI.
The joint auditors shall also discuss and document the nature, timing, and the extent of the audit
procedures for common and specific allotted areas of audit to be performed by each of the joint
auditors and the same shall be communicated to those charged with governance.
After identification and allocation of work among the joint auditors, the work allocation document
shall be signed by all the joint auditors and the same shall be communicated to those charged with
governance of the entity.

CA Final Audit - By CA. Sarthak Niraj Jain 21


Auditing Standards, Statements and Guidance Notes - An Overview

Hence, in respect of audit work divided among the joint auditors, each joint auditor shall be
responsible only for the work allocated to such joint auditor including proper execution of the audit
procedures.
In the instant case. Dice Ltd. appointed two MN & Associates and PQ & Co. as joint auditor for conducting
audit. As observed during the course of audit that there is a major understatement in the value of
inventory and the inventory valuation work was looked after by MN & Associates.
(iii) Conclusion: -In view of SA 299 MN & Associate will be held responsible for the same as inventory
valuation work was looked after by MN & Associates only. Further, there is violation of SA 299 as
the division of work has not been documented.
31. Reporting Responsibility
Excellent Bank Ltd. is a Public Limited Company. The said Bank has various branches all over India. The Bank
appoints 3 Joint Auditors for the financial year ending 31/03/2022. All the 3 Joint Auditors divide the work
with mutual consent. Verification of Consolidation, however, remained undivided. All branches and zones were
divided amongst the 3 Joint Auditors. During audit of zones, CA. Z, one of the Joint auditors expressed a concern
about internal control in one of the large corporate branches situated. in his zone. The irregularity was not
reported in the final accounts as the other 2 Joint Auditors were not in favour of reporting and decision of not
reporting the same was taken on the basis of majority. Subsequently, fraud was detected in the said branch
which was audited by CA. Z. The Bank seeks your advice about the responsibility of the 3 Joint Auditors in the
above situation. (Nov-2019-Old)
Ans. (i) As per SA 299, “Joint Audit of Financial Statements” where joint auditors are appointed, they should,
by mutual discussion, divide the audit work among themselves
 The division of the work would usually be in terms of audit identifiable units or specified area. In
respect of the audit work divided among the joint auditors, each joint auditor is responsible only


FAST
for the work allocated to him, whether or not he has prepared a separate audit of the work
performed by him.
On the other hand all the joint auditors are jointly and severally responsible for the audit work

CA.Sarthak Niraj Jain


which is not divided among the joint auditors and. is carried out jointly and matters which are
brought to the notice of the joint auditors by any one of them and on which there is an agreement
among the joint auditors.
(ii) In the instant case: - Excellent Bank Ltd. Appoints 3 joint auditor for the financial year ending
31.03.2019. All the joint auditors divided the work with mutual consent. The only work which
remained undivided was verification of Consolidation. In accordance with SA 299, all the joint
auditors are responsible for the same.
(iii) Analysis of the case
 Further, during audit of zone, CA Z, one of the joint auditors expressed a concern about internal
control in one of the large corporate branches situated in his zone, however, this irregularity
was not reported as 2 of the joint auditors (i.e. majority of the joint auditors ) were not in favour of
the same.
 Later on, fraud has been detected in the same branch which was audited by CA. Z. As per SA 299,
Before finalizing their audit report, the joint auditors shall discuss and communicate with each
other their respective conclusions that would form the content of the audit report.
 Further, a joint auditor is not bound by the views of the majority of the joint auditors regarding
the opinion or matters to be covered in the audit report and shall express opinion formed by the
said joint auditor in separate audit report in case of disagreement.
(iv) Conclusion: - in the present scenario, CA. Z brought this matter in the notice of the other 2 joint
auditors and the decision for not reporting was taken on majority basis and no separate opinion was
expressed through separate audit report pointing out irregularity. Thus, all the 3 joint auditors will
be held responsible for the fraud detected in the branch audited by CA. Z as per SA 299.

22 Ab Audit Hoga Sabse Scoring - By CA. Sarthak Niraj Jain


Auditing Standards, Statements and Guidance Notes - An Overview

SA-300
Planning an Audit of Financial Statements
32. Changes in Audit Plan
Mr. S & Mr. J are a senior and junior articled assistant respectively, in a renowned audit firm. Both were
assigned statutory audit of a manufacturing company. Mr. S instructed his junior to draft an audit plan by
taking reference from a similar client (a partnership firm) who was engaged in the same business. Mr. J was
confused as to how that reference could suit in this case, since the nature and extent of planning would vary for
both clients. After few days, the audit work commenced. During the course of the audit, certain events took
place, which made Mr. J to rethink about the audit plan initially designed. He approached Mr. S and enquired
about when would an audit plan require a change. Comment about both the situations face by Mr. J in the above
situation. (MTP-Dec-2021)
Ans. SA 200 “Overall Objectives of the Independent Auditor and the Conduct of an Audit in accordance with
Standards on Auditing” states that in order to achieve the overall objectives of the audit, the auditor shall use
the objectives stated in relevant SAs in planning and performing the audit. Without a careful plan, the overall
objective of an audit may not be achieved. The audit planning is necessary to conduct an effective audit, in an
efficient and timely manner. So far as the nature of planning is concerned, it would vary according to-
(i) Size and Complexity of the Auditee - If the size and complexity of organization of which audit is to be
conducted is large, then much more planning activities would be required as compared to an entity whose
size and complexity is small.
(ii) Past Experience & Expertise - The key engagement team members’ previous experience & expertise
also contributes towards variation in planning activities.
(iii) Change in Circumstances - Another factor contributing towards variation in planning activities is

FAST
change in circumstances.
Changes to Audit Planning: The auditor should update and change the overall audit strategy and audit plan
as necessary during the course of the audit. The auditor may need to modify the overall audit strategy and audit

CA.Sarthak Niraj Jain


plan due to the factors such as:
(i) result of unexpected events,
(ii) changes in conditions, or
(iii) the audit evidence obtained from the results of audit procedures.
Further, the auditor would also have to modify the nature, timing & extent of further audit procedures,
based on the revised considerations of assessed risks. This may be the case when information coming to the
auditor differs significantly from the information when he planned the audit process. In addition to the
above, there may be possibilities of change in - law, notifications, government policies, which warrants
updation of overall audit strategy and audit plan.
SA -315
Identifying and Assessing the Risks of Material Misstatement through Understanding
the Entity and its Environment
33. Points of Indication - Risks Identified may be Significant
You are engaged by M/s. Real Ltd. as an internal auditor for the financial year 2020-2021. While applying risk
assessment procedures of inquiring from management and various analytical procedures, you have identified
some risks which in your opinion may lead to material misstatement at the financial level and assertion level.
Which factors as an auditor will you consider while exercising judgement as to whether such risks are
significant risks or not? (Jan-2021-New)
OR
The identified risks are assessed by Auditor as to its significance on account of its likely impact, by way of
material misstatement appearing in financial statements or by affecting internal control system. What may be
the points of indication that may direct the Auditor to judge that the risks identified may be significant?
(Nov-2018-New)

CA Final Audit - By CA. Sarthak Niraj Jain 23


Auditing Standards, Statements and Guidance Notes - An Overview

Ans. (i) The internal auditor of Real Ltd. has identified some risks while he was applying risk assessment
procedures and various analytical procedures.
(ii) As per SA 315 “Identifying and Assessing the Risks of Material Misstatement through
Understanding the Entity and Its Environment”, in exercising judgment as to which risks are
significant risks, the auditor shall consider at least the following:
 Whether the risk is a risk of fraud;
 Whether the risk is related to recent significant economic, accounting, or other developments
like changes in regulatory environment, etc., and, therefore, requires specific attention;
 The complexity of transactions;
 Whether the risk involves significant transactions with related parties;
 The degree of subjectivity in the measurement of financial information related to the risk,
especially those measurements involving a wide range of measurement uncertainty; and
 Whether the risk involves significant transactions that are outside the normal course of
business for the entity, or that otherwise appear to be unusual.
SA-320
Materiality in Planning and Performing an Audit
34. Use of Benchmark in determining Materiality
You are being appointed as the auditor of X Ltd. for the first time. You want to determine the materiality level
and for that you have applied percentage to choose benchmark as a starting point in determining materiality
for the financial statements as a whole. What are the factors that may affect the identification of materiality
while auditing? (Nov-2020-Old)
OR

FAST
Mr. X was appointed as the auditor of M/s Easygo Ltd. and intends to apply the concept of materiality for the
financial statements as a whole. Please guide him as to the factors that may affect the identification of an
appropriate benchmark for this purpose. (MTP-May-2018)
Ans.
CA.Sarthak Niraj Jain
(i) Provision: -SA 320 “Materiality in Planning and Performing an Audit” prescribes :
 The use of Benchmarks in Determining Materiality for the Financial Statements as a Whole.
 Determining materiality involves the exercise of professional judgment.
 A percentage is often applied to a chosen benchmark as a starting point in determining
materiality for the financial statements as a whole.
(ii) As auditor of X Ltd., for determining the materiality level, Factors that may affect the
identification of an appropriate benchmark include the following:
 The elements of the financial statements (for example, assets, liabilities, equity, revenue,
expenses);
 Whether there are items on which the attention of the users of the particular entity’s financial
statements tends to be focused (for example, for the purpose of evaluating financial performance
users may tend to focus on profit, revenue or net assets);
 The nature of the entity, where the entity is at in its life cycle, and the industry and economic
environment in which the entity operates;
 The entity’s ownership structure and the way it is financed (for example, if an entity is financed
solely by debt rather than equity, users may put more emphasis on assets, and claims on them, than
on the entity’s earnings); and
 The relative volatility of the benchmark.
35. Re-evaluation of the Materiality Concept
As an auditor of RST Ltd. Mr. P applied the concept of materiality for the financial statements as a whole. On the
basis of obtaining additional information of significant contractual arrangements that draw attention to a
particular aspect of a company's business, he wants to re-evaluate the materiality concept. Please, guide him.
(Study Material)

24 Ab Audit Hoga Sabse Scoring - By CA. Sarthak Niraj Jain


Auditing Standards, Statements and Guidance Notes - An Overview

OR
Y & Co., Chartered Accountants have come across in the course of audit of a company that certain machinery,
had been imported for production of new product. Although the Auditors have applied the concept of
materiality for the Financial Statements as a whole, they now want to re-evaluate the materiality Concept for
this transaction involving foreign exchange. Give your views in this regard. (MTP-May-2019)(May-2018-New)
Ans. (i) In the instant case: - Mr. P, as an auditor of RST Ltd. has applied the concept of materiality for the
financial statements as a whole. But he wants to re-evaluate the materiality concept on the basis of
additional information of significant contractual arrangements which draws attention to a particular
aspect of the company’s business.
(ii) SA 320 on Materiality For FS as a whole, Separate for Individual Assertions if Needed
 As per SA 320 “Materiality in Planning and Performing an Audit”, while establishing the overall
audit strategy, the auditor shall determine materiality for the financial statement as a whole.
 He should set the benchmark on the basis of which he performs his audit procedure. If, in the
specific circumstances of the entity, there is one or more particular classes of transactions, account
balances or disclosures for which misstatements of lesser amounts than the materiality for the
financial statements as a whole could reasonably be expected to influence the economic
decisions of users taken on the basis of the financial statements,
 The auditor shall also determine the materiality level or levels to be applied to those particular
classes of transactions, account balances or disclosures.
(iii) Revision of Materiality Level
 The auditor shall revise materiality for the financial statements in the event of becoming aware
of information during the audit that would have caused the auditor to have determined a different


FAST
amount (or amounts) initially.
If the auditor concludes a lower materiality for the same, then he should consider the fact that
whether it is necessary to revise performance materiality and whether the nature, timing and

CA.Sarthak Niraj Jain


extent of the further audit procedures remain appropriate.
(iv) Conclusion: -Thus, Mr. P can re-evaluate the materiality concepts after considering the necessity of
such revision.

SA-330
The Auditor’s Responses to Assessed Risks
36. Factors Warranting Re-Test of Controls
In the course of audit of Ambika Ltd, its auditor wants to rely on audit evidence obtained in previous audit in
respect of effectiveness of internal controls instead of retesting the same during the current audit. As an
advisor to the auditor kindly caution him about the factors that may warrant a re-test of controls.
(RTP-May-2022)(MTP-Nov-2019)(RTP-Nov-2018)
Ans.  As per SA 330 on “The Auditor’s Responses to Assessed Risks”, changes may affect the relevance of
the audit evidence obtained in previous audits such that there may no longer be a basis for continued
reliance.
 The auditor’s decision on whether to rely on audit evidence obtained in previous audits for control is a
matter of professional judgment. In addition, the length of time between retesting such controls is
also a matter of professional judgment.
 Factors that may warrant a re-test of controls are-
 A deficient control environment.
 Deficient monitoring of controls.
 A significant manual element to the relevant controls.

CA Final Audit - By CA. Sarthak Niraj Jain 25


Auditing Standards, Statements and Guidance Notes - An Overview

 Personnel changes that significantly affect the application of the control.


 Changing circumstances that indicate the need for changes in the control.
 Deficient general IT-controls.
SA-402
Audit Considerations Relating to an Entity Using a Service Organization
37. Understanding of the User Entity
G Ltd. is a mobile phone operating company. Barring the marketing function it had outsourced the entire
operations like maintenance of mobile infrastructure, customer billing, payroll, accounting functions, etc. Assist
the auditor of G Ltd. as to how he can obtain an understanding of how G Ltd. uses the services of the outsourced
agency in its operations. (MTP-Nov-2019) (RTP-Nov-2018)
OR
In the course of the audit of R Ltd., the audit manager of ABC & Co. observed that R Ltd. has outsourced certain
activities to an outsourcing agency. As the engagement partner guide the audit manager in the assessment of
services provided by the outsourcing agency in relation to the audit. (May-2018)
Ans. Provision: - As per SA 402 on Audit Considerations Relating to an Entity Using a Service Organisation: -
When obtaining an understanding of the user entity in accordance with SA 315 “Identifying and Assessing
the Risks of Material Misstatement through Understanding the Entity and its Environment”, the user
auditor shall obtain an understanding of how a user entity uses the services of a service organisation in the
user entity’s operations, including:
(i) The nature of the services provided by the service organisation and the significance of those services
to the user entity, including the effect thereof on the user entity’s internal control;
(ii)

(iii)
FAST
The nature and materiality of the transactions processed or accounts or financial reporting processes
affected by the service organisation;
The degree of interaction between the activities of the service organisation and those of the user

(iv)
CA.Sarthak Niraj Jain
entity; and
The nature of the relationship between the user entity and the service organisation, including the
relevant contractual terms for the activities undertaken by the service organisation.
38. Controls at a Sub-Service Organisation
When a sub-service organization performs services for a service organization, there are two alternative
methods of presenting the description of controls. The service organization determines which method will be
used. As a user auditor what information would you obtain about controls at a sub-service organization?
(Study Material)(May-2015)
Ans. (i) Provision: -As per SA 402 - Audit Considerations relating to an Entity Using a Service Organisation
 A user entity may use a service organisation that in turn uses a sub -service organisation to
provide some of the services provided to a user entity that are part of the user entity’s
information system relevant to financial reporting.
 The sub -service organisation may be a separate entity from the service organisation or may be
related to the service organisation.
(ii) User auditor shall obtain an understanding of how a user entity uses the services of a sub-service
organisation in the user entity’s operations, including:
 The nature of the services provided by the sub-service organisation and the significance of those
services to the user entity, including the effect thereof on the user entity’s internal control;
 The nature and materiality of the transactions processed or accounts or financial reporting
processes affected by the sub-service organisation;
 The degree of interaction between the activities of the sub-service organization, service
organisation and those of the user entity; and

26 Ab Audit Hoga Sabse Scoring - By CA. Sarthak Niraj Jain


Auditing Standards, Statements and Guidance Notes - An Overview

 The nature of the relationship between the user entity and the service organization and service
organisation, including the relevant contractual terms for the activities undertaken by the sub-
service organisation.
(iii) Reporting: -Type 1 / 2 Reports
 If the user auditor is unable to obtain a sufficient understanding from the user entity, the user
auditor shall obtain that understanding by application of the following two methods of
presenting description of internal controls i.e.
 Type 1 report; or
 Type 2 report.
 If a service organisation uses a subservice organisation, the service auditor's report may either
include or exclude the subservice organisation's relevant control objectives and related controls
in the service organisation's description of its system and in the scope of the service auditor's
engagement. These two methods of reporting are known as the inclusive method and the carve -
out method respectively.
 In either method, the service organisation includes in its description of controls a description of
the functions and nature of the processing performed by the sub - service organisation.
 If the Type 1 or Type 2 report excludes the control at a subservice organization and the services
provided by the subservice organization are relevant to the audit of the user entity’s financial
statements, the user auditor is required to apply the requirements of the SA 402 in respect of the
subservice organization.
39. Reporting by the User Auditor
ENN Limited is availing the services of APP Private Limited for its payroll operations. Payroll cost accounts for

FAST
65% of total cost for ENN Limited. APP Limited has provided the type 2 report as specified under SA 402 for its
description, design and operating effectiveness of control. APP Private Limited has also outsourced a material
part of payroll operation M/s SMP & Associates in such a way that M/s SMP & Associates is sub-service

CA.Sarthak Niraj Jain


organization to ENN Limited. The Type 2 report which was provided by APP Private Limited was based on
carve-out method as specified under SA 402. CA Raman while reviewing the unmodified audit report drafted by
his assistant found that, a reference has been made to the work done by the service auditor. CA Raman hence
asked his assistant to remove such reference and modify report accordingly. Comment whether CA Raman is
correct in removing the reference of the work done by service auditor?
(MTP-May-2021)(RTP-Nov-2020)(MTP-Dec-2021)
Ans. (i) Provision: - As per SA 402, “Audit Considerations Relating to an Entity Using a Service
Organisation”:
 The user auditor shall modify the opinion in the user auditor’s report in accordance with SA 705,
“Modifications to the Opinion in the Independent Auditor’s Report”, if the user auditor is
unable to obtain sufficient appropriate audit evidence regarding the services provided by the
service organisation relevant to the audit of the user entity’s financial statements.
 The user auditor shall not refer to the work of a service auditor in the user auditor’s report
containing an unmodified opinion unless required by law or regulation to do so.
 If such reference is required by law or regulation, the user auditor’s report shall indicate that the
reference does not diminish the user auditor’s responsibility for the audit opinion.
(ii) Conclusion:- Thus, in view of above, contention of CA. Raman in removing reference of the work
done by service auditor is in order as in case of unmodified audit report, user auditor cannot refer to
the work done by service auditor.
SJ Note – Further explanation on matters to be reported in the report, in case where user auditor issues
modified opinion, should also be given

CA Final Audit - By CA. Sarthak Niraj Jain 27


Auditing Standards, Statements and Guidance Notes - An Overview

SA-450
Evaluation of Misstatements Identified during the Audits
40. Consideration of Uncorrected & Unidentified Misstatements
The auditor should take into account the aggregate of all uncorrected misstatements including those involving
estimates in his assessment of materiality in audit.
Ans. (i) Determination of Materiality for FS as a Whole
 SA 450 “Evaluation of Misstatements identified during the Audit” requires that in forming his
opinion on the financial information, the auditor should consider all material aspects, either
individual or in aggregate which are relatively important for true and fair view of financial
statements.
 In this context, the auditor should consider whether the effect of aggregate uncorrected
misstatements on the financial information is material.
 Qualitative considerations also influence an auditor in reaching a conclusion as to whether the
misstatements are material.
(ii) Composition of Aggreggate Uncorrected Misstatement: -As per SA 320, the aggregate of
uncorrected misstatements comprises:
 specific misstatements identified by the auditor including the net effect of uncorrected
misstatements identified during the audit of previous periods; and
 the auditor’s best estimate of other misstatements which cannot be specifically identified (that is,
projected errors).


FAST
The analytical procedures employed by the auditor may give him some indication about the
existence of misstatements, which can be further substantiated by him through estimates
process.

CA.Sarthak Niraj Jain


 When an auditor uses audit sampling to test an account balance or class of transactions, he
projects the amount of known misstatements identified by him in his sample to the items in
the balance or class from which his sample was selected.
 That projected misstatement, along with the results of other substantive tests,
contributes to the auditor’s assessment of aggregate misstatement in the balance or class.
(iii) Impact of Misstatements & Assessment of Materiality:
 If the aggregate of the uncorrected misstatements that the auditor has identified approaches the
materiality level, or if auditor determines that the aggregate of uncorrected misstatements
causes the financial information to be materially misstated, he should consider requesting the
management to adjust the financial information or extending his audit procedures.
(iv) Reporting
 If the management refuses to adjust the financial information and the results of extended audit
procedures do not enable the auditor to conclude that the aggregate of uncorrected
misstatements is not material, the auditor should express a qualified or adverse opinion, as
appropriate.
SA-500
Audit Evidence
41. Using the Work of Management Expert
Gap Ltd. possesses some investment for which there is no ready market and to assess its fair market value it
hires an expert, the result of which it can use in preparing its financial statement. Being an Auditor of the
Company, state the matters which may affect the nature, timing and extent of audit procedure to be adopted by
you in the instant case. (May-2018-Old)

28 Ab Audit Hoga Sabse Scoring - By CA. Sarthak Niraj Jain


Auditing Standards, Statements and Guidance Notes - An Overview

Ans. (i) As per SA 500 “Audit Evidence” if any information is prepared by the entity using the work of
management expert and auditor has to consider that information as audit evidence, he is required to
evaluate the competence, capability and objectivity of that expert.
(ii) Matters which may affect nature, timing and extent of audit procedures in such a case are:
 The nature and complexity of the matter to which the management’s expert relates.
 The risks of material misstatement in the matter.
 The availability of alternative sources of audit evidence.
 The nature, scope and objectives of the management’s expert’s work.
 Whether the management’s expert is employed by the entity, or is a party engaged by it to
provide relevant services.
 The extent to which management can exercise control or influence over the work of the
management’s expert.
 Whether the management’s expert is subject to technical performance standards or other
professional or industry requirements.
 The nature and extent of any controls within the entity over the management’s expert’s work.
 The auditor’s knowledge and experience of the management’s expert’s field of expertise.
 The auditor’s previous experience of the work of that expert.
42. Evaluating the Work of Management’s Expert
PDJ Ltd. has engaged an actuary to ascertain actuarial valuation of defined benefit obligations viz. Gratuity and
Leave Encashment liabilities. As an auditor of PDJ Ltd. you would like to use the report of the actuary as audit
evidence. How would you evaluate the work of the actuary? (Jan-2021-New)
OR

FAST
CA. Pointer had been appointed as an Auditor of Textile Ltd. During the course of audit, it was observed that
inventory including work-in-process has been valued by the Management by using experts hired by them.
Analyse relevant factors to decide as to whether CA. Pointer should accept or not accept the findings from the

CA.Sarthak Niraj Jain


work of Management expert in valuation of inventories.
OR
(MTP-Nov-2020)

CA. Needle had been appointed as an auditor of M/s Fabric Ltd. in the course of audit, it had been observed that
inventory including workin-process had been valued by management by using experts hired by them. Analyse
relevant factors to decide as to whether or not to accept the findings from the work of management expert in
valuation of inventories. (May-2018-New)
OR
Y Ltd. engaged an actuary to ascertain its employee cost, gratuity and leave encashment liabilities. As the
auditor of Y Ltd., you would like to use the report of the actuary as an audit evidence. How do you evaluate the
work of the actuary? (May-2016)
Ans.  As per SA 500 “Audit Evidence”, when information to be used as audit evidence has been prepared
using the work of a management’s expert, the auditor shall, to the extent necessary, having regard to
the significance of that expert’s work for the auditor’s purposes-
(1) Evaluate the competence, capabilities and objectivity of that expert;
(2) Obtain an understanding of the work of that expert; and
(3) Evaluate the appropriateness of that expert’s work as audit evidence for the relevant assertion.
 The auditor may obtain information regarding the competence, capabilities and objectivity of a
management’s expert from a variety of sources, such as personal experience with previous work of
that expert; discussions with that expert; discussions with others who are familiar with that
expert’s work; knowledge of that expert’s qualifications; published papers or books written by
that expert.
 Aspects of the management’s expert’s field relevant to the auditor’s understanding may include
what assumptions and methods are used by the management’s expert, and whether they are
generally accepted within that expert’s field and appropriate for financial reporting purposes.

CA Final Audit - By CA. Sarthak Niraj Jain 29


Auditing Standards, Statements and Guidance Notes - An Overview

 The auditor may also consider the following while evaluating the appropriateness of the
management’s expert’s work as audit evidence for the relevant assertion:
(i) The relevance and reasonableness of that expert’s findings or conclusions, their consistency
with other audit evidence, and whether they have been appropriately reflected in the financial
statements;
(ii) If that expert’s work involves use of significant assumptions and methods, the relevance and
reasonableness of those assumptions and methods; and
(iii) If that expert’s work involves significant use of source data, the relevance, completeness,
and accuracy of that source data.
43. Using the Work of Management Expert
The auditor of SS Ltd. accepted the gratuity liability valuation based on the certificate issued by a qualified
actuary. However, the auditor noticed that the retirement age adopted is 65 years as against the existing
retirement age of 60 years. The company is considering a proposal to increase the retirement age. Comment.
(MTP-May-2018)
Ans. (i) SA 500 (Revised), “Audit Evidence” states that the
 Auditor has to evaluate the work of management expert, say, actuary, before adopting the same.
 The work of management expert is required to be evaluated in terms of following:
(a) Relevance and reasonableness of that expert findings and conclusion.
(b) Relevance and reasonableness of assumptions and methods used; and
(c) Relevance, completeness and accuracy of source data.
 There is no doubt that appropriateness, reasonableness of assumptions and methods used are
the responsibility of the expert, but the auditor has to determine whether they are reasonable

FAST
based on the auditor’s knowledge of the client’s business and result of his audit procedures.
(ii) In the instant case -a qualified actuary has issued a certificate for gratuity liability valuation, for
which retirement age adopted is 65 years against the existing retirement age of 60 years; however,

CA.Sarthak Niraj Jain


the company is considering a proposal to increase the retirement age.
(iii) Conclusion:
 In view of provisions of SA 500 as discussed above, the assumption made by actuary has no
relevance and reasonableness as presently retiring age is of 60 years.
 Hence the auditor is required to bring out the facts to the notice of management and advice the
modification accordingly. In case of failure of compliance of the same the auditor may qualify the
report.
SA-501
Audit Evidence - Specific Considerations for Selected Items
44. Impracticable to Attend Physical Inventory Count
Coccyx Ltd. supplies navy uniforms across the country. The Company has 3 warehouses at different locations
throughout the India and 5 warehouses at the borders. The major stocks are generally supplied from the
borders. Coccyx Ltd. appointed M/s OPAQE & Co. to conduct its audit for the financial year 2021-22. Mr. P,
partner of M/s OPAQE & Co., attended all the physical inventory counting conducted throughout the India but
could not attend the same at borders due to some unavoidable reason.
You are required to advise M/s OPAQE & Co.,
(I) How sufficient appropriate audit evidence regarding the existence and condition of inventory may be
obtained?
(II) How is an auditor supposed to deal when attendance at physical inventory counting is impracticable?
(RTP-May-2021),(May-2018-Old),(RTP-May-2018)
Ans. (i) Special Consideration with Regard to Inventory: As per SA 501 “Audit Evidence- Specific
Considerations for Selected Items”, when inventory is material to the financial statements, the auditor
shall obtain sufficient appropriate audit evidence regarding the existence and condition of inventory
by:

30 Ab Audit Hoga Sabse Scoring - By CA. Sarthak Niraj Jain


Auditing Standards, Statements and Guidance Notes - An Overview

(a) Attendance at physical inventory counting, unless impracticable, to:


 Evaluate management’s instructions and procedures for recording and controlling the
results of the entity’s physical inventory counting;
 Observe the performance of management’s count procedures;
 Inspect the inventory; and
 Perform test counts; and
(b) Performing audit procedures over the entity’s final inventory records to determine whether
they accurately reflect actual inventory count results.

(ii) Attendance at Physical Inventory Counting Not Practicable:


 In some cases: - Attendance at physical inventory counting may be impracticable. This may be
due to factors such as the nature and location of the inventory,
 For example: - Where inventory is held in a location that may pose threats to the safety of the
auditor. The matter of general inconvenience to the auditor, however, is not sufficient to
support a decision by the auditor that attendance is impracticable.
 As per SA 200 “Overall Objectives of the Independent Auditor and the Conduct of an Audit in
Accordance with Standards on Auditing”: The matter of difficulty, time, or cost involved is
not in itself a valid basis for the auditor to omit an audit procedure for which there is no
alternative or to be satisfied with audit evidence that is less than persuasive.
 Where attendance is impracticable: - Alternative audit procedures, for example, inspection
of documentation of the subsequent sale of specific inventory items acquired or purchased prior
to the physical inventory counting, may provide sufficient appropriate audit evidence about

 FAST
the existence and condition of inventory.
Reporting: -In some cases, though, it may not be possible to obtain sufficient appropriate
audit evidence regarding the existence and condition of inventory by performing alternative

CA.Sarthak Niraj Jain


audit procedures. In such cases, SA 705 on Modifications to the Opinion in the Independent
Auditor’s Report, requires the auditor to modify the opinion in the auditor’s report as a result of
the scope limitation.
45. Unable to Attend Physical Inventory Count
RIM Private Ltd is engaged in the business of manufacturing of water bottles and is experiencing significant
increase in turnover year on year. During the financial year ended 31st March 2022, the company carried out a
detailed physical verification of its inventory and property, plant and equipment. You are the auditor of RIM
Private Ltd. The inventory as at the end of the year was ` 2.25 crores. Due to unavoidable circumstances, you
could not be present at the time of annual physical verification. Under the above circumstances how would you
ensure that the physical verification conducted by the management was in order?
(Study Material)(RTP-May-2020)
OR
You are the auditor of Easy Communications Ltd. for the year 2021–22. The inventory as at the end of the year
i.e. 31.3.22 was ` 2.25 crores. Due to unavoidable circumstances, you could not be present at the time of annual
physical verification. Under the above circumstances how would you ensure that the physical verification
conducted by the management was in order?
Ans.  SA 501 “Audit Evidence – Specific Considerations for Specific Items”, requires from the auditor that
when inventory is material to the financial statements, he shall obtain sufficient appropriate audit
evidence regarding the existence and condition of inventory by attendance at physical inventory
counting, unless impracticable, to:
 Evaluate management’s instructions and procedures for recording and controlling the results of
the entity’s physical inventory counting;
 Observe the performance of management’s count procedures;

CA Final Audit - By CA. Sarthak Niraj Jain 31


Auditing Standards, Statements and Guidance Notes - An Overview

 Inspect the inventory; and


 Perform test counts.
 When unable to present at the physical inventory count: -SA 501 provides that if the auditor is
unable to be present at the physical inventory count on the date planned due to unforeseen
circumstances, the auditor should take or observe some physical counts on an alternative date and
perform audit procedures on intervening transactions to assess whether the changes in inventory
between the date of physical count and the period end date are correctly recorded.
 The auditor would also verify the procedure adopted and treatment given for the discrepancies
noticed during the physical count. The auditor would also ensure that appropriate cut off procedures
were followed by the management.
 The auditor may also seek management’s written representation on
 the completeness of information provided regarding the inventory, and
 assurance with regard to adherence to laid down procedures for physical inventory count.
 Conclusion: - By following the above procedure it will be ensured that the physical verification
conducted by the management was in order.
46. Inventory under the Custody and Control of a Third Party
Your firm has been appointed as the statutory auditors of GBM Private Limited for the financial year 2021-22.
While verification of company’s inventories as on 31st March 2022, you found that the significant amount of
inventories belonging to the company are held by other parties. However, the company has kept all the
records of the inventories maintained by other parties. What is your duty as an auditor in order to ensure that
third parties are not such with whom the stock should not be held and the stock as disclosed in company’s
records actually belongs to them? (RTP-Nov-2019)
Ans.
FAST
As per SA 501, “Audit Evidence—Specific Considerations for Selected Items” when inventory under the
custody and control of a third party is material to the financial statements, the auditor shall obtain
sufficient appropriate audit evidence regarding the existence and condition of that inventory by

(i) CA.Sarthak Niraj Jain


performing one or both of the following:
Request confirmation from the third party as to the quantities and condition of inventory held on
behalf of the entity.
(ii) Perform inspection or other audit procedures appropriate in the circumstances, for example where
information is obtained that raises doubt about the integrity and objectivity of the third party, the
auditor may consider it appropriate to perform other audit procedures instead of, or in addition to,
confirmation with the third party.
Examples of other audit procedures include:
 Attending, or arranging for another auditor to attend, the third party’s physical counting of
inventory, if practicable.
 Obtaining another auditor’s report, or a service auditor’s report, on the adequacy of the third party’s
internal control for ensuring that inventory is properly counted and adequately safeguarded.
 Inspecting documentation regarding inventory held by third parties, for example, warehouse
receipts.
 Requesting confirmation from other parties when inventory has been pledged as collateral.
47. Inventory Counting at a Date other than the Date of the FSs
Moon Ltd. is a dealer in electronic appliances. The Company has a centralised warehouse at the outskirts of
Mumbai. The Auditors of the company M/s JK Associates normally attend the physical verification of stocks
carried out by the Management at the end of the financial year. However, on account of certain disturbances in
the region, the physical inventory counting could not be carried out at the year end. The stock taking is decided
to be done by management at some other date subsequently, after a month. In the light of the above facts:
Enumerate the audit procedures to be considered by M/s JK Associates, if physical inventory counting is
conducted at a date other than the date of the financial statements with reference to the relevant Standard on
Auditing. (Nov-2020-New)

32 Ab Audit Hoga Sabse Scoring - By CA. Sarthak Niraj Jain


Auditing Standards, Statements and Guidance Notes - An Overview

Ans. (i) As per SA 501 “Audit Evidence- Specific Considerations for Selected Items”
 When inventory is material to the financial statements, the auditor shall obtain sufficient
appropriate audit evidence regarding the existence and condition of inventory.
 For practical reasons, the physical inventory counting may be conducted at a date, or dates,
other than the date of the financial statements.
 (This may be done irrespective of whether management determines inventory quantities by an
annual physical inventory counting or maintains a perpetual inventory system.)
 (In either case, the effectiveness of the design, implementation and maintenance of controls over
changes in inventory determines whether the conduct of physical inventory counting at a date, or
dates, other than the date of the financial statements is appropriate for audit purposes.)
(ii) If physical inventory counting is conducted at a date other than the date of the financial statements,
the auditor, JK Associates, shall perform the following procedures:
(A) Attendance at physical inventory counting, unless impracticable, to:
o Evaluate management’s instructions and procedures for recording and controlling the
results of the entity’s physical inventory counting;
o Observe the performance of management’s count procedures;
o Inspect the inventory; and
o Perform test counts; and
(B) Performing audit procedures over the entity’s final inventory records to determine whether
they accurately reflect actual inventory count results.
(iii) Additional Audit Procedure:

FAST
In addition to above, auditor shall also perform audit procedures to obtain audit evidence about
whether changes in inventory between the count date and the date of the financial statements are
properly recorded.

CA.Sarthak Niraj Jain


 Relevant matters for consideration when designing audit procedures to obtain audit evidence
about whether changes in inventory amounts between the count date, or dates, and the final
inventory records are properly recorded include:
 Whether the perpetual inventory records are properly adjusted.
 Reliability of the entity’s perpetual inventory records.
 Reasons for significant differences between the information obtained during the physical
count and the perpetual inventory records.
SA -505
External Confirmations
48. Management Refusal to allow Auditor to Send Confirmation Request
Your firm has been appointed as the statutory auditors of AGM Private Limited for the financial year 2021-22.
While verification of company’s trade receivables as on 31st March 2022, accountant of AGM Ltd. has requested
you, not to send balance confirmations to a particular group of trade receivables since the said balances are
under dispute and the matter is pending in the Court. As a Statutory Auditor, how would you deal in this
situation? (Study Material)(RTP-May-2020)
Ans. SA 505 “External Confirmations”, establishes standards on the auditor’s use of external confirmation as a
means of obtaining audit evidence. If the management refuses to allow the auditor to a send a confirmation
request, the auditor shall:
(i) Inquire as to Management’s reasons for the refusal, and seek audit evidence as to their validity and
reasonableness,
(ii) Evaluate the implications of management’s refusal on the auditor’s assessment of the relevant risks
of material misstatement, including the risk of fraud, and on the nature, timing and extent of other
audit procedures, and
(iii) Perform alternative audit procedures designed to obtain relevant and reliable audit evidence.

CA Final Audit - By CA. Sarthak Niraj Jain 33


Auditing Standards, Statements and Guidance Notes - An Overview

If the auditor concludes that management’s refusal to allow the auditor to send a confirmation request is
unreasonable or the auditor is unable to obtain relevant and reliable audit evidence from alternative audit
procedures, the auditor shall communicate with those in charge of governance and also determine its
implication for the audit and his opinion.
49. Use of Negative Confirmations
M/s ABC & Co. LLP are appointed auditors of Sharp Company Ltd. for the year ended 31 st March. 2022. As part
of the audit process. They want to use confirmation procedures as audit evidence during the course of audit. In
view of the fact that positive confirmations are not responded favourably, the firm also intends to use negative
confirmation requests. What are The factors to be considered for the same ? (May-2019-Old)
Ans.  As per SA 505 “External Confirmations” factors to consider when designing confirmation requests
include
(i) Assertion being addressed.
(ii) Specific identified ROMM.
(iii) Layout & presentation of request.
(iv) Prior experience on the audit of similar engagements.
(v) Method of communication.
(vi) Management authorisation/encouragement to Confirming Party to respond to auditor.
(vii) Ability of Confirming Party to provide/confirm requested information.
 Factors to be considered for Negative Confirmation requests:
 As per SA 505, “External Confirmation”, Negative Confirmation is a request that the confirming
party respond directly to the auditor only if the confirming party disagrees with the



FAST
information provided in the request.
Negative confirmations provide less persuasive audit evidence than positive confirmations.
Accordingly, the auditor shall use negative confirmation requests as the sole substantive audit

CA.Sarthak Niraj Jain


procedure only when all of the following conditions are present:
(a) Low Risk of material misstatement and auditor has obtained sufficient appropriate audit
evidence regarding the operating effectiveness of controls.
(b) The population comprises a large number of small, homogeneous, account balances or
transactions.
(c) A very low exception rate is expected.
(d) The auditor is not aware of circumstances or conditions that would cause recipients of
negative confirmation requests to disregard such requests.
50. Response to Negative Confirmation Request
During the course of audit of Star Limited the auditor received some of the confirmation of the balances of
trade payables outstanding in the balance sheet through external confirmation by negative confirmation
request. In the list of trade payables, there are number of trade payables of small balances except one, old
outstanding of ` 15 Lacs, of whom, no confirmation on the credit balance received. Comment with respect to
Standard of Auditing. (Study Material)(RTP-May-2018)
Ans. (i) As per SA 505, “External Confirmation”
 Negative Confirmation is a request that the confirming party respond directly to the auditor only
if the confirming party disagrees with the information provided in the request.
 Negative confirmations provide less persuasive audit evidence than positive confirmations.
 In case of negative confirmation request, confirming parties may be more likely to respond
indicating their disagreement with a confirmation request when the information in the request is
not in their favor, and less likely to respond otherwise.

34 Ab Audit Hoga Sabse Scoring - By CA. Sarthak Niraj Jain


Auditing Standards, Statements and Guidance Notes - An Overview

(ii) In the instant case: - The auditor sent the negative confirmation requesting the trade payables
having outstanding balances in the balance sheet while doing audit of Star Limited. One of the old
outstanding of rupees 15 lacs has not sent the confirmation on the credit balance.
(iii) Non-response for negative confirmation request does not means that there is some misstatement as
negative confirmation request itself is to respond to the auditor only if the confirming party
disagrees with the information provided in the request.
(iv) Further Audit Procedure: -In the present case, considering the materiality of the account balance, the
auditor may examine subsequent cash disbursements or correspondence from third parties, and
other records, such as goods received notes.
51. Auditor’s Duties in case of Non-Availability of External Confirmation
Moon Limited replaced its statutory auditor for the Financial year 2021-22. During the course of audit, the new
auditor found a credit item of ` 5 lakhs. On enquiry, the company explained him that it is, a very old credit
balance. The trade payable had neither approached for the payment nor he is traceable. Under the
circumstances no confirmation of the credit balance is available.
Ans. (i) SA 505 “External Confirmations” provides that:
 If the auditor has determined that a response to a positive confirmation request is necessary to
obtain sufficient appropriate audit evidence, and
 Alternative audit procedures will not provide the audit evidence the auditor requires, he should
determine the implications for the audit and the auditor’s opinion in accordance with SA 705.
(ii) In the present case: - The identities of trade payables are not traceable to confirm the credit balance
as appearing in the financial statement of the company. It is also not a case of pending litigation. It
might be a case that an income of ` 5 lakhs had been hidden in previous year/s.


FAST
(iii) Audit Procedure:
The statutory auditor should examine the validity of the credit balance as appeared in the

CA.Sarthak Niraj Jain



company’s financial statements.
He should obtain sufficient evidence in support of the balance.
 He should apply alternative audit procedures to get documentary proof for the transaction/s and
should not rely entirely on the management representation.
 Finally, he should include the matter by way of a qualification in his audit report to the members.

SA-510
Initial Audit Engagements-Opening Balances
52. Initial Audit Engagement - Audit Procedures
Mr. X has been appointed as an auditor of M/s ABC Ltd., Mr. X wants to be satisfied about the sufficiency and
appropriateness of 'Opening Balances' to ensure that they are free from misstatements. Lay down the audit
procedure, Mr. X should follow, in the initial audit engagement of M/s ABC Ltd. Also suggest the approach to be
followed regarding mention in the audit report if Mr. X is not satisfied about the correctness of 'Opening
Balances'? (Nov-2019-New)
Ans.  As per SA 510 the auditor shall obtain sufficient appropriate audit evidence about whether the
opening balances contain misstatements that materially affect the current period’s financial
statements by:
 Determining whether the prior period’s closing balances have been correctly brought forward to
the current period or, when appropriate, any adjustments have been disclosed as prior period
items in the current year’s Statement of Profit and Loss;
 Determining whether the opening balances reflect the application of appropriate accounting
policies; and

CA Final Audit - By CA. Sarthak Niraj Jain 35


Auditing Standards, Statements and Guidance Notes - An Overview

 Performing one or more of the following:


o Where the prior year financial statements were audited, perusing the copies of the audited
financial statements including the other relevant documents relating to the prior period
financial statements;
o Evaluating whether audit procedures performed in the current period provide evidence
relevant to the opening balances; or
 Approach to be followed regarding mention in the Audit Report:
Performing specific audit procedures to obtain evidence regarding the opening balances:
 If the auditor is unable to obtain sufficient appropriate audit evidence regarding the opening
balances, the auditor shall express a qualified opinion or a disclaimer of opinion, as appropriate.
 Further, If the auditor concludes that the opening balances contain a misstatement that
materially affects the current period’s financial statements, and the effect of the misstatement is
not properly accounted for or not adequately presented or disclosed, the auditor shall express
a qualified opinion or an adverse opinion.
SA-520
Analytical Procedures
53. Investigating Results of Analytical Procedures
In audit of DEF Limited, the Auditor had made use of certain analytical procedures with regard to certain key
data in the Statement of Profit and Loss. The results obtained showed inconsistencies with other relevant
information. State the course of action that the Auditor should take to ensure that the risk of material
misstatement would be contained to a low level fixed as per materiality level. (Nov-2018-New)
Ans. 
FAST
As per SA 520, “Analytical Procedures”, if analytical procedures performed in accordance with this SA
identify fluctuations or relationships that are inconsistent with other relevant information or that
differ from expected values by a significant amount,
 CA.Sarthak Niraj Jain
The auditor shall investigate such differences by:
 Inquiring of management and obtaining appropriate audit evidence relevant to management’s
responses; and
 Performing other audit procedures as necessary in the circumstances.
 Audit evidence relevant to management’s responses may be obtained by evaluating those responses
taking into account the auditor’s understanding of the entity and its environment, and with other
audit evidence obtained during the course of the audit.
 The need to perform other audit procedures may arise when,
For example, management is unable to provide an explanation, or the explanation, together with the
audit evidence obtained relevant to management’s response, is not considered adequate.
54. Analytical Procedures as Substantive Procedures
You have been appointed as an auditor of M/s Excellent Hotels Ltd. As a senior partner, you want to use
analytical procedures in respect of room rentals as well as payroll expenses. Discuss. (May-2019-Old)
OR
In the audit of Hotel Great Stay Ltd., its auditor wants to use the analytical procedure as substantive procedure
in respect of room rental income as well as payroll costs. Guide him as to how it can be done.
Ans. (i) Analytical Procedures:
 SA 520 “Analytical Procedures” deals with the auditor’s use of analytical procedures as
substantive procedures and as procedures near the end of the audit that assist the auditor when
forming an overall conclusion on the financial statements.
 in some cases, even an unsophisticated predictive model may be effective as an analytical
procedure.

36 Ab Audit Hoga Sabse Scoring - By CA. Sarthak Niraj Jain


Auditing Standards, Statements and Guidance Notes - An Overview

(ii) Analytical Procedures in case of Payroll cost:


 Where an entity has a known number of employees at fixed rates of pay throughout the period, it
may be possible for the auditor to use this data to estimate the total payroll costs for the period
with a high degree of accuracy,
 Thereby providing audit evidence for a significant item in the financial statements and reducing
the need to perform tests of details on the payroll.
(iii) Analytical Procedures in case of Room Rental Income of Hotel-
 Different types of analytical procedures provide different levels of assurance.
 Analytical procedures involving the prediction of total rental income in case of Hotel taking the
room tariff rates, the number of rooms and vacancy rates into consideration, can provide
persuasive evidence and may eliminate the need for further verification by means of tests of
details, provided the elements are appropriately verified.
SA-530
Audit Sampling
55. Risk Factors while applying Sampling Techniques
Write short note on: Sampling Risk.
OR
While planning the audit of S Ltd. you want to apply sampling techniques. What are the risk factors you should
keep in mind?
Ans. (i) As per SA 530 “Audit Sampling”
 Sampling risk is the risk that the auditor’s conclusion based on a sample may be different from

(ii) FAST
the conclusion if the entire population were subjected to the same audit procedure.
Sampling risk can lead to two types of erroneous conclusions-

CA.Sarthak Niraj Jain
Controls are more effective than they actually are
 In the case of a test of controls, that controls are more effective than they actually are, or in
the case of tests of details, that a material misstatement does not exists when in fact it does.
 The auditor is primarily concerned with this type of erroneous conclusion because it
affects audit effectiveness and is more likely to lead to an inappropriate audit opinion.
 Controls are less effective than they actually are
 In the case of test of controls, the controls are less effective than they actually are, or in the
case of tests of details, that a material misstatements exists when in fact it does not.
 This type of erroneous conclusion affects audit efficiency as it would usually lead to additional
work to establish that initial conclusions were incorrect.
56. Characteristics of Sampling Errors
“An auditor while analysing the errors in a sample need not consider the qualitative aspects of errors detected.”
Please comment.
Ans. (i) SA 530, “Audit Sampling”, requires that while evaluating sample results, the auditor should :
 Analyse any errors detected in the samples having regard to appropriateness of the audit
objective
 An auditor while evaluating the errors detected in a sample selected by him should analyse the
nature of the errors, the projected errors in the total population and the sampling risk attached
to it.
 While designing an audit sample, the auditor would also need to define the conditions that
constitute the error keeping in view the audit objectives.

CA Final Audit - By CA. Sarthak Niraj Jain 37


Auditing Standards, Statements and Guidance Notes - An Overview

(ii) The auditor also need to consider the qualitative aspects of the errors detected by him
 This will include the nature and reasons for the error and its possible effect on other phases.
 In case a repetitive pattern emerges from such analysis, for example, type of transaction,
location, product line or period of time, the auditor would need to identify all items in the
population, which contain such errors, resulting in a total population.
 The auditor would then need to carry out a separate analysis based on the examination carried
out by him for each such sub -population.
(iii) Conclusion: -Accordingly, the auditor cannot be satisfied by detecting errors only but also would
need to consider the qualitative aspects of such errors.
SA-540
Auditing Accounting Estimates, Including Fair Value Accounting Estimates, and
Related Disclosures
57. Evaluation of Financial Statement Assertions
While auditing Z Ltd., you observe certain material financial statement assertions have been based on estimates
made by the management. As the auditor how do you identify & assess the risk of material misstatements?
(Study Material)
OR
While auditing REAL Ltd., you observe certain material financial statement assertions have been based on
estimates made by the management. As the auditor how do you minimize the risk of material misstatements?
(MTP-May-2021)
Ans.
FAST
As per SA 540 “Auditing Accounting Estimates, Including Fair Value Accounting Estimates, and Related
Disclosures” - The auditor shall obtain an understanding of the following in order to provide a basis for the
identification and assessment of the risks of material misstatements for accounting estimates:

CA.Sarthak Niraj Jain
The requirements of the applicable financial reporting framework relevant to the accounting
estimates, including related disclosures.
 How Management identifies those transactions, events and conditions that may give rise to the need
for accounting estimates to be recognized or disclosed, in the financial statements. In obtaining this
understanding, the auditor shall make inquiries of management about changes in circumstances that
may give rise to new, or the need to revise existing, accounting estimates.
 The estimation making process adopted by the management including-
 The method, including where applicable the model, used in making the accounting estimates.
 Relevant controls.
 Whether management has used an expert?
 The assumption underlying the accounting estimates.
 Whether there has been or ought to have been a change from the prior period in the methods for
making the accounting estimates, and if so, why; and
 Whether and, if so, how the management has assessed the effect of estimation uncertainty.
58. High Estimation Uncertainty - Examples
With reference to the Standards on Auditing state the examples of accounting estimates that may have a high
estimation uncertainty. (Nov-2016)
Ans. (i) Provision: -As per SA 540, “Auditing Accounting Estimates, Including Fair Value Accounting
Estimates and Related Disclosures”, the auditor shall determine whether, in the auditor’s judgment,
any of those accounting estimates that have been identified as having high estimation uncertainty give
rise to significant risks.

38 Ab Audit Hoga Sabse Scoring - By CA. Sarthak Niraj Jain


Auditing Standards, Statements and Guidance Notes - An Overview

(ii) Examples of accounting estimates that may have high estimation uncertainty:
 Accounting estimates that are highly dependent upon judgment,
For example, judgments about the outcome of pending litigation or the amount and timing of
future cash flows dependent on uncertain events many years in the future.
 Accounting estimates that are not calculated using recognised measurement techniques.
 Accounting estimates where the results of the auditor’s review of similar accounting estimates
made in the prior period financial statements indicate a substantial difference between the original
accounting estimate and the actual outcome.
 Fair value accounting estimates for which a highly specialised entity-developed model is used or
for which there are no observable inputs.
59. Lower Level of ROMM - Accounting Estimates
During the Audit of Data Solutions Ltd., a listed company, your audit manager observed that several estimates
are made by the Company. He seeks your guidance to know areas of accounting estimates that may give rise to
lower level of risk of material misstatement. Guide him with examples. (Nov-2019-Old)
Ans. (i) Provision: -As per SA 540 “Auditing Accounting Estimates, Including Fair Value Accounting
Estimates, and Related Disclosures”, some accounting estimates involve relatively low estimation
uncertainty and may give rise to lower risks of material misstatements,
(ii) Examples of accounting estimates that may give rise to lower level of risk of material
misstatement.
 Accounting estimates arising in entities that engage in business activities that are not complex.
 Accounting estimates that are frequently made and updated because they relate to routine

 FAST
transactions.
Accounting estimates derived from data that is readily available, such as published interest rate
data or exchange-traded prices of securities. Such data may be referred to as “observable” in the

CA.Sarthak Niraj Jain



context of a fair value accounting estimate.
Fair value accounting estimates where the method of measurement prescribed by the applicable
financial reporting framework is simple and applied easily to the asset or liability requiring
measurement at fair value.
 Fair value accounting estimates where the model used to measure the accounting estimate is
well-known or generally accepted, provided that the assumptions or inputs to the model are
observable.
60. Accounting Assumptions & Estimates - Written Representations
Mr. L while conducting the audit of ABC Ltd., observed that a substantial amount is recognized in respect of
obsolescence of inventory and warranty obligation in the financial statements. Mr. L wants to obtain written
representation from the management to determine whether the assumptions and estimates used are
reasonable. Guide Mr. L with reference to the relevant Standard on Auditing. (Study Material)(Nov-2019-New)
Ans. (i) Provision: -As per SA 540, “Auditing Accounting Estimates, Including Fair Value Accounting
Estimates, and Related Disclosures”,
 The auditor shall obtain written representations from management and, where appropriate,
TCWG
 Whether they believe significant assumptions used in making accounting estimates are
reasonable.
(ii) SA 580, “Written Representations” discusses the use of written representations.
 Depending on the nature, materiality and extent of estimation uncertainty, written
representations about accounting estimates recognised or disclosed in the financial statements
may include representations:

CA Final Audit - By CA. Sarthak Niraj Jain 39


Auditing Standards, Statements and Guidance Notes - An Overview

a) About the appropriateness of the measurement processes, including related assumptions


and models, used by management in determining accounting estimates in the context of the
applicable financial reporting framework, and the consistency in application of the processes.
b) That the assumptions appropriately reflect management’s intent and ability to carry out
specific courses of action on behalf of the entity, where relevant to the accounting estimates
and disclosures.
c) That disclosure related to accounting estimates are complete and appropriate under the
applicable financial reporting framework.
d) That no subsequent event requires adjustment to the accounting estimates and disclosures
included in the financial statements.
61. Review of Outcome of Accounting Estimates
A Pvt Ltd is engaged in the business of real estate. The auditor of the company requested the information from
the management to review the outcome of accounting estimates (like estimated costs considered for
percentage completion etc) included in the prior period financial statements and their subsequent re-
estimation for the purpose of the current period. The management has refused the information to the auditor
saying that the review of prior period information should not be done by the auditor. Please advise.
(Study Material)(RTP-May-2019)(MTP-Dec-2021)
Ans. (i) Provision: - As per SA 540, “Auditing Accounting Estimates, Including Fair Value Accounting
Estimates, and Related Disclosures”
 The auditor shall review the outcome of accounting estimates included in the prior period
financial statements, or, where applicable, their subsequent re - estimation for the purpose of the
current period.
 FAST
The nature and extent of the auditor’s review takes account of the nature of the accounting
estimates, and whether the information obtained from the review would be relevant to identifying

(ii)
CA.Sarthak Niraj Jain
and assessing risks of material misstatement of accounting estimates made in the current period
financial statements.
The outcome of an accounting estimate will often differ from the accounting estimate recognised in
the prior period financial statements.
By performing risk assessment procedures to identify and understand the reasons for such
differences, the auditor may obtain:
 Information regarding the effectiveness of management’s prior period estimation process, from
which the auditor can judge the likely effectiveness of management’s current process.
 Audit evidence that is pertinent to the re-estimation, in the current period, of prior period
accounting estimates.
 Audit evidence of matters, such as estimation uncertainty, that may be required to be disclosed in
the financial statements.
(iii) The review of prior period accounting estimates may also assist the auditor
 In the current period, in identifying circumstances or conditions that increase the susceptibility
of accounting estimates to, or indicate the presence of, possible management bias.
 The auditor’s professional skepticism assists in identifying such circumstances or conditions and
in determining the nature, timing and extent of further audit procedures.
 However, the review is not intended to call into question the judgments made in the prior periods
that were based on information available at that time.
(iv) In the given case: - The management is not correct in refusing the relevant information to the
auditor.

40 Ab Audit Hoga Sabse Scoring - By CA. Sarthak Niraj Jain


Auditing Standards, Statements and Guidance Notes - An Overview

62. Accounting Estimate - Significant Risk


M/s. HK & Co. was appointed as an auditor of GSB Limited, a company operating its business in telecom sector.
As per spectrum allocation agreement with Government, GSB Limited is required to pay certain percentage of
its annual revenue as license fee. GSB Limited paid the license fee on its core business for last two years. At the
end of third year, the communication was received from Government that it needs to pay agreed percentage on
its total revenues and not only on core business revenues. Matter was disputed a nd went to court of law. On
prudence basis, GSB Limited made a provision on estimated business in its books of accounts of agreed
percentage on non-core business receipts also. The amount of provision was of such huge amount that the GSB
Limited's profit a nd loss account for that quarter reflected loss due to that provision. How you as an auditor
can evaluate this accounting estimate which involves significant risk and what if Management has not
addressed the effects of estimation uncertainty on provision made? (Jan-2021-New)
Ans. (i) In the given case: - HK & Co. was appointed as an auditor of GSB Ltd., operating in Telecom sector. GSB
Ltd paid the license fee on its core business revenue whereas Govt. required it to pay on non-core
business receipts as well. Consequently, the amount of provision was of such a huge amount that GSB
Ltd.’s profit and loss account reflected a loss due to that provision. As an auditor evaluation would be
done as under:
(ii) For accounting estimates that give rise to significant risks, in addition to other substantive
procedures performed to meet the requirements of SA 330, the auditor shall evaluate the following:
 How management has considered alternative assumptions or outcomes, and why it has rejected
them, or
 How management has otherwise addressed estimation uncertainty in making the accounting
estimate.
 Whether the significant assumptions used by management are reasonable.

 FAST
Where relevant to the reasonableness of the significant assumptions used by management or
The appropriate application of the applicable financial reporting framework, management’s
intent to carry out specific courses of action and its ability to do so.

CA.Sarthak Niraj Jain


 If, in the auditor’s judgment, management has not adequately addressed the effects of estimation
uncertainty on the accounting estimates that give rise to significant risks, the auditor shall, if
considered necessary, develop a range with which to evaluate the reasonableness of the
accounting estimate.
63. Assumptions in Accounting Estimates
Assumptions are integral components of accounting estimates. State the matters that the auditor may
consider in obtaining an understanding of the assumptions underlying the accounting estimates with
reference to relevant SAs. (July-2021-Old)
Ans. As per SA 540, “Auditing Accounting Estimates, Including Fair Value Accounting Estimates, and Related
Disclosures”,
(a) How management makes the accounting estimates, and an understanding of the data on which they
are based, including assumptions underlying the accounting estimates.
(b) Assumptions are integral components of accounting estimates. Matters that the auditor may consider
in obtaining an understanding of the assumptions underlying the accounting estimates include, for
example:
• The nature of the assumptions, including which of the assumptions are likely to be significant
assumptions.
• How management assesses whether the assumptions are relevant and complete (that is, that all
relevant variables have been taken into account).
• Where applicable, how management determines that the assumptions used are internally
consistent.

CA Final Audit - By CA. Sarthak Niraj Jain 41


Auditing Standards, Statements and Guidance Notes - An Overview

• Whether the assumptions relate to matters within the control of management (for example,
assumptions about the maintenance programs that may affect the estimation of an asset’s useful
life), and how they conform to the entity’s business plans and the external environment, or to
matters that are outside its control (for example, assumptions about interest rates, mortality rates,
potential judicial or regulatory actions, or the variability and the timing of future cash flows).
• The nature and extent of documentation, if any, supporting the assumptions.
Assumptions may be made or identified by an expert to assist management in making the accounting
estimates. Such assumptions, when used by management, become management’s assumptions.
SA-550
Related Parties
64. Possible Sources of Related Party Information
JY & Co. is appointed as auditor of Breeze Ltd. JY & Co. seeks your guidance for reviewing the records and
documentation of the company regarding ‘related party transactions in the normal course of business’.
Describe the steps to be followed. (MTP-May-2019)(Nov-2015)
OR
In the course of audit of Q Ltd, its statutory auditor wants to be sure of the adequacy of related party
disclosures? Kindly guide the auditor in identifying the possible source of related party information.
(MTP-Nov-2018)
Ans. (i) Provision: -As per SA 550 “Related Parties” the auditor shall remain alert, when inspecting records
or documents with respect to arrangements or information indicating the existence of related party
relationships or transactions, not previously identified or disclosed to the auditor.
(ii) During the audit, the auditor may inspect records or documents that may provide information


FAST
about related party relationships and transactions, for example:
 Entity income tax returns.
Information supplied by the entity to regulatory authorities.

CA.Sarthak Niraj Jain




Shareholder registers to identify the entity’s principal shareholders.
Statements of conflicts of interest from management and TCWG.
 Records of the entity’s investments and those of its pension plans.
 Contracts and agreements with key management or TCWG.
 Significant contracts and agreements not in the entity’s ordinary course of business.
 Specific invoices and correspondence from the entity’s professional advisors.
 Life insurance policies acquired by the entity.
 Significant contracts re-negotiated by the entity during the period.
 Internal auditors’ reports.
 Documents associated with the entity’s filings with a securities regulator (for example,
prospectuses).
(iii)Further Audit Procedure:
 Auditor should also obtain further information on significant transactions outside the entity’s
normal course of business. It enables him to evaluate whether fraud risk factors, if any, are
present.
 In addition, the auditor needs to be alert for transactions which appear unusual in the
circumstances and which may indicate the existence of previously unidentified related parties.
 For example: Complex equity transactions such as corporate restructurings or acquisitions,
transactions with offshore entities in jurisdictions with weak corporate laws, the leasing of premises
etc.
(iv) Written Representation: - Finally, the auditor should also obtain a written representation from the
management concerning the completeness of information provided regarding the identification of
related parties.

42 Ab Audit Hoga Sabse Scoring - By CA. Sarthak Niraj Jain


Auditing Standards, Statements and Guidance Notes - An Overview

65. Identification of Significant Related Party Transaction


Write a short note on auditor’s response on Identification of significant related party transaction outside the
normal course of business. (May-2016)
Ans. As per SA 550 on “Related Parties”, for identified significant related party transactions outside the entity’s
normal course of business, the auditor shall-
(i) Inspect the underlying contracts or agreements, if any, and evaluate whether:
(a) The business rationale (or lack thereof) of the transactions suggests that they may have been
entered into to engage in fraudulent financial reporting or to conceal misappropriation of
assets;
(b) The terms of the transactions are consistent with management’s explanations; and
(c) The transactions have been appropriately accounted for and disclosed in accordance with the
applicable financial reporting framework; and
(ii) Obtain audit evidence that the transactions have been appropriately authorized and approved.
66. Related Party Transaction –Biased
In the course of your audit you have come across a related party transaction which prima facie appears to be
biased. How would you deal with this?
Ans. (i) Provision of AS-18/Ind-AS-24 & SA 550
 In the course of your audit you have come with regard to reporting of transactions with related
parties as required by Accounting Standard 18 are given in SA 550 on Related Parties.
 As per SA 550 on, “Related Parties”, the auditor should review information provided by the
management of the entity identifying the names of all known related parties.

FAST
Since it is the management, which is primarily responsible for identification of related parties, SA
550 requires that to identify names of all known related parties, the auditor may inspect records
or documents that may provide information about related party relationships and transactions.
(ii)
CA.Sarthak Niraj Jain
In the given case: - The auditor is finding a related party transaction which prima facie appears to be
biased. So the auditor is required to confirm the same.
(iii) For identified significant RPTs outside the entity’s normal course of business
The auditor shall inspect the underlying contracts or agreements, if any, and evaluate whether:
(a) The business rationale (or lack thereof) of the transactions suggests that they may have been
entered into to engage in fraudulent financial reporting or to conceal misappropriation of assets,
(b) The terms of the transactions are consistent with management’s explanations; and
(c) The transactions have been appropriately accounted for and disclosed in accordance with
the applicable financial reporting framework.
(d) The auditor should also obtain audit evidence that the transactions have been appropriately
authorised and approved.
(iv) Written Representation:
The auditor shall obtain written representations from management and, where appropriate, those
charged with governance that they have disclosed to the auditor the identity of the entity’s related
parties and all the related party relationships and transactions of which they are aware; and they have
appropriately accounted for and disclosed such relationships and transactions in accordance with the
requirements of the framework.
(v) Reporting:
Finally, the auditor should report on the basis of this fact that the related party relationships and
transactions:
(a) prevent the financial statements from achieving true and fair presentation(for fair presentation
frameworks); or
(b) cause the financial statements to be misleading (for compliance frameworks).

CA Final Audit - By CA. Sarthak Niraj Jain 43


Auditing Standards, Statements and Guidance Notes - An Overview

67. Disclosure of Related Party Transaction


Mr. D is a director of X Ltd. and Y Ltd. On 30th June, 2021, Mr.D resigned from directorship of Y Ltd. X Ltd. sold
goods to Y Ltd., during the entire year at the same price and conditions as to any other customer. X Ltd.
discloses only the sales for the first quarter ending 30th June, 2021 as related party transactions.
Ans. (i) As per SA 550 ‘Related Parties’: - In examining the identified related party transactions, the auditor
should obtain sufficient appropriate audit evidence as to whether these transactions have been
properly recorded and disclosed.
(ii) As per AS 18 ‘Related Party Disclosures’: - Transactions of X Ltd. for the first quarter with Y Ltd. up
to 30th June, 2015 only are required to be disclosed as related party transactions. The transactions for
the period in which related party relationship did exist need not to be disclosed as related party
transactions.
(iii) Reporting under CARO-2020: -Even though X Ltd. has correctly identified and disclosed the related
party transactions, the auditor need to report under
 Clause (xiii) of CARO, 2020 about whether all transactions with the related parties are in
compliance with sections 177 and 188 of Companies Act, 2013 where applicable and the details
have been disclosed in the Financial Statements etc., as required by the applicable accounting
standards.
68. Reporting of Significant Related Party Matters
Whilst the Audit team has identified few matters, they need your advice to conclude on the same. Engagement
Partner have asked them to review the Board minutes and other secretarial/ regulatory records based on
which the following additional matters were brought to the attention of the Partner:-
(i) The long term borrowings from the parent company has no written terms and neither the interest nor

(ii)
FAST
the principal has been repaid so far.
Certain computers were received from the parent company free of cost, the value of which is ` 0.23 lac

(iii) CA.Sarthak Niraj Jain


and no accounting or disclosure of the same has been made in the notes to accounts.
An amount of ` 3.25 Lakhs per month is paid to M/s. WE CARE Associates, a partnership firm, which is a
'related party' in accordance with the provisions of the Companies Act, 2013 for the marketing services
rendered by them. Based on an independent assessment, the consideration paid is higher than the arm's
length pricing by `0.25 Lakhs per month. Whilst the transaction was accounted in the financial
statements based on the amounts' paid, no separate disclosure of this related party transaction has been
made in the notes to accounts forming part of the financial statements highlighting the same as a 'related
party' transaction.
Audit Manager has reported that she had asked certain information relating to another 'related party'
transaction (amounting to approx. ` 47 lac) but the CFO refused to provide the same since the same is
perceived to be confidential and cannot be shared with the Auditors.
You are required to advise about items to be reported to those charged with governance, where applicable,
based on your audit findings in the given situation. (MTP-Nov-2020)
Ans. (i) Provision: -As per SA 550, Related Parties:
 Communicating significant matters arising during the audit in connection with the entity’s related
parties helps the auditor to establish a common understanding with those charged with
governance of the nature and resolution of these matters.
 Examples of significant related party matters include:
 Non-disclosure (whether intentional or not) by management to the auditor of related parties
or significant related party transactions, which may alert those charged with governance to
significant related party relationships and transactions of which they may not have been
previously aware;
 The identification of significant related party transactions that have not been
appropriately authorised and approved, which may give rise to suspected fraud; etc.

44 Ab Audit Hoga Sabse Scoring - By CA. Sarthak Niraj Jain


Auditing Standards, Statements and Guidance Notes - An Overview

(ii) Involvement of TCWG in MGT: -It may be noted that unless all of TCWG are involved in managing
the entity, the auditor shall communicate with TCWG significant matters arising during the audit in
connection with the entity’s related parties.
(iii) Compliance of AS/Ind-AS: - The auditor is also required to ensure the compliance of Ind AS 24 / AS 18
Related Party Disclosures.
(iv) Reporting: -In view of above in the given scenario, the auditor is required to prepare a brief summary
of following items to be reported to TCWG in accordance with SA 260 Communication with TCWG:
 One of related party transaction amounting 3.25 lac per month i.e. in lieu of marketing services
has been noticed of which amount ` 0.25 lac per month is exceeds the arm’s length price has not
been disclosed highlighting the same as related party transactions as per Ind- AS 24 / AS 18 Related
Party Disclosures.
 Refusal by CFO of, as denying for the related part details of ` 47 lac is imposing limitation of scope
of auditor in view of SA 705.
 Receipt of free of cost Computers and long-term borrowing (on no agreed terms and repayment
of interest and principal) from the Parent Company need separate disclosure in financial
statements as per Ind AS 24 / AS 18 Related Party Disclosures.
 Further, in case of all the above cases: - The auditor would also need to assess his reporting
requirements under the clauses (xiii) of Paragraph 3 of CARO 2016 with respect to related party
transactions that whether all transactions with the related parties are in compliance with
sections 177 and 188 of Companies Act, 2013 where applicable and the details have been
disclosed in the Financial Statements etc., as required by the applicable Accounting Standards.
69. Audit Procedure - Operative Effectiveness of Control related to RPT

FAST
While formulating the audit plan and responding to the risks of material misstatement identified and assessed
in related party transaction and relationships, Ms. K the engagement manager of the audit team of ABC
Limited, decided to rely upon the internal controls placed for identification and disclosure of related party

CA.Sarthak Niraj Jain


relationships and transactions in accordance with the applicable financial reporting framework.
You are requested to guide Ms. K regarding the necessity to test the controls to obtain sufficient and
appropriate audit evidence. Also guide, whether Ms. K can use the audit evidence obtained, regarding
operative effectiveness of control on identification and disclosure of related party relationships and
transactions, in the interim period. (RTP-Dec-2021)
Ans. (i) As per SA 550, “Related Parties”, according to para on “Responses to the risks of material
misstatement associated with related party relationships and transactions”
 The auditor should design and performs further audit procedures to obtain sufficient appropriate
audit evidence about the assessed risks of material misstatement associated with related party
relationships and transactions.
 Further, as per SA 330, “The Auditor’s Responses to Assessed Risks”, the auditor shall design and
perform tests of controls to obtain sufficient appropriate audit evidence as to the operating
effectiveness of relevant controls when:
(a) the auditor’s assessment of risks of material misstatement at the assertion level includes an
expectation that the controls are operating effectively (i.e., the auditor intends to rely on the
operating effectiveness of controls in determining the nature, timing and extent of substantive
procedures); or
(b) Substantive procedures alone cannot provide sufficient appropriate audit evidence at the
assertion level.
(ii) In designing and performing tests of controls
 The auditor shall obtain more persuasive audit evidence the greater the reliance the auditor
places on the effectiveness of a control.

CA Final Audit - By CA. Sarthak Niraj Jain 45


Auditing Standards, Statements and Guidance Notes - An Overview

 Moreover, the auditor shall test controls for the particular time, or throughout the period, for
which the auditor intends to rely on those controls, subject to when the auditor obtains audit
evidence about the operating effectiveness of controls during an interim period, and the timing
of test of controls over significant risks, in order to provide an appropriate basis for the auditor’s
intended reliance.
(iii) When the auditor obtains audit evidence about the operating effectiveness of controls during an
interim period, the auditor shall:

(a) Obtain audit evidence about significant changes to those controls subsequent to the interim
period; and
(b) Determine the additional audit evidence to be obtained for the remaining period.
(iv) Conclusion
 In the current case Ms. K shall design and perform tests of controls to obtain sufficient
appropriate audit evidence as to the operating effectiveness of relevant controls as she intends to
rely on the operating effectiveness of controls in determining the nature, timing and extent of
substantive procedures.
 Further, she is also required to obtain the audit evidence about significant changes to those
controls subsequent to the interim period along with the additional audit evidence to be obtained
for the remaining period in accordance with the requirements of Standards on Auditing as discussed
above.
SA-560
Subsequent Events
70. FAST
Audit Procedures on Subsequent Events
You are the auditor of PQR Ltd. which is in the business of supplying food products to various airline companies

CA.Sarthak Niraj Jain


operating aircrafts in domestic circle only. As per terms of agreement with airlines, the company needs to stock
various non- perishable food items for coming one month (average holding of inventory to the tune of INR 75
Crores). Also the payment terms have been settled and the company receives payment in 45 days after the
supply of goods. Everything was going-on well till the end of March 2020 when pandemic Covid hit the world
and everything came to a standstill. Aviation sector was hit hard and there were no flights from April 2020
onwards. Consequently, the business of PQR Ltd. also got severely affected and the scheduled supplies of goods
to airlines also were not made. Also, the liquidity position of airline companies got hit and the schedule d
payments were also not received on due dates. As the auditor of PQR Ltd. what audit procedures would you
perform to ensure that all subsequent events are considered, so that financial statements for the year ended
31.03.2020 represent true and fair view? (Nov-2020-New)
OR
Briefly explain Audit procedures on subsequent events.
Ans. (i) Subsequent Events
 As per SA 560 “Subsequent Events”, the auditor shall perform audit procedures designed to
obtain sufficient appropriate audit evidence that all events occurring between the date of the
financial statements and the date of the auditor’s report that require adjustment of, or
disclosure in, the financial statements have been identified.
 The auditor is not, however, expected to perform additional audit procedures on matters to which
previously applied audit procedures have provided satisfactory conclusions.
 The auditor shall perform the procedures required in above paragraph so that they cover the period
from the date of the financial statements to the date of the auditor’s report, or as near as
practicable thereto.

46 Ab Audit Hoga Sabse Scoring - By CA. Sarthak Niraj Jain


Auditing Standards, Statements and Guidance Notes - An Overview

(ii) Audit Procedure: Being the auditor of PQR Ltd, to ensure that all subsequent events are considered
so that financial statements for the year ending 31.03.2020 represent true and fair view, the auditor shall
take into account the auditor’s risk assessment in determining the nature and extent of such audit
procedures, which shall include the following:
a. Obtaining an understanding of any procedures management has established to ensure that
subsequent events are identified.
b. Inquiring of management and, where appropriate, those charged with governance as to
whether any subsequent events have occurred which might affect the financial statements.
c. Reading minutes, if any, of the meetings, of the entity’s owners, management and those
charged with governance, that have been held after the date of the financial statements and
inquiring about matters discussed at any such meetings for which minutes are not yet available.
d. Reading the entity’s latest subsequent interim financial statements, if any.
When, as a result of the procedures performed as required above, the auditor identifies events that require
adjustment of, or disclosure in, the financial statements, the auditor shall determine whether each such
event is appropriately reflected in those financial statements.
71. Specific Inquiries to Evaluate Subsequent Events
M/s LMP Associates, Chartered Accountants while conducting the audit of PQR Ltd want to conduct an inquiry
of management and those charged with governance as to whether any subsequent event have occurred which
might affect the financial statements. Guide M/s LMP Associates with the matters where specific enquiry may
be conducted to evaluate subsequent events. (May-2019-Old)
OR
Inquiry from Management is helpful for Auditor to evaluate subsequent events. Discuss specific enquiries in
reference of SA 560, which might have effect on the financial statements.
Ans. 
FAST
As per SA 560, “Subsequent Events”, in inquiring of management and, where appropriate, those
charged with governance, as to whether any subsequent events have occurred that might affect the
financial statements,

CA.Sarthak Niraj Jain
The auditor may inquire as to the current status of items that were accounted for on the basis of
preliminary or inconclusive data and may make specific inquiries about the following matters:
 Whether new commitments, borrowings or guarantees have been entered into.
 Whether sales or acquisitions of assets have occurred or are planned.

 Whether there have been increases in capital or issuance of debt instruments, such as the issue
of new shares or debentures, or an agreement to merge or liquidate has been made or is planned.
 Whether any assets have been appropriated by government or destroyed, for example, by fire
or flood.
 Whether there have been any developments regarding contingencies.
 Whether any unusual accounting adjustments have been made or are contemplated.
 Whether any events have occurred or are likely to occur that will bring into question the
appropriateness of accounting policies used in the financial statements, as would be the case, for
example, if such events call into question the validity of the going concern assumption.
 Whether any events have occurred that are relevant to the measurement of estimates or
provisions made in the financial statements.
 Whether any events have occurred that are relevant to the recoverability of assets.
72. Treatment of Subsequent Events
A Co. Ltd. has not included in the Balance Sheet as on 31-03-2022 a sum of ` 1.50 crores being amount in the
arrears of salaries and wages payable to the staff for the last 2 years as a result of successful negotiations which
were going on during the last 18 months and concluded on 30-04-2022. The auditor wants to sign the said
Balance Sheet and give the audit report on 31-05-2022. The auditor came to know the result of the negotiations
on 15-05-2022. Comment.

CA Final Audit - By CA. Sarthak Niraj Jain 47


Auditing Standards, Statements and Guidance Notes - An Overview

Ans. (i) As per SA 560 “Subsequent Events”


 In respect of events occurring between the date of F.S. and date of the Audit Report, the auditor
shall perform audit procedures to obtain sufficient & appropriate audit evidence to ensure that
events which require adjustments or disclosure in the F.S. have been identified.
(ii) AS-4 & AS-29
 This case requires attention to AS 4 “Contingencies and Events occurring after the Balance Sheet
Date” and AS 29 “Provisions, Contingent liabilities and Contingent Assets”.
 As per AS 4 “Contingencies and Events occurring after the Balance Sheet Date”, adjustments to
assets and liabilities are required for events occurring after the balance sheet date that provide
additional information materially affecting the determination of the amounts relating to
conditions existing at the balance sheet date.
 Similarly as per AS 29 “Provisions, Contingent liabilities and Contingent Assets”, future events that
may affect the amount required to settle an obligation should be reflected in the amount of a
provision where there is sufficient objective evidence that the will occur.
(iii) In the instant case: - The amount of ` 1.50 crores is a material amount and it is the result of an event,
which has occurred after the Balance Sheet date. The facts have become known to the auditor before
the date of issue of the Audit Report and Financial Statements.
(iv) Audit Procedure:
 The auditor has to perform the procedure to obtain sufficient, appropriate evidence covering the
period from the date of the financial statements i.e. 31-3-2022 to the date of Auditors Report ie.31-
05-2022.
 It will be observed that as a result of long pending negotiations a sum of ` 1.50 cores representing
arrears of salaries of the year 2020-21 and 2021-22 have not been included in the financial

FAST
statements. It is quite clear that the obligation requires provision for outstanding expenses as
per AS 4 and AS 29.
(v) Conclusion: -So the auditor should request the management to adjust the sum of ` 1.50 crores by

73.
CA.Sarthak Niraj Jain
making provision for expenses. If the management does not accept the request the auditor should
qualify the audit report.
Facts Which Become Known to the Auditor After the Date of the Auditor’s Report but Before the Date
the Financial Statements are Issued
Amudhan & Co., are the Auditors of XYZ Company Ltd., for the year ended on 31/03/2022. The Audit Report for
that year was signed by the Auditors on 04/05/2022. The Annual General Meeting was decided to be held
during the month of August 2022. On 06/05/2022, the Company had received a communication from the
Central Government that an amount of ` 5800 crore kept pending on account of incentives pertaining to
Financial Year 2021-22 had been approved and the amount would be paid to the Company before the end of
May 2022. To a query to Chief Financial officer of the Company by the Board, it was informed that this amount
had not been recognised in the Audited Financial Statements in view of the same not being released' before the
close of the Financial Year and due to uncertainty of receipt. Now, having received the amount, the Board of
Directors wished to include this amount in the Financial Statements of the Company for the Financial Year
ended on 31/03/2022. On 08/05/2022, the Board amended the accounts, approved the same and requested
the Auditor to consider this event and issue a fresh Audit Report on the Financial Statements for the year ended
on 31/03/2022. Analyse the issues involved and give your views as to whether or not the Auditors could
accede to the request of the Board of Directors. (Nov-2018- New)
Ans. (i) Provision:
 As per SA 560, “Subsequent Events”, the auditor has no obligation to perform any audit
procedures regarding the financial statements after the date of the auditor’s report.
 However, when, after the date of the auditor’s report but before the date the financial
statements are issued, a fact becomes known to the auditor that, had it been known to the auditor
at the date of the auditor’s report, may have caused the auditor to amend the auditor’s report,

48 Ab Audit Hoga Sabse Scoring - By CA. Sarthak Niraj Jain


Auditing Standards, Statements and Guidance Notes - An Overview

 The auditor shall


 Discuss the matter with management and, where appropriate, TCWG.
 Determine whether the financial statements need amendment and, if so,
 Inquire how management intends to address the matter in the financial statements.
(ii) Audit Procedure & New Audit Report:
 If management amends the financial statements, the auditor shall carry out the audit procedures
necessary in the circumstances on the amendment.
 Further, the auditor shall extend the audit procedures and provide a new auditor’s report on the
amended financial statements.
 However, the new auditor’s report shall not be dated earlier than the date of approval of the
amended financial statements.
(iii) In the instant case:
 XYZ Company Ltd. received an amount of rupees 5800 crore on account of incentives pertaining to
year 2021-22 in the month of May 2022 i.e. after finalisation of financial statements and signing of
audit report.
 Board of Directors of XYZ Ltd. amended the accounts, approved the same and requested the
Amudhan & Co. (auditor) to consider this event and issue a fresh audit report on the financial
statements for the year ended on 31.03.2022.
(iv) Conclusion: -After applying the conditions given in SA 560, Amudhan & Co. can issue new audit
report subject to date of audit report which should not be earlier than the date of approval of the

74.
FAST
amended financial statements.
Subsequent Events & Emphasis of Matter Paragraph

CA.Sarthak Niraj Jain


The audit report of Kolsi (P) Ltd. for F.Y. 2021-22 was issued by Bishnoi & Co. on 25 th July, 2022. However, a
case was filed against Kolsi (P) Ltd. on 4th August, 2022, with the Civil Court, with respect to an incident
caused in its factory on 17th January, 2022, the outcome of which may result in paying heavy penalty by Kolsi
(P) Ltd.
Mr. Raj Bishnoi, the partner of Bishnoi & Co., discussed the said matter with the management and it was
determined to amend the financial statements for F.Y. 2021-22. Further, Mr. Raj inquired how the
management intended to address the said matter in the financial statements to which he was told that the said
matter was going to be disclosed as a “Contingent Liability for a Court case” to the foot note in the balance
sheet with no additional disclosures.
The management told Mr. Raj that such disclosure was enough as he would further going a description of the
said court case and its outcome in the ‘Emphasis of Matter’ paragraph in his amended audit report.
In the context of aforesaid case scenario, please answer the following questions:-
(a) Whether Mr. Raj on behalf of Bishnoi & Co., has properly adhered to his responsibilities in accordance
with SA 560, on becoming aware of the court case filed against Kolsi (P) Ltd.?
(b) Whether the contention of management of Kolsi (P) Ltd. is valid with respect to the disclosure of the
court case in the financial statements? (RTP-Dec-2021)
Ans. (a) Auditor’s responsibility in case of Subsequent Event
(i) Provision: -As per SA 560, ‘Subsequent Events’, the auditor has no obligation to perform any
audit procedures regarding the financial statements after the date of the auditor’s report.
However, when, after the date of the auditor’s report but before the date the financial
statements are issued, a fact becomes known to the auditor that, had it been known to the auditor
at the date of the auditor’s report, may have caused the auditor to amend the auditor’s report, the
auditor shall:

CA Final Audit - By CA. Sarthak Niraj Jain 49


Auditing Standards, Statements and Guidance Notes - An Overview

(1) Discuss the matter with management and, where appropriate, those charged with
governance.
(2) Determine whether the financial statements need amendment and, if so,
(3) Inquire how management intends to address the matter in the financial statements.
(ii) In the given case: - On becoming aware of the court case filed against Kolsi (P) Ltd., Mr. Raj
discussed the said matter with the management and it was determined to amend the financial
statements. Also, he inquired how the management intended to address the said matter in the
financial statements.
(iii) Conclusion/Comment
 However, If management does not take the necessary steps to ensure that anyone in receipt
of the previously issued financial statements is informed of the situation and does not amend
the financial statements in circumstances where Mr. Raj (hereinafter referred as ‘the
auditor’) believes they need to be amended,
 The auditor shall notify management and, those charged with governance (unless all of
those charged with governance are involved in managing the entity), that the auditor will
seek to prevent future reliance on the auditor’s report.
 If despite such notification the management or those charged with governance do not take
these necessary steps, the auditor shall take appropriate action to seek to prevent reliance
on the auditor’s report in accordance with SA 560.
(b) Disclosure of the court case in the financial statements
(i) Provision: -As per SA 706, ‘Emphasis of Matter Paragraphs and Other Matter Paragraphs in

FAST
the Independent Auditor’s Report’, an Emphasis of Matter paragraph is not a substitute for:
(a) A modified opinion in accordance with SA 705 (Revised) when required by the

CA.Sarthak Niraj Jain


circumstances of a specific audit engagement;
(b) Disclosures in the financial statements that the applicable financial reporting framework
requires management to make, or that are otherwise necessary to achieve fair presentation; or
(c) Reporting in accordance with SA 570 (Revised) when a material uncertainty exists relating
to events or conditions that may cast significant doubt on an entity’s ability to continue as a
going concern.
(ii) In the given case: - The management of Kolsi (P) Ltd. has presumed that as the auditor was going
to provide a description of the said court case and its outcome in the ‘Emphasis of Matter’
paragraph in his amended audit report, there was no further need for it to provide additional
disclosures about the court case in the financial statements.
(iii) Conclusion: -The said contention of management of Kolsi (P) Ltd. is not valid as ‘Emphasis of
Matter’ paragraph cannot be used as a substitute for disclosures required to be made in the
financial statements as per the applicable financial reporting framework or that is otherwise
necessary to achieve fair presentation, which is the responsibility of the management.

SA-570
Going Concern
75. Additional Audit Procedures when Events or Conditions are Identified:
Mr. Ram, an auditor, identified some events that cast significant doubt on the entity’s ability to continue as a
going concern. What are the additional procedures he should perform as per the related Standard on Auditing?
(Nov-2016)

50 Ab Audit Hoga Sabse Scoring - By CA. Sarthak Niraj Jain


Auditing Standards, Statements and Guidance Notes - An Overview

Ans. (i) As per SA 570 “Going Concern”, when events or conditions have been identified that may cast
significant doubt on the entity’s ability to continue as a going concern, the auditor shall obtain
sufficient appropriate audit evidence to determine whether or not a material uncertainty exists
through performing additional audit procedures, including consideration of mitigating factors.
(ii) These procedures shall include:
 When management has not yet performed an assessment of the entity’s ability to continue as a
going concern, requesting management to make its assessment.
 Evaluating management’s plans for future actions in relation to its going concern assessment,
whether the outcome of these plans is likely to improve the situation and whether management’s
plans are feasible in the circumstances.
 When the entity has prepared a cash flow forecast, and analysis of the forecast is a significant
factor in considering the future outcome of events or conditions in the evaluation of
management’s plans for future action:
(a) Evaluating the reliability of the underlying data generated to prepare the forecast; and
(b) Determining whether there is adequate support for the assumptions underlying the
forecast.
 Considering whether any additional facts or information have become available since the date on
which management made its assessment.
 Requesting written representations from management and, where appropriate, TCWG, regarding
their plans for future action and the feasibility of these plans.
76. Evaluation of Appropriateness of Going Concern Basis

FAST
MNO Limited is one of the prominent players in the chemicals industry. The company is a public company
domiciled in India and listed on BSE and NSE. The Company was facing extreme liquidity constraints and there
were multiple indicators that casted doubt over the company’s ability to continue as a going concern.

CA.Sarthak Niraj Jain


The Company was led into insolvency proceedings by consortium of banks led by PNB and the NCLT ordered
the commencement of corporate insolvency process against the Company on 31 August 2021. The company
invited prospective lenders, investors and others to submit their resolution plans to the Resolution
Professional (RP) latest by 1 January 2022. The RP reviewed the resolution plans and ensured conformity with
Insolvency and Bankruptcy Code 2016. The compliant plans were presented to Committee on Creditors (CoC)
on 2 February 2022 and the resolution plan submitted by PQR Ltd. was evaluated as highest evaluated
Compliant Resolution Plan. CoC of MNO Ltd. approved the Resolution Plan submitted by PQR Ltd. on 2 March
2022. The approval of NCLT was finally obtained on 4 May 2022.
PQR Ltd. submitted detailed plans and commitments as part of the resolution plan including clearance of all
outstanding debts which were leading to negative cash flows. Please suggest how would you deal with this
situation as the auditors of MNO Ltd. (RTP-Nov-2019)(MTP-May-2019)
Ans. (i) As per SA 570 “Going Concern”, when events or conditions have been identified that may cast
significant doubt on the entity’s ability to continue as a going concern, the auditor shall obtain
sufficient appropriate audit evidence to determine whether or not a material uncertainty exists
through performing additional audit procedures, including consideration of mitigating factors.
(ii) These procedures shall include:
 When management has not yet performed an assessment of the entity’s ability to continue as a
going concern, requesting management to make its assessment.
 Evaluating management’s plans for future actions in relation to its going concern assessment,
whether the outcome of these plans is likely to improve the situation and whether management’s
plans are feasible in the circumstances.
 When the entity has prepared a cash flow forecast, and analysis of the forecast is a significant
factor in considering the future outcome of events or conditions in the evaluation of
management’s plans for future action:

CA Final Audit - By CA. Sarthak Niraj Jain 51


Auditing Standards, Statements and Guidance Notes - An Overview

(a) Evaluating the reliability of the underlying data generated to prepare the forecast; and
(b) Determining whether there is adequate support for the assumptions underlying the forecast.
 Considering whether any additional facts or information have become available since the date on
which management made its assessment.
 Requesting written representations from management and, where appropriate, TCWG, regarding
their plans for future action and the feasibility of these plans.
(iii) Further Audit Procedure
 The auditor shall evaluate whether sufficient appropriate audit evidence has been obtained
regarding, and shall conclude on, the appropriateness of management’s use of the going concern
basis of accounting in the preparation of the financial statements.
 If events or conditions have been identified that may cast significant doubt on the entity’s ability
to continue as a going concern but, based on the audit evidence obtained the auditor concludes
that no material uncertainty exists,
 The auditor shall evaluate whether, in view of the requirements of the applicable financial
reporting framework, the financial statements provide adequate disclosures about these events
or conditions.
(iv) In the instant case:
 The approval of the resolution plan is a significant mitigating factor to counter the going
concern issues of MNO Ltd. PQR Ltd. has submitted a detailed plan and commitments that has
been given as part of the resolution plan which includes clearance of all outstanding debts which
were leading to negative cash flows.
(v) Conclusion:
 Therefore, it can be said that the events and conditions are mitigated effectively and there is no
material uncertainty in relation to the ability of the company to continue as a going concern.

77. FAST
 Company should disclose events or conditions & mitigating factors and auditor to give EOM.
Circumstances affects Going Concern
Star Ltd. is a power generating company which uses coal as raw material for its power generating plant. The
CA.Sarthak Niraj Jain
Company has been allotted coal blocks in the state of Jharkhand and Odisha. During the FY 2021-22, a scam
regarding allotment of coal blocks was unveiled leading to a ban on the allotment of coal blocks to various
companies including Star Ltd. This happened in the month of December 2021 and as such entire power
generation process of Star Ltd, came to a halt in that month. As a result of such ban, and the resultant stoppage
of the production process, many key managerial personnel of the company left the Company. There were
delays in the of payment of wages and salaries and the banks from whom the Company had taken funds for
project financing also decided not to extend further finance or to fund further working capital requirements of
the Company. Further, when discussed with the management, the statutory auditor understood that the
Company had no action plan to mitigate such circumstances. Further, all such circumstances were not reflected
the financial statements of Star Ltd. What course of action should the statutory auditor of the Company
consider in such situation? (RTP-May-2021)(Study Material)
Ans. (i) SA 570- “Going Concern”: - Deals with the auditor’s responsibilities in the audit of financial
statements relating to going concern and the implications for the auditor’s report.
 The auditor’s responsibilities are to obtain SAAE regarding, and
 Conclude on, the appropriateness of management’s use of the going concern basis of accounting
in the preparation of the FSs, and
 To conclude, based on the audit evidence obtained, whether a material uncertainty exists about
the entity’s ability to continue as a going concern.
(ii) When the use of going concern basis of accounting is inappropriate:
 If the financial statements have been prepared using the going concern basis of accounting but, in
the auditor’s judgment, management’s use of the going concern basis of accounting in the
preparation of the financial statements is inappropriate,
 The auditor shall express an adverse opinion.

52 Ab Audit Hoga Sabse Scoring - By CA. Sarthak Niraj Jain


Auditing Standards, Statements and Guidance Notes - An Overview

(iii) When adequate disclosure of a material uncertainty is not made in the FSs
 The auditor shall express a qualified opinion or adverse opinion, as appropriate, in accordance
with SA 705 (Revised); and
 In the Basis for Qualified (Adverse) Opinion section of the auditor’s report, state that a material
uncertainty exists that may cast significant doubt on the entity’s ability to continue as a going
concern and that the financial statements do not adequately disclose this matter.
(iv) In the present case: - The following circumstances indicate the inability of Star Ltd. to continue as a
going concern:
 Ban on the allotment of coal blocks
 Halt in power generation
 Key Managerial Personnel leaving the Company.
 Banks decided not to extend further finance and not to fund the working capital requirements of
the Company.
 Non availability of sound action plan to mitigate such circumstances.
(v) Conclusion: -Therefore, considering the above factors it is clear that the going concern basis is
inappropriate for the Company. Further, such circumstances are not reflected in the FSs of the
Company. As such, the statutory auditor of Star Ltd. should:
 Express an adverse opinion in accordance with SA 705 (Revised) and
 In the Basis of Opinion paragraph of the auditor’s report, The statutory auditor should state that:
 a material uncertainty exists that may cast significant doubt on the entity’s ability to
continue as a going concern and that the financial statements do not adequately disclose this
matter.
78. Appropriateness of using Going Concern Assumption in case of Material Uncertainty Exists

FAST
OM Ltd. is a company engaged in the business of manufacture of spare parts. Shanti & Associates are the
statutory auditors of the company for the FY 2021-22. During the course of audit, CA Shanti noticed that the
company had a major customer, namely, Korean Mart from South Korea. Owing to an outbreak of war and

CA.Sarthak Niraj Jain


subsequent destruction leading to government ban on import and export in South Korea, the demand from
Korean Mart for the products of OM Ltd. ended for an unforeseeable time period. When discussed with the
management, CA Shanti was told that the company is in the process of identifying new customers for their
products. CA Shanti understands that though the use of going concern assumption is appropriate but a material
uncertainty exists with respect to the identification of new customers. This fact is duly reflected in the financial
statements of OM Ltd. for the FY 2021-22. How should CA Shanti deal with this matter in the auditor’s report
for the FY 2021-22 in accordance with relevant Standard on Auditing? (MTP-Dec-2021)(Study Material)
Ans. (i) As per SA 570, “Going Concern” - loss of a major market or a key customer is one of the operating
indicators that may cast significant doubt on the company’s ability to continue as a going concern.
(ii) In the present case:
 OM Ltd. has a key customer in South Korea from which the demand for its products has ended on
account of outbreak of war, subsequent destruction and government ban on import and export in
South Korea.
 Further, the company has not yet identified new customers and is in the process of doing the same.
 As such, the identification of new customer is a material uncertainty that cast a significant doubt on
the company’s ability to continue as a going concern.
 However, this matter is duly disclosed by the management of OM Ltd. in the financial statements
for the year ended 31.03.2021.
 As such, considering that the going concern assumption is appropriate but a material
uncertainty exists with respect to identification of new customer, CA Shanti should:
(1) Express an unmodified opinion and
(2) Include in his audit report, a separate section under the heading “Material Uncertainty Related
to Going Concern” to:

CA Final Audit - By CA. Sarthak Niraj Jain 53


Auditing Standards, Statements and Guidance Notes - An Overview

(i) Draw attention to the note in the financial statements that discloses the matters and
(ii) State that these events or conditions indicate that a material uncertainty exists that may
cast significant doubt on the entity’s ability to continue as a going concern and that the
auditor’s opinion is not modified in respect of the matter.
(iii) Conclusion: -Thus, CA Shanti should deal with this matter in his auditor’s report in the above
mentioned manner.
79. Uncertainty Clamping on the Going Concern – Reporting
M/s Airlift Ltd., carrying on the business of Passenger Transportation by air is running into continuous
financial losses as well as reduction in Sales due to stiff competition and frequent break down of its own
aircrafts. The Financial Statements for the Year ended on 31/03/2022 are to be now finalized. The
Management is quite uncertain as to its ability to continue in near future and has informed the Auditors that
having seized of this matter, it had constituted a committee to study this aspect and to give suggestions for
recovery, if any, from this bad situation. Till the study is completed, according to the Management, the issue
involves uncertainty as to its ability to continue its business and it informs the Auditor that the fact of
uncertainty clamping on the, 'Going Concern" would suitably be disclosed in notes to accounts. State the
reporting requirement if any, in the Independent Auditor's Report in respect of this matter. (Nov-2018-New)
Ans. (i) As per SA 570 “Going Concern”
 if the auditor concludes that management’s use of the going concern basis of accounting is
appropriate in the circumstances but a material uncertainty exists, the auditor shall determine
whether the financial statements :
 Adequately disclose the principal events or conditions that may cast significant doubt on the
entity’s ability to continue as a going concern and management’s plans to deal with these

FAST
events or conditions; and
 Disclose clearly that there is a material uncertainty related to events or conditions that may
cast significant doubt on the entity’s ability to continue as a going concern and, therefore, that

CA.Sarthak Niraj Jain


it may be unable to realize its assets and discharge its liabilities in the normal course of
business.
(ii) If adequate disclosure about the material uncertainty is made in the FS
 The auditor shall express an unmodified opinion and the auditor’s report shall include a separate
section under the heading “Material Uncertainty Related to Going Concern” to:
 Draw attention to the note in the financial statements that discloses the matters set out
above; and
 State that these events or conditions indicate that a material uncertainty exists that may
cast significant doubt on the entity’s ability to continue as a going concern and that the
auditor’s opinion is not modified in respect of the matter.
(iii) In the instant case:
 M/s Aircraft Ltd. is running into continuous financial losses as well as reduction in sales due to
stiff competition and frequent break down of its own aircrafts and management of Aircraft Ltd. is
uncertain as of its ability to continue in near future.
 Therefore, a committee has been constituted to study this aspect and till the time study is
completed management accordingly decided to suitable disclose this aspect in notes to accounts.
(v) Conclusion: -
 Therefore, the auditor should disclose about the material uncertainty and express an
unmodified opinion and
 In his audit report shall include a separate section under the heading “Material Uncertainty
Related to Going Concern” to draw attention to the note in the financial statements that discloses
the matters set out above; and
 State that these events or conditions indicate that a material uncertainty exists that may cast
significant doubt on the entity’s ability to continue as a going concern and that the auditor’s
opinion is not modified in respect of the matter.

54 Ab Audit Hoga Sabse Scoring - By CA. Sarthak Niraj Jain


Auditing Standards, Statements and Guidance Notes - An Overview

80. Contingent Liability & Impact on Going Concern


ABC Company files a law suit against Unlucky Company for ` 5 crores. The Attorney of Unlucky Company feels
that the suit is without merit, so Unlucky Company merely discloses the existence of the law suit in the notes
accompanying its financial statements. As an auditor of Unlucky Company, how will you deal with the
situation? (Study Material)
Ans. (i) As per AS 29 "Provisions, Contingent liabilities and Contingent Assets"
 A contingent liability is a possible obligation that arises from past events and the existence of
which will be confirmed only by the occurrence or non-occurrence of one or more uncertain
future events not wholly within the control of the enterprise.
 Further, future events that may affect the amount required to settle an obligation should be
reflected in the amount of a provision where there is sufficient objective evidence that the event
will occur.
(ii) As per SA 570 “Going Concern”
 There are certain examples of events or conditions that, individually or collectively, may cast
significant doubt about the going concern assumption.
 Pending legal or regulatory proceedings against the entity that may, if successful, result in claims
that the entity is unlikely to be able to satisfy is one of the example of such event.
(iii) Auditor’s procedure: -When the auditor concludes that the use of the going concern assumption is
appropriate in the circumstances but a material uncertainty exists, the auditor shall determine
 Whether the financial statements adequately describe the principal events or conditions that
may cast significant doubt on the entity’s ability to continue as a going concern and management’s
plans to deal with these events or conditions; and
 Disclose clearly that there is a material uncertainty related to events or conditions that may

FAST
cast significant doubt on the entity’s ability to continue as a going concern and, therefore, that it may
be unable to realize its assets and discharge its liabilities in the normal course of business.
(iv) In the instant case: - ABC Company has filed a law suit against Unlucky Company for ` 5 crores.

CA.Sarthak Niraj Jain


Though, the attorney of Unlucky Company feels that the suit is without merit so the company merely
discloses the existence of law suit in the notes accompanying its financial statements.
(v) Conclusion:
 Auditor should evaluate the source data on which basis the opinion is formed and evaluate the
appropriateness of use of going concern assumption.
 If the auditor finds the uncertainty, he may request the management to adjust the sum of ` 5
crore by making provision for expenses as per AS 29.
 If the management does not accept the request the auditor should qualify the audit report.
81. Reporting Responsibilities of Statutory Auditor
Yummy Ltd., dealing in manufacturing and trading of milk butter, has a benchmark in its product for so many
years. Tasty Ltd., a rival company to Yummy Ltd., has introduced its new product, peanut butter. Due to being
health conscious, the consumers have shifted from milk butter to peanut butter within few months. This has
result into massive loss during the year to Yummy Ltd. due to non -selling of perishable milk products. The
company has also started having negative net worth. It's production head, finance head and marketing head
have also left the company. The company has no sound action plan to mitigate these situations. Kindly guide the
auditor of Yummy Ltd., how he should deal with the situation.
OR
R & Co. is the statutory auditor of S Ltd. For the financial year ended on 31st March 2022, S Ltd had disclosed in
the notes (Note No. X) “The state pollution control board had ordered the closure of the company’s only
manufacturing plant on the ground that it is environmentaly damaging, which the company had challenged in a
law suit. Pending the outcome of the law suit the financial statements are prepared on a going concern basis”.
Further the financial statements prepared by the management of S Ltd include financial statements of certain
branches which are audited by other auditors. What are the reporting responsibilities of R & Co.?

CA Final Audit - By CA. Sarthak Niraj Jain 55


Auditing Standards, Statements and Guidance Notes - An Overview

Ans. This question involves two broad aspect with respect to reporting requirements i.e.
 one which deals with Going Concern aspect where the company has gone for legal suit and
 other is work done by other auditors.
(i) As per facts of the case:
 The State Pollution Control Board has issued the closure order for S Ltd., on account of
environmental damaging by its only manufacturing plant.
 However, S Ltd had challenged the same by way of a law suit. Due to pendency of the outcome of
the legal suit, the company has prepared its financial statements on going concern basis.
(ii) As per SA 570 “Going Concern”
 Under the going concern assumption, an entity is viewed as continuing in business for the
foreseeable future.
 General purpose financial statements are prepared on a going concern basis, unless
management either intends to liquidate the entity or to cease operations, or has no realistic
alternative but to do so.
Management’s assessment of the entity’s ability to continue as a going concern involves making
a judgment, at a particular point in time, about inherently uncertain future outcomes of events or
conditions.
(iii) Auditor’s Responsibility:
 The auditor’s responsibility is to obtain sufficient appropriate audit evidence about the
appropriateness of management’s use of the going concern assumption in the preparation and
presentation of the financial statements and
 To conclude whether there is a material uncertainty about the entity’s ability to continue as a

FAST
going concern. For this the auditor may take the help of expert.
(iv) As per SA 620, “Using the Work of an Auditor’s Expert”
 If expertise in a field is necessary to obtain sufficient appropriate audit evidence, he may

CA.Sarthak Niraj Jain


determine to use the work of an auditor’s expert. On the basis of expert’s opinion he may decide
to rely or not on assessment of management.
(v) As per SA 570 “Going Concern”
 Pending legal proceedings is a condition that, individually, may cast significant doubt about the going
concern assumption. Existence of above condition signifies that a material uncertainty exists.
 Further, when the auditor concludes that the use of the going concern assumption is appropriate
in the circumstances but a material uncertainty exists, the auditor shall determine whether the
financial statements:
 Adequately describe the principal events or conditions that may cast significant doubt on
the entity’s ability to continue as a going concern and management’s plans to deal with these
events or conditions; and
 Disclose clearly that there is a material uncertainty related to events or conditions that
may cast significant doubt on the entity’s ability to continue as a going concern and,
therefore, that it may be unable to realise its assets and discharge its liabilities in the normal
course of business.
(vi) Reporting in Audit Report
 If adequate disclosure is made in the financial statements, the auditor shall express an
unmodified opinion and include an Emphasis of Matter paragraph as per SA 706 “Emphasis of
matter paragraphs and other matter paragraphs in the Independent Auditor’s Report”, in the
auditor’s report to:
 Highlight the existence of a material uncertainty relating to the event or condition that may
cast significant doubt on the entity’s ability to continue as a going concern; and
 Draw attention to the note in the financial statements that discloses the matters

56 Ab Audit Hoga Sabse Scoring - By CA. Sarthak Niraj Jain


Auditing Standards, Statements and Guidance Notes - An Overview

(vii) Conclusion: -In the present situation, management of S Ltd. had disclosed the above fact in the
financial statement. Further, use of the going concern assumption is appropriate but a material
uncertainty exists so assuming the assessment and disclosure of S Ltd. in order, R & Co. should include
an Emphasis of Matter paragraph in the auditor’s report.
(viii) Further, as per SA 600 “Using the work of Another Auditor”
 When the principal auditor has to base his opinion on the financial information of the entity as a
whole relying upon the statements and reports of the other auditors,
 His report should state clearly the division of responsibility for the financial information of the
entity by indicating the extent to which the financial information of components audited by the
other auditors have been included in the financial information of the entity, e.g., the number of
divisions/ branches/ subsidiaries or other components audited by other auditors.
SJ Note: Alternative answer is possible assuming that company is not going concern, hence auditor should issue
adverse report.
SA-580
Written Representations
82. Date of and Period covered by Written Representation
Mokshda & Co is the statutory auditor of Get My Trip Ltd. The company is in the business of tours and travels.
Annual turnover of the company is INR 2765 crore and profits are INR 285 crore. During the planning meeting
of the management and the auditors, it was discussed that the management needs to provide written
representation letter to the auditors for the preparation of the financial statements and for the completeness of
the information provided to the auditor. At the time of closure of the audit, there has been some confusion
about the requirements of the written representation letter. Management argued that representation need not

FAST
be written, it can also be verbal which has been provided to the audit team during the course of their audit.
Auditors have completed their documentation and hence in a way, representation based on verbal discussions
with the auditors has also got documented. Auditors explained that this is mandatory to obtain written

CA.Sarthak Niraj Jain


representation in accordance with the requirements of SA 580. However, still some confusion remains
regarding the date and period covered by the written representation. You are required to advise about the date
of and period covered by written representation in view of SA 580. (MTP-May-2021)
OR
PRSH & Co is the statutory auditor of Make My Journey Ltd. The company is in the business of tours and travels.
Annual turnover of the company is INR 2000 crores and profits are INR 190 crores. During the planning
meeting of the management and the auditors, it was discussed that the management needs to provide written
representation letter to the auditors for the preparation of the financial statements and for the completeness of
the information provided to the auditor. At the time of closure of the audit, there has been some confusion
about the requirements of the written representation letter. Management argued that representation need not
be written, it can also be verbal which has been provided to the audit team during the course of their audit.
Auditors have completed their documentation and hence in a way, representation based on verbal discussions
with the auditors has also got documented. Auditors explained that this is mandatory to obtain written
representation in accordance with the requirements of SA 580. However, still some confusion remains
regarding the date and period covered by the written representation. You are required to advise about the
date of and period covered by written representation in view of SA 580. (RTP-May-2019)
Ans. (i) As per SA 580, “Written Representations – Date of WR
 As written representations are necessary audit evidence, the auditor’s opinion cannot be
expressed, and the auditor’s report cannot be dated, before the date of the written
representations.
 Furthermore, because the auditor is concerned with events occurring up to the date of the
auditor’s report that may require adjustment to or disclosure in the financial statements,
 The written representations are dated as near as practicable to, but not after, the date of the
auditor’s report on the financial statements.

CA Final Audit - By CA. Sarthak Niraj Jain 57


Auditing Standards, Statements and Guidance Notes - An Overview

(ii) WR for specific assertions


 In some circumstances it may be appropriate for the auditor to obtain a WR about a specific
assertion in the financial statements during the course of the audit. Where this is the case, it may be
necessary to request an updated WR.
(iii) Period covered in WR
 The written representations are for all periods referred to in the auditor’s report because
management needs to reaffirm that the written representations it previously made with respect to
the prior periods remain appropriate.
 The auditor and management may agree to a form of written representation that updates written
representations relating to the prior periods by addressing whether there are any changes to such
written representations and, if so, what they are.
(iv) Management’s Responsibility:
 Situations may arise where current management were not present during all periods referred
to in the auditor’s report.
 Such persons may assert that they are not in a position to provide some or all of the written
representations because they were not in place during the period.
 This fact, however, does not diminish such persons’ responsibilities for the financial statements
as a whole.
 Accordingly, the requirement for the auditor to request from them written representations that
cover the whole of the relevant period(s) still applies.
83.
FAST
Validity of Written Representation
The Auditor of PQR Pvt. Ltd. having turnover of ` 12 crore, was not able to get the confirmation about the
existence and value of certain stock. However, a certificate from the Management has been obtained regarding

CA.Sarthak Niraj Jain


the existence and value of the stock at the year end. The auditor relied on the same and without any further
procedure, signed the Audit Report. Is he right in his approach?
OR
An auditor of Mohan Ltd. was not able to get the confirmation about the existence and value of certain
machineries. However, the management gave him a certificate to prove the existence and value of the
machinery as appearing in the books of account. The auditor accepted the same without any further procedure
and signed the audit report. Is he right in his approach?
OR
While auditing a lawyer company, Mr. X, the statutory auditor of the company, was unable to get the
confirmation about the existence and value of certain books existed in the library worth ` 35 lakhs. However,
the management gave him a certificate to prove the existence and value of the books as appearing in the books
of account. The auditor accepted the same without any further procedure and signed the audit report. Is he
right in his approach?

Ans. (i) Responsibility of Management & Auditor


 The physical verification of stock is the primary responsibility of the management.
 The auditor, however, is required to examine the verification programme adopted by the
management. He must satisfy himself about the existence and valuation of stock.
(ii) In the case of PQR Pvt. Ltd.:
 The auditor has not been able to verify the existence and value of certain stock despite the
verification procedure followed in routine audit.
 He accepted the certificate given to him by the management without making any further enquiry.

58 Ab Audit Hoga Sabse Scoring - By CA. Sarthak Niraj Jain


Auditing Standards, Statements and Guidance Notes - An Overview

(iii) As per SA 580 “Written Representations”


 When representation relate to matters which are material to the financial information, then the
auditor should seek corroborative audit evidence for other sources inside or outside the entity.
 He should evaluate whether such representations are reasonable and consistent with other
evidences and should consider whether individuals making such representations can be expected
to be well informed on the matter.
 “Written Representations” cannot be a substitute for other audit evidence that the auditor
could reasonably expect to be available.
(iv) Conclusion:
 If the auditor is unable to obtain sufficient appropriate audit evidence that he believes would
be available regarding a matter, which has or may have a material effect on the financial information,
this will constitute a limitation on the scope of his examination even if he has obtained a
representation from management on the matter.
 Therefore, the approach adopted by the auditor is not tenable.
84. Refusal to provide WR
In the course of audit of K Ltd., its auditor Mr. ‘N’ observed that there was a special audit conducted at the
instance of the management on a possible suspicion of a fraud and requested for a copy of the report to enable
him to report on the fraud aspects. Despite many reminders it was not provided. In absence of the special audit
report, Mr. ‘N’ insisted that he be provided with at least a written representation in respect of fraud on/by the
company. For this request also, the management remained silent. Please guide Mr. ‘N’.
(Study Material)(MTP-Nov-2018)(RTP-May-2018)
Ans. (i)
FAST
Responsibility of Management & Auditor
 As per SA 240, “The Auditor’s Responsibilities relating to Fraud in an Audit of Financial

CA.Sarthak Niraj Jain



Statements”, the primary responsibility for the prevention and detection of fraud rests with both
TCWG of the entity and management.
In addition an auditor conducting an audit in accordance with SAs is responsible for obtaining
reasonable assurance that the financial statements taken as a whole are free from material
misstatement, whether caused by fraud or error?
(ii) Fraud Detection Risk
 The risk of not detecting a material misstatement resulting from fraud is higher than the risk of
not detecting one resulting from error.
 This is because fraud may involve sophisticated and carefully organized schemes designed to
conceal it, such as forgery, deliberate failure to record transactions, or intentional
misrepresentations being made to the auditor.
(iii) As per SA 580, “Written Representations”
If management modifies or does not provide the requested written representations, it may alert the
auditor to the possibility that one or more significant issues may exist. Further, If the management does
not provide one or more of the requested written representations, the auditor shall:
 Discuss the matter with management,
 Re-evaluate the Integrity of the management and evaluate the effect that this may have on the
reliability of representations (oral or written) and audit evidence in general, and
 Take appropriate actions, including determining the possible effect on the opinion in the auditor’s
report.
The auditor shall disclaim an opinion on the financial statements if the auditor concludes that there is
sufficient doubt about the integrity of management such that the written representations are not
reliable; or management does not provide the written representations.

CA Final Audit - By CA. Sarthak Niraj Jain 59


Auditing Standards, Statements and Guidance Notes - An Overview

(iv) Facts of the case:


 In the instant case, in the course of audit of K Ltd., its auditor Mr. N observed that there was a
special audit conducted at the instance of the management on a possible suspicion of fraud.
 Therefore, the auditor requested for special audit report, which was not provided by the
management despite of many reminders.
 Mr. N also insisted for written representation in respect of fraud on/by the company. For this
request also management remained silent.
(v) Conclusion:
 Hence, the fact is required to be reported and the auditor should also disclaim an opinion on the
financial statements.
85. Duty of an Auditor if Management Refuses to provide WRs
In the course of audit of ABC Ltd. its management refuses to provide written representations. As an auditor
what is your duty?
Ans. (i) As per SA 580 “Written Representations”
If the management does not provide one or more of the requested written representations, the
auditor shall:
 Discuss the matter with management,
 Re-evaluate the Integrity of the management and evaluate the effect that this may have on the
reliability of representations (oral or written) and audit evidence in general, and
 Take appropriate actions, including determining the possible effect on the opinion in the auditor’s
report.
(ii) Effect on Opinion
 The auditor should disclaim an opinion on the financial statements if management does not

FAST
provide written representations in accordance with SA 705 “Modifications to the Opinion in the
Independent Auditor’s Report”.
SA-600

86.
CA.Sarthak Niraj Jain Using the Work of Another Auditor
Principal Auditor’s Right to Review the WPs of Branch Auditors
B is the Principal Auditor of ABC Co. Ltd., with 8 branches audited by 8 Branch Auditors. B wanted to ensure
that the works of Branch Auditors were adequate for the purpose of his audit. Hence he insisted on Branch
Auditors to get familiar with a check list he prepared for branches and, besides, required them to share the
working papers compiled by them for his review and return. Is Principal Auditor within his right in asking for
such sharing of working papers? (May-2018-New)
Ans. (i) Using the Work of Another Auditor-Principals
 SA 600 “Using the Work of Another Auditor” guides principal auditor regarding the procedures
to be performed when he is using the work of another auditor.
 As per SA 600, when principal auditor plans to use the work of branch auditor, he should
consider the professional competence of the other auditor in the context of specific assignment if
the other auditor is not a member of the ICAI.
 He should perform procedures to obtain sufficient appropriate audit evidence, that the work of
the other auditor is adequate for the principal auditor's purposes, in the context of the specific
assignment.
(ii) Audit Documentation
 As per SA 230 “Audit Documentation” and SQC 1 “Quality Control for Firms that Perform Audits and
Reviews of Historical Financial Information, and Other Assurance and Related Services
Engagements”, unless otherwise specified by law or regulation, audit documentation is the
property of the auditor.
 The Principal auditors of an enterprise do not have right of access to the audit working papers of
the branch auditors.

60 Ab Audit Hoga Sabse Scoring - By CA. Sarthak Niraj Jain


Auditing Standards, Statements and Guidance Notes - An Overview

(iii) In the present case: - Mr. B requires the branch auditors to share their working papers with him for
the purpose of review.
(iv) Conclusion: considering the requirements of SA 600, SA 230 and SQC 1, principal auditor is not right
in asking for sharing of working papers.
87. Role of Auditor in case of Parent Company and Subsidiary Company
BETA Ltd is the Subsidiary company of ALPHA Ltd. PQR & Associates has been appointed as auditor of ALPHA
Ltd. for the Financial Year 2021-22 and MNO & Associates has been appointed as auditor of BETA Ltd for the
year 2021-22. Explain the role of PQR & Associates and MNO & Associates as auditors of the parent company
and subsidiary company respectively. (MTP-Nov-2020)
OR
B Ltd. is the Subsidiary company of A Ltd. ABC & Associates has been appointed as auditor of A Ltd. for the
Financial year 2021-22 and XYZ & Associates has been appointed as auditor of B Ltd for the year 2021-22.
Explain the role of ABC & Associates and XYZ & Associates as auditors of the parent company and subsidiary
respectively. (Nov-2016)
Ans. (i) As per SA 600 “Using the Work of Another Auditor”, there should be sufficient liaison between the
principal auditor (hereinafter referred as auditor of Parent Company) and the other auditor
(hereinafter referred as auditor of Subsidiary Company).
(ii) Role of Principal Auditor (PQR & Associates- Auditor of Parent Company):
 It is necessary to issue written communication(s) as a principal auditor to the other auditor.
 The principal auditor should advise the other auditor of any matters that come to his attention
that he thinks may have an important bearing on the other auditor’s work.

FAST
When considered necessary by him, the principal auditor may require the other auditor to answer a
detailed questionnaire regarding matters on which the principal auditor requires information for
discharging his duties.

CA.Sarthak Niraj Jain


(iii) Role of Other Auditor (MNO & Associates- Auditor of Subsidiary Company):
 The other auditor, knowing the context in which his work is to be used by the principal auditor,
should co-ordinate with the principal auditor. For example, by bringing to the principal
auditor’s immediate attention to any significant findings requiring to be dealt with at entity level,
adhering to the time-table for audit of the component, etc.
 He should ensure compliance with the relevant statutory requirements.
 The other auditor should respond to the questionnaire sent by Principal Auditor on a timely basis.
88. Qualification in Branch Auditor's Report-Subsequent Events
As an auditor, comment on You are a Principal Auditor of Sri Company Limited which has three branches the
accounts of which are subject to audit by qualified branch auditors. One of the branch auditors qualified his
report for non-provision of doubtful debts which he considered to be material for the company as a whole.
Subsequent to their reporting, but before you could sign the audit report on the accounts of the company as a
whole, the management informed you that the debt under the subject-matter of qualification in Branch
Auditor’s report had been fully recovered.
Ans.  The Branch auditor had qualified on non-provision of a major debt. After his report but before the
issue of report by Principal auditor an event happened which has thrown new light on the facts that
existed as on the date of balance sheet date.
 This is a subsequent event within the meaning of SA 560 “Subsequent Events” i.e. event that had
taken place between the date of balance sheet date and the date of signing the audit report.
 In relation to the cases where the component (i.e. branch) is audited by another auditor
 The subsequent event would include events that had taken place between the date of audit
report of the component and the date of signing the audit report of the entity as a whole by the
principal auditor.

CA Final Audit - By CA. Sarthak Niraj Jain 61


Auditing Standards, Statements and Guidance Notes - An Overview

 On becoming aware of the subsequent events, the auditor should consider whether the accounts
had been drawn so as to give effect to the facts of subsequent events.
 Conclusion: - Accordingly, the auditor should omit qualification as the debt is no more doubtful in
view of its recovery in full. However, the auditor may check that it has in fact been received by a
substantial vouching of the subsequent events which had been considered by him to make himself
fully satisfied about his report in the matter.
SA-610
Using the Work of Internal Auditors
89. Using the Work of Internal Auditor
CA Mr. X, a practicing chartered accountant has been appointed as an internal auditor of Textile Ltd. He
conducted the physical verification of the inventory at the year-end and handed over the report of such
verification to CA Mr. Y, the statutory auditor of the Company, for his view and reporting. Can CA Mr. Y rely on
such report? (MTP-Nov-2018)
OR
CA. Amboj, a practicing chartered accountant has been appointed as an internal auditor of Textile Ltd. He
conducted the physical verification of the inventory at the year-end and handed over the report of such
verification to CA. Kishore, the statutory auditor of the Company, for his view and reporting. Can CA. Kishore
rely on such report?
Ans. (i) As per SA 610 “Using the Work of Internal Auditors”, while determining whether the work of the
internal auditors can be used for the purpose of the audit, the external auditor shall evaluate-
 The extent to which the internal audit function’s organizational status and relevant policies and
procedures support the objectivity of the internal auditors;


FAST
The level of competence of the internal audit function; and
Whether the internal audit function applies a systematic and disciplined approach, including

(ii) CA.Sarthak Niraj Jain


quality control.
the external auditor shall not use the work of the internal audit function if the external auditor
determines that:
 The function’s organizational status and relevant policies and procedures do not adequately
support the objectivity of internal auditors;
 The function lacks sufficient competence; or
 The function does not apply a systematic and disciplined approach, including quality control.
(iii) Conclusion In the instant case CA. Kishore should ascertain
 The internal auditor’s scope of verification, area of coverage and method of verification.
 He should review the report on physical verification taking into consideration these factors.
 If possible, he should also test check few items and he can also observe the procedures performed
by the internal auditors.
 If the statutory auditor is satisfied about the appropriateness of the verification, he can rely on
the report but if he finds that the verification is not in order, he has to decide otherwise.
 The final responsibility to express opinion on the financial statement remains with the statutory
auditor.
90. Direct Assistance from Internal Auditor
OPQ Ltd is in the software consultancy business. The company had large balance of accounts receivables in the
past years which have been assessed as area of high risk. For the year ended 31 March 2022, in respect of the
valuation of accounts receivable, the statutory auditor was assigned with the checking of accuracy of the aging
of the accounts receivables and provision based on ageing, to the internal auditor providing direct assistance to
him. Comment. (MTP-Nov-2019)(RTP-May-2019)(MTP-May-2019)

62 Ab Audit Hoga Sabse Scoring - By CA. Sarthak Niraj Jain


Auditing Standards, Statements and Guidance Notes - An Overview

Ans. (i) Provision: -As per SA 610 Using the Work of Internal Auditor, the external auditor (Statutory
Auditor) shall not use internal auditors to provide direct assistance to perform procedures that:
 Involve making significant judgments in the audit;
 Relate to higher assessed risks of material misstatement where the judgment required in
performing the relevant audit procedures or evaluating the audit evidence gathered is more
than limited;
 Relate to work with which the internal auditors have been involved and which has already been,
or will be, reported to management or TCWG by the internal audit function; or
 Relate to decisions the external auditor makes in accordance with this SA regarding the internal
audit function and the use of its work or direct assistance.
(ii) In the present case: - Statutory auditor assigned internal auditor to provide direct assistance
regarding checking of the accuracy of the aging of the accounts receivables and provision based on
ageing. Valuation of accounts receivable is assessed as an area of higher risk.
(iii) Conclusion: -Statutory auditor could assign the checking of the accuracy of the aging to an internal
auditor providing direct assistance. However, because the evaluation of the adequacy of the
provision based on the aging would involve more than limited judgment, it would not be appropriate
to assign that latter procedure to an internal auditor providing direct assistance.
91. Using Direct Assistance of Internal Auditor & Use in External Confirmation Procedure
Mr. A is appointed as statutory auditor of XYZ Ltd. XYZ Ltd is required to appoint internal auditor as per
statutory provisions given in the Companies Act, 2013 and appointed Mr. B as its internal auditor. The external
auditor Mr. A asked internal auditor to provide direct assistance to him regarding evaluating significant
accounting estimates by the management and assessing the risk of material misstatements.
(a)
FAST
Discuss whether Mr. A, statutory auditor, can ask direct assistance from Mr. B, internal auditor as stated
above in view of auditing standards.
(b)
CA.Sarthak Niraj Jain
Will your answer be different, if Mr. A ask direct assistance from Mr. B, internal auditor with respect to
external confirmation requests and evaluation of the results of external confirmation procedures?
(RTP-May-2020)
Ans. (i) Provision: -As per SA 610 Using the Work of Internal Auditor, the external auditor (Statutory
Auditor) shall not use internal auditors to provide direct assistance to perform procedures that:
 Involve making significant judgments in the audit;
Significant judgments include the following:
 Assessing the risks of material misstatement;
 Evaluating the sufficiency of tests performed;
 Evaluating the appropriateness of management’s use of the going concern assumption;
 Evaluating significant accounting estimates; and
 Evaluating the adequacy of disclosures in the financial statements, and other matters
affecting the auditor’s report
 Relate to higher assessed risks of material misstatement where the judgment required in
performing the relevant audit procedures or evaluating the audit evidence gathered is more
than limited;
 Relate to work with which the internal auditors have been involved and which has already been,
or will be, reported to management or TCWG by the internal audit function; or
 Relate to decisions the external auditor makes in accordance with this SA regarding the internal
audit function and the use of its work or direct assistance.
(ii) In the present case: - Mr. A asked internal auditor to provide direct assistance regarding evaluating
significant accounting estimates and assessing the risk of material misstatements.

CA Final Audit - By CA. Sarthak Niraj Jain 63


Auditing Standards, Statements and Guidance Notes - An Overview

(iii) Conclusion: Evaluation of Accounting estimates and assessing Risk of Material Misstatements
involve significant judgements, hence Mr. A should not insists for direct assistance in these areas.
Direct Assistance from Internal Auditor in case of External Confirmation Procedures:
 SA 610 “Using the Work of Internal Auditor”, provide relevant guidance in determining the
nature and extent of work that may be assigned to internal auditors.
In determining the nature of work that may be assigned to internal auditors, the external auditor
is careful to limit such work to those areas that would be appropriate to be assigned.
 In accordance with SA 505, “External Confirmation” the external auditor is required to maintain
control over external confirmation requests and evaluate the results of external confirmation
procedures, it would not be appropriate to assign these responsibilities to internal auditors.
 Conclusion: It would not be appropriate to use direct assistance w.r.t. obtaining external
confirmation requests and their evaluation. Assistance may be used in assembling information
necessary for the external auditor to resolve exceptions in confirmation responses
92. Using Direct Assistance of Internal Auditor
Mr. Sheetal is appointed as a statutory auditor of Mahi Ltd. Mahi Ltd is required to appoint an internal auditor
as per statutory provisions given in the Companies Act, 2013 and appointed Mr. Kunthu as its internal auditor.
The external auditor Mr. Sheetal asked internal auditor to provide direct assistance to him regarding
evaluating the sufficiency of tests performed and the adequacy of disclosures in the financial statements and
other matters affecting the auditor’s report. Discuss whether Mr. Sheetal, statutory auditor, can ask direct
assistance from Mr. Kunthu, internal auditor as stated above in view of relevant Standard on Auditing.
(MTP-May-2021)
Ans. As per SA 610 “Using the Work of Internal Auditor”, the external auditor shall not use internal auditors to

FAST
provide direct assistance to perform procedures that Involve making significant judgments in the audit.
Since the external auditor has sole responsibility for the audit opinion expressed, the external auditor needs to
make the significant judgments in the audit engagement.

CA.Sarthak Niraj Jain


Significant judgments include the following:
 Assessing the risks of material misstatement;
 Evaluating the sufficiency of tests performed;
 Evaluating the appropriateness of management’s use of the going concern assumption;
 Evaluating significant accounting estimates; and
 Evaluating the adequacy of disclosures in the financial statements, and other matters affecting the
auditor’s report.
Conclusion: -In view of above, Mr. Sheetal cannot ask direct assistance from internal auditors regarding
evaluating the sufficiency of tests performed and the adequacy of disclosures in the financial statements
and other matters affecting the auditor’s report.
SA-620
Using the Work of an Auditor’s Expert
93. Agreement with Auditor’s Expert
X Ltd had a net worth of INR 1300 crores because of which Ind AS became applicable to them. The company
had various derivative contracts – options, forward contracts, interest rate swaps etc. which were required to
be fair valued for which company got the fair valuation done through an external third party. The statutory
auditors of the company involved an auditor’s expert to audit valuation of derivatives. Auditor and auditor’s
expert were new to each other i.e. they were working for the first time together but developed a good bonding
during the course of the audit. The auditor did not enter into any formal agreement with the auditor’s expert.
Please advise. (MTP-May-2021)(Study Material)(MTP-May-2019)(RTP-May-2019)(MTP-Nov-2018)

64 Ab Audit Hoga Sabse Scoring - By CA. Sarthak Niraj Jain


Auditing Standards, Statements and Guidance Notes - An Overview

Ans. (i) As per SA 620 “Using work of Auditor’s Expert”


The auditor shall agree, in writing when appropriate, on the following matters with the auditor’s
expert:
 Nature, scope & objectives of Auditor’s Expert work (may include relevant technical standards or
other professional and industry requirements).
 Respective roles & responsibilities of auditors & Auditor Expert.
 Nature, timing & extent of communication, including form of report.
 Need for Auditor Expert to observe confidentiality requirements under ethical requirements or
Law and regulation.
(ii) In the instant case: - X Ltd. had various derivative contracts – options, forward contracts, interest
rate swaps etc. which were required to be fair valued for which company got the fair valuation done
through an external third party. The statutory auditors of the company involved an auditor’s expert to
audit valuation of derivatives.
(iii) Conclusion: Considering the complexity involved in the valuation and volume of derivatives and
also due to the fact that the auditor and auditor’s expert were new to each other, auditor should have
signed a formal agreement/ engagement letter with the auditor’s expert in respect of the work
assigned to him in accordance with SA 620.
94. Evaluating the Adequacy of the Auditor’s Expert’s Work
CA Dahu has been appointed as an auditor of M/s MAP Technocraft Ltd. to conduct statutory audit. While
conducting audit, he came across some difficulties which the management could not explain to him properly
and. therefore, he decided to take services of Mr. Jay, an engineering consultant. Mr. Jay performed his work
and submitted details to CA Dahu. State the specific procedure which CA Dahu should follow to evaluate the

Ans. (i)
FAST
adequacy of work performed by Mr. lay. (May-2019-New)
As per SA 620 Using the work of an Auditor’s Expert, the auditor shall evaluate the adequacy of the
auditor’s expert’s work for the auditor’s purposes, including the relevance and reasonableness of

(ii)
CA.Sarthak Niraj Jain
that expert’s findings or conclusions, and their consistency with other audit evidence, etc.
Specific procedure to evaluate the adequacy of the auditor’s expert’s work are –
 Enquiries of the auditor’s expert.
 Reviewing the auditor’s expert’s working papers and reports
 Corroborative procedure such as-
(a) Observing the auditor’s expert’s work
(b) Examining the published data, such as statistical reports from reputed source
(c) Confirming the relevant matters with third parties
(d) Performing detailed analytical procedure to see whether principles of materiality aspects
considered
(e) Re performing calculations
 Discussions with another expert with relevant expertise when, for example, the findings or the
conclusion of the auditor’s expert are not consistent with other audit evidence.
 Discussing the expert’s report with the management.
(iii) Therefore, as per SA 620 on “Using the Work of an Auditor’s Expert”,
The auditor shall evaluate the adequacy of the auditor’s expert’s work for the auditor’s purposes,
including:
 The relevance and reasonableness of that expert’s findings or conclusions, and their consistency
with other audit evidence;
 If that expert’s work involves use of significant assumptions and methods, the relevance and
reasonableness of those assumptions and methods in the circumstances; and
 If that expert’s work involves the use of source data that is significant to that expert’s work, the
relevance, completeness, and accuracy of that source data.

CA Final Audit - By CA. Sarthak Niraj Jain 65


Auditing Standards, Statements and Guidance Notes - An Overview

(iv) Additional Audit Procedure


If the auditor determines that the work of the auditor’s expert is not adequate for the auditor’s
purposes, the auditor shall:
 Agree with that expert on the nature and extent of further work to be performed by that
expert; or
 Perform further audit procedures appropriate to the circumstances.
95. Auditor’s Expert and Joint Auditors
KRP Ltd., at its annual general meeting, appointed Mr. X, Mr. Y and Mr. Z as joint auditors to conduct auditing
for the financial year 2021-22. For the valuation of gratuity scheme of the company, Mr. X, Mr. Y and Mr. Z
wanted to refer their own known Actuaries. Due to difference of opinion, all the joint auditors consulted their
respective Actuaries. Subsequently, major difference was found in the actuary reports. However, Mr. X agreed
to Mr. Y’s actuary report, though, Mr. Z did not. Mr. X contends that Mr. Y’s actuary report shall be considered
in audit report due to majority of votes. Now, Mr. Z is in dilemma.
Explain the responsibility of auditors, in case, report made by Mr. Y’s actuary, later on, found faulty.
(Study Material)
Ans. (i) As per SA 620 “Using the Work of an Auditor’s Expert”
 The expertise of an expert may be required in the actuarial calculation of liabilities associated
with insurance contracts or employee benefit plans etc., however, the auditor has sole
responsibility for the audit opinion expressed, and that responsibility is not reduced by the
auditor’s use of the work of an auditor’s expert.

FAST
The auditor shall evaluate the adequacy of the auditor’s expert’s work for the auditor’s
purposes, including the relevance and reasonableness of that expert’s findings or conclusions, and
their consistency with other audit evidence as per SA 500.

CA.Sarthak Niraj Jain


 Further, in view of SA 620, if the expert’s work involves use of significant assumptions and
methods, then the relevance and reasonableness of those assumptions and methods must be
ensured by the auditor and
 If the expert’s work involves the use of source data that is significant to that expert’s work, the
relevance, completeness, and accuracy of that source data in the circumstances must be verified
by the auditor.
(ii) In the instant case:
Mr. X, Mr. Y and Mr. Z, jointly appointed as an auditor of KRP Ltd., referred their own known Actuaries
for valuation of gratuity scheme. Actuaries are an auditor’s expert as per SA 620. Mr. Y’s referred
actuary has provided the gratuity valuation report, which later on found faulty. Further, Mr. Z is not
agreed with this report therefore he submitted a separate audit report specifically for such gratuity
valuation.
(iii) Duty of Joint Auditors
 In such situation, it was duty of Mr. X, Mr. Y and Mr. Z, before using the gratuity valuation report
of Actuary, to ensure the relevance and reasonableness of assumptions and methods used.
 They were also required to examine the relevance, completeness and accuracy of source data
used for such report before expressing their opinion.
(iv) Conclusion:
 Mr. X and Mr. Y will be held responsible for grossly negligence and using such faulty report
without examining the adequacy of expert actuary’s work whereas
 Mr. Z will not be held liable for the same due to separate opinion expressed by him.

66 Ab Audit Hoga Sabse Scoring - By CA. Sarthak Niraj Jain


Auditing Standards, Statements and Guidance Notes - An Overview

96. Auditor’s Reporting Responsibility


O Ltd. is in the business of manufacturing of steel. The manufacturing process requires raw material as iron
ore for which large stock was maintained by the company at year end – 31st March 2022. The nature of raw
material is such that its physical verification requires involvement of an expert. Management hired their expert
for stock take and auditors also involved auditor’s expert for the stock take.
The auditor observed that the work of the auditor’s expert was not adequate for auditor’s purposes and the
auditor could not resolve the matter through additional audit procedures which included further work
performed by both the auditor’s expert and the auditor.
Basis above, the auditor concluded that it would be necessary to express a modified opinion in the auditor’s
report because the auditor has not obtained sufficient appropriate audit evidence. However, the auditor
issued a clean report and included the name of the expert in his report to reduce his responsibility for the audit
opinion. Comment. (MTP-May-2019)
Ans. (i) As per SA 620, Using the work of an Auditor’s Expert,
 If the auditor concludes that the work of the auditor’s expert is not adequate for the auditor’s
purposes and the auditor cannot resolve the matter through the additional audit, which may
involve further work being performed by both the expert and the auditor, or include employing
or engaging another expert, it may be necessary to express a modified opinion in the auditor’s
report in accordance with SA 705 because the auditor has not obtained sufficient appropriate audit
evidence
 In addition, the auditor shall not refer to the work of an auditor’s expert in an auditor’s report
containing an unmodified opinion unless required by law or regulation to do so.
 If such reference is required by law or regulation, the auditor shall indicate in the auditor’s
report that the reference does not reduce the auditor’s responsibility for the audit opinion.

FAST
 If the auditor makes reference to the work of an auditor’s expert in the auditor’s report
because such reference is relevant to an understanding of a modification to the auditor’s opinion,
 The auditor shall indicate in the auditor’s report that such reference does not reduce the

CA.Sarthak Niraj Jain


auditor’s responsibility for that opinion.
(ii) Conclusion:
 In the given case, the auditor cannot reduce his responsibility by referring the name of auditor’s
expert and thereby issuing a clean report.
 Auditor should have issued a modified report and could have given reference to the work of an
auditor’s expert in that report if such reference was relevant to understanding of a
modification to the auditor’s opinion but even in that case the auditor should have indicated in his
report that such reference of auditor’s expert does not reduce his responsibility for that opinion.
SA-700
Forming an Opinion and Reporting on Financial Statements
97. Process of Judgement Formation by Auditor
What is the basis on which the judgement is formed by Auditor to express his opinion after the audit?
(May-2018-Old)
Ans. (i) After the audit, the opinion that the auditor expresses is the result of exercise of judgement on facts,
evidence and circumstances which he comes across in the course of audit.
(ii) The judgement is formed on the following basis-
a. Identification of the assertions to be examined.
b. Evaluation of the assertion as to relative importance.
c. Collection of the information or evidence about the assertions to enable him to give an informed
opinion.
d. Evaluation of evidence as valid or invalid, pertinent or not pertinent, sufficient or insufficient.
e. Formulation of judgement as to the fairness of the assertions under consideration.

CA Final Audit - By CA. Sarthak Niraj Jain 67


Auditing Standards, Statements and Guidance Notes - An Overview

98. Auditor’s Report for Audits Conducted in accordance with Both SA’s Issued by ICAI and ISA
KPI Ltd is a joint venture of KPI Inc, a company based in US, and OPQ Ltd, a company based in Japan
(hereinafter referred to as ‘JV partners’). KPI Ltd was registered in India and is operating as a marketing
support company for KPI Inc. All the costs of KPI Ltd are incurred in India and entire revenue of KPI Inc is
generated in USD. The entire funding requirements of KPI Ltd are taken care of by the JV partners. Since KPI
Ltd is based in India, hence it is also required to get its financial statements audited.
The company appointed new auditors for the audit of the financial statements for the year ended 31 March
2020 after doing all appointment formalities wherein auditors are required to ensure compliance with
Standards on Auditing and Internal Standards on Auditing.
As an expert you are required to advise the auditor about the requirements regarding auditor’s report for
audits conducted in accordance with both Standards on Auditing issued by ICAI and International Standards on
Auditing. (Study Material)
OR

KPI Ltd. is a company on which International Standards on Auditing are applicable along with Standard on
Auditing issued by the ICAI. The company appointed new auditors for the audit of the financial statements for
the year ended 31st March 2022 after doing all appointment formalities. In the auditor’s report, auditor
referred the International Standard on Auditing in addition to the Standard on Auditing issued by the ICAI. As
an expert, you are required to advise the auditor regarding auditor’s report for audits conducted in accordance
with both the Standards. (RTP-Nov-2019)
Ans. Auditor’s Report for Audits Conducted in Accordance with Both Standards on Auditing Issued by ICAI
and International Standards on Auditing:
As per SA 700, “Forming an Opinion and Reporting on Financial Statements”,

FAST
 An auditor may be required to conduct an audit in accordance with the International Standards on
Auditing, in addition to the Standards on Auditing issued by ICAI.
 If this is the case, the auditor’s report may refer to Standards on Auditing in addition to the

CA.Sarthak Niraj Jain


International Standards on Auditing, but the auditor shall do so only if:
(a) There is no conflict between the requirements in the International Auditing Standards and those in
SAs that would lead the auditor:
 to form a different opinion, or
 not to include an Emphasis of Matter paragraph or Other Matter paragraph that, in the
particular circumstances, is required by SAs; and
(b) The auditor’s report includes, at a minimum, each of the elements set out in Auditor’s Report
Prescribed by Law or Regulation discussed above when the auditor uses the layout
or wording specified by the Standards on Auditing.
When the auditor’s report refers to both the ISAs and the Standards on Auditing issued by ICAI,
the auditor’s report shall clearly identify the same including the jurisdiction of origin of the other
auditing standards.
SA-701
Communicating Key Audit Matters in the Independent Auditor’s Report
99. Determining Kay Audit Matter
Mr. Hemant Ramsey was appointed as the engagement partner for conducting the audit of Kshetra Lap Ltd. for
F.Y. 2021-22, on behalf of Ramsey & Associates. Mr. Vishay Tyagi was appointed as the engagement quality
control reviewer by the firm for the said audit. During F.Y. 2021-22, there was an implementation of ERP
system in a phased manner, in Kshetra Lap Ltd. due to which some of its business processes got automated. As
a result of the implementation of such a system, there was a significant effect on the auditor’s overall audit
strategy. Mr. Hemant discussed the implementation of such a system with Mr. Vishay and also told him that
such a matter may be a key audit matter to be reported in the audit report.
Mr. Vishay considered the significance of such matter but however he was of the opinion that such a matter did
not appear to link with the matters disclosed in the financial statements and so there was no need to disclose
such matter as a key audit matter. Whether the contention of Mr. Vishay is proper with respect to the matters
to be communicated as a key audit matter? (RTP-Dec-2021)

68 Ab Audit Hoga Sabse Scoring - By CA. Sarthak Niraj Jain


Auditing Standards, Statements and Guidance Notes - An Overview

Ans. (i) As per SA 701, ‘Communicating Key Audit Matters in the Independent Auditor’s Report’
The auditor shall determine, from the matters communicated with those charged with governance,
those matters that required significant auditor attention in performing the audit. In making this
determination, the auditor shall take into account the following:
(i) Areas of higher assessed risk of material misstatement, or significant risks identified in
accordance with SA 315.
(ii) Significant auditor judgments relating to areas in the financial statements that involved significant
management judgment, including accounting estimates that have been identified as having high
estimation uncertainty.
(iii) The effect on the audit of significant events or transactions that occurred during the period.
(ii) Audit Procedure & Analysis
a) The auditor shall determine which of the aforesaid matters considered were of most significance
in the audit of the financial statements of the current period and therefore are the key audit
matters.
b) These aforesaid considerations focus on the nature of matters communicated with those charged
with governance. Such matters are often linked to matters disclosed in the financial statements and
are intended to reflect areas of the audit of the financial statements that may be of particular interest
to intended users.
c) The fact that these considerations are required is not intended to imply that matters related to
them are always key audit matters; rather, matters related to such specific considerations are key
audit matters only if they are determined to be of most significance in the audit
d) In addition to matters that relate to the specific required considerations, there may be other

e)
FAST
matters communicated with those charged with governance that required significant auditor
attention and that therefore may be determined to be key audit matters.
Such matters may include, for example: - Matters relevant to the audit that was performed that

CA.Sarthak Niraj Jain


may not be required to be disclosed in the financial statements.
For example: - The implementation of a new IT system (or significant changes to an existing IT system)
during the period may be an area of significant auditor attention, in particular if such a change had a
significant effect on the auditor’s overall audit strategy or related to a significant risk (e.g., changes to a
system affecting revenue recognition).
(iii) In the given case: - There was implementation of ERP system in the company due to which some of its
business processes got automated and which had a significant effect on the auditor’s overall audit
strategy during the period. Accordingly, such a matter can be considered as a key audit matter if
according to Mr. Hemant, such a matter required significant attention that had affected his overall audit
strategy.
(iv) Conclusion: - Thus, the contention of Mr. Vishay is not proper as matters that do not link with the
matters disclosed in the financial statements can also be considered as a key audit matter if it required
significant attention of the auditor which had an impact on its audit.
100. KAM in Case of Closure of Business
TBR Limited has plan of its business being closed due to huge loss incurred due to the recent outbreak of global
pandemic, within a short period from the close of the accounting year ended on 31st March, 2022. The
Financial Statements for the said year have been prepared on the same basis as in earlier periods with an
additional note that the business of the Company shall cease in near future and the assets shall be disposed off
in accordance with a plan of disposal as decided by the Management. The Statutory Auditors of the Company
indicated this aspect in Key Audit Matters only by a reference as to a possible cessation of business and making
of adjustments, if any, to be made at the time of cessation only. Comment on the reporting by the Statutory
Auditor as above (Nov-2020-Old)

CA Final Audit - By CA. Sarthak Niraj Jain 69


Auditing Standards, Statements and Guidance Notes - An Overview

OR
Choti Limited had definite plan of its business being closed within a short period from the close of the
accounting year ended on 31st March, 2022. The Financial Statements for the year ended 31/03/2022 had been
prepared on the same basis as it had been in earlier periods with an additional note that the business of the
Company shall cease in near future and the assets shall be disposed off in accordance with a plan of disposal as
decided by the Management. The Statutory Auditors of the Company indicated this aspect in Key Audit Matters
only by a reference as to a possible cessation of business and making of adjustments, if any, thereto to be made
at the time of cessation only. Comment on the reporting by the Statutory Auditor as above. (MTP-May-2020)
OR
Toddle Limited had definite plan of its business being closed within a short period from the close of the
accounting year ended on 31st March, 2022. The Financial Statements for the year ended 31/03/2022 had
been prepared on the same basis as it had been in earlier periods with an additional note that the business of
the Company shall cease in near future and the assets shall be 'disposed, off in accordance with a plan of
disposal as decided by the Management. The Statutory Auditors of the Company indicated this aspect in Key
Audit Matters only by a reference as to a possible cessation of business and making of adjustments, if any,
thereto to be made at the time of cessation only. Comment on the reporting by the Statutory Auditor as above.
(May-2018-New)
Ans. (i) As per SA 570 “Going Concern”
 Management intentions to liquidate the entity or to cease operations is one of the event or
condition that may cast significant doubt on the entity’s ability to continue as going concern.
 As per SA 570, if events or conditions have been identified that may cast significant doubt on the
entity’s ability to continue as a going concern but, based on the audit evidence obtained the

 FAST
auditor concludes that no material uncertainty exists,
The auditor shall evaluate whether, in view of the requirements of the applicable financial
reporting framework, the financial statements provide adequate disclosures about these events

CA.Sarthak Niraj Jain



or conditions.
Even when no material uncertainty exists, it requires the auditor to evaluate whether, in view of
the requirements of the applicable financial reporting framework, the financial statements provide
adequate disclosure about events or conditions that may cast significant doubt on the entity’s
ability to continue as a going concern.
(ii) As per SA 701 “Communicating Key Audit Matters in the Independent Auditor’s Report”
 when matters relating to going concern may be determined to be key audit matters, and explains
that a material uncertainty related to events or conditions that may cast significant doubt on the
entity’s ability to continue as a going concern is, by its nature, a key audit matter.
 SA 701 also emphasises on auditor’s responsibility to communicate key audit matters in the
auditor’s report.
(iii) As per the facts given in the case
Intention of TBR Limited has a plan of its business being closed as huge loss is incurred due to the
recent outbreak of global pandemic, within short period from 31st March 2022. However, financial
statements for the year ended 31.03.2022 have been prepared on the same basis as it had been in
earlier periods with an additional note.
(iv) Conclusion:
 Thus, management intentions to liquidate the entity or to cease operations is one of the event or
condition that may cast significant doubt on the entity’s ability to continue as going concern is a
key audit matter.
 Therefore, the auditor is required to Communicate the Key Audit Matters in accordance with
SA 570 in above stated manner.
 Simple reference as to a possible cessation of business and making of adjustments, if any, be
made at the time of cessation only by the auditor in his report is not sufficient.

70 Ab Audit Hoga Sabse Scoring - By CA. Sarthak Niraj Jain


Auditing Standards, Statements and Guidance Notes - An Overview

101. Placement of the KAMs in Audit Report


Where should the placement of the key audit matters section be in the auditor’s report? (RTP-May-2022)
Ans.  Generally, the Key Audit Matters section is required to be placed after the Basis for Opinion
paragraph and before the Management’s Responsibility paragraph.
 In case, ‘Material uncertainty relating to going concern’ section is required as per SA 570 (Revised),
then KAM section is placed after that section.
 Further, regarding placement of KAM section, SA 706 (Revised), “Emphasis of Matter Paragraphs and
Other Matter Paragraphs in the Independent Auditor’s Report” provides as under: When a Key Audit
Matters section is presented in the auditor’s report, an Emphasis of Matter (EOM) paragraph may be
presented either directly before or after the Key Audit Matters section, based on the auditor’s judgment
as to the relative significance of the information included in the Emphasis of Matter paragraph.
 The auditor may also add further context to the heading “Emphasis of Matter”, such as “Emphasis of
Matter – Subsequent Event”, to differentiate the Emphasis of Matter paragraph from the individual
matters described in the Key Audit Matters section.
SA-705
Modifications to the Opinion in the Independent Auditor’s Report
102. Limitation after the Auditor has Accepted the Engagement
After accepting the statutory audit M/s All in One Ltd., a departmental store, you became aware of the fact that
management of the company have imposed certain limitations on the scope of your assurance function which
may and result in your inability to bruin sufficient appropriate audit evidence to discharge your responsibility
required by the statute. Indicate the consequences and your response to the limitations imposed by the
management on your scope. (May-2019-New)
Ans. (i)
FAST
As per SA 705, Modification to the Opinion in the Independent Auditor’s Report”
 If, after accepting the engagement, the auditor becomes aware that management has imposed a
limitation on the scope of the audit that the auditor considers likely to result in the need to

CA.Sarthak Niraj Jain


express a qualified opinion or to disclaim an opinion on the financial statements, the auditor shall
request that management remove the limitation.
 If management refuses to remove the limitation, the auditor shall communicate the matter to
TCWG, unless all of TCWG are involved in managing the entity, and determine whether it is
possible to perform alternative procedures to obtain SAAE.
(ii) Unable to obtain SAAE
If the auditor is unable to obtain sufficient appropriate audit evidence, the auditor shall determine
the implications as follows:
(a) If the auditor concludes that the possible effects on the financial statements of undetected
misstatements, if any, could be material but not pervasive, the auditor shall qualify the opinion;
or
(b) If the auditor concludes that the possible effects on the financial statements of undetected
misstatements, if any, could be both material and pervasive so that a qualification of opinion
would be inadequate to communicate the gravity of the situation, the auditor shall:
 Withdraw from the audit, where practicable and not prohibited by law or regulation; or
 If Withdrawal from the audit before issuing the auditor’s report is not practicable or possible,
disclaim an opinion on the financial statements
If the auditor withdraws as discussed above paragraph, before withdrawing, the auditor shall
communicate to TCWG any matters regarding misstatements identified during the audit that would
have given rise to a modification of the opinion
103. Limitation on the Auditor
While conducting audit of VED Ltd., you as an auditor are not only prevented in completing certain audit
procedures but also are not able to obtain audit evidence even by performing alternative procedures. How you
will deal with this situation? (Jan-2021-Old)

CA Final Audit - By CA. Sarthak Niraj Jain 71


Auditing Standards, Statements and Guidance Notes - An Overview

Ans.  As per SA 705, “Modifications to the Opinion in the Independent Auditor’s Report”, if, after accepting
the engagement, the auditor becomes aware that management has imposed a limitation on the scope of
the audit that the auditor considers likely to result in the need to express a qualified opinion or to
disclaim an opinion on the financial statements, the auditor shall request that management remove the
limitation.
 If management refuses to remove the limitation, the auditor shall communicate the matter to those
charged with governance, unless all of those charged with governance are involved in managing the
entity, and determine whether it is possible to perform alternative procedures to obtain sufficient
appropriate audit evidence. If the auditor is unable to obtain sufficient appropriate audit evidence, the
auditor shall determine the implications as follows:
(1) If the auditor concludes that the possible effects on the financial statements of undetected
misstatements, if any, could be material but not pervasive, the auditor shall qualify the opinion; or
(2) If the auditor concludes that the possible effects on the financial statements of undetected
misstatements, if any, could be both material and pervasive so that a qualification of the opinion
would be inadequate to communicate the gravity of the situation, the auditor shall:
(i) Withdraw from the audit, where practicable and possible under applicable law or regulation;
or
(ii) If withdrawal from the audit before issuing the auditor’s report is not practicable or possible,
disclaim an opinion on the financial statements.
If the auditor withdraws, before withdrawing, the auditor shall communicate to those charged with
governance any matters regarding misstatements identified during the audit that would have given rise to a
modification of the opinion.
104. Circumstances in which a Modified Opinion may be issued

FAST
ADKS & Co LLP are the newly appointed statutory auditors of PKK Ltd. During the course of audit, the
statutory auditors have come across certain significant observations which they believe could lead to material
misstatement of financial statements. Management has a different view and does not concur with the view of

CA.Sarthak Niraj Jain


the statutory auditors. Considering this the statutory auditors are determining as to how to address these
observations in terms of their reporting requirement. Please advise.
OR
(Study Material) (MTP-May-2019)

Under the applicable Standards on Auditing, in what circumstances does the report of the statutory auditor
require modifications? What are the types of modifications possible to the said report? (Study Material)
OR
Explain the circumstances which require a modification to the Auditor’s Opinion. (Nov-2017)
Ans. (i) As per SA 705-“Modifications to the Opinion in the Independent Auditor’s Report”
 If the auditor concludes that, based on the audit evidence obtained, the financial statements as a
whole are not free from material misstatement or the auditor is unable to obtain sufficient
appropriate audit evidence to conclude that the financial statements as a whole are free from
material misstatement, the auditor shall modify the opinion in his report.
(ii) The auditor in such a case needs to determine the modification as follows:
(a) Qualified Opinion: The auditor shall express a qualified opinion when:
 The auditor, having obtained SAAE, concludes that misstatements, individually or in the
aggregate, are material, but not pervasive, to the financial statements; or
 The auditor is unable to obtain SAAE on which to base the opinion, but the auditor
concludes that the possible effects on the FSs of undetected misstatements, if any, could be
material but not pervasive.
(b) Adverse Opinion:
 The auditor shall express an adverse opinion when the auditor, having obtained SAAE,
concludes that misstatements, individually or in the aggregate, are both material and
pervasive to the financial statements

72 Ab Audit Hoga Sabse Scoring - By CA. Sarthak Niraj Jain


Auditing Standards, Statements and Guidance Notes - An Overview

(c) Disclaimer of Opinion:


 The auditor shall disclaim an opinion when the auditor is unable to obtain SAAE on which
to base the opinion, and the auditor concludes that the possible effects on the FSs of
undetected misstatements, if any, could be both material and pervasive.
 The auditor shall disclaim an opinion when, in extremely rare circumstances involving
multiple uncertainties, the auditor concludes that, notwithstanding having obtained SAAE
regarding each of the individual uncertainties, it is not possible to form an opinion on
the FSs due to the potential interaction of the uncertainties and their possible cumulative
effect on the financial statements.
 If, after accepting the engagement, the auditor becomes aware that management has
imposed a limitation on the scope of the audit that the auditor considers likely to result in
the need to express a qualified opinion or to disclaim an opinion on the financial
statements, the auditor shall request that management remove the limitation.
 If management refuses to remove the limitation, the auditor shall communicate the
matter to TCWG, unless all of TCWG are involved in managing the entity, and determine
whether it is possible to perform alternative procedures to obtain SAAE.
105. Adverse Opinion –Drafting
As an auditor of XYZ Limited, in view of given circumstances, you are required to draft Adverse Opinion and
basis for adverse opinion due to a Material Misstatement of the Consolidated Financial Statements.
• Audit of a complete set of consolidated financial statements of a listed company (incorporated under the
Companies Act, 2013) using a fair presentation framework. The audit is a group audit of an entity with
subsidiaries (i.e., SA 600 applies).

FAST
The consolidated financial statements are prepared by management of the entity in accordance with the
Accounting Standards prescribed under section 133 of the Companies Act, 2013 (a general purpose
framework).


CA.Sarthak Niraj Jain
The terms of the audit engagement reflect the description of management’s responsibility for the
consolidated financial statements in SA 210.
The consolidated financial statements are materially misstated due to the non-consolidation of a
subsidiary. The material misstatement is deemed to be pervasive to the consolidated financial statements.
The effects of the misstatement on the consolidated financial statements have not been determined
because it was not practicable to do so (i.e., an adverse opinion is appropriate).
• The relevant ethical requirements that apply to the audit are the ICAI’s Code of Ethics and the provisions of
the Companies Act, 2013.
• Based on the audit evidence obtained, the auditor has concluded that a material uncertainty does not exist
related to events or conditions that may cast significant doubt on the entity’s ability to continue as a going
concern in accordance with SA 570 (Revised).
• SA 701 applies; however, the auditor has determined that there are no key audit matters other than the
matter described in the Basis for Adverse Opinion section.
• Those responsible for oversight of the consolidated financial statements differ from those responsible for
the preparation of the consolidated financial statements.
• In addition to the audit of the consolidated financial statements, the auditor has other reporting
responsibilities required under the Companies Act, 2013. (MTP-May-2018)
Ans. Adverse Opinion
We have audited the accompanying consolidated financial statements of XYZ Company Limited
(hereinafter referred to as the “Holding Company”) and its subsidiaries (the Holding Company and its
subsidiaries together referred to as “the Group”), its associates and jointly controlled entities, which
comprise the consolidated balance sheet as at March 31, 2XXX, the consolidated statement of profit and

CA Final Audit - By CA. Sarthak Niraj Jain 73


Auditing Standards, Statements and Guidance Notes - An Overview

Loss, (consolidated statement of changes in equity){where applicable} and the consolidated statement of
cash flows for the year then ended, and notes to the consolidated financial statements, including a summary of
significant accounting policies (hereinafter referred to as the “consolidated financial statements”).
In our opinion and to the best of our information and according to the explanations given to us, because
of the significance of the matter discussed in the Basis for Adverse Opinion section of our report, the
accompanying consolidated financial statements do not give a true and fair view in conformity with the
accounting principles generally accepted in India, of their consolidated state of affairs of the Group, its
associates and jointly controlled entities, as at March 31, 20XX, of its consolidated profit/loss, (consolidated
position of changes in equity) ){where applicable} and the consolidated cash flows for the year then ended.
Basis for Adverse Opinion
As explained in Note X, the Group has not consolidated subsidiary PQR Company that the Group acquired
during 20XX because it has not yet been able to determine the fair values of certain of the subsidiary’s
material assets and liabilities at the acquisition date.
This investment is therefore accounted for on a cost basis. Under the accounting principles generally
accepted in India, the Group should have consolidated this subsidiary and accounted for the acquisition based
on provisional amounts.
Had PQR Company been consolidated, many elements in the accompanying consolidated financial
statements would have been materially affected. The effects on the consolidated financial statements of the
failure to consolidate have not been determined.
We conducted our audit in accordance with Standards on Auditing (SAs) specified under section
143(10) of the Companies Act, 2013. Our responsibilities under those Standards are further described in the
Auditor’s Responsibilities for the Audit of the Consolidated Financial Statements section of our report.

FAST
We are independent of the Group, its associates and jointly controlled entities, in accordance with the
Code of Ethics and provisions of the Companies Act, 2013 that are relevant to our audit of the consolidated
financial statements in India under the Companies Act, 2013, and we have fulfilled our other ethical

CA.Sarthak Niraj Jain


responsibilities in accordance with the Code of Ethics and the requirements under the Companies act, 2013.
We believe that the audit evidence we have obtained is sufficient and appropriate to provide a basis for
our adverse opinion
106. Qualified Opinion-Drafting
As an auditor of ABC Limited, in view of given circumstances, you are required to draft qualified opinion and
basis for qualified opinion due to the departure from the applicable Financial Reporting Framework:
• Audit of a complete set of financial statements of a company other than a listed company (registered under
the Companies Act, 2013) using a fair presentation framework.
• The financial statements are prepared by management of the entity in accordance with the Accounting
Standards prescribed under section 133 of the Companies Act, 2013 (a general purpose framework).
• The terms of the audit engagement reflect the description of management’s responsibility for the financial
statements in SA 210.
• A departure from the applicable financial reporting framework resulted in a qualified opinion. • The
relevant ethical requirements that apply to the audit are the ICAI’s Code of Ethics and the provisions of the
Companies Act, 2013.
• Based on the audit evidence obtained, the auditor has concluded that a material uncertainty does not exist
related to events or conditions that may cast significant doubt on the entity’s ability to continue as a going
concern in accordance with SA 570 (Revised).
• Between the date of the financial statements and the date of the auditor’s report, there was a fire in the
entity’s production facilities, which was disclosed by the entity as a subsequent event. In the auditor’s
judgment, the matter is of such importance that it is fundamental to users’ understanding of the financial
statements. The matter did not require significant auditor attention in the audit of the financial statements
in the current period.

74 Ab Audit Hoga Sabse Scoring - By CA. Sarthak Niraj Jain


Auditing Standards, Statements and Guidance Notes - An Overview

• The auditor is not required, and has otherwise not decided, to communicate key audit matters in
accordance with SA 701.
• Those responsible for oversight of the financial statements differ from those responsible for the
preparation of the financial statements.
• In addition to the audit of the financial statements, the auditor has other reporting responsibilities required
under the Companies Act, 2013. (MTP-Nov-2018)
Ans. Qualified Opinion:
We have audited the standalone financial statements of ABC Limited (“the Company”), which comprise the
balance sheet as at March 31, 20X1, and the statement of Profit and Loss, (statement of changes in
equity) and the statement of cash flows for the year then ended, and notes to the financial statements,
including a summary of significant accounting policies and other explanatory information (in which are
included the Returns for the year ended on that date audited by the branch auditors of the Company’s branches
located at (location of branches))2.
In our opinion and to the best of our information and according to the explanations given to us, except
for the effects of the matter described in the Basis for Qualified Opinion section of our report, the
aforesaid financial statements present fairly, in all material respects, or give a true and fair view in
conformity with the accounting principles generally accepted in India of the state of affairs of the
Company as at March 31st, 2XXX and profit/loss, (changes in equity) and its cash flows for the year ended on
that date.
Basis for Qualified Opinion
The Company’s short-term marketable securities are carried in the statement of financial position at xxx.
Management has not marked these securities to market but has instead stated them at cost, which

FAST
constitutes a departure from the Accounting Standards prescribed in section 133 of the Companies Act,
2013. The Company’s records indicate that had management marked the marketable securities to market,
the Company would have recognized an unrealized loss of ` xxx in the statement of comprehensive

CA.Sarthak Niraj Jain


income for the year. The carrying amount of the securities in the statement of financial position would have
been reduced by the same amount at March 31, 20X1, and income tax, net income and shareholders’
equity would have been reduced by ` xxx, ` xxx and ` xxx, respectively.
We conducted our audit in accordance with Standards on Auditing (SAs).Our responsibilities under those
standards are further described in the Auditor’s Responsibilities for the Audit of the Financial Statements
section of our report. We are independent of the Company in accordance with the ethical requirements that
are relevant to our audit of the financial statements under the provisions of the Companies Act, 2013, and
we have fulfilled our other ethical responsibilities in accordance with these requirements and the ICAI’s Code
of Ethics.
We believe that the audit evidence we have obtained is sufficient and appropriate to provide a basis for
our qualified opinion.
107. Contingent Liability & Reporting
MN & Associates, Chartered Accountants have been appointed as statutory Auditors of Cotton Ltd. for the F.Y.
2021-2022. The Company is into the business of yarn manufacturing. For this purpose, cotton ginning is also
done within the factory premises. Raw cotton is purchased from local market and processed in-house. The
Company received a notice from the State Government to deposit market development fee for the last 5 years
to the tune of ` 10.00 crores. The Company and all other organizations in the same business has not deposited
the market development fee, taking shelter of an old circular issued by the Government. The trade association
met with the government officials to resolve the matter and agreed to deposit the same prospectively.
However, the matter relating to payment of development fee for the last 5 years is pending before the
Government as at the end of the financial year. The Company, however, disclosed the same in notes to accounts,
as contingent liability, without quantifying the effect and proper explanation. If the liability is provided in the
books of accounts, entire reserves will be wiped off. Auditor seeks your guidance as to how this disclosure
affects them while forming an opinion on financial statements (July-2021-New)
CA Final Audit - By CA. Sarthak Niraj Jain 75
Auditing Standards, Statements and Guidance Notes - An Overview

Ans. (i) As per Ind AS 37: - “Provisions, Contingent Liabilities and Contingent Assets”, an entity should
disclose for each class of contingent liability at the end of the reporting period a brief description of the
nature of the contingent liability and, where practicable .
(a) an estimate of its financial effect, measured in the standard;
(b) an indication of the uncertainties relating to the amount or timing of any outflow; and
(c) the possibility of any reimbursement.
(ii) As per SA – 700 - Forming an opinion and reporting on financial statements:
The auditor shall evaluate whether in view of the requirements of the applicable financial reporting
framework –
(i) The financial statements adequately disclose the significant accounting policies selected and
applied;
(ii) The accounting policies selected and applied are consistent with the applicable financial reporting
framework and are appropriate;
(iii) The accounting estimates made by the management are reasonable;
(iv) The information presented in the financial statements is relevant, reliable, comparable and
understandable;
(v) The financial statements provide adequate disclosures to enable the intended users to understand
the effect of material transactions and events on the information conveyed in the financial
statements.
If financial statements prepared in accordance with the requirements of a fair presentation framework
do not achieve fair presentation, the auditor shall discuss the matter with management and, depending
on the requirements of the applicable financial reporting framework and how the matter is resolved,
shall determine whether it is necessary to modify the opinion in the auditor’s report in accordance with
SA 705.
(iii) Conclusion: -In the present case, auditor may consider modifying his opinion considering the financial

FAST
effect of liability not disclosed properly.
SA-710

108.
CA.Sarthak Niraj Jain
Comparative Information – Corresponding Figures and
Comparative Financial Statements
Auditor’s Responsibilities w.r.t. Corresponding Figures:
For the year ended 31st March, 2021, the audit report of Avinash Ltd., contained a qualification regarding non-
provision for diminution in the value of investments to the extent of ` 50 lacs. As an Auditor of the Company for
the year 2021-22, how would you report, if:
(i) The Company does not make provision for diminution in the value of investments in the year 2021-2022.
(ii) The Company makes adequate provision for diminution in the year 2020-2021. (May-2018-Old)
OR
The audit report of P Ltd. for the year 2020-21 contained a qualification regarding non-provision of doubtful
debts. As the statutory auditor of the company for the year 2021-22, how would you report, if:
(i) The company does not make provision for doubtful debts in 2021-22?
(ii) The company makes adequate provision for doubtful debts in 2021-22?
Ans. (i) Auditor’s Responsibilities when modification resolved
 In cases where audit report for an earlier year is qualified is given in SA 710 “Comparative
Information – Corresponding Figures and Comparative Financial Statements”.
 As per SA 710, When the auditor’s report on the prior period, as previously issued, included a
qualified opinion, a disclaimer of opinion, or an adverse opinion and the matter which gave rise
to the modified opinion is resolved and properly accounted for or disclosed in the financial
statements in accordance with the applicable financial reporting framework, the auditor’s
opinion on the current period need not refer to the previous modification.
(ii) When modification un-resolved
 SA 710 further states that if the auditor’s report on the prior period, as previously issued, included a
qualified opinion and the matter which gave rise to the modification is unresolved, the auditor
shall modify the auditor’s opinion on the current period’s financial statements.

76 Ab Audit Hoga Sabse Scoring - By CA. Sarthak Niraj Jain


Auditing Standards, Statements and Guidance Notes - An Overview

(iii) In the Basis for Modification paragraph in the auditor’s report, the auditor shall either:
 Refer to both the current period’s figures and the corresponding figures in the description of
the matter giving rise to the modification when the effects or possible effects of the matter on the
current period’s figures are material; or
 In other cases, explain that the audit opinion has been modified because of the effects or
possible effects of the unresolved matter on the comparability of the current period’s figures and
the corresponding figures.
(iv) Conclusion/Opinion: -In the instant Case, if P Ltd. does not make provision for doubtful debts the
auditor will have to modify his report for both current and previous year’s figures as mentioned above.
If however, the provision is made, the auditor need not refer to the earlier year’s modification.
109. Impact of Modification in the Predecessor Auditor’s Report
It was observed from the modified audit report of the financial statements of ULFA Ltd. for the year ended 31st
March, 2021 that depreciation of ` 4.25 crore for the year 2020-2021 had been charged off to the Statement of
Profit and Loss instead of including it in "carrying value of asset under construction". State in relation to the
audit for the year ended 31st March 2022, whether such modification in the previous year's audit report would
have any audit implication for the current year i.e. FY 2021-22 and if yes, how the auditor is required to deal
with the same in his audit report for the current year? (MTP-Nov-2020)(Nov-2018-Old)
Ans. (i) Auditor’s Responsibilities when modification resolved
 In cases where audit report for an earlier year is qualified is given in SA 710 “Comparative
Information – Corresponding Figures and Comparative Financial Statements”.
 As per SA 710, When the auditor’s report on the prior period, as previously issued, included a
qualified opinion, a disclaimer of opinion, or an adverse opinion and the matter which gave rise
to the modified opinion is resolved and properly accounted for or disclosed in the financial
statements in accordance with the applicable financial reporting framework, the auditor’s
opinion on the current period need not refer to the previous modification.
(ii) When modification un-resolved

FAST
 SA 710 further states that if the auditor’s report on the prior period, as previously issued, included
a qualified opinion and the matter which gave rise to the modification is unresolved, the auditor
shall modify the auditor’s opinion on the current period’s financial statements.
(iii) In the Basis for Modification paragraph in the auditor’s report, the auditor shall either:

CA.Sarthak Niraj Jain


 Refer to both the current period’s figures and the corresponding figures in the description of
the matter giving rise to the modification when the effects or possible effects of the matter on the
current period’s figures are material; or
 In other cases, explain that the audit opinion has been modified because of the effects or
possible effects of the unresolved matter on the comparability of the current period’s figures and
the corresponding figures.
(iv) Conclusion:
 In the instant case, if Ulfa Ltd. does not correct the treatment of depreciation extent of rupees
4.25 crore for previous year, the auditor will have to modify his report for both current and
previous year’s figures as mentioned above.
 If however, the figures and provisions are corrected, the auditor need not refer to the earlier
year’s modification.
SA-720
The Auditor’s Responsibility in Relation to Other Information
110. Examples of Amounts or Other Items
LMP Associates, Chartered Accountants, conducting the audit of PQR Ltd., a listed Company for the year ended
31st March 2022 is concerned with the auditor's responsibilities relating to other information, both financial
and non-financial, included in the Company’s annual report. While reading other information, LMP Associates
considers whether there is a material inconsistency between other information and the financial statements. As
a basis for the consideration the auditor shall evaluate their consistency, compare selected amounts or other
items in the other information with such amounts or other items in the financial statements. Guide LMP
Associates with examples of "Amounts" or "other items" that may be included in the "other information" with
reference to SA 720. (MTP-May-2021)(Study Material)(Nov-2019-New)(May-2019-Old)
Ans. (i) As per SA 720 “The Auditor’s Responsibility in Relation to Other Information”
The following are examples of amounts and other items that may be included in other information.
This list is not intended to be exhaustive.

CA Final Audit - By CA. Sarthak Niraj Jain 77


Auditing Standards, Statements and Guidance Notes - An Overview

(ii) Examples of Amounts


 Items in a summary of key financial results, such as net income, earnings per share, dividends,
sales and other operating revenues, and purchases and operating expenses.
 Selected operating data, such as income from continuing operations by major operating area, or
sales by geographical segment or product line.
 Special items, such as asset dispositions, litigation provisions, asset impairments, tax
adjustments, environmental remediation provisions, and restructuring and reorganization expenses.
 Liquidity and capital resource information, such as cash, cash equivalents and marketable
securities; dividends; and debt, capital lease and minority interest obligations.
 Capital expenditures by segment or division etc.
(iii) Examples of Other Items
 Explanations of critical accounting estimates and related assumptions.
 Identification of related parties and descriptions of transactions with them.
 Articulation of the entity’s policies or approach to manage commodity, foreign exchange or
interest rate risks, such as through the use of forward contracts, interest rate swaps, or other
financial instruments.
 Descriptions of the nature of off-balance sheet arrangements.
 Descriptions of guarantees, indemnifications, contractual obligations, litigation or
environmental liability cases, and other contingencies, including management’s qualitative
assessments of the entity’s related exposures etc.
 Explanations of specific factors influencing the entity’s profitability in specific segments
111. Responding when the Auditor concludes that a Material Misstatement of the Other Information exists
ING Associates, Chartered Accountants, conducting the audit of XYZ Ltd., a listed Company for the year ended
31st March 2022 is concerned with the auditor's responsibilities relating to misstatements in other
information, both financial and non-financial, included in the Company’s annual report. While reading other

FAST
information, ING Associates considers whether there is any material misstatement of the other information in
the Company. After performing their procedures, the auditor concludes that a material misstatement of the
other information exists. ING Associates discussed with the Management about the other information that

CA.Sarthak Niraj Jain


appeared to be materially misstated to the auditor and also requested management to provide evidence for the
basis of management’s statements in the other information along with supporting documents. Guide ING
Associates as to how to respond to that material misstatement of other information obtained prior to the date
of auditor’s report. Will your answer be different in case ING Associates conclude the same after the date of
auditor’s report? (MTP-Nov-2020)
Ans. (i) Provision: -As per SA 720, “The Auditor’s Responsibility in Relation to Other Information”,
if the auditor concludes that a material misstatement of the other information exists, the auditor shall
request management to correct the other information. If management:
 Agrees to make the correction, the auditor shall determine that the correction has been made; or
 Refuses to make the correction, the auditor shall communicate the matter with TCWG and
request that the correction be made.
(ii) If the auditor concludes that a material misstatement exists in other information obtained prior to
the date of the auditor’s report, and the other information is not corrected after communicating with
TCWG, the auditor shall take appropriate action, including:
 Considering the implications for the auditor’s report and communicating with TCWG about how
the auditor plans to address the material misstatement in the auditor’s report;
 Withdrawing from the engagement, where withdrawal is possible under applicable law or
regulation.
(iii) If the auditor concludes that a material misstatement exists in other information obtained after the
date of the auditor’s report, the auditor shall:
 If the other information is corrected, perform the procedures necessary in the circumstances; or
 If the other information is not corrected after communicating with those charged with
governance, take appropriate action considering the auditor’s legal rights and obligations, to seek
to have the uncorrected material misstatement appropriately brought to the attention of users for
whom the auditor’s report is prepared.

78 Ab Audit Hoga Sabse Scoring - By CA. Sarthak Niraj Jain


Auditing Standards, Statements and Guidance Notes - An Overview

112. Responding when the Auditor concludes that a Material Misstatement of the Other Information exists
GS & Co., Chartered Accountants, have been appointed Statutory Auditors of MAP Ltd. for the F.Y 2021-22. The
audit team has completed the audit and is in the process of preparing audit report Management of the company
has also prepared draft annual report. Audit in-charge was going through the draft annual report and observed
that the company has included an item in its Annual Report indicating downward trend in market prices of key
commodities/raw material as compared to previous year. However, the actual profit margin of the company as
reported in financial statements has gone in the reverse direction. Audit Manager discussed this issue with
partner of the firm who in reply said that auditors are not covered with such disclosures made by the
management in its annual report, it being the responsibility of the management.
Do you think that the partner is correct in his approach on this issue. Discuss with reference to relevant
Standard on Auditing the Auditor's duties with regard to reporting. (Nov-2020-New)(MTP-Dec-2021)
Ans. (i) As per SA 720, “The Auditor’s Responsibility in Relation to Other Information”, Descriptions of
trends in market prices of key commodities or raw materials is an example of amounts or other Items
that may be Included in the other information.
 The auditor’s discussion with management about a material inconsistency (or other information
that appears to be materially misstated) may include requesting management to provide support
for the basis of management’s statements in the other information.
 Based on management’s further information or explanations, the auditor may be satisfied that
the other information is not materially misstated.
 For example, management explanations may indicate reasonable and sufficient grounds for
valid differences of judgment.
(ii) Auditor’s duties with regard to reporting in the given case are given hereunder:
As per SA 720, “The Auditor’s Responsibility in Relation to Other Information”, if the auditor concludes

FAST
that a material misstatement of the other information exists, the auditor shall request management
to correct the other information. If management:
 Agrees to make the correction, the auditor shall determine that the correction has been made; or
 Refuses to make the correction, the auditor shall communicate the matter with TCWG and

CA.Sarthak Niraj Jain


request that the correction be made. Contention of the partner of the firm that auditors are not
concerned with such disclosures made by the management in its annual report, is incorrect.

Notes

CA Final Audit - By CA. Sarthak Niraj Jain 79


Audit Planning, Strategy & Execution

Chapter
2
AUDIT PLANNING, STRATEGY & EXECUTION

Past Exams Weightage


5
4
3
2
1
0 Marks

May-19 Nov-19 Nov-20 Jan-21 Jul-21 Dec-21


Marks 0 5 0 0 0 0

2.1 - Audit Planning -Content & Objective


1. Audit Plan - Development
A & Co. was appointed as auditor of Great Airways Ltd. As the audit partner what factors shall be considered

Ans.
FAST
in the development of overall audit plan?
Factors to be considered in development of overall audit plan
(Study Material)

CA.Sarthak Niraj Jain


Overall plan is basically intended to provide direction for audit work programming and includes the
determination of timing, manpower development and co-ordination of work with the client, other
auditors and other experts. The auditor should consider the following matters in developing his overall plan
for the expected scope and conduct of the audit:
1. Terms of his engagement and any statutory responsibilities.
2. Nature and timing of reports or other communications.
3. Applicable Legal or Statutory requirements.
4. Accounting policies adopted by the clients and changes, if any, in those policies.
5. The effects of new accounting and auditing pronouncement on the audit.
6. Identification of significant audit areas.
7. Setting of materiality levels for the audit purpose.
8. Conditions requiring special attention such as the possibility of material error or fraud or
involvement of parties in whom directors or persons who are substantial owners of the entity are
interested and with whom transactions are likely.
9. Degree of reliance to be placed on the accounting system and internal control.
10. Possible rotation of emphasis on specific audit areas.
11. Nature and extent of audit evidence to be obtained.
12. Work of the internal auditors and the extent of reliance on their work, if any in the audit.
13. Involvement of other auditors in the audit of subsidiaries or branches of the client and involvement of
experts.
14. Allocation of works to be undertaken between joint auditors and the procedures for its control and
review.
15. Establishing and coordinating staffing requirements.
SJ Note: Any eight points may be answered in Exam

80 Ab Audit Hoga Sabse Scoring - By CA. Sarthak Niraj Jain


Audit Planning, Strategy & Execution

2. Audit Plan - Abnormal Wastage of Raw Material


XY Ltd. is a manufacturing company, provided following details of wastages of raw materials in percentage,
for various months. You have been asked to enquire into causes of abnormal wastage of raw materials. Draw
out an audit plan.
Wastage percentage are:
July 2019 - 1.5% Aug 2019 - 1.7%
Sep 2019 - 1.4% Oct 2019 - 4.1% (MTP–May-2018)
Ans. Audit Plan to inquire causes of abnormal wastage of raw material:
Before planning for detailed investigation, the auditor is required to understand the manufacturing
operations from the initial stage of purchase of materials, issue of material, consumption of material and
conversions into finished goods.
To locate the reasons for the abnormal wastage, the audit plan may include the following:
(i) Obtain a list of raw materials used in the production process mentioning therein the names and
detailed characteristics of each raw material.
(ii) Ascertain the basis of computation of normal wastage figures and its consistency as compared to
previous months.
(iii) Obtain internal control reports in respect of manufacturing costs incurred with reference to
predetermined standards or budgets.
(iv) Examine the stock records so as to determine the raw material purchased, material issued to
production.
(v) Examine the production records so as to determine the material received from the stores and actual
material consumed in the production and waste produced in the production process.

FAST
(vi) Study the Maintenance Programme of machinery to ensure that the machinery does not consume
more raw material and quality of goods manufacture is not of sub-standard nature.
(vii) Employees training ascertain whether employees engaged in production are properly trained and

CA.Sarthak Niraj Jain


working efficiently.
(viii) Examine inventory controls in respect of storage, pilferage, issues etc.
(ix) Obtain a statement showing break up of wastage figures in storage, handling and production for the
period under reference and compare the results of the analysis for each of the month.
(x) Consider the existence of following situations that may also cause the abnormal wastage:
 Purchase of poor quality of raw materials.
 Machinery breakdown or power failure.
 High rate of rejections of finished goods
 Deterioration of raw material lying in godowns
 Abnormal wastages in storage and handling.
 Commencing new production line.
2.2 - Audit Strategy-Formulation, Factors & Benefits
3. Audit Strategy- Formulation & Specific factors for Online Shopping Venture
As an auditor of garment manufacturing company for the last five years, you have observed that new venture
of online shopping has been added by the company during current year. What factors would be considered
by you in formulating the audit strategy of the company? (Study Material)
Ans. Formulation of Audit Strategy & Specific factors for Online Shopping Venture
(i) Factors to be considered in establishing audit strategy SA 300 “Planning an Audit of Financial
Statements”:-Requires that the auditor shall establish an overall audit strategy that sets the scope,
timing and direction of the audit, and that guides the development of the audit plan.
In establishing the overall audit strategy, the auditor shall:
a) Identify the characteristics of the engagement that define its scope;

CA Final Audit - By CA. Sarthak Niraj Jain 81


Audit Planning, Strategy & Execution

b) Ascertain the reporting objectives of the engagement to plan the timing of the audit and the
nature of the communications required;
c) Consider the factors that are significant in directing the engagement team’s efforts;
d) Consider the results of preliminary engagement activities and, where applicable, whether
knowledge gained on other engagements performed by the engagement partner for the entity
is relevant; and
e) Ascertain the NTE of procedures necessary to perform.
(ii) Specific Factors for Online Shopping: The auditor shall also obtain an understanding of the
information system including the related business processes due to new venture of online shopping in
the following areas:
a) Nature of transactions significant to the financial statements;
b) Procedures, within both information technology and manual systems, by which transactions
are initiated, recorded, processed, corrected (if necessary), transferred to the general ledger
and reported in the financial statements;
c) System by which the information system captures events and conditions, other than
transactions, that are significant to the financial statements;
d) Controls used to record non-recurring, unusual transactions or adjustments.
4. Audit Strategy - Team Resources
The process of establishing the overall audit strategy assists the auditor to determine certain matters with
respect of team resource. Explain those matters.
Ans. Matters determined by Audit Strategy with respect to team resources
(i) Employment of Qualitative Resources:
• Audit strategy helps in deploying the appropriate resources for specific audit areas,

FAST
Such as the use of experienced team members for high risk areas or the involvement of
experts on complex matters like review of valuation, hedging transactions. .

CA.Sarthak Niraj Jain


(ii) Allocation of Quantity of Resources:


Audit strategy helps in allocating the appropriate number of resources to specific audit areas
Such as the number of team members assigned to observe the inventory count at material
locations.
(iii) Timing of Deployment of Resources:
• Audit strategy helps in determining the timing of deploying the resources
• Such as whether at an interim audit stage or at key cut-off dates.
(iv) Management of Resources:
• Audit strategy helps in managing, directing, supervising the resources
• Such as when team briefing and debriefing meetings are expected to be held,
• How engagement partner and manager reviews are expected to take place (for example, on-
site or off-site), and
• Whether to complete engagement quality control reviews.
2.3 - Audit Programme –Drafting
5. Audit Programme -Adequacy of Records
CA. Dev, a recently qualified practicing Chartered Accountant got his first internal audit assignment of a large
manufacturing concern Growth Limited. As an internal auditor for Growth Limited, CA. Dev is required to
verify whether there are adequate records for identification and value of Plant and Machinery, tools and dies
and whether any of these items have become obsolescent and not in use. Draft a suitable audit programme
for the above. (RTP-May-2021)
OR
As an internal auditor for a large manufacturing concern, you are asked to verify whether there are adequate
records for identification and value of Plant and Machinery, tools and dies and whether any of these items
have become obsolescent and not in use. Draft a suitable audit programme for the above. (MTP-May-2018)

82 Ab Audit Hoga Sabse Scoring - By CA. Sarthak Niraj Jain


Audit Planning, Strategy & Execution

Ans. Audit Programme to determine adequacy of records w.r.t. Plant and Machinery:
The Internal Audit Programme in connection with Plant and Machinery and Tools and Dies may be on
the following lines:
(i) Internal control aspects: The following may be incorporated in the audit programme to check the
internal control aspects-
 Maintaining separate register for hired assets, leased asset and jointly owned assets.
 Maintaining register of fixed asset and reconciling to physical inspection of fixed asset and to
nominal ledger.
 All movements of assets are accurately recorded.
 Authorisation be obtained for –
 a declaring a fixed asset scrapped.
 selling a fixed asset.
 Check whether additions to fixed asset register are verified and checked by authorised person.
 Proper recording of all additions and disposal.
 Examining procedure for the purchase of new fixed assets, including written authority, work
order, voucher and other relevant evidence.
 Regular review of adequate security arrangements.
 Periodic inspection of assets is done or not.
(ii) Assets register: To review the registers and records of plant, machinery, etc. showing clearly the
date of purchase of assets, cost price, location, depreciation charged, etc.
(iii) Cost report and journal register: To review the cost relating to each plant and machinery and to
verify items which have been capitalised.
(iv) Code register: To see that each item of plant and machinery has been given a distinct code

FAST
number to facilitate identification and verify the maintenance of Code Register.
(v) Physical verification: To see physical verification has been conducted at frequent intervals.
(vi) Movement register: To verify

CA.Sarthak Niraj Jain


 whether Movement Register for movable equipments and
 log books in case of vehicles, etc. are being maintained properly.
(vii) Assets disposal register: To review whether assets have been disposed off after proper technical
and financial advice and sales/disposal/retirement, etc. of these assets are governed by
authorisation, sales memos or other appropriate documents.
(viii) Spare parts register: To examine the maintenance of a separate register of tools, spare parts for
each plant and machinery.
(ix) Review of maintenance: To scrutinise the programme for an actual periodical servicing and
overhauling of machines and to examine the extent of utilisation of maintenance department
services.
(x) Review of obsolescence: To scrutinise whether expert’s opinion have been obtained from time to
time to ensure purchase of technically most useful efficient and advanced machinery after a thorough
study.
(xi) Review of R&D: To review R&D activity and ascertain the extent of its relevance to the operations of
the organisation, maintenance of machinery efficiency and prevention of early obsolescence.
6. Audit Programme - Multiplex Cinema House
Entertainment Paradise, a movie theatre complex, is the foremost theatre located in Chennai. Along with the
sale of tickets over the counter and online booking, the major proportion of income is from the cafe, shops,
pubs etc. located in the complex. Its other income includes advertisements exhibited within/outside the
premises such as hoardings, banners, slides, short films etc. The facility for parking of vehicles is also
provided in the basement of the premises. Entertainment Paradise appointed your firm as the auditor of the
entity. Being the head of the audit team, you are, therefore, required to draw an audit programme initially in
respect of its revenue and expenditure considering the above mentioned facts along with other relevant
points relating to a complex. (MTP-May-2021)

CA Final Audit - By CA. Sarthak Niraj Jain 83


Audit Planning, Strategy & Execution

OR
Cineplex, a movie theatre complex, is the foremost theatre located in Delhi. Along with the sale of tickets over
the counter and online booking, the major proportion of income is from the cafe, shops, pubs etc. located in
the complex. It’s other income includes advertisements exhibited within/outside the premises such as
hoardings, banners, slides, short films etc. The facility for parking of vehicles is also provided in the basement
of the premises. Cineplex appointed your firm as the auditor of the entity. Being the head of the audit team,
you are therefore required to draw an audit programme initially in respect of its revenue and expenditure
considering the above-mentioned facts along with other relevant points relating to a complex.
(Study Material)(Nov-2019-New)
OR
You have been appointed as the auditor of a Multiplex Cinema House. Draw an audit programme in respect of
its Revenue and Expenditure. (RTP-May-2018)
Ans. Audit Programme for Revenue and Expenditure of Multiplex:
(i) Study the MOA and AOA:- of the entity so as to ensure that the object clause of MOA permits the
entity to engage in this business.
(ii) In respect of income from sale of tickets, perform the following:
 Examine the internal control system to ensure proper accounting of sale of tickets.
 Examine the system of on line booking and realization of money.
 Examine the system of reconciliation of collections with the number of seats available.
(iii) Examine the existence and effectiveness of internal control system relating to the income from
cafes shops, pubs etc., located within the multiplex.
(iv) Examine the control exercised relating to the income receivable from advertisements exhibited

FAST
within the premises.
(v) Examine the system of collection from the parking areas.
(vi) Examine the system of payment to the distributors which may be either through out right payment

CA.Sarthak Niraj Jain


or percentage of collection.
(vii) Examine the system of payment of salaries and other benefits to the employees and ensure
compliance of statutory requirements.
(viii) Examine the expenses incurred in respect of the maintenance of the building and ensure the same is
in order.
(ix) Verify the insurance premium paid and ensure it covers the entire assets.
2.4 - Audit Execution -Key phases
7. Key Phases in the Audit Execution
Key phases in the audit execution stage are Execution Planning, Risk and Control Evaluation, Testing and
Reporting. Explain. (RTP-Nov-2018)
OR
Briefly explain the key phases in the audit execution.
Ans. Key phases in the audit execution stage are Execution Planning, Risk and Control Evaluation, Testing
and Reporting.
Phase-I: Execution Planning
 In order to carry out the audit in an effective, efficient & timely manner
 Auditors need to plan the work and a detailed audit program should be prepared
 Covering the audit objectives, scope and audit approach.
 During execution planning, auditor should consider the manpower requirement, qualification of
members of ET, time factor etc.

84 Ab Audit Hoga Sabse Scoring - By CA. Sarthak Niraj Jain


Audit Planning, Strategy & Execution

Phase-II: Risk and Control Evaluation


 Auditor need to conduct a detailed assessment of risk and control as per requirements of SA 315.
Steps involved in assessment of risk are:
 List the risk that need to be reviewed for each segment of audit.
 Capture for each risk the controls that exist or those that are needed
 Determine the steps required to test the effectiveness of each controls.

Phase-III: Testing
 As required by SA 330- auditor should test the operating effectiveness of the controls to determine
whether controls are operating as designed.
 Auditor should perform appropriate substantive procedures (Tests of Details and SAP) so as to collect
sufficient appropriate audit evidences w.r.t. completeness, accuracy and validity of accounting data.

Phase-IV: Reporting
 The auditor should review and assess the conclusions drawn from the audit evidence obtained as the
basis for the expression of an opinion on the F.S.
 The opinion so farmed should be expressed in the form of audit report as required by SA 700.
 Auditor’s report should contain a clear written expression of opinion on the financial statements taken
as a whole.

Notes
FAST
CA.Sarthak Niraj Jain

CA Final Audit - By CA. Sarthak Niraj Jain 85


Risk Assessment & Internal Control

Chapter
3
RISK ASSESSMENT & INTERNAL CONTROL

Past Exams Weightage


5

0
MAY-19 NOV-19 NOV-20 JAN-21 JUL-21 DEC-21

May-19 Nov-19 Nov-20 Jan-21 Jul-21 Dec-21


Marks 5 5 0 5 5 5

1.
FAST
Audit Risk & RoMM
3.1 - Risk Assessment

Compute the overall Audit Risk if looking to the nature of business there are chances that 40% bills of

CA.Sarthak Niraj Jain


services provided would be defalcated, inquiring on the same matter management has assured that internal
control can prevent such defalcation to 75%. At his part the Auditor assesses that the procedure he could
apply in the remaining time to complete Audit gives him satisfaction level of detection of frauds & error to an
extent of 60%. Analyse the Risk of Material Misstatement and find out the overall Audit Risk.
(MTP-May-2019) (MTP-Nov-2020)(Study Material)
Ans. Determination of Audit Risk
(i) As per SA-200:-“Overall Objectives of the Independent Auditor and the Conduct of an Audit in
Accordance with Standards on Auditing”, the Audit Risk is a risk that Auditor will issue an
inappropriate opinion while Financial Statements are materially misstated.
(ii) Audit Risk is a function of two components:
Risk of material Misstatement and Detection Risk, i.e. Audit Risk = Risk of material Misstatement X
Detection Risk
(iii) Risk of Material Misstatement:-
It is anticipated risk that a material Misstatement may exist in Financial Statement before start of the
Audit. It has two components Inherent risk and Control risk. The relationship can be defined as:
Risk of material Misstatement = Inherent risk X control risk
 Inherent Risk:- The susceptibility of an assertion about account balance; class of transaction,
disclosure towards misstatements which may be either individually or collectively with other
Misstatement becomes material before considering any related internal control.
 In the given case:- Inherent Risk is 40%.
 Control Risk:- It is the risk that material misstatement will not be prevented or detected and
corrected on a timely basis by the internal control system.
 In the given case :- Control risk is 25% (100%-75%).
Risk of material Misstatement = 40% X 25 % = 10%

86 Ab Audit Hoga Sabse Scoring - By CA. Sarthak Niraj Jain


Risk Assessment & Internal Control

(iv) Detection Risk:- It is the risk that the substantive procedures performed by the auditor fails to detect
material misstatement.
• In the given case :- Detection Risk is 100-60 = 40%
• Hence overall Audit Risk = Risk of Material Misstatement X Detection Risk = 10X40% = 4%
2. Assessment of Risk and Acceptable Level
While commencing the statutory audit of B Company Limited, the auditor undertook the risk assessment and
found that the detection risk relating to certain class of transactions cannot be reduced to acceptance level.
Explain. (May-2017)
Ans.  SA 315 “Identifying and Assessing the Risk of Material Misstatement Through Understanding the
Entity and its Environment” and SA 330 “The Auditor’s Responses to Assessed Risks”
 Establishes standards on the procedures to be followed to obtain an understanding of the accounting
and internal control systems and on audit risk and its components.
 SA 315 and SA 330
 Require that the auditor should use professional judgement to assess risk of material misstatement
and to design audit procedures to ensure that it is reduced to an acceptably low level.
 Risk of Material Misstatements comprises of Inherent risk and Control Risk.
 “Detection risk” is the risk that an auditor’s substantive procedures will not detect a misstatement
that exists in an account balance or class of transactions that could be material
 The higher the risk of material misstatement, the more audit evidence the auditor should obtain from
the performance of substantive procedures.
 When both inherent and control risks are assessed as high, the auditor needs to consider whether
substantive procedures can provide sufficient appropriate audit evidence to reduce detection risk, and

 FAST
therefore audit risk, to an acceptably low level.
The auditor should use his professional judgement to assess audit risk and to design audit
procedures to ensure that it is reduced to an acceptably low level. If it cannot be reduced to an

3.
CA.Sarthak Niraj Jain
acceptable level, the auditor should express a qualified opinion or a disclaimer of opinion as may be
appropriate.
Audit Procedures to Address the Validity of Account Balance Level
During the process of extracting the exception reports, the auditors noted numerous purchase entries
without valid purchase orders. In terms of percentage, about 40% of purchases were made without valid
purchase orders whereas few purchase orders were validated after the actual purchase. Also, there was no
reconciliation between the goods received and the goods ordered. You are required to briefly explain the
audit procedures to address the validity of account balance level. (MTP-May-2021)
Ans. (i) In the given scenario:
 The auditors noted numerous purchase entries without valid purchase orders during the
process of extracting the exception reports.
 Further, in terms of percentage, about 40% of purchases were made without valid purchase
orders and also few purchase orders were validated after the actual purchase.
 Also there was no reconciliation between the goods received and the goods ordered.
(ii) Audit Procedures: The following procedures may address the validity of the account balance:
 Make a selection of the purchases, review correspondence with the vendors, purchase
requisitions (internal document) and reconciliations of their accounts.
 Review Vendor listing along with the ageing details. Follow up the material amounts paid before
the normal credit period and analyse the reasons for exceptions.
 Meet with the company's Purchase officer and obtain responses to our inquiries regarding the
purchases made without purchase orders.
 Discuss the summary of such issues with the client.

CA Final Audit - By CA. Sarthak Niraj Jain 87


Risk Assessment & Internal Control

3.2 - Internal Control System


4. Control Objectives
As auditor of ZED Ltd., you would like to limit your examination of account balance tests. What are the
control objectives you would like the accounting control system to achieve to suit your purpose?
(MTP-May-2021) (Study Material)
Ans. The basic accounting control objectives which are sought to be achieved by any accounting
control system are:
1. Whether all transactions are recorded;
2. whether recorded transactions are real;
3. Whether all recorded transactions are properly valued;
4. Whether all transactions are recorded timely;
5. Whether all transactions are properly posted;
6. Whether all transactions are properly classified and disclosed;
7. Whether all transactions are properly summarized.
5. Key Components - Internal Control Structure
XYZ Hospital Private Ltd. is engaged in running a hospital of 200 Beds since last 20 years. Revenue Track of
the hospital for last 3 years is as under:
2019-20 : ` 20 Crores
2020-21 : ` 25 Crores
2021-22 : ` 35 Crores
Hospital has its own Pharmacy, Laboratory, Blood Bank, Radiology & General Stores. Its management
suspects that leakages/theft is happening in Pharmacy, Radiology, Laboratory and General Stores
departments. It seeks advice of RST & Co., Internal Auditors of the Company, as to how it can
FAST
Institute/Improve its Internal Control. In this context, Management wants to understand the concept of
components of Internal Control Structure in detail. Advise. (MTP-Nov-2018)
Ans. 
CA.Sarthak Niraj Jain
The Internal Control structure in an organization is referred to as the policies and procedures
established by the entity to provide reasonable assurance that the objectives are achieved. The
control structure in an organization basically has the following components:
1. Control Environment - Control environment covers the effect of various factors like management
attitude; awareness and actions for establishing, enhancing or mitigating the effectiveness of
specific policies and procedures.
2. Accounting System - Accounting system means the series of task and records of an entity by which
transactions are processed for maintaining financial records. Such system identifies, assemble,
analyze, calculate, classify, record, summarize and report transactions and other events.
3. Control Procedure - Policies and procedures means those policies and procedures in addition to
the control environment and accounting systems which the management has established to achieve
the entity’s specific objectives.
 In this regard, the management is responsible for maintaining an adequate accounting system
incorporating various internal controls to the extent that they are appropriate to the size and nature of
the business.
 There should be reasonable assurance for the auditor that the accounting system is adequate and that
all the accounting information required to be recorded has in fact been recorded. Internal controls
normally contribute to such assurance.
 The auditor should gain an understanding of the accounting system and related internal controls and
should study and evaluate the operation of those internal controls upon which he wishes to rely in
determining the nature, timing and extent of other audit procedures.
 Where the auditor concludes that he can rely on certain internal controls, he could reduce his
substantive procedures which otherwise may be required and may also differ as to the nature and
timing.

88 Ab Audit Hoga Sabse Scoring - By CA. Sarthak Niraj Jain


Risk Assessment & Internal Control

 Specific Requirement under SA 315 - “Identifying and Assessing the Risks of Material Misstatement
through Understanding the Entity and its Environment” deals with the auditor’s responsibility to
identify and assess the risks of material misstatement in the financial statements, through
understanding the entity and its environment, including the entity’s internal control.
3.3 - Components of Internal Control
6. Integrity and Ethical Values
The effectiveness of controls cannot rise above the integrity and ethical values of the people who create,
administer, and monitor them. Explain. (Study Material)
Ans.  The effectiveness of controls cannot rise above the integrity and ethical values of the people who
create, administer, and monitor them.
 Integrity and ethical behavior are the product of the entity’s ethical and behavioral standards, how
they are communicated, and how they are reinforced in practice.
 The enforcement of integrity and ethical values includes, for example, management actions to
eliminate or mitigate incentives or temptations that might prompt personnel to engage in dishonest,
illegal, or unethical acts.
 The communication of entity policies on integrity and ethical values may include the communication of
behavioral standards to personnel through policy statements and codes of conduct and by example.
7. Risk Assessment Process
Briefly describe the various stages of a risk assessment process. (Study Material)
Ans. (i) Risk Assessment is one of the most critical components of Enterprise Risk Management.
(ii)


FAST
The risk assessment process involves considerations for :
Qualitative and quantitative factors,
Definition of key performance and risk indicators,

CA.Sarthak Niraj Jain




Risk appetite,
Risk scores,
 Scales and maps,
 Use of data & metrics and benchmarking.
(iii) The various stages in a Risk Assessment process are as follows
1. Define Business Objectives and Goals;
2. Identify events that affect achievement of business objectives;
3. Assess likelihood and impact;
4. Respond and mitigate risks;
5. Assess residual risk.
8. Circumstances in which - Risks can Arise or Change
The Entity’s Risk Assessment Process includes how management identifies business risks relevant to the
preparation of financial statements in accordance with the entity’s applicable financial reporting framework,
estimates their significance, assesses the likelihood of occurrence and decides upon actions to respond to and
manage them and the results thereof. Elucidate the circumstances in which risks can arise or change.
(Study Material)(Nov-2019-New)
OR
For financial reporting purposes, the entity's risk assessment process includes how management identifies
business risks relevant to the preparation of financial statements in accordance with the entity's applicable
financial reporting framework, estimates their significance, assesses the likelihood of their occurrence, and
decides upon actions to respond to and manage them and the results thereof. Explain the circumstances
under which risk can arise or change. (Nov-2020-Old)

CA Final Audit - By CA. Sarthak Niraj Jain 89


Risk Assessment & Internal Control

Ans. Risks can arise or change due to circumstances such as the following:
(i) Changes in operating environment
Changes in the regulatory or operating environment can result in changes in competitive pressures
and significantly different risks.
(ii) New personnel
New personnel may have a different focus on or understanding of internal control.
(iii) New or revamped information systems.
Significant and rapid changes in information systems can change the risk relating to internal control.
(iv) Rapid growth
Significant and rapid expansion of operations can strain controls and increase the risk of a
breakdown in controls.
(v) New technology
Incorporating new technologies into production processes or information systems may change the
risk associated with internal control.
(vi) New business models, products, or activities
Entering into business areas or transactions with which an entity has little experience may
introduce new risks associated with internal control.
(vii) Corporate restructurings
Restructurings may be accompanied by staff reductions and changes in supervision and
segregation of duties that may change the risk associated with internal control.
(viii) Expanded foreign operations

FAST
The expansion or acquisition of foreign operations carries new and often unique risks that may
affect internal control, for example, additional or changed risks from foreign currency transactions.
(ix) New accounting pronouncements

CA.Sarthak Niraj Jain


Adoption of new accounting principles or changing accounting principles may affect risks in
preparing financial statements.
9. Deficiencies in ICS & General Conditions pertaining to Internal Check
New life Hospital is a multi-specialty hospital which has been facing a lot of pilferage and troubles regarding
their inventory maintenance and control. On investigation into the matter it was found that the person in
charge of inventory inflow and outflow from the store house is also responsible for purchases and
maintaining inventory records. According to you, which basic system control has been violated? Also list
down the other general conditions pertaining to such system which needs to be maintained and checked by
the management. (Study Material)(Nov-2015)
OR
Navjeevan Hospital is a multi-specialty hospital which has been facing a lot of pilferage and troubles
regarding their inventory maintenance and control. On investigation into the matter it was found that the
person in charge of inventory inflow and outflow from the store house is also responsible for purchases and
maintaining inventory records. According to you, which basic system control has been violated? Also list
down the other general conditions pertaining to such system which needs to be maintained and checked by
the management. (RTP-May-2020)
Ans. Basic system of Control:
 Internal Checks and Internal Audit are important constituents of Accounting Controls. Internal
check system implies organization of the overall system of book-keeping and arrangement of Staff
duties in such a way that no one person can carry through a transaction and record every aspect
thereof.

90 Ab Audit Hoga Sabse Scoring - By CA. Sarthak Niraj Jain


Risk Assessment & Internal Control

 In the given case of New Life Hospital: - The person-in-charge of inventory inflow and outflow
from the store house is also responsible for purchases and maintaining inventory records.
 Thus, one of the basic system of control i.e. internal check which includes segregation of duties or
maker and checker has been violated where transaction processing are allocated to different persons
in such a manner that no one person can carry through the completion of a transaction from start
to finish or the work of one person is made complimentary to the work of another person.

 General Conditions pertaining to internal check:


a) No single person should have an independent control over any important aspect of the
business operation. Every employee’s action should come under the review of another person.
b) Staff duties should be rotated from time to time so that members do not perform the same
function for a considerable length of time.
c) Every member of the staff should be encouraged to go on leave at least once in a year.
d) Persons having physical custody of assets must not be permitted to have access to the books
of account.
e) There should exist an accounting control in respect of each class of assets, in addition, there
should be periodical inspection so as to establish their physical condition.
f) Mechanical devices should be used, where ever practicable to prevent loss or
misappropriation of cash.
g) Budgetary control would enable the management to review from time to time the progress of
trading activities.
h) For inventory taking, at the close of the year, trading activities should, if possible be

FAST
suspended, and it should be done by staff belonging to several sections of the organization.
i) The financial and administrative powers should be distributed very judiciously among
different officers and the manner in which those are actually exercised should be reviewed

CA.Sarthak Niraj Jain


periodically.
j) Procedures should be laid down for periodical verification and testing of different sections of
accounting records to ensure that they are accurate.
10. General Conditions pertaining to Internal Check
BSF Limited is engaged in the business of trading leather goods. You are the internal auditor of the company
for the year 2021-22. In order to review internal controls of the sales department of the company, you visited
the department and noticed the work division as follows:
(1) An officer was handling the sales ledger and cash receipts.
(2) Another official was handling dispatch of goods and issuance of Delivery challans.
(3) One more officer was there to handle customer/ debtor accounts and issue of receipts.
(a) As an internal auditor you are required to briefly discuss the general condition pertaining to the
internal check system.
(b) Do you think that there was proper division of work? If not, why?
(Study Material)(RTP-Nov-2019)(MTP-Nov-2019)
Ans. (a) General Conditions pertaining to internal check:
Considerations for effectiveness of a system of internal check:
 No single person should have an independent control over any important aspect of the
business operation. Every employee’s action should come under the review of another person.
 Staff duties should be rotated from time to time so that members do not perform the same
function for a considerable length of time.
 Every member of the staff should be encouraged to go on leave at least once in a year.
 Persons having physical custody of assets must not be permitted to have access to the books
of account.

CA Final Audit - By CA. Sarthak Niraj Jain 91


Risk Assessment & Internal Control

 There should exist an accounting control in respect of each class of assets, in addition, there
should be periodical inspection so as to establish their physical condition.
 Mechanical devices should be used, where ever practicable to prevent loss or
misappropriation of cash.
 Budgetary control would enable the management to review from time to time the progress of
trading activities.
 For inventory taking, at the close of the year, trading activities should, if possible be
suspended, and it should be done by staff belonging to several sections of the organization.
 The financial and administrative powers should be distributed very judiciously among
different officers and the manner in which those are actually exercised should be reviewed
periodically.
 Procedures should be laid down for periodical verification and testing of different sections of
accounting records to ensure that they are accurate.
(b) Conclusion:- In the present condition company has not done proper division of work as:
 The receipts of cash should not be handled by the official handling sales ledger and
 Delivery challans should be verified by an authorised official other than the officer handling
dispatch of goods.
3.4 - Internal Control Assessment & Evaluation
11. SOPs of Assessment and Evaluation of Control
A newly qualified professional has received his first appointment as auditor of a large company and is very
much concerned about the effectiveness of internal control and wants to assess and evaluate the control
environment as· part of his audit program. Towards achieving his objective, he seeks your help in knowing

Ans.
FAST
the Standard Operating Procedures (SOPs) of assessment and evaluation of control.
Standard Operating Procedures (SOPs) of assessment and evaluation of control:
(May-2019-New)

(i)
CA.Sarthak Niraj Jain
Enterprise risk management:
Organization having robust processes to identify & mitigate risks across the entity & its periodical
review will assist in early identification of weaknesses in internal control and taking effective
control measures. In such entities, surprises of failures in controls is likely to be few.
(ii) Segregation of job responsibilities:
Segregation of duties is an important element of control which ensures that no two commercial
activities should be conducted by the same person.
(iii) Job rotation in sensitive areas:
In key commercial functions, job rotation is regularly followed to avoid degeneration of controls.
(iv) Documents of delegation of financial powers:
Document on delegation of powers allows controls to be clearly operated without being dependant
on individuals.
(v) IT based controls:
In an IT Environment, it is much easier to embed controls through the system instead of being
human dependant. The failure rate for IT embedded controls is likely to be low, is likely to have better
audit trail & is thus easier to monitor
3.5 - Reporting to clients on internal control weaknesses
12. Letter of Weaknesses-Drafting
During the course of his audit, the auditor noticed material weaknesses in the internal control system and he
wishes to communicate the same to the management. You are required to elucidate the important points the
auditor should keep in the mind while drafting the letter of weaknesses in internal control system.
(Study Material)(RTP-May-2020)(MTP-May-2020)

92 Ab Audit Hoga Sabse Scoring - By CA. Sarthak Niraj Jain


Risk Assessment & Internal Control

Ans. Points to be considered while drafting letter of weaknesses:


(i) As per SA 265, “Communicating Deficiencies in Internal Control to Those who Charged with
Governance and Management” The auditor shall include in the written communication of significant
deficiencies in internal control –
 A description of the deficiencies and an explanation of their potential effects; and
 Sufficient information to enable those charged with governance and management to understand
the context of the communication.
This communication should be, preferably, in writing through a letter of weakness.
(ii) Important points with regard to such a letter are as follows:
a) It lists down the area of weaknesses in the internal control system and recommends suggestions
for improvement.
b) It should clearly indicate that this letter covers only weaknesses which have come to the
attention of the auditor during his evaluation of internal control for the purpose of determining
nature, timing and extent of further audit procedures.
c) Letter should clearly indicate that his examination of internal control has not been designed to
determine the adequacy of internal control for management.
d) This letter serves as a significant means for management and governing body for the purpose of
improving the system and its strict implementation.
e) The letter may also serve to minimize legal liability in the event of a major defalcation or other
loss resulting from a weakness in internal control.
3.6 - Risk Based Audit (RBA)
13. Risk Based Audit
What are the general steps in the conduct of risk based audit?

FAST OR
(Study Material)(MTP-Nov-2020)(May-2016)

ST Ltd is a growing company and currently engaged in the business of manufacturing of tiles. The company is

CA.Sarthak Niraj Jain


planning to expand and diversify its operations. The management has increased the focus on the internal
controls to ensure better governance. The management had a discussion with the statutory auditors to
ensure the steps required to be taken so that the statutory audit is risk based and focused on areas of
greatest risk to the achievement of the company’s objectives. Please advise the management and the auditor
on the steps that should be taken for the same. (Study Material)(MTP-May-2019)(RTP-May-2019)
OR
Audit should be risk-based or focused on areas of greatest risk to the achievement of the audited entity's
objectives. Risk based Audit (RBA) is an approach to audit that analyzes audit risks, sets materiality
thresholds based on audit risk analysis and develops audit programmes that allocate a larger portion of audit
resources to high-risk areas. What are the general steps for conducting Risk Based Audit? (Nov-2020-Old)
OR
What are the main phases in the conduct of risk-based audit? (May-2017)
Ans. (i) Definition: -Risk Based audit is an audit approach that analyzes audit risks, sets materiality
thresholds based on audit risk analysis, and develops audit programmes that allocate a larger
portion of audit resources to high-risk areas.
(ii) General Steps in the Conduct of Risk Based Audit:
 The auditor’s objective in a risk-based audit is to obtain reasonable assurance that no material
misstatements whether caused by fraud or errors exist in the financial statements.
 This involves the following three key steps:
 Assessing the risks of material misstatement in the financial statements
 Designing and performing further audit procedures that respond to assessed risks and
reduce the risks of material misstatements in the financial statements to an acceptably low
level; and
 Issuing an appropriate audit report based on the audit findings.

CA Final Audit - By CA. Sarthak Niraj Jain 93


Risk Assessment & Internal Control

 The risk-based audit process is presented in three distinct phases:


(a) Risk assessment.
The risk assessment phase of the audit involves the following steps:
1) Performing client acceptance or continuance procedures;
2) Planning the overall engagement;
3) Performing risk assessment procedures to understand the business and identify
inherent and control risks;
4) Identifying relevant internal control procedures and assessing their design and
implementation (those controls that would prevent material misstatements from
occurring or detect and correct misstatements after they have occurred);
5) Assessing the risks of material misstatement in the financial statements; Identifying
the significant risks that require special audit consideration and those risks for which
substantive procedures alone are not sufficient;
6) Communicating any material weaknesses in the design and implementation of internal
control to management and those charged with governance; and
7) Making an informed assessment of the risks of material misstatement at the financial
statement level and at the assertion level.
(b) Risk response: -This phase of the audit is to design and perform further audit
procedures that respond to the assessed risks of material misstatement and will provide
the evidence necessary to support the audit opinion.
Some of the matters the auditor should consider when planning the audit procedures
include:

FAST 1. Assertions that cannot be addressed by substantive procedures alone. This can occur
where there is highly automated processing of transactions with little or no manual
intervention.

CA.Sarthak Niraj Jain


2. Existence of internal control that, if tested, could reduce the need/scope for other
substantive procedures.
3. The potential for substantive analytical procedures that would reduce the
need/scope for other types of procedures.
4. The need to incorporate an element of unpredictability in procedures performed.
5. The need to perform further audit procedures to address the potential for
management override of controls or other fraud scenarios.
6. The need to perform specific procedures to address “significant risks” that have been
identified.
(c) Reporting.
The final phase of the audit is to assess the audit evidence obtained and determine
whether it is sufficient and appropriate to reduce the risks of material misstatement in
the financial statements to an acceptably low level.
It is important at this stage to determine:
1. If there had been a change in the assessed level of risk;
2. Whether conclusions drawn from work performed are appropriate; and
3. If any suspicious circumstances have been encountered. Any additional risks should
be appropriately assessed, and further audit procedures performed as required. When
all procedures have been performed and conclusions reached:
4. Audit findings should be reported to management and those charged with
governance; and
5. An audit opinion should be formed, and a decision made on the appropriate wording
for the auditor’s report.

94 Ab Audit Hoga Sabse Scoring - By CA. Sarthak Niraj Jain


Risk Assessment & Internal Control

14. Risk Based Audit & Risk Assessment Phase


PADHAM Ltd is engaged in the business of manufacturing of carpets. The company is planning to expand and
diversify its operations. The management has increased the focus on internal controls to ensure better
governance. The management discussed with the statutory auditors to ensure the steps required to be taken
so that the statutory audit is risk based and focused on areas of greatest risk to the achievement of the
company’s objectives.
(a) Name the key steps and phases involved in Risk Based Audit.
(b) Also, discuss the steps to be taken for the risk assessment phase of the audit. (RTP-May-2022)
Ans. (a) The auditor’s objective in a risk-based audit is to obtain reasonable assurance that no material
misstatements whether caused by fraud or errors exist in the financial statements.
This involves the following three key steps:
• Assessing the risks of material misstatement in the financial statements
• Designing and performing further audit procedures that respond to assessed risks and reduce the
risks of material misstatements in the financial statements to an acceptably low level; and
• Issuing an appropriate audit report based on the audit findings. The risk-based audit process is
presented in three distinct phases: Risk assessment. Risk response; and Reporting.
(b) The risk assessment phase of the audit involves the following steps:
• Performing client acceptance or continuance procedures;
• Planning the overall engagement;
• Performing risk assessment procedures to understand the business and identify inherent and
control risks;
• Identifying relevant internal control procedures and assessing their design and

FAST
implementation (those controls that would prevent material misstatements from occurring or
detect and correct misstatements after they have occurred);
• Assessing the risks of material misstatement in the financial statements;

CA.Sarthak Niraj Jain


• Identifying the significant risks that require special audit consideration and those risks for which
substantive procedures alone are not sufficient;
• Communicating any material weaknesses in the design and implementation of internal control to
management and those charged with governance; and
• Making an informed assessment of the risks of material misstatement at the financial statement
level and at the assertion level.
3.7 - International Internal Control Frameworks & Other Topics
15. Corporate Responsibility – SOX, 2002 & Co. Act,2013
The corporate responsibility under the SOX, 2002 and the Corporate Social Responsibility under the
Companies Act, 2013 are referring to the same concept. Please evaluate this statement critically.
(Nov-2020-Old)
Ans. Corporate Responsibility under the SOX 2002 and the Corporate Responsibility under the Companies
Act, 2013 – Referring to the same concept:
The Sarbanes Oxley Act of 2002 established corporate accountability and civil and criminal penalties
for white – collar crimes. The title three of SOX 2002 deals with the Corporate Responsibility whereas
Section 135 of the Companies Act, 2013 deals with Corporate Social Responsibility.
Both refer the same concepts such as:
(i) More independence be given to Audit Committee and auditor.
(ii) Strict reporting by an auditor on insider trading.
(iii) Additional disclosures imposed on financial reporting.
(iv) If there is any conflict between company and its auditor, the Audit Committee should be
empowered to resolve the same.

CA Final Audit - By CA. Sarthak Niraj Jain 95


Risk Assessment & Internal Control

(v) Higher penalties and criminal prosecution on financial frauds.


(vi) To include effectiveness of Internal Control System in the financial reporting.
(vii) More responsibilities must be imposed on managerial personal with higher penalties and
prosecutions on the breach.
(viii) Strict action against white collar crime.
(ix) Disclosers of the % of shareholdings by Directors, Executive Officers and principal shareholders. In
view of above, it can be concluded that Corporate Responsibility under SOX and Corporate Social
Responsibility under Companies Act, 2013 are referring to the same concept.
16. System towards Collection of Money
Y Co. Ltd. has five entertainment centers to provide recreational facilities for public especially for children
and youngsters at 5 different locations in the peripheral of 200 kilometers. Collections are made in cash.
Specify the adequate system towards collection of money.
(Study Material)(MTP-May-2018)
Ans. Control System over Selling and Collection of Tickets: In order to achieve proper internal control
over the sale of tickets and its collection by the Y Co. Ltd., following system should be adopted -
(a) Printing of tickets: Tickets should be serially numbered and pre-printed Serial numbers should not
be repeated during a reasonable period, say a month or year depending on the turnover.
(b) Sale of Tickets: Tickets should be sold from the Central ticket office at each of the 5 centers,
preferably through machines.
(c) Reconciliation of daily cash: Cash collection at each office should be reconciled with the number of
tickets sold.
(d) Banking of daily cash collection: Daily collection should be deposited in the bank on next working

FAST
day of the bank. Till that time, the cash should be in the custody of properly authorized person.
(e) Cancellation of Entrance ticket: Entrance tickets should be cancelled at the entrance gate when
public enters the centre.

CA.Sarthak Niraj Jain


(f) Advance booking: If advance booking of facility is made available, the system should ensure that all
advance booked tickets are paid for.
(g) Discounts and free pass: The discount policy should be such that the concessional rates should be
properly authorized and signed forms for such authorization should be preserved.
(h) Surprise checks: Internal audit system should carry out periodic surprise checks for cash counts,
daily banking, reconciliation and stock of unsold tickets etc.

Notes

96 Ab Audit Hoga Sabse Scoring - By CA. Sarthak Niraj Jain


Special Aspects of Auditing in an Automated Environment

Chapter SPECIAL ASPECTS OF AUDITING IN AN


4 AUTOMATED ENVIRONMENT

Past Exams Weightage


5

0
MAY-19 NOV-19 NOV-20 JAN-21 JUL-21 DEC-21

May-19 Nov-19 Nov-20 Jan-21 Jul-21 Dec-21


Marks 5 4 4 0 5 5

4.1 -Auditing in Real Environment


1.
FAST
Identification of Risks Associated with the IT systems (Statutory Audit)
A Company is using ERP for all its business processes including Procurement, Sales, Finance and Reporting. You
are required to explain the Statutory Auditor’s approach to identify the risks associated with the IT systems.

Ans. CA.Sarthak Niraj Jain


The Statutory Auditor’s approach to identify the risks associated with the IT systems
(i)
(Study Material)

The Auditor should understand and document each of the business processes in form of narratives and
/ or flowcharts.
(ii) The next process will be to identify areas / events that can lead to risks, viz. manual Invoicing and
accounting once goods are dispatched could lead to incorrect Invoicing and accounting and hence is a
‘risk’.
(iii) The Auditor should also analyse the risks i.e. the impact it will have if materializes.
(iv) Next will be prioritization in terms of probability of how often the risks will materialize.
2. Considerations of Automated Environment in Different Stages of Audit
In a risk-based audit, the audit approach can be classified into three broad phases comprising of planning,
execution, and completion. You are required to briefly explain the relevant considerations for every phase in
above audit approach in case of an automated environment. (Study Material)
OR
In a controls-based audit, the audit approach can be classified into three broad phases comprising of planning,
execution, and completion. You are required to briefly explain the relevant considerations for every phase in
above audit approach in case of an automated environment. (Nov-2019-New)
OR
“The audit cycle consists of Planning, Execution and Completion. The automation in processing of business
transactions has considerations to be weighed by auditor at every phase of this cycle” – Enumerate the focal
points of such considerations when auditing in automated environment. (Nov-2018-New)
OR
In a controls-based audit, the audit approach can be classified into three broad phases comprising of planning,
execution, and completion. In this approach, the considerations of automated environment will be relevant at
every phase. Comment. (RTP-May-2018)(MTP-Nov-2018)

CA Final Audit - By CA. Sarthak Niraj Jain 97


Special Aspects of Auditing in an Automated Environment

Ans. In a controls-based audit, the audit approach can be classified into three broad phases comprising of
planning, execution, and completion. In this approach, the considerations of automated environment will be
relevant at every phase as given below:
(i) during risk assessment, the auditor should consider risk arising from the use of IT systems at the
company;
(ii) when obtaining an understanding of the business process and performing walkthroughs the use of IT
systems and applications should be considered;
(iii) while assessing the entity level controls the aspects related to IT governance need to be understood and
reviewed;
(iv) pervasive controls including segregation of duties, general IT controls and applications should be
considered and reviewed;
(v) during testing phase, the results of general IT controls would impact the nature, timing and extent of
testing;
(vi) when testing of reports and information produced by the entity (IPE) generated through IT systems and
applications;
(vii) at completion stage, evaluation of control deficiencies may require using data analytics and CAATs.

Alternative Answer (Given in Suggested Answers of ICAI)


In a controls-based audit, the audit approach can be classified into three broad phases comprising of
planning, execution, and completion. In this approach, the considerations of automated environment will be
relevant at every phase as given below:
(i) Risk Assessment


FAST
Identify significant accounts and disclosures
Qualitative and Quantitative considerations

CA.Sarthak Niraj Jain





Relevant financial statement assertions
Identify likely sources of misstatement
Consider risk arising from use of IT system
(ii) Understand and Evaluate
 Document understanding of business process using Flowchart/Narratives
 Prepare Risk Control Matrix (RCM)
 Understand design of controls by performing walk through of end-by-end process
 Process wide considerations for entity level controls, segregation of duties
 IT general controls, application controls
(iii) Test for Operating Effectiveness
 Assess nature timing and extent of control testing
 Assess reliability of key reports and spreadsheet
 Sample testing
 Consider competence and independence of staff/team performing control testing
(iv) Reporting
 Evaluate control deficiencies
 Significant deficiencies, material weakness
 Remediation of control weakness
 Internal control memo (ICM) or management letter
 Auditor’s Report

98 Ab Audit Hoga Sabse Scoring - By CA. Sarthak Niraj Jain


Special Aspects of Auditing in an Automated Environment

4.2 - IT Related Risk & Controls


3. Direct & Indirect Entity Level Controls (ELCs)
While evaluating the risks and controls at entity level, the Auditor should take cognizance of the prevalent direct
and indirect entity level controls operating in the entity. Explain what they pertain to with few examples.
(May-2018)
OR
Distinguish between: Direct entity level controls and indirect entity level controls.
Ans. Entity Level Risk & Controls
(i) Direct ELCs operate at a level higher than business activity or transaction level such as a business
process or sub-process level, account balance level, at a sufficient level of precision, to prevent, detect or
correct a misstatement in a timely manner.
Examples of Direct ELCs are:
• Business performance reviews;
• Monitoring of effectiveness of control by Internal Audit function.
(ii) Indirect ELCs do not relate to any specific business process, transaction or account balance and
hence, cannot prevent or detect misstatements. However, they contribute indirectly to the effective
operation of direct ELC and other control activities.
Examples of Indirect ELCs are:
• Company code of conduct & ethics policies;
• Human resource policies;
• Employee Job roles & responsibilities.
4. Identification of Controls
Identify the controls which are automated, manual or IT dependent manual for the below mentioned cases?
(i)
FAST
Price master configured in the sales master can only be edited by authorised personnel in the system.
(ii) Invoice cannot be booked in SAP in case Purchase orders are not approved.
(iii) Inventory ageing report is pulled out from the system based on which provisioning is calculated after

CA.Sarthak Niraj Jain


analyzing the future demand by the inventory personnel and approved by the controller.
(iv) All invoices are signed by warehouse personnel before the goods are dispatched to the customer.
(v) Credit limit is assigned to the customer and goods cannot be sold in excess of credit limit configured in the
system.
(vi) All changes to the credit limit is approved manually by sales manager.
(vii) Ageing report is pulled out from SAP based on which provisioning is calculated by accounting personnel
and approved by financial controller.
(viii) PO, CRN (Good received note) and invoice are matched by the system before it is posted in the financial
records. (MTP-May-2018)
Ans. (i) Automated control as there is inbuilt control which allows editing in sales master by only authorised
personnel.
(ii) Automated control as there is inbuilt control which doesn’t allow approval of invoice in case of non
approval of purchase order.
(iii) IT dependent manual control as inventory ageing report is pulled out from the system after which
provision for inventory is manually approved.
(iv) Manual control as sign off is required to be done for the invoice before the dispatch of the goods.
(v) Automated control as there is inbuilt control that doesn’t allow goods to be sold if credit limit
assigned to the customer has been crossed.
(vi) Manual control as sign off is required for every change to the credit limit.
(vii) IT dependent manual control as ageing report is relied upon for calculation of provisioning for
debtors.
(viii) Automated control as PO, GRN and invoice is matched by the system before recording of the invoice
to the vendor account.

CA Final Audit - By CA. Sarthak Niraj Jain 99


Special Aspects of Auditing in an Automated Environment

5. Auditing Virtually - Pros and Cons


Auditing virtually through remote working model has its own pros and cons for the Auditors. Elucidate any four
pros and cons with reasons. (Nov-2020-Old)
Ans. Auditing virtually has several benefits as well as disadvantages for the Auditors, the details of which are
summarised as under:
Particulars Remarks
Pros of auditing virtually through remote working model:
Heightened use of Virtual audit is possible only with the use of technology.
technology for Hence embracing technology helps in better audit delivery and in making
better execution auditors tech savvy to prepare for the future.
Improved audit Use of technology helps in improving the audit quality.
quality
Greater audit With the interplay of technology and the availability of quality time — the
coverage audit coverage could be enhanced significantly.
Audit efficiencies Through proper planning, upfront engagement with the client, audit
efficiencies are possible.
Lesser cost of The cost of audit delivery would be comparatively lesser through remote
delivery working.
Higher scalability The possibility of capacity ramp up is feasible without any restrictions on
availability of space etc.
High speed Information can be generated very quickly. Even complex reports in specific

FAST report format can be generated for audit purposes without much loss of time.
This cuts down the time enabling the auditor to extend their analytical review
with high speed of operation.

CA.Sarthak Niraj Jain


Low clerical error Computerised operation being a systematic and sequential programmed
course of action the changes of commission of error is considerably reduced.
Clerical error is highly minimised.
Concentration of Computer programs perform more than one set of activities at a time thereby
duties restricting the duty of personnel involved in the work.
Control distribution Auditor being able to control distribution of the output and ensuring the
of the output and timely correction of errors, for example, if the wrong input file were to be used.
timely corrections
of errors
Cons of auditing virtually through remote working model:
Lack of conclusive Aspects such as physical verification of inventory etc. requires physical
evidence presence and the same cannot be done virtually.
High level of Dependency on technology and various infrastructure associated with it
dependency would pose challenges.
Increased time in Time spent in coordination is very high.
coordination
Security and The entire audit process is carried out with the help of technology which is prone
compliance issues to various security concerns. Further, aspects relating to confidentiality, data
protection etc. are very critical for a successful virtual audits. Further, the
possibility of non identification of frauds/ reliance on fake scanned documents
etc. is very high in a virtual audit environment.
Feel of audit and In the absence of face to face interactions, the real feel of the audit and the
client engagement client engagement could be a challenge.

100 Ab Audit Hoga Sabse Scoring - By CA. Sarthak Niraj Jain


Special Aspects of Auditing in an Automated Environment

Non-availability of The audit team should have sufficient knowledge to plan, execute and use the
sufficient IT results of the same. In the absence of sufficient knowledge it would be difficult
knowledge and to audit through remote working model.
expertise:
Non-availability of The auditor may plan to use other computer facilities when the use of remote
suitable computer working model on an entity’s computer is uneconomical or impractical, for
facilities and data in example, because of an incompatibility between the auditor’s package
suitable format: programme and entity’s computer.
Impracticability of Some audit procedures may not be possible to perform manually because they
manual tests due to rely on complex processing (for example, advanced statistical analysis) or
lack of evidence: involve, amounts of data that would overwhelm any manual procedure.
Time constraints in Time constraints in certain data, such as transaction details, are often kept for a
certain data short time and may not be available in machine-readable form by the time
auditor wants them. Thus, the auditor will need to make arrangements for the
retention of data required, or may need to alter the timing of the work that
requires such data.
SJ Note:- Any four Pros & Cons can be written in exam
6. Whistle-Blower Policy
The auditors are required to understand, evaluate and validate the entity level controls as a part of audit
engagement, the result of which has an impact on the nature, timing and extent of other audit procedures. In
evaluating the effect of such control, existence, effectiveness and assessment of the whistle-blower policy in the
company is very important. Specify the procedure you would perform for an understanding and evaluation of

Ans. (i) FAST


such whistle-blower policy.
Procedure for understanding and evaluation of whistle-blower policy

(July-2021-New)

Auditors are required to understand, evaluate and validate the entity level controls as a part of an

CA.Sarthak Niraj Jain



audit engagement.
The results of testing entity level controls could have an impact on the nature, timing and extent
of other audit procedures including testing of controls.
 For example, when the entity level controls at a company are effective, the auditor may consider
reducing the number of samples in the test of controls and where the auditor finds the entity
level controls ineffective, the auditor may consider to increase the rigo ur of testing by
increasing sample sizes.
 In small and less complex companies, the entity level controls may not formally defined or
documented. In such situations, the auditor should design audit procedures accordingly to obtain
evidence of the existence and effectiveness of entity level controls.

(ii) The following example shows how the auditor performs an understanding and evaluation of the
whistle-blower policy in a company:
 Does the company have a whistle-blower policy?
 Is this policy documented and approved?
 Has the whistle-blower policy been communicated to all the employees?
 Are employees aware of this policy and understand its purpose and their obligations?
 Has the company taken measures viz., training, to make the employees understand the contents
and purpose of the policy?
 Does the company monitor effectiveness of the policy from time-to-time?
 How does the company deal with deviations and non-compliance?

CA Final Audit - By CA. Sarthak Niraj Jain 101


Special Aspects of Auditing in an Automated Environment

4.3 - Data Analytics


7. Data Analytics & Its Application
You came to know that the data stored and processed in systems can be used to get various insights into the way
business operates in an automated environment. This data can be useful for the preparation of management
information system (MIS) reports and electronic dashboards that give a high-level snapshot of business
performance. In view of above you are required to briefly discuss the meaning of data analytics and example of
circumstances when auditing in an automated environment, auditors can apply the concepts of data analytics.
(RTP-May-2022)
OR
In an automated environment, the data stored and processed in systems can be used to get various insights into
the way business operates. This data can be useful for preparation of management information system (MIS)
reports and electronic dashboards that give a high-level snapshot of business performance. In view of above you
are required to briefly discuss the meaning of data analytics and example of circumstances when auditing in an
automated environment, auditors can apply the concepts of data analytics. (Study Material)(RTP-May-2019)
OR
What is Data Analytics, When auditing in an automated environment, auditors can apply the concepts of data
analytics for several aspects of an audit. State those aspects
Ans. (i) Data Analytics:
 Generating and preparing meaningful information from raw system data using processes, tools,
and techniques is known as Data Analytics.
 The data analytics methods used in an audit are known as Computer Assisted Auditing Techniques
or CAATs.


FAST
(ii) Auditors can apply the concept of data analytics for several aspects of an audit including the following:
 Preliminary Analytics;
Risk Assessment;

CA.Sarthak Niraj Jain




Control Testing;
Non-Standard Journal Analysis;
 Evaluation of Deficiencies;
 Fraud Risk assessment.
8. Approach to get the Benefit from the use of CAATs
“Generating and preparing meaningful information from raw system data using processes, tools, and techniques
is known as Data Analytics and the data analytics methods used in an audit are known as Computer Assisted
Auditing Techniques or CAATs.” You are required to give a suggested approach to get the benefit from the use of
CAATs. (Study Material)(MTP-Nov-2020)(RTP-Nov-2019)
Ans. There are several steps that should be followed to achieve success with CAATs and any of the supporting
tools. A suggested approach to benefit from the use of CAATs is given in the illustration below:
Step 1: Understand Business Environment including IT;
Step 2: Define the Objectives and Criteria;
Step 3: Identify Source and Format of Data;
Step 4: Extract Data;
Step 5: Verify the Completeness and Accuracy of Extracted Data;
Step 6: Apply Criteria on Data Obtained;
Step 7: Validate and Confirm Results;
Step 8: Report and Document Results and Conclusions [SA 230]

102 Ab Audit Hoga Sabse Scoring - By CA. Sarthak Niraj Jain


Company Audit

Chapter
5
COMPANY AUDIT
\

Past Exams Weightage


10
9
8
7
6
5
4
3
2
1
0
MAY-19 NOV-19 NOV-20 JAN-21 JUL-21 DEC-21

May-19 Nov-19 Nov-20 Jan-21 Jul-21 Dec-21

FASTMarks 8 10 10 0 5 10

CA.Sarthak Niraj Jain


UNIT-1 : AUDIT & AUDITORS (Section 139-148)

5.1 - Section -139 Appointment of Auditor


1. Audit Firm having Common Partner
M/s. ABC & Company is an audit firm having partners Mr. A, Mr. B and Mr. C, whose tenure as statutory
auditor in R Ltd. a listed entity, has expired as per the Companies Act, 2013. M/s. XY is another audit firm
which is appointed as the statutory auditor of R Ltd. for the subsequent year. Mr. A joins M/s XY as partner, 3
months after it was appointed the statutory auditor of R Ltd. Comment. (May-2017)
Ans. (i) Provision:- Section 139(2) of the Companies Act, 2013 provides that as on the date of
appointment, no audit firm having a common partner or partners to the other audit firm, whose
tenure has expired in a company immediately preceding the financial year, shall be appointed as
auditor of the same company for a period of five years.
(ii) In the given case:- R Ltd. is a listed entity, thus, rotational provisions are applicable. Mr. A is a
partner in M/s. ABC &Co., whose tenure as statutory auditor in R Ltd. has expired. He joined M/s.
XY as partner, 3 months after it was appointed as statutory auditor of R Ltd.
 It may be noted that that there should not be a common partner in the firms as on the date of
appointment, however, in the given case, Mr. A has joined M/s. XY after 3 months of its
appointment as statutory auditor of R Ltd.
(iii) Conclusion:- M/s. XY may continue as statutory auditor of R Ltd. for the subsequent term.

CA Final Audit - By CA. Sarthak Niraj Jain 103


Company Audit

2. Rotation of Auditors
CA. Rock is a partner in M/s Ajay & Associates and M/s Vijay & Associates simultaneously. M/s Ajay &
Associates has completed its tenure of 10 years as an auditor in Sanjay Ltd. immediately preceding the
current financial year. It may be noted that the provisions for applicability of rotation of auditors are
applicable to Sanjay Ltd. Now, the company wants to appoint M/s Vijay & Associates as auditor for 5 years
(a) Whether M/s Vijay & Associates is allowed to accept the appointment as auditor of Sanjay Ltd.?
(b) Would your answer be different from above if CA. Rock, being in-charge of M/s Ajay& Associates and
certifying authority of financial statements of Sanjay Ltd., retires from the partnership in M/s Ajay &
Associates and joins M/s Vijay & Associates? (Assume 10 years = 10 AGMs) (RTP-May-2016)
Ans. (i) Provision:- Section 139(2) of the Companies Act 2013
 No listed company or a company belonging to such class or classes of companies as
mentioned above, shall appoint or re-appoint.
 an individual as auditor for more than one term of 5 consecutive years; and
 an audit firm as auditor for more than two terms of 5 consecutive years.
 Section 139(2) of the Companies Act, 2013 provides that as on the date of appointment, no
audit firm having a common partner or partners to the other audit firm, whose tenure has
expired in a company immediately preceding the financial year, shall be appointed as
auditor of the same company for a period of five years.
 Companies (Audit and Auditors) Rules, 2014:- For the purpose of rotation of auditors, if a
partner, who is in-charge of an audit firm and also certifies the financial statements of the
company, retires from the said firm and joins another firm of chartered accountants, such other
firm shall also be ineligible to be appointed for a period of 5 Years
(ii) In the given case:- CA. Rock is a common partner in M/s Ajay & Associates and M/s Vijay &

FAST
Associates. The tenure of 10 years of M/s Ajay & Associates as an auditor expired in Sanjay Ltd.
immediately preceding financial year.
(iii) Conclusion:-

CA.Sarthak Niraj Jain


(a) Ajay & Associates shall not be eligible for re-appointment as auditor in the same company for
5 years from the completion of such term. Therefore, M/s Vijay & Associates will also be
disqualified to be appointed as auditor of Sanjay Ltd. for next 5 financial years as CA. Rock is a
common partner in M/s Ajay & Associates and M/s Vijay & Associates.
(b) M/s Vijay & Associates will also be disqualified to be appointed as auditor of Sanjay Ltd. for
next 5 financial years if newly joined partner CA. Rock has joined Vijay & Associates after
retiring from M/s Ajay & Associates as he was in-charge of the audit firm and also certified the
financial statements of the company in immediately preceding the current financial year.
3. Section 2(45) & 139 - Audit of Government Company & Role of C&AG
VM Ltd., a company wholly owned by Central Government was disinvested during the previous year,
resulting in 45% of the shares being held by public. The shares were also listed on the BSE. Since the shares
were listed, all the listing requirements were applicable, including publication of quarterly results,
submission of information to the BSE etc. Gautam, the Finance Manager of the Company is of the opinion that
now the company is subject to stringent control by BSE and the markets, therefore the auditing requirements
of a limited company in private sector under the Companies Act 2013 would be applicable to the company
and the C&AG will not have any role to play. Comment. (MTP-May-2021)
Ans. (i) Provision: -Section 2(45) of the Companies Act, 2013, defines a “Government Company” as a
company in which not less than 51% of the paid-up share capital is held by the Central Government
or by any State Government or Governments or partly by the Central Government and partly by
one or more State Governments, and includes a company which is a subsidiary company of such a
Government company.
(ii) Appointment of Auditors: -The auditors of these government companies are firms of Chartered
Accountants, appointed by the Comptroller & Auditor General, who gives the auditor directions on
the manner in which the audit should be conducted by them.

104 Ab Audit Hoga Sabse Scoring - By CA. Sarthak Niraj Jain


Company Audit

(iii) In the given scenario: - VM Ltd., a company wholly owned by Central Government was disinvested
during the previous year, resulting in 45% of the shares being held by public. Since, shares were
listed on the BSE therefore all the listing requirements were applicable.
(iv) Conclusion: -Opinion of Finance Manager of the Company Mr. Gautam that since company is subject
to stringent control by BSE and the markets, therefore the auditing requirements of a limited
company in private sector under the Companies Act 2013 would be applicable to the Company and the
C&AG will not have any role to play, is not correct as listing of company’s shares on a stock
exchange is irrelevant for this purpose.
5.2 - Section-140 Removal & Resignation of Auditor
4. Sec. 140 - Appointment of Sole Auditor
M/s. PQR, audit firm has been re-appointed as sole statutory auditor of a listed company in the AGM, where
till last year M/s. LMN, audit firm was also one of the joint auditors along with M/s. PQR. One tenure of
consecutive five years of both the firms get completed in the mentioned AGM. Mention the steps that should
be taken by M/s. PQR before commencing the audit. (RTP-Nov-2015)
OR
You have been appointed the sole auditor of a company where you were one of the joint auditors for the
immediately preceding year and the said joint auditors is not re-appointed.
Ans. When one of the joint auditors of the previous years is considered for reappointment by the members as
the sole auditor for the next year, it is similar to non-re-appointment of one of the retiring joint auditors.
(i) Provision:- As per Section 140(4) of the Companies Act, 2013, special notice shall be required for a
resolution at an annual general meeting appointing as auditor a person other than a retiring
auditor, or providing expressly that a retiring auditor shall not be re-appointed.
(ii) Exception:- Except where the retiring auditor has completed a consecutive tenure of five years or
ten years, as the case may be, as provided u/s 139(2)
(iii) Steps to comply:- Accordingly, provisions of the Companies Act, 2013 to be complied with areas

FAST
under:
 Special Notice:- Ascertain that special notice u/s 140(4) of the Companies Act, 2013 was
received by the company from such number of members holding not less than one percent of

CA.Sarthak Niraj Jain


total voting power or holding shares on which an aggregate sum of not less than ` 5 lakh has
been paid up on the date of the notice not earlier than 3 months but at least 14 days before the
AGM date.
 Sending copy of notice:- Check whether the said notice has been sent to all the members at least
7 days before the date of the AGM.
 Contents of Notice:- Verify the notice contains an express intention of a member for proposing
the resolution for appointing a sole auditor in place of both the joint auditors who retire at the
meeting but are eligible for re-appointment.
 Notice to Auditor:-The notice is also sent to the retiring auditor as per Section 140(4)(ii) of the
Companies Act, 2013.
 Sending the Representation:-Verify whether any representation, received from the retiring
auditor was sent to the members of the company.
 Consideration of representation:- Verify from the minutes book whether the representation
received from the retiring joint auditor was considered at the AGM.
5. Power of Tribunal in case Auditor acted in a Fraudulent Manner
Vivan Ltd is a company engaged in the business of software development. It is one of the largest companies in
this sector with a turnover of INR 25,000 crores. The operations of the company are increasing constantly,
however, the focus of the management is more on cost cutting in the coming years to improve its
profitability. In respect of the financial statements of the company which are used by various stakeholders,
some fraud was observed in respect of assets reported therein due to which those stakeholders suffered
damages. As a result, those stakeholders applied to Tribunal for change of auditor on the basis that auditor is
colluded in the fraud.
Elucidate the power of Tribunal to change the auditor of a company if found acting in a fraudulent manner as
provided under sub-section (5) of section 140 of the Companies Act, 2013.
(MTP-Nov-2019, RTP-Nov-2019, Nov-2019-Old)

CA Final Audit - By CA. Sarthak Niraj Jain 105


Company Audit

Ans. (i) Provision:-Section 140(5) of the Companies Act, 2013, the Tribunal either suo motu or on an
application made to it by the Central Government or by any person concerned, if it is satisfied that
the auditor of a company has, whether directly or indirectly, acted in a fraudulent manner or
abetted or colluded in any fraud by, or in relation to, the company or its directors or officers, it may,
by order, direct the company to change its auditors.
(ii) When application made by CG:- if the application is made by the Central Government and the
Tribunal is satisfied that any change of the auditor is required, it shall within fifteen days of
receipt of such application, make an order that he shall not function as an auditor and the Central
Government may appoint another auditor in his place.
(iii) When final order passed:- It may be noted that an auditor, whether individual or firm, against
whom final order has been passed by the Tribunal under this section shall not be eligible to be
appointed as an auditor of any company for a period of five years from the date of passing of the
order and the auditor shall also be liable for action under section 447of the said Act.
(iv) In case of Firm:- It is hereby clarified that in the case of a firm, the liability shall be of the firm and
that of every partner or partners who acted in a fraudulent manner or abetted or colluded in any
fraud by, or in relation to, the company or its director or officers.
5.3 - Section-141 Qualification & Disqualification of Auditor
6. Auditor’s Disqualifications as to Business Relationship
PQ and Co. is an audit firm with P and Q as partners. For the financial year 2021-22, the firm has been
appointed as statutory auditor of M/s Mango Orchards Hotel Ltd. The audit firm is a regular customer of the
hotel and the partners usually stay in the same hotel at various locations in the course of travelling for their
various professional assignments. Normally, payments for such stay are settled against quarterly bills raised
by the company. Give you comment with respect to the Companies Act, 2013. (May-2019-New)
Ans. (i) FAST
Provision:-
 As per Sec. 141(3)(d)(ii) of the Companies Act, 2013 read with Rule 10 of Companies (Audit and

CA.Sarthak Niraj Jain



Auditor’s) Rules, 2014, a person who is indebted to the company for an amount exceeding ₹5
Lacs shall be disqualified to be appointed as auditor.
As per section 141(3)(e) of Companies Act, 2013, a person or a firm who, whether directly or
indirectly, has business relationship with the company, or its subsidiary, or its holding or
associate company or subsidiary of such holding company or associate company of such nature as
may be prescribed, is disqualified to be appointed as auditor of that company.
(ii) As per Rule 10 of Companies (Audit and Auditor’s) Rules, 2014:- The term “business
relationship” shall be construed as any transaction entered into for a commercial purpose,
except:
 Commercial transactions which are in the nature of professional services permitted to be
rendered by an auditor or audit firm under the Act and the Chartered Accountants Act, 1949 and
the rules or the regulations made under those Acts;
 Commercial transactions which are in the ordinary course of business of the company at arm’s
length price - like sale of products or services to the auditor, as customer, in the ordinary course
of business, by companies engaged in the business of telecommunications, airlines, hospitals,
hotels and such other similar businesses.
(iii) In the present case:- Audit firm is a regular customer of the client running a hotel and the partners
usually stay in the same hotel at various locations in the course of travelling for their various
professional assignments. Normally, payments for such stay are settled against quarterly bills raised by
the company.
(iv) Conclusion:- No disqualification arises as the services availed are in ordinary course of business of
client and cannot be considered as business relationship. (It is assumed that outstanding does not
exceed ₹5 Lacs.)

106 Ab Audit Hoga Sabse Scoring - By CA. Sarthak Niraj Jain


Company Audit

7. Implications of Relatives' Securities Holding on the Appointment of the Auditor


Mr. Y, a practising Chartered Accountant, has been appointed as an auditor of M/s Z Ltd on 12th June, 2021
for the year ended 31st March, 2022. The following persons have done following transactions in securities of
M/s Z Ltd.:
 Daughter of Mr. Y: Purchase of Securities on 10th September, 2021 of face value of ` 45,000(market value
` 90,000)
 Husband of daughter of Mr. Y: Purchase of Securities on 10 th December 2021 of face value of ` 90,000
(market value ` 1,90,000)
 All the above securities were sold on 10th March, 2022 for ` 3,00,000. Discuss the implications of the
above on the appointment of Mr. Y. (May-2019-Old)
Ans. (i) Provision:- As per Section 141(3)(d)(i) of the Companies Act, 2013 along with Rule 10 of the
Companies (Audit and Auditors) Rule, 2014, a person shall not be eligible for appointment as an
auditor of a company,
 Who, or his relative or partner
 Is holding any security of or interest in the company or
 Its subsidiary, or of its holding or associate company or a subsidiary of such holding company.
 Exception:- Provided that the relative may hold security or interest in the company of face value
not exceeding rupees one lakh.
 Section 141(4) provides that an auditor who incurs any of the disqualifications mentioned
in section 141(3) after his appointment, he shall vacate his office as such auditor. and such
vacation shall be deemed to be a casual vacancy in the office of the auditor.
 Further the definition of relative also includes daughter and a daughter's husband. Both
are covered in the definition of relative as defined by the Companies Act 2013.

FAST
 Thus, the disqualifications will be applicable as the relative/s are holding securities of face
value of more than ` 100,000 and market value is not important.
 It is also to note that in the event of acquiring any security or interest by a relative above the

CA.Sarthak Niraj Jain


threshold prescribed, the corrective action to maintain the limits as specified above can
be taken by the auditor within 60 days of such acquisition or interest. The same has
however not been done.
(ii) In the instant case:- Daughter of Mr. Y purchased the securities on 10th September 2021 of face
value of rupees 45,000 and husband of daughter of Mr. Y purchased the securities on 10th of
December, 2021 of face value of rupees 90,000. Aggregating the value of holding of securities
exceeds the limits mentioned in proviso to section 141(3)(d)(i) i.e. rupees 1,00,000.
Further, corrective action taken by Husband of Daughter of Mr. Y on 10th March, is also not in
accordance with prescribed grace period of 60 days.
(iii) Conclusion:- Therefore, CA. Y will be disqualified for appointment as an auditor of M/s. Z Ltd. as
per section 141(3)(d)(i) and he shall vacate his office.
8. Ceiling Limit for Holding Company Audits
KSY & Company Chartered Accountants is an audit firm having two partners CA K and CA Y.KSY & Company
is already holding appointment as auditors of 36 public companies.
KSY & Company seeks your advice in the following situations:
(a) KSY & Company has been offered the appointment as Auditors of 7 more Private Limited Companies.
Of the seven, one is a company with a paid up share capital of ` 150 crores, five are "Small companies"
as per the Act and one is a "Dormant Company”.
(b) Would your answer be different, if out of those 7 Private Companies, 3 Companies have paid up capital
of ` 90 crores each? Answer in light of CA, 2013 (May-2016)
Ans. (i) Provision:- As per section 141(3)(g) of the Companies Act, 2013,a person shall not be eligible for
appointment as an auditor if he:
 Is in full time employment elsewhere or

CA Final Audit - By CA. Sarthak Niraj Jain 107


Company Audit

 A person or a partner of a firm holding appointment as its auditor if such person or partner is at
the date of such appointment or reappointment holding appointment as auditor of more than
twenty companies other than:
 One person companies,
 Dormant companies,
 Small companies and
 Private companies having paid-up share capital less than ` 100 crore.
 Which has not committed default in filing its financial statements under section 137 or annual
return under section 92 of the Companies Act with the Registrar.
 In the case of firm of auditors:- It has been further provided that specified number of
companies shall be construed as the number of companies specified for every partner of the firm
who is not in full time employment elsewhere.
(ii) In the instant case:- KSY & Co. is an audit firm having two partners, namely, CA. K and CA. Y. The
total number of company audits that can be accepted by the firm is 40 (2 partners x 20 companies
each partner). However, the firm is already holding appointment as auditors of 36 public companies.
Thus, the remaining number of audits that can be accepted by the firm is of 4 more companies.
(iii) Conclusion:-
 KSY & Co. can accept audit of all 7 Private companies, because 5 small companies and one
dormant company will not be considered for the purpose of ceiling limit. Total number of audits
after acceptance of all seven audits remains at 37.
 Answer will remain same, as the private companies having paid up capital less than 100
Crores are not considered for the purpose of ceiling limit. Total number of audits after acceptance
of all seven audits remains at 40 assuming that other four companies having paid up capital in
excess of ` 100 Crores.

9.
FAST
SJ Note: The answer has been given assuming none of the partners hold any company audits in their
personal capacity or as partners with another firm.
Disqualification Due to Holding of Securities

CA.Sarthak Niraj Jain


Mr. Pratiq, a practicing Chartered Accountant, has been appointed as an auditor of Opus Ltd. He is holding
securities of the company having face value of ` 89,000 only.
(a) You are required to state, whether Mr. Pratiq is qualified to be appointed as an auditor of Opus Ltd.
(b) Would your answer be different, if instead of Mr. Pratiq; Mr. Quresh, the step- father of Mr. Pratiq, is
holding the securities?
Ans. (i) Provision:- As per Section 141(3)(d)(i) of the Companies Act, 2013 along with Rule 10 of the
Companies (Audit and Auditors) Rule, 2014, a person shall not be eligible for appointment as an
auditor of a company,
 Who, or his relative or partner
 Is holding any security of or interest in the company or
 its subsidiary, or of its holding or associate company or a subsidiary of such holding
company.
 Exception:- Provided that the relative may hold security or interest in the company of face
value not exceeding rupees one lakh.
 Section 141(4) provides that an auditor who incurs any of the disqualifications mentioned
in section 141(3) after his appointment, he shall vacate his office as such auditor. and such
vacation shall be deemed to be a casual vacancy in the office of the auditor.
(ii) Definition of Relative :- The term “relative” has been defined under the Companies Act, 2013 which
means anyone who is related to another as members of a Hindu Undivided Family;
 Husband and wife;
 Father (including step- father),
 Mother (including step-mother),

108 Ab Audit Hoga Sabse Scoring - By CA. Sarthak Niraj Jain


Company Audit

 Son (including step- son),


 Son’s wife, Daughter, Daughter’s husband,
 Brother (including step- brother),
 Sister (including step- sister).
(iii) Conclusion:- In the present situation,
 Mr. Pratiq is holding securities in Opus Ltd., which is not allowed as per the provisions of
section 141(3)(d)(i) of the Act. Therefore, Mr. Pratiq will be disqualified to be appointed as an
auditor of Opus Ltd.
 Mr. Quresh, the step-father of Mr. Pratiq, is holding the securities in Opus Ltd. It may be noted that
step-father is included in the definition of the term “relative” as per the Companies Act, 2013.
Further, proviso to section 141(3)(d)(i) of the Act allows a relative of the auditor to hold securities
in the company of face value not exceeding of ` 1,00,000.Here, Mr. Quresh is holding securities for
face value of ` 89,000 which is below the limit as prescribed under the said proviso. Therefore,
Mr. Pratiq will not be disqualified to be appointed as an auditor of Opus Ltd.
5.4 - Section-142 Remuneration of Auditors
10. Restricting Scope of Audit
Y, is the auditor of X Pvt. Ltd. in which there are four shareholders only, who are also the Directors of the
company. On account of bad trade and for reducing the expenses in all directions, the directors asked Y to
accept a reduced fee and for that he has been offered not to carry out such full audit as he has done in the
past. Y accepted the suggestions of the directors.
Ans. (i) Provision:
 Section 142: Remuneration of Auditor

FAST
 Further, remuneration is a matter of arrangement between the auditor and the shareholders.
 Section 142 specifies the remuneration of an auditor, shall be fixed by the company in
general meeting or

CA.Sarthak Niraj Jain


 In such manner as the company in general meeting may determine.
 His duties may not necessarily commensurate with his remuneration.
 Section 143: Power & Duties of Auditor
 Auditor’s duties are governed by the provisions of Section 143 of Companies Act 2013,
 Which cannot be restricted either by the director or even by the entire body of
shareholders?
(ii) Conclusion:- Y, therefore, should not accept the suggestions of the directors regarding the scope of
the work to be done. Even if Y accepts the suggestions of the directors regarding the scope of work to
be done, it would not reduce his responsibility as an auditor under the law. Under the
circumstances, Y is violating the provisions of the Companies Act, 2013.
5.5 - Section-143 Power & Duties of Auditor
11. Audit Report - Inquiry into Propriety Matters
While doing the audit, X, the Statutory Auditor of ABC Ltd. observes that certain loans and advances were
made without proper securities, certain trade receivables and trade payables were adjusted inter se, and
personal expenses were charged to revenue. Comment. (Nov-2010, RTP-May-2015, Nov-2019-New)
Ans. (i) Provision:- Section 143(1) of the Companies Act, 2013 requires the auditor to conduct inquiry into
certain matters and if the auditor finds answer of any of these matters in adverse, auditor is
required to report, otherwise no reporting is required.
(ii) In relation to observations stated in the question, auditor should inquire as follows:
 Clause (a) of Section 143(1) requires the auditor to inquire “Whether loans and advances made
by the company on the basis of security have been properly secured and whether the terms on
which they have been made are prejudicial to the interests of the company or its members”.

CA Final Audit - By CA. Sarthak Niraj Jain 109


Company Audit

 Clause (b) of section 143(1), requires the auditor to inquire “whether transactions of the
company which are represented merely by book entries are prejudicial to the interests of the
company”.
 Clause (e) of section 143(1), requires the auditor to inquire “Whether personal expenses have
been charged to revenue account”.
(iii) Audit Procedure:-
 If the auditor finds that the loans and advances have not been properly secured, he may enter
an adverse comment in the report without modifying opinion on financial statements if the
loans and advances are properly described and presented in terms of Part I of Schedule III to
the Companies Act.
 If relation to his observation regarding inter se adjustment of trade receivables and trade
payables, being a book entry, auditor should have inquired into the legitimate interests of the
company. If appears prejudicial, he may enter adverse comment in the report.
 Regarding charging of personal expenses to revenue account auditor should inquire whether
such expenses are incurred on the basis of the company’s contractual obligations, or in
accordance with accepted business practice. If personal expenses incurred by the company are
not covered by contractual obligations or by accepted business practice and charged to revenue
account, it would be the duty of the auditor to report thereon.
(iv) Conclusion:- Mr. X, the statutory auditor of ABC Ltd., needs to enquire in light of above provisions,
as a result of the enquiries if he is satisfied then there is no further duty to report on these matters.
12. Enquiring into Propriety Matters of the Company as required by Section 143(1)
Mr. Raj, the engagement partner of R.O.K. & Co., in connection with statutory audit of Waria Ltd., had assigned

FAST
the responsibility of enquiring into propriety matters of the Company as required by section 143(1) of the
Companies Act, 2013, to Mr. Samay, an engagement team member. Mr. Samay while making such enquiries,
was having following queries, as tabulated below, which he ought to get resolved from Mr. Raj, as follows:-

1 CA.Sarthak Niraj Jain


Sr. No. Query of Mr. Samay
What documents to be seen in case of loan given by the company in lieu of hypothecation of
goods from lender as a security for the purpose of reporting as per clause (a) of section 143(1)
of the Companies Act, 2013?
2 What shall be the cost of Debentures and Bonus Shares sold by the company for which the cost
is not ascertainable for the purpose of reporting as per clause (c) of section 143(1) of the
Companies Act, 2013?
3 Whether the shares allotted by Waria Ltd. against a loan taken by it from a NBFC can be
considered to be allotted for cash for the purpose of reporting as per clause (f) of section 143(1)
of the Companies Act, 2013?
Assuming that you are Mr. Raj the engagement partner, please provide answer to the queries of Mr. Samay?
(MTP-May-2021)
Ans. As a engagement partner I will provide answer to queries as follows:
Sr. No. Query of Mr. Samay Response to Query
1 What documents to be seen in case of Mr. Samay should see deed of Hypothecation or
loan given by the company in lieu of other document creating the charge, together
hypothecation of goods from lender as with a statement of stocks held at the balance sheet
a security for the purpose of reporting date in order.
as per clause (a) of section 143(1) of
the Companies Act, 2013?
2 What shall be the cost of Debentures (i) For Debentures sold: Where the cost of
and Bonus Shares sold by the company debentures sold is not ascertainable, the book
for which the cost is not ascertainable value thereof at the date of sale may be treated
as the cost for the purposes of this clause.

110 Ab Audit Hoga Sabse Scoring - By CA. Sarthak Niraj Jain


Company Audit

for the purpose of reporting as per (ii) For Bonus Shares sold: When bonus shares
clause (c) of section 143(1) of the are received, the number of shares in the
Companies Act, 2013? portfolio would be increased by the bonus
shares while the cost of the total portfolio
would remain the same as before. The result
would be that the average cost per unit of the
total holding would come down
proportionately. The usual accounting practice
for apportioning the cost of a part of the total
holding on the sale thereof is to take it at its
average cost.
3 Whether the shares allotted by Waria The law on the subject has hitherto been that, where
Ltd. against a loan taken by it from a the consideration for the issue of shares is an
NBFC can be considered to be allotted adjustment against a bona fide debt payable in
money on demand by the company, the shares are
for cash for the purpose of reporting as
deemed to have been subscribed in cash
per clause (f) of section 143(1) of the
(vide the decision in Spargo’s Case – 1873, 8, Ch.
Companies Act, 2013? A. 407). According to the legal opinion obtained by
the ICAI, the expression “shares allotted for cash”
may also include shares allotted against a debt.
Therefore, in cases which are covered by the
decision in Spargo’s case, no comment is required
by the auditor, even though the company may have
in the Return of Allotment under Section 75,
shown such shares as allotted against adjustment of
a debt.

FAST Thus, the shares allotted by Waria Ltd. against a


loan taken by it from a NBFC can be considered
to be allotted for cash.
13.
CA.Sarthak Niraj Jain
Audit of Government Companies
What are the relevant sections of the Companies Act, 2013 and steps involved in auditor of Government
Companies? (Nov-2016, MTP-May/Nov-2018, MTP-May-2019, Nov-2019-New)
OR
Ceta Ltd is a company in which 58% of the paid up share capital is held by Rajasthan Government. The
company is engaged in the business of providing consultancy services in relation to construction projects.
The audit of the financial statements of Ceta Ltd for the financial year ended 31st March 2022 got completed
with lot of intervention of Comptroller & Auditor General of India, wherein C&AG was giving directions to the
auditors on the manner in which audit should be conducted in respect of certain areas. Further, it also
received comments from C&AG on the audit report of the auditors. Ceta Ltd. is seeking advice to go against
C&AG so that they can avoid unnecessary interference of C&AG. You are required to advise Ceta Ltd. with
respect to role of C&AG in the audit of a Government company. (MTP-May-2020)
Ans. Relevant Sections and Steps involved in Audit of Government Companies: Section 143(5), 143(6) and
143(7) of the Companies Act, 2013 are relevant sections in case of Audit of Government Companies.
The following steps are involved in the audit of government companies:
(i) Appointment of Auditors under Section 139(5) and 139(7) read with section 143(5) of the
Companies Act, 2013 - Statutory auditors of Government Company are appointed or re-appointed
by the Comptroller and Auditor General of India. The C&AG may direct the appointed auditor the
manner in which the accounts of the Government company are required to be audited and thereupon
the auditor so appointed shall submit a copy of the audit report to the Comptroller and Auditor-
General of India which, among other things, include the directions, if any, issued by the Comptroller
and Auditor-General of India, the action taken thereon and its impact on the accounts and financial
statement of the company.

CA Final Audit - By CA. Sarthak Niraj Jain 111


Company Audit

(ii) Supplementary audit under section 143(6)(a) of the Companies Act, 2013 - The Comptroller and
Auditor-General of India shall within 60 days from the date of receipt of the audit report have a
right to conduct a supplementary audit of the financial statement of the company by such person or
persons as he may authorize in this behalf; and for the purposes of such audit, require information or
additional information to be furnished to any person or persons, so authorised, on such matters, by
such person or persons, and in such form, as the Comptroller and Auditor-General of India may direct.
(iii) Comment upon or supplement such Audit Report under section 143(6)(b) of the Companies Act,
2013 - Any comments given by the Comptroller and Auditor-General of India upon, or supplement
to, the audit report shall be sent by the company to every person entitled to copies of audited
financial statements under section 136(1) of the said Act i.e. every member of the company, to
every trustee for the debenture-holder of any debentures issued by the company, and to all persons
other than such member or trustee, being the person so entitled and also be placed before the annual
general meeting of the company at the same time and in the same manner as the audit report.
(iv) Test audit under section 143(7) of the Companies Act, 2013 - Without prejudice to the provisions
relating to audit and auditor, the Comptroller and Auditor- General of India may, in case of any
company covered under section 139(5) or (7) of the said Act, if he considers necessary, by an order,
cause test audit to be conducted of the accounts of such company and the provisions of the
Comptroller and Auditor-General's (Duties, Powers and Conditions of Service) Act, 1971, shall
apply to the report of such test audit.
14. Branch Audit
D. Ltd., a Delhi based company having turnover of ` 25 crores, has a branch at USA having a turnover of ` 10
lakhs (as converted from US dollars). The area where the branch office is located in USA was severely
affected by storms and the office along with all accounting records was completely destroyed. Due to the

FAST
unavailability of records, the financial statements of D. Ltd, for the financial year 2021-22 did not include the
figures pertaining to the said branch. As the statutory auditor of D. Ltd., how will you report on the same?
(Nov-2017)
Ans. (i)
CA.Sarthak Niraj Jain
Provision:- As per section 143(8) of the Companies Act, 2013 if a company has a branch office, the
accounts of that office shall be audited either by:
 The auditor appointed for the company (herein referred to as the company's auditor) under this
Act or
 Any other person qualified for appointment as an auditor of the company under this Act and
appointed as such under section 139, or
 Where the branch office is situated in a country outside India, the accounts of the branch office
shall be audited either by
 the company's auditor or
 by an accountant or
 by any other person duly qualified to act as an auditor of the accounts of the branch office in
accordance with the laws of that country.
(ii) In the given case:- D Ltd. is a Delhi based company, having total turnover of ` 25 crores. The
company is having a branch office in USA (having a turnover of rupees 10 lakhs i.e. as converted
from US dollars) in an area which is recently affected by storm and the office along with all
accounting records was completely destroyed. Due to unavailability of records, the financial
statements of D Ltd. for the financial year 2021 -22 did not include the figures pertaining to the said
branch.
(iii) Conclusion:- Under such a circumstance beyond the control of the entity when the entity’s
accounting records have been destroyed the auditor’s opinion has to be modified. The auditor has
also to mention in his report the effect on the financial statements due to non-inclusion of financial
data pertaining to the branch.

112 Ab Audit Hoga Sabse Scoring - By CA. Sarthak Niraj Jain


Company Audit

5.6 - Section 146 Auditor’s Duty to attend General Meeting


15. Auditor’s Duties w.r.t. Subsequent Events
An auditor became aware of a matter regarding a company, only after he had issued his audit opinion. Had he
become aware of the same prior to his issuing the audit report, he would have issued a different opinion.
(Further refer SA 560)
Ans. (i) Provision:- As per Section 146 of the Companies Act, 2013, the auditor of a company are under an
obligation to attend any general meeting of the company and not only those meetings at which the
accounts audited by them are to be presented and discussed.
(ii) Analysis of the provision:
 Provision required the auditors of a company to attend the general meeting of the company
unless otherwise exempted by the company.
 Auditor shall have the right to be heard at such meeting on any part of the business which
concerns him as auditors.
 The discovery of a fact after issuance of the financial statements that existed at the date of the
audit report which would have caused the revision of the audit report, requires the auditor to
bring this to the notice of shareholders.
 Further SA 560 “Subsequent Event” also prescribes the procedures which the auditor is required
to perform.
(iii) Conclusion:-It will be advisable for the auditor to attend the meeting with a view to bringing to the
notice of the shareholders the matter which came to his knowledge subsequent to his signing the
report and perform procedures are per the requirement of SA 560.
5.7 - Section 147 Penalty in case of Contravention
16.
FAST
Contravention by Management & Auditor in case of Appointment
AB & Co. were appointed auditors for NOME Limited, a listed company, for the term of two five consecutive
years from 2011-12, 2012-13, 2013-14, 2014-15, 2015-16, 2016-17, 2017-18, 2018-19, 2019-20, 2020-21 As

CA.Sarthak Niraj Jain


per the provision of the section 139(2)(b) “No listed company or a company belonging to such class or classes
of companies as may be prescribed, shall appoint or re-appoint an audit firm as auditor for more than two
terms of five consecutive years”.
Hence, Management of NOME Limited reached out (based on the recommendation of Audit Committee) to BCD
& Co. for their nomination as the appointment of Statutory Auditor for the financial year 2021-22. However,
BCD & Co. did not provide any written consent to such appointment neither they provided a certificate that
the appointment, if made, shall be in accordance with the conditions laid in the Act and Rules therein.
Still the management went ahead and proposed an appointment in AGM and BCD & Co. were appointed as an
auditor for the financial year 2021-22. Post appointment, those charged with governance identified that
majority of the partners in the BCD & Co. are same which were there in AB & Co. Now, fearing the
contravention of the provision of Companies Act, 2013. Management, on guidance of those charged with
governance, decided to file a complaint with tribunal under section 140(5) of the Companies Act against
statutory auditors.
You are required to guide the BCD & Co. regarding the contravention of the provisions of the Companies Act,
2013 with respect to appointment of Auditor. (RTP-Dec-2021)
Ans. (i) Provisions:
 As per section 139(1) of the Companies Act, 2013, every company shall, at the first annual
general meeting, appoint an individual or a firm as an auditor who shall hold office from the
conclusion of that meeting till the conclusion of its sixth annual general meeting and thereafter
till the conclusion of every sixth meeting and the manner and procedure of selection of auditors by
the members of the company at such meeting shall be such as may be prescribed.
 It may be noted further that before such appointment is made, the written consent of the
auditor to such appointment, and a certificate from him or it that the appointment, if made, shall
be in accordance with the conditions as may be prescribed, shall be obtained from the auditor.

CA Final Audit - By CA. Sarthak Niraj Jain 113


Company Audit

 It may also be noted that the certificate shall also indicate whether the auditor satisfies the
criteria provided in section 141 of the Companies Act, 2013.
 Further, as per section 139(2), No listed company or a company belonging to such class or
classes of companies as may be prescribed, shall appoint or re-appoint:
(a) an individual as auditor for more than one term of five consecutive years; and
(b) an audit firm as auditor for more than two terms of five consecutive years.
 It may also be noted further that as on the date of appointment no audit firm having a
common partner or partners to the other audit firm, whose tenure has expired in a company
immediately preceding the financial year, shall be appointed as auditor of the same company for a
period of five years:”
(ii) In the current case:
 While appointing the auditors of the company a written consent of the auditor to such
appointment was not obtained.
 Moreover a certificate from him that the appointment if made shall be in accordance with the
conditions laid down in the Act and Rules was also not obtained.
 Further, majority of the partners of AB & Co. were partners in BCD & Co. AB & Co. already
served two terms of five consecutive years i.e., from 2012-13 to 2020-21 as a statutory auditor of
the company.
(iii) Conclusion:
 Hence, BCD & Co. were not eligible to be appointed as an auditor of NOME Limited as all
partners of BCD & Co are partner of AB & Co. who have already served two terms of five
consecutive years as an auditor of NOME Limited.
 Since, before the appointment of Statutory Auditor, the management should have obtained the

FAST
required certification and written consent from BCD & Co., therefore, in this case both, the
management and the auditors have contravened the provision of the Companies Act, 2013
(iv) Penalty- (Section 147)
 As per section 147 of Companies Act will be applicable i.e. if any of the provisions of sections
CA.Sarthak Niraj Jain
139 to 146 (both inclusive) is contravened,
 the company shall be punishable with fine which shall not be less than twenty-five thousand
rupees but which may extend to five lakh rupees and
 every officer of the company who is in default shall be punishable with fine which shall not be
less than ten thousand rupees but which may extend to one lakh rupees.
 If an auditor of a company contravenes any of the provisions of section 139, section 144 or
section 145, the auditor shall be punishable with fine which shall not be less than twenty-five
thousand rupees, but which may extend to five lakh rupees or four times the remuneration of
the auditor, whichever is less.
It may be noted that if an auditor has contravened such provisions knowingly or willfully with the
intention to deceive the company or its shareholders or creditors or tax authorities, he shall be punishable
with imprisonment for a term which may extend to one year and with fine which shall not be less than
fifty thousand rupees, but which may extend to twenty-five lakh rupees or eight times the remuneration
of the auditor, whichever is less.
5.8 - Section 148 Cost Audit & Cost Audit Rules
17. Cost Audit Rules
Pearl Ltd. is an exporter of precious and semi-precious stones. The turnover of the company is ` 150 crore,
out of which ` 105 crore is from export business and remaining ` 45 crore from domestic sales. Amount
received from export business is all in foreign currency. Directors of Pearl Ltd. are of the opinion that cost
audit is not applicable to their company as maximum revenue has been generated from export business. Give
your opinion. (May-2019-New, Study Material)

114 Ab Audit Hoga Sabse Scoring - By CA. Sarthak Niraj Jain


Company Audit

Ans. (i) Provisions:- Provisions relating to cost audit are governed by section 148 of the Companies Act,
2013 read with the Companies (Cost Records and Audit) Rules, 2014.
(a) Applicability of Cost Audit:- Rule 4 of Companies (Cost Records and Audit) Rules, 2014
requires audit of cost records in case of non-regulated sector industries if
 Annual turnover from all products and services in immediately preceding financial year
is ` 100 Cr. or more and
 The turnover of individual product or service is ` 35 Cr. or more.

(b) Exception:- The requirement for cost audit shall not be applicable to a company
 Whose revenue from exports, in foreign exchange, exceeds 75 per cent of its total
revenue,
 Which is operating from SEZ and
 Which is engaged in the generation of electricity for captive consumption through
captive generating plant.
(ii) In the instant case:- Pearl Ltd. is an exporter of precious and semi-precious stones and the turnover
of the companyis rupees 150 crore out of which rupees 105 crore i.e. 70% is from export business
and remaining rupees 45 crore i.e. 30% from domestic sales. It is neither operating from SEZ nor
involved in captive power generation.
(iii) Conclusion:- Thus, opinion of director is not tenable as revenue from exports in foreign
exchanges is below prescribed limit. Therefore, cost audit is applicable on Pearl Ltd. as per Rule 4 of
the Companies (Cost Records and Audit) Rules, 2014. Pearl Ltd. has to appoint costauditor to get the
cost accounts of the company audited.
18.
FAST
Cost Audit Rules
Mithas Ltd Is a top sugar manufacturer and exporter in India operating from Noida Specific Economic Zone,
Uttar Pradesh Its revenue from sale/export for the current year is given below:

CA.Sarthak Niraj Jain


Sate within India
Sale outside India (Export)
Total Revenue
`153 lakhs
`357 lakhs
`510 lakhs
Mr X, the statutory auditor of Mithas Ltd, is of the view that the company is mandatorily required to include
cost records in their books of account and consequently conduct cost audit He also suggested the name of his
friend, who is a Cost accountant, for the purpose of such cost audit However, the 'management is of the view
that the company neither required including cost records in their books of account nor conduct cost audit
Being an expert in cost records and audit rules, you are required to guide the management in this regard
(RTP-Nov-2015)
Ans. (i) Provision: - The provisions relating to cost records and audit are governed by section 148 of the
Companies Act, 2013 read with the Companies (Cost Records and Audit) Rules, 2014. The audit
conducted under this section shall be in addition to the audit conducted under section 143.
(ii) Rule 3 of the Companies (Cost Records and Audit) Rules, 2014: - Provides the classes of
companies, engaged in the production of goods or providing services, required to include cost
records in their books of account. The said rule has divided the list of companies into regulated
sectors and non-regulated sectors.
Company belonging to sugar industry is one of the types of companies prescribed under the
regulated sectors. However, the requirement for cost audit under these rules shall not be
applicable to a company which is covered under Rule 3, and,
(1) whose revenue from exports, in foreign exchange, exceeds 75 percent of its total revenue; or
(2) which is operating from a special economic zone.
(iii) In the given case: -Mithas Ltd., a sugar manufacturer and exporter in India, is operating from
Noida Specific Economic Zone, Uttar Pradesh.

CA Final Audit - By CA. Sarthak Niraj Jain 115


Company Audit

(iv) Conclusion:- Therefore, Mithas Ltd. is required to include cost records in their books of account in
accordance with Rule 3 of the Companies (Cost Records and Audit) Rules, 2014. However, the
company is not required to conduct cost audit as it is operating from a special economic zone. The
facts given on revenue are not relevant here.
19. Cost Auditor - Certificate
On 30.08.2021, the Board of SRE Ltd. proposed to appoint Mr. Elex, a Cost accountant, for conducting cost
audit for the financial year 2021-22. The management came to know about the certificate which needs to be
obtained from the auditor before such appointment is made. However, the management is unaware about
what certification is required from the auditor. Please guide.
Ans. As per Rule 6 of Companies (Cost Records and Audit) Rules, 2014:- Companies required to get the cost
records audited, shall within 180 Days of the commencement of every financial year, appoint a cost
auditor. Before such appointment is made, the written consent of cost auditor to such appointment and a
certificate from him or it shall be obtained.
The Cost Auditor appointed shall submit certificate that-
(a) The individual or the firm, as the case may be, is eligible for appointment and is not disqualified
for appointment under the Act, the Cost and Works Accountants Act, 1959 and the Rules or
regulations made there under.
(b) The individual or the firm, as the case may be, satisfies the criteria provided in Sec. 141, so far as
may be applicable.
(c) The proposed appointment is within the limits laid down by or under the authority of the Act.
(d) The list of proceedings against the cost auditor or audit firm or any partner of the audit firm
pending with respect to professional matters of conduct, as disclosed in the certificate, is true and

Notes
FAST
correct.

CA.Sarthak Niraj Jain

116 Ab Audit Hoga Sabse Scoring - By CA. Sarthak Niraj Jain


Company Audit

UNIT-2 : BOOK OF ACCOUNTS (SECTION 128 TO 138)


5.9 -Section 128 Books of Account
20. Preparation of Financial Statements
Ganga-Kaveri Project Ltd. was incorporated on 1.7.2022. During the year ended 31.3.2023 there was no
manufacturing or trading activity except raising of share capital, purchase of land, acquisition of plant and
machinery and construction of factory sheds. Therefore the Chief Accountant of the company contends that
for the relevant year there was no need to prepare a statement of profit or loss or any other statement except
a Balance Sheet as at 31.3.2023. Comment as an auditor. (Date restated)
Ans. (i) Provision:- Section 128 of the Companies Act, 2013 requires
 Every company to prepare its financial statements for every financial year
 Which gives a true and fair view of the state of the affairs of the company and
 Such books shall be kept on accrual basis, and
 According to the double entry system of accounting.
(ii) Financial Statement definition:- As per section 2(40) of the Companies Act, 2013, includes
 balance-sheet,
 statement of profit and loss,
 cash flow statement,
 a statement of change in equity, if applicable and explanatory note as annexure.
(iii) In the given case:- Ganga-Kaveri Project Ltd. did not carry any manufacturing or trading activity
except raising of share capital, purchase of land, acquisition of plant and machinery, etc.
Though the company did not carry any manufacturing or trading activity, the company has carried on

FAST
certain activities like construction of factory shed, acquisition of plant and machinery etc. In such a
case, it is necessary to provide for depreciation and other expenses.
(iv) Conclusion:- The mere fact that there was no manufacturing or trading activity cannot be the basis

21.
CA.Sarthak Niraj Jain
for not preparing the Statement of Profit and Loss. Therefore, the contention of the Chief
Accountant is not correct.
Right to Lien
Write short notes on - Auditor's right to Lien as per Companies Act, 2013. (May-2017)
Ans. (i) Auditor’s Right to Lien & Books of accounts
 Definition:- Lien refers to the right of a person for lawful possession of somebody’s else property
on which he has worked.
 Right of lien is exercised for non-payment of his dues for the work done.
 The auditors can exercise right of lien on the client’s books and documents in his possession for
non-payment of fees by the client, for the work done on the books and documents.
 In respect of auditor exercising the lien, The Institute of Chartered Accountants of England and
Wales has expressed a similar view subject to the following conditions:
(a) Documents must belong to the client who owes the money,
(b) These documents must come to the possession of the auditor on the client’s authority.
(c) The auditor can retain such documents, only if he has done work on such documents, on
which fees have not been paid.

(ii) As per Provision:- Under section 128 of the Act, books of account of a company must be kept at the
registered office. These provisions ordinarily make it impracticable for the auditor to have
possession of the books and documents. The company provides reasonable facility to auditor for
inspection of the books of account by directors and others authorised to inspect under the Act.

(iii) As per Ethical Standard Board of ICAI:- A chartered accountant cannot exercise lien over the
client documents/records for non-payment of his fees.

CA Final Audit - By CA. Sarthak Niraj Jain 117


Company Audit

(iv) Conclusion:- Taking an overall view of the matter, it seems that though legally, auditor may exercise
right of lien in cases of companies, it is mostly impracticable for legal and practicable constraints.
His working papers being his own property, the question of lien, on them does not arise.
5.10 -Section 129 Financial Statements
22. Applicability of AS/Ind AS
As an auditor of a company registered under section 8 of the Companies Act, 2013 you find that as per the
notification of the ministry of Corporate Affairs regarding applicability of Indian Accounting Standards (Ind-
AS). The company has to prepare its financial statements for the year ended 31st March. 2022 under Ind-AS.
The management of the company is however of the strong view that being a section 8 company having
charitable objects. Ind-AS cannot apply to the company. The financial statements are therefore prepared by
the management under the earlier GAAP and a note for the same is given in the financial statements. How
would you report on these financial statements? (Nov-2017), (Nov-2019)
Ans. (i) Provision:- Section 129(1) of the Companies Act, 2013, governs the requirements to be satisfied by
financial statements. The provisions there under which should be complied with are:
 Financial statements shall, give a true and fair view of the state of affairs of the company or
companies as at the end of financial year, comply with the notified accounting standards under
section 133 and be in such form or forms specified in Schedule III to the Companies Act, 2013
and
 The items contained in such financial statements shall be in accordance with the accounting
standards.
(ii) AS notified by Central Government:- As per section 133 of the Companies Act, 2013, the Central
Government has notified Companies (Indian Accounting Standards) Rules, 2015 dated 16.02.2015

FAST
in exercise of the powers conferred by section 133.
(iii) Companies (Indian Accounting Standards) Rules, 2015:-
 The said rules list the Indian Accounting Standards (Ind AS) and the class of companies required

CA.Sarthak Niraj Jain



to comply with the Ind AS while preparation of their financial statements.
Here, it may be noted that the companies covered under Section 8 are required to comply the
provisions of the Companies Act, 2013, unless and until any exemption is provided.
 Companies registered under Section 8 are not exempted from the requirements of section 133
and section 129 of the Companies Act, 2013.

(iv) Conclusion:- In the given case, only contention of management that being a section 8 company
having charitable object, Ind-AS cannot apply to the company, therefore financial statements
prepared under the earlier GAAP and a note for the same is given, is not tenable.
However, the auditor is required to ensure the applicable monetary limits w.r.t Ind-AS and need to advise
the management to prepare the financial statements as per Ind-AS accordingly. In case of non-compliance
the auditor should report accordingly.
23. Audit report - Wrong anticipation of Tax Liability & Provision not made
ABC Ltd. has been dealing in tyres since 1995. The Company envisaged to expand its business and wanted to
manufacture the tyres besides trading. Accordingly, the machinery imported, installed and manufacturing
operations commenced. The Government also gave certain incentives like power subsidy, land acquisition
subsidy, etc. After 2 years of operations, Company received a notice from the Income Tax authorities to pay tax
on incentive received in the form of power subsidy. The demand notice was served for ` 150.00 Lakhs. The
Company, however filed an appeal with higher tax authorities against the demand and the matter is undecided
as on 31.03.2022.
Legal team of the Company anticipated that tax liability might mature. The Company has not made a provision
of anticipated tax liability. Considering the provisions of Companies Act, 2013, how an auditor of ABC Ltd.
should see this matter and report in audit report, if required? (July-2021-New)

118 Ab Audit Hoga Sabse Scoring - By CA. Sarthak Niraj Jain


Company Audit

Ans. The Council of the Institute of Chartered Accountants of India has taken note of the fact that there is a
practice prevalent whereby companies do not make provision for tax even when such a liability is
anticipated. It has expressed the view that on an overall consideration of the relevant provisions of law, non-
provision for tax (where a liability is anticipated) would amount to contravention of the provisions of
Sections 128 and 129 of the Companies Act, 2013. Accordingly, it is necessary for the auditor to qualify his
report and such qualification should bring out the manner in which the accounts do not disclose a “true and
fair” view of the state of affairs of the company and the profit or loss of the company.
Conclusion -Applying the above to the facts given in the question: - auditor should qualify his report.
An example of the manner in which the report on the balance sheet and the Statement of Profit and Loss
may be qualified in this respect is given below:
“The company has not provided for taxation in respect of its profits and the estimated aggregate amount of
taxation not so provided for is ` ............including.............for the Year ended on ..............To the extent of such non-
provision for the year, the profits of the Company for the financial year under report have been overstated and to
the extent of such aggregate non-provision, the reserves of the company appearing in the said balance sheet have
been over-stated and the current liabilities and provisions appearing in the said balance sheet have been
understated”
5.11 -Section 132 NFRA
24. Companies and Bodies Corporate governed by NFRA
M/s PC & Co., Chartered Accountants are the statutory auditors of various categories of companies and
bodies corporate. In exercise of the powers conferred under sub-sections (2) and (4) of section 132, of the
Companies Act, 2013 the Central Government made the National Financial Reporting Authority Rules, 2018
(NFRA Rules) (MCA Notification dated 13 November 2018). The audit firm seeks your guidance on the
applicability of those categories of companies and bodies corporate which are covered by NFRA Rules.

Ans. (a)
FAST (Nov-2020-New)
Companies whose securities are listed on any stock exchange in India or outside India;
(b)
CA.Sarthak Niraj Jain
Unlisted public companies
 having paid-up capital of not less than ₹500 crores or
 having annual turnover of not less than ₹1,000 crores or
 having, in aggregate, outstanding loans, debentures and deposits of not less than ₹500 crores
as on the 31st March of immediately preceding financial year;
(c) Insurance companies, banking companies, companies engaged in the generation or supply of
electricity, companies governed by any special Act for the time being in force or bodies corporate
incorporated by an Act in accordance with clauses (b), (c), (d), (e) and (f) of sub-section (4) of section 1
of the Act;
Explanation – Banking company includes ‘corresponding new bank’ as defined in respective acts.
(d) Any body corporate or company or person, or any class of bodies corporate or companies or
persons, on a reference made to the Authority by the Central Government in public interest; and
(e) A body corporate incorporated or registered outside India, which is a subsidiary or associate
company of any company or body corporate incorporated or registered in India as referred to in
clauses (a) to (d), if the income or net worth of such subsidiary or associate company exceeds 20% of
the consolidated income or consolidated net worth of such company or the body corporate, as the
case may be, referred to in clauses (a) to (d).
25. Applicability of NFRA
You are doing audit of unlisted public limited company, Perfect Limited As per National Financial Reporting
Authority Rules, 2018, NFRA has power to monitor and enforce compliance with accounting standards and
auditing standards, oversee the quality of service under sub-section (2) of section 132 or undertake
investigation under sub-section (4) of such section, of the auditors. Discuss about the class of companies and
bodies corporate, where NERA is applicable. (July-2021-Old)

CA Final Audit - By CA. Sarthak Niraj Jain 119


Company Audit

Ans. As per NFRA rules, NFRA shall have power to monitor and enforce compliance with accounting standards
and auditing standards, oversee the quality of service under subsection (2) of section 132 or undertake
investigation under sub-section (4) of such section of the auditors of the following class of companies and
bodies corporate:
(a) Companies whose securities are listed on any stock exchange in India or outside India;
(b) Unlisted public companies having:
 Paid-up capital of not less than rupees five hundred crores or
 Having annual turnover of not less than rupees one thousand crores or
 Having, in aggregate, outstanding loans, debentures and deposits of not less than rupees five
hundred crores as on the 31st of March of immediately preceding financial year;
(c) Insurance companies, banking companies, companies engaged in the generation or supply of
electricity, companies governed by any special Act for the time being in force or bodies corporate
incorporated by an Act in accordance with clauses (b), (c), (d), (e) and (f) of section 1 (4) of the
Companies Act, 2013; “Explanation.- For the purpose of this clause, “banking company” includes
‘corresponding new bank’
(d) Any body corporate or company or person, or any class of bodies corporate or companies or persons,
on a reference made to the NFRA by the Central Government in public interest; and
(e) A body corporate incorporated or registered outside India, which is a subsidiary or associate
company of any company or body corporate incorporated or registered in India as referred to in clauses
(a) to (d) above, if the income or net-worth of such subsidiary or associate company exceeds 20% of the
consolidated income or consolidated net-worth of such company or the body corporate, as the case may
be, referred to in clauses (a) to (d) above.

26. FAST
Authentication of Financial Statements
5.12 - Section 134 Board Report

Discuss the provisions of Section 134 of the Companies Act, 2013 regarding the authentication of financial

Ans. (i)
CA.Sarthak Niraj Jain
statements. (May-2016)
Provision:- Section 134(1) of the Companies Act,2013 provides that the financial statements,
including consolidated financial statement, if any, shall be approved by the board of directors
before they are signed on behalf of the board at least by the following:
 Chairperson of the company where he is authorized by the Board; or
 By two directors out of which one shall be Managing Director, if any and the Chief Executive
Officer; and
 The Chief Financial Officer and the Company Secretary of the company, wherever they are
appointed.
(ii) In the case of a one person company:- The financial statement shall be signed by only one
director, for submission to the auditor for his report thereon.
(iii) Distribution/Circulation:- According to section 134(7) of the Companies Act, 2013, a signed copy of
every financial statement, including consolidated financial statement, if any, shall be issued,
circulated or published along with a copy each of -
 Any notes annexed to or forming part of such financial statement;
 The auditor’s report; and
 The board’s report
27. Explain the terms
(i) ‘Internal Financial Control’ and
(ii) ‘Internal Financial Controls over Financial Reporting’

120 Ab Audit Hoga Sabse Scoring - By CA. Sarthak Niraj Jain


Company Audit

Ans. (i) Internal Financial Control:


 Definition:- As per explanation to section 134(5) of Companies Act, 2013
The term Internal Financial Controls means the policies and procedures adopted by the
company for ensuring:
 Safeguarding of its assets,
 Orderly and efficient conduct of its business, including adherence to Company’ s policies,
 Prevention and detection of frauds and errors,
 Accuracy and completeness of the accounting records, and
 Timely preparation of reliable financial Information.

(ii) Internal Financial Control over Financial Reporting:


Guidance Note issued by ICAI:- Guidance Note on Audit of Internal Financial Controls Over
Financial Reporting”
 Definition
“A Process designed to provide reasonable assurance regarding the reliability of financial
reporting and the preparation of financial statements for external purposes In accordance
with generally accepted accounting principles”.
 Policy & Procedures
A Company’s internal financial control over financial reporting includes those policies and
procedures:
 Pertain to the maintenance of the records that, in reasonable detail, accurately and fairly
reflect the transactions and dispositions of the assets of the company
 It provides reasonable assurance that transactions are recorded as necessary to permit
FAST
preparation of financial statement in accordance with generally accepted accounting
principles, and those receipts and expenditures of the company are being made only in

CA.Sarthak Niraj Jain


accordance with authorizations of management and director of the company.
 Provide reasonable assurance regarding prevention or timely detection of unauthorized
acquisition, use or disposition of the company’s assets that could have a material effects of
the financial statement.
5.13 -Section 138 Internal Audit
28. Applicability of Provisions of Internal Audit
MWF (P) Ltd. is a private company having ` 50 lacs paid up capital during the preceding financial year. The
company had turnover of last three consecutive financial years, immediately preceding the financial year
under audit, being ` 210 crores, ` 205 crores and ` 200 crores (latest), but does not have any internal audit
system. In view of the management, internal audit system is not mandatory. You are required to state the
provisions related to applicability of internal audit as per the Companies Act, 2013 and comment upon the
contention of the management of the company.
Ans. (i) Provision:- As per section 138 of the Companies Act, 2013, following class of companies
(prescribed in Rule 13 of Companies (Accounts) Rules, 2014) shall be required to appoint an
internal auditor or a firm of internal auditors or Body Corporate,
(ii) Applicability (Rule 13): Companies required to appoint internal auditor / firm of internal
auditors:
(a) Every listed company;
(b) Every UNLISTED PUBLIC COMPANY having-
I. >= 25 Crores - Outstanding deposits at any point of time during the
preceding financial year; or
II. >= 50 Crores - Paid up share capital during the preceding financial year; or
III. > 100 Crores - Outstanding loans / borrowings from banks or public
financial institutions at any point of time during the
preceding financial year; or

CA Final Audit - By CA. Sarthak Niraj Jain 121


Company Audit

IV. >= 200 Crores - Turnover during the preceding financial year
(c) Every PRIVATE COMPANY having-
I. > 100 Crores - Outstanding loans / borrowings from banks or public
financial institutions at any point of time during the
preceding financial year; or
II. >= 200 Crores - Turnover during the preceding financial year
IFSC public company (unlisted) and IFSC Private Company Section 138 applicable with AOA
provide for it.
(ii) In the instant case:- MWF (P) Ltd. is a private company having turnover of ` 200 crores during the
preceding financial year which is covered under the limit prescribed for the applicability of provisions
on internal audit.
(iii) Conclusion:- Hence, the company has the statutory liability to appoint an Internal Auditor and
mandatorily conduct internal audit. Consequently, the contention of the management of the
company is not tenable.

Notes

FAST
CA.Sarthak Niraj Jain

122 Ab Audit Hoga Sabse Scoring - By CA. Sarthak Niraj Jain


Company Audit

UNIT - 3 : DIVIDENDS, LLP & OTHER PROVISIONS


5.14 – Section 123 Declaration of Dividend
29. Deferred Dividend
X Ltd. is good in profits, but suffers temporarily in liquidity. It proposes to declare dividend of 10% in annual
general meeting, but the Board proposes to defer payment of dividend by two months from the date of
annual general meeting by getting a resolution passed in AGM. Comment as an auditor.
Ans. Declaration of Dividends:
(i) Provision:- As per law, dividend once declared cannot be revoked.
(ii) Facts of the case:- X Ltd. has not declared the dividends so far because Board proposed to
recommend declaration of 10% dividend in annual general meeting but in view of liquidity
problem it proposes to defer the payment of dividend by two months from the date of annual
general meeting by getting a resolution passed in AGM.
(iii) Conclusion:- As per the question it is clear that the Board has only made a proposal and the same
has not been passed by the shareholders. Therefore, in such a case, X Ltd. may declare dividends
at a subsequent general meeting.
30. Decision to Rescind the Recommended Dividend
The Board of Directors of ACP Ltd. has recommended the dividend of 15% on paid up share capital of ` 450
crore for the year ended 31st March, 2022, at their meeting held on 1st of May, 2022 when the accounts for
the financial year 2021-22 were approved. The Board of Directors when they met on 7th July, 2022 for the
review of first quarter accounts, they realized that results were negative for the first quarter. Therefore, the
Board has decided to rescind their decision to recommend dividend. The notice for AGM to be held on
14.8.2022 was sent on 15th July, 2022 without any recommendation for dividend.

Ans. (i)
FAST
At the AGM, the members asked the management how they can rescind the declaration of dividend once
recommended. Comment. (May-2016, MTP-Nov-2018)
As per Regulation 80 of Table-F:- Dividend is firstly recommended by the Board. Thereafter, the

CA.Sarthak Niraj Jain


members in the Annual General Meeting (AGM) may declare the dividend by passing ordinary
resolution. The members may reducethe rate or amount recommended by the Board, but they
cannot increase it.
(ii) Provisions:-
 Section 123 of the Companies Act, 2013, provides that the dividend shall be declared or paid
by a company for any financial year out of the profits of the company for that year arrived at after
providing for depreciation in prescribed manner.
 As per section 127 of the Act, dividends once declared become the liability of the company and
must be paid within 30 days from the date of declaration. Any failure to do so attract a penalty
for the various persons associated with the management.
(iii) In the instant case:- Board of Directors of ACP Ltd. has recommended the dividend in their meeting.
Such dividend is not declared in AGM. Further, Board has decided to rescind the decision before the
date of Annual General Meeting.
(iv) Conclusion:- Thus, the dividend which is only recommended and not declared does not attract penal
provisions. Therefore, Board of Directors may rescind their decision to recommend dividend.
5.15 – Limited Liability Partnership & Other Provisionse
31. Maintenance of Books of Accounts & Audit of LLP
AMc LLP is a newly set up LLP (Limited Liability Partnership). The operations of the LLP have been picking
up and management is currently in the process of setting up processes and procedures in place. As per the
understanding of the management of the LLP, its accounts would not be required to be audited mandatory
because of its operations but still the management has decided that they would get the accounts audited
voluntarily. In this regard, the management would like to understand some of the aspects which they should
consider not only limited to audit but also about the maintenance of books of accounts as per the relevant
laws. Please advise. (RTP-May-2022)(RTP-May-2019)

CA Final Audit - By CA. Sarthak Niraj Jain 123


Company Audit

OR
Write a short note on: Statutory provisions as to Audit of Limited Liability partnerships and also briefly
describe the auditor’s duty regarding audit of LLP.
Ans. (i) Books of accounts:-
As per Section 34 of LLP Act, 2008:- An LLP shall be under obligation to maintain annual accounts
reflecting true and fair view of its state of affairs.
LLPs are required to maintain books of accounts which shall contain -
 Particulars of all sums of money received and expended by the LLP and the matters in respect of
which the receipt and expenditure takes place,
 A record of the assets and liabilities of the LLP,
 Statements of costs of goods purchased, inventories, WIP, FGs and COGS,
 Any other particulars which the partners may decide.
The books of account which a LLP is required to keep shall be preserved for eight years from the
date on which they are made.
(ii) Audit:- As per Section 34 of LLP Act, 2008:
 Applicability:- The accounts of every LLP shall be audited in accordance with Rule 24 of LLP
Rules 2009. Such rules, inter-alia, provides that:
 Any LLP, whose turnover does not exceed, in any financial year, 40 lakh rupees, or
 Whose contribution does not exceed 25 lakh rupees, is not required to get its accounts
audited. However, if the partners of such limited liability partnership decide to get the
accounts of such LLP audited, the accounts shall be audited only in accordance with such rule.
 Eligibility for auditor:- A person shall not be qualified for appointment as an auditor of a LLP

 FAST
unless he is a Chartered Accountant in practice.
Period of Appointment:- Auditor of a LLP shall be appointed for each financial year of the LLP
for auditing its accounts.

CA.Sarthak Niraj Jain


 Appointment of Auditor: The auditor may be appointed by the designated partners of the
LLP–

1. At any time for the first financial year but before the end of first financial year,
2. At least 30 days prior to the end of each financial year (other than the first financial year),
3. To fill the causal vacancy in the office of auditor,
4. To fill the casual vacancy caused by removal of auditor.
The partners may appoint the auditors if the designated partners have failed to appoint them.
 Tenure of Auditor: Auditor shall hold office in accordance with the terms of his or their
appointment and shall continue to hold such office till the period the new auditors are
appointed, or they are re-appointed.
(iii) Auditor’s duty regarding Audit of LLP
 Auditor should obtain instructions in writing as to the work to be performed by him.
 The auditor should read the LLP agreement & note the following provisions
(a) Nature of the business of the LLP
(b) Amount of capital contributed by each partner
(c) Interest – in respect of additional capital contributed
(d) Duration of partnership
(e) Drawings allowed to the partners
(f) Salaries, commission etc payable to partners
(g) Borrowing powers of the LLP
(h) Rights & duties of partners

124 Ab Audit Hoga Sabse Scoring - By CA. Sarthak Niraj Jain


Company Audit

(i) Method of settlement of accounts between partners at the time of admission, retirement,
admission etc.
(j) Any loans advanced by the partners
(k) Profit sharing ratio.
 Auditor should report
(a) Whether the records reflects true and fair view
(b) Whether he obtains all information & explanation
(c) Whether any restriction/limitation imposed upon him.
 If minute book is being maintained, auditor shall refer it for any resolution passed regarding
the accounts.
5.16 – Miscellaneous Provision
32. Section 52 - Application of Securities Premium Account
Pirana Ltd. issued 10,000 shares of face value of 10 each at a premium of ` 490 each in May, 2022. The
company received the stated minimum amount in the prospectus and transferred a sum equal to the
aggregate amount of the premium received on shares (i.e. ` 49 lakhs) to the ‘Securities Premium Account’.
Unfortunately, in the month of July, the godown of the company caught fire and stock worth ` 45 lakhs burnt
to ashes. Now, the management desires to adjust the loss due to fire against the said premium account.
(RTP-Nov-2015, RTP-Nov-2018)
OR
Miranda spinning mills Ltd. is a sick company and has accumulated losses of ` 10 crores. The company has `
12 crores in its Share Premium Account. The management desires to adjust the accumulated losses against
the share premium balance. Advise the company giving your reasons.
Ans. (i) Provision:- Section 52 of the Companies Act, 2013 deals with the application of premium received

FAST
on issue of shares.
 The said section provides that where a company issues shares at a premium, whether for cash
or otherwise, a sum equal to the aggregate amount of the premium received on those shares

CA.Sarthak Niraj Jain


shall be transferred to an account called “Securities Premium Account” and
 The provisions of this Act relating to reduction of share capital of a company except as provided
in this section shall apply as if the securities premium account was the paid up share capital of the
company.
(ii) Application of Securities Premium Account:- As per section 52, the securities premium account may
be applied for the following purposes:
 towards the issue of fully paid bonus shares;
 in writing off the preliminary expenses;
 in writing off the expenses of, or the commission paid or discount allowed on, any issue of shares
or debentures;
 in providing for the premium payable on the redemption of any redeemable preference shares or
debentures; or
 for the purchase of its own shares or other securities under section 68 of the Companies Act,
2013.
(iii) In the given case:- the management of Pirana Ltd. desires to adjust the loss due to fire against the
securities premium account.
(iv) Conclusion:- In view of the above provisions of the Companies Act, 2013, it may be noted that the
company is not permitted to adjust its loss against the securities premium account.
33. Failure to Detect Untrue and Incorrect Financial Position of a Company
C Ltd. declared dividend amounting to ` 5 lacs out of Profits for the year ended 31.3.2022. Subsequently, it
was noticed that company had failed to make provisions for outstanding expenses of ` 7.80 lacs and closing
stock was also overvalued, which was not reported by auditors of the company. Management of C Ltd. held
auditors responsible for this situation. It was established that auditor was grossly negligent in conduct of his
duties.

CA Final Audit - By CA. Sarthak Niraj Jain 125


Company Audit

Ans. (i) In the given case:- Profit of the company has been inflated by non-provisioning of outstanding
expenses of ` 7.80 lacs and by overvaluation of closing stock and based on such inflated profit the
company has declared and paid dividend of ` 5.00 lacs. Thus it can be said that dividend has been
paid out “inflated profit” and not out of “real profit”. If there is insufficient profit after above
adjustment of outstanding expenses and correction of stock valuation and there is no past reserve, it
would amount to payment of dividend out of capital.
(ii) Auditor’s responsibility:- It was the duty of auditor to ascertain whether the Balance Sheet and
Statement of Profit and Loss of the company show a true and fair view of the financial position and
revenue earning capacity.
For that he has to exercise proper audit procedure of substantive test (i.e. vouching and
verification) and valuation of various items of Balance Sheet and Statement of Profit and Loss.
 The auditor should have checked whether all the outstanding expenses have been provided or
not and
 Whether closing stock has been properly valued as per AS-2.
 If he was not satisfied, he should have issued a qualified report or adverse report.
(iii) Conclusion:- In the instant case he has failed to do so, he will be guilty of gross negligence in the
performance of his duty.
(v) Case Law -The facts of the case are similar to the established judgement on “The Leeds Estate
Building & Investment Co. Ltd vs Shepherd (1887)”, where, it was held, that it was an auditor’s
duty to ascertain that the accounts, he certifies, are correct and that if he fails in his duty, he is
liable for damages for dividends wrongly paid by the company out of capital.

Notes

FAST
CA.Sarthak Niraj Jain

126 Ab Audit Hoga Sabse Scoring - By CA. Sarthak Niraj Jain


Company Audit

UNIT-4: CARO, 2020


5.17 – Applicability of CARO
34. Applicability of CARO, 2020
Sanjana Private limited reports the following position as on 31st March 2022:
Paid up capital `55Lacs
Revaluation reserves `25Lacs
Capital reserves `26Lacs
P & L A/c (Dr. Balance) `8Lacs
The management of the company contends that CARO 2020 is not applicable to it.
OR
Ramya Private Limited Company reports the following position as on 31st March, 2022:
Paid up Capital ` 75 Lacs
Revaluation Reserve ` 25 Lacs
Capital Reserve ` 20 Lacs
Profit & Loss (Dr.) Balance ` 25 Lacs
The Management of the Company contends that CARO, 2020 is not applicable to it. Comment.
Ans. (i) As per CARO-2020:- The Companies (Auditor’s Report) Order, 2020, exempts private limited
companies, not being a subsidiary or holding of a public company, from its application which
fulfills all the following conditions:
 its paid-up capital and reserves are not more than ` 1 Cr. as on Balance Sheet date, and
 its total borrowings any bank or financial institution are not more than ` 1 cr. at any point of

FAST
time during the financial year; and
 its total revenue as disclosed in Schedule III (including revenue from discontinuing operations)
does not exceed ` 10 Cr. during the financial year as per the financial statements.

CA.Sarthak Niraj Jain


(ii) As per Guidance Note on CARO, 2020 issued by ICAI:
 While computing paid up capital and reserves, capital reserves, revenue reserves, revaluation
reserves and credit balance of Profit and loss account are to be considered in aggregate as
reduced by debit balance in the profit and loss account, if any
(iii) In the present case:- Paid up capital and reserves after deducting debit balance of Profit & Loss
Account amounts to ` 98 Lacs (55 + 25 + 26 –8).
(iv) Conclusion:- CARO is not applicable as paid up capital and reserves does not exceed ` 1 Cr.
Note:- assuming that other conditions as to borrowings and revenue also satisfied
35. Applicability of CARO, 2020
H Private Ltd (not a small company) had taken overdrafts from two banks with a limit of ` 40 lacs each
against the security of fixed deposit it had with those banks and an unsecured overdraft from a financial
institution of ` 36 lacs. The said loans were outstanding as at 31st March 2022. The paid-up capital and
reserves of the company as at that date was ` 80 lacs and its revenue during the financial year ended on 31st
March 2022 was ` 6 crores. The management of the company is of the opinion that CARO, 2020 is not
applicable to it because turnover and paid up capital were within the limits prescribed and loans taken
against the fixed deposits cannot be considered. The company further contended that loan limit is to be
reckoned per bank or financial institution and not cumulatively. Comment.
Ans. (i) As per CARO-2020:- The Companies (Auditor’s Report) Order, 2020, exempts private limited
companies, not being a subsidiary or holding of a public company, from its application which
fulfills all the following conditions:
 its paid-up capital and reserves are not more than ` 1 Cr. as on Balance Sheet date, and
 its total borrowings any bank or financial institution are not more than ` 1 cr. at any point of
time during the financial year; and
 itstotal revenue as disclosed in Schedule III (including revenue from discontinuing operations)
does not exceed ` 10 Cr. during the financial year as per the financial statements.

CA Final Audit - By CA. Sarthak Niraj Jain 127


Company Audit

(ii) As per Guidance Note on CARO, 2020 issued by ICAI:


 While computing total borrowings from banks and financial statements, loans against Fixed
deposits are to be taken into consideration.
 Further loans from banks and financial institutions are to be taken cumulatively not
individually.
(iii) In the present case:- Paid-up capital is less than ` 1 Cr., Revenue is less than ` 10 Crores but its
total borrowings from banks and financial institution is ` 1.16 Cr.
(iv) Conclusion:- Contention of H Ltd. is not correct as total borrowings exceeds` 1 Cr., hence reporting
under CARO, 2020 will be required.
36. Applicability of CARO, 2020
TI NEXT Pvt. Ltd. is a holding company of C21 Ltd. Whether CARO is applicable to TI NEXT Pvt. Ltd.?
Ans. (i) As per CARO-2020:- The Companies (Auditor’s Report) Order, 2020, exempts private limited
companies, not being a subsidiary or holding of a public company, from its application which
fulfills all the following conditions:
 its paid-up capital and reserves are not more than ` 1 Cr. as on Balance Sheet date, and
 its total borrowings any bank or financial institution are not more than ` 1 cr. at any point of time
during the financial year; and
 its total revenue as disclosed in Schedule III (including revenue from discontinuing operations)
does not exceed ` 10 Cr. during the financial year as per the financial statements.
(ii) In the present case:- TI NEXT Pvt. Ltd. is a holding company of C21 Ltd., hence reporting under
CARO is required.
(iii) Conclusion:- As per the facts given above CARO is applicable on TI NEXT Pvt. Ltd.
37. Applicability of CARO, 2020
FAST
Grow More Pvt. Ltd.’s paid up Capital & Reserves are less than `1 cr. and it has no outstanding loan exceeding
` 1 Cr. from any bank or financial institution. Its sales are `12 crores before deducting Trade discount ` 20

CA.Sarthak Niraj Jain


lakhs and Sales returns `1.90 Cr. The services rendered by the company amounted to `20 lakhs. The
company contends that reporting under Companies Auditor’s Reports Order (CARO) is not applicable.
Discuss.
Ans. (i) As per CARO-2020:- The Companies (Auditor’s Report) Order, 2020, exempts private limited
companies, not being a subsidiary or holding of a public company, from its application which
fulfills all the following conditions:
 its paid-up capital and reserves are not more than ` 1 Cr. as on Balance Sheet date, and
 its total borrowings any bank or financial institution are not more than ` 1 cr. at any point of
time during the financial year; and
 its total revenue as disclosed in Schedule III (including revenue from discontinuing operations)
does not exceed ` 10 Cr. during the financial year as per the financial statements.
(ii) As per Schedule-III:
 Revenue from operations shall consists of revenue from sale of products, sale of services, and
other operating revenues.
 While computing total revenue, trade discount as well as sales returns are required to be
deducted.
(iii) In the present case:- The turnover of the company including value of service rendered after
deducting trade discount and sales returns amounts to ` 10.10 crores (i.e. 12 - 0.20 – 1.90 + 0.20
crore).
(iv) Conclusion:- Contention of the company that CARO is not applicable is not correct, as Total
Revenue exceeds ` 10 Cr.
38. Applicability of CARO, 2020 - Kind of Loan to be considered
For the purpose of availing exemption from CARO what kind of loan to be considered? (Study Material)

128 Ab Audit Hoga Sabse Scoring - By CA. Sarthak Niraj Jain


Company Audit

Ans.  Borrowings from banks or financial institutions can be long term or short term and are normally in
the form of term loans, demand loans, export credits, cash credits, overdraft facilities, bills purchased
or discounted.
 Outstanding balances of such borrowings should be considered as borrowing outstanding for the
purpose of computing the limit of rupees one crore. Non-fund based credit facilities, to the extent
such facilities have devolved and have been converted into fund-based credit facilities, should also be
considered as outstanding borrowings.
 The figures of outstanding borrowing would also include the amount of bank guarantees issued by
the company where such guarantee(s) has (have) been invoked and encashed or where, say, a letter of
credit has been devolved on the company.
 In case of term loans, interest accrued and due is considered as a borrowing whereas interest accrued
but not due is not considered as a borrowing. Further, in case the company enjoys a facility, say, a cash
credit facility, whose balance is fluctuating in nature, the Order would apply to the company in case on
any day during the financial year concerned, the amount outstanding in the cash credit facility exceeds
` one crore as per books of the company along with other borrowings.
 The aggregate borrowings disclosed in the financial statements would need to be considered based
on applicable generally accepted accounting principles in India (Ind AS/ AS).
5.18 - Matters to be Reported in CARO-2020 & Disclosures in Audit Report
39. Title Deeds of Immovable Property in the Name of Bank
ABC Ltd. owns a piece of Land and Building situated at IP road, Mumbai which was purchased before 30
years. The title deeds for the same are deposited with State Bank of India for obtaining credit facilities by the
company. As the statutory auditor of the company for the year ended 31st March, 2022, what are the audit

Ans. FAST
procedures to be followed and what is the reporting under CARO 2020? (May-2017)
The auditor is required to specifically include certain matters as per CARO, 2020 under section 143 of
the Companies Act, 2013.

CA.Sarthak Niraj Jain


(i) As per Clause (i)(c) of Para-3 of CARO-2020:
 Whether the title deeds of all the immovable properties (other than properties where the
company is the lessee and the lease agreements are duly executedin favour of the lessee)
disclosed in the financial statements are held in the name of the company, if not, provide the
details thereof in the prescribed format which includes:
 Description of property
 Gross carrying value
 Held in name of
 Whether promoter, director or their relative or employee
 Period held – indicate range, where appropriate
 Reason for not being held in name of company (also indicate if in dispute)
(ii) Audit Procedure:
 The auditor should verify the title deeds available and reconcile the same with the PPE register.
 The scrutiny of the title deeds of the immovable property may reveal a number of
discrepancies between the details in the PPE register and the details available in the title deeds.
 The auditor should carry out a detailed examination of the Registered sale deed and related
documents of Land and Building. Where the title deeds of the immovable property have been
mortgaged with the Banks/ Financial Institutions, etc., for securing the borrowings and loan
raised by the company, a confirmation about the same should be sought from the respective
institution to this effect.
 The auditor may also consider verifying this information from the online records, if available,
of the relevant State.

CA Final Audit - By CA. Sarthak Niraj Jain 129


Company Audit

(iii) In the given case:- ABC Ltd. owns a piece of land and building situated at IP road Mumbai and title
deeds for the same are deposited with State Bank of India for obtaining credit facilities by the
company.
(iv) Conclusion:- Thus, the auditor needs to follow the audit procedure as mentioned above and shall
report on the same under Clause (i)(c) of Paragraph 3 of the CARO, 2020.
40. PPE Valuation, Working Capital limit & Quarterly Returns
Mr. Arjun was appointed as the engagement partner on behalf of Bhism & Co., a Chartered Accountant Firm,
for conducting statutory audit assignment of Sinwar Ltd., unlisted public company.
Mr. Brijesh, one of the senior engagement team members, was given the responsibility to audit the matters as
per the requirements of CARO, 2020 and in that connection, he made the following observations, that may be
relevant for reporting as per the said Order:-
Sr. No. Observations
(a) One of the Plant and Equipment taken on a lease (‘right of use’ asset) by Sinwar Ltd. was
revalued based on the valuation by a registered valuer and the net carrying value of Plant and
Equipment in aggregate was changed from 4 crore to 4.45 crore.
(b) During the year under consideration, cash credit limit of 5.5 crore was sanctioned to Sinwar
Ltd. by DMC Bank based on the security of current assets which was reduced to 4.5 crore after
6 months. In this connection, quarterly returns have been filed by the company with the DMC
bank which are in agreement with Books of Accounts.
You are required to examine the contention of Mr. Brijesh regarding reporting of the above observations in
accordance with CARO 2020. (RTP-May-2022)
Ans. Matters to be reported by Mr. Brijesh as per CARO, 2020 are as follows:

FAST
(a) According to Clause (i) (d) of Para 3 of CARO 2020
• The auditor is required to report whether the company has revalued its Property, Plant and
Equipment (including Right of Use assets) or intangible assets or both during the year and,

CA.Sarthak Niraj Jain


• If so, whether the revaluation is based on the valuation by a Registered Valuer; specify the amount
of change, if the change is 10% or more in the aggregate of the net carrying value of each class of
Property, Plant and Equipment or intangible assets;
• In the given situation: - Sinwar Ltd. has revalued one of the Plant and Equipment taken on a
lease (‘right of use’ asset) based on the valuation by a registered valuer. The amount of change in the
value of such Plant and Equipment is 45 lakh. As the net carrying value of Plant and Equipment in
aggregate was changed from 4 crore to 4.45 crore i.e. change was 10% or more.
• Conclusion: -Thus, the auditor is required to report the amount of change of 45 lakh in
accordance with Clause (i)(d) of Para 3 of CARO 2020.

(b) As per Clause (ii) (b) of Para 3 of CARO 2020


• The auditor is required to report whether during any point of time of the year, the company has
been sanctioned working capital limits in excess of five crore rupees, in aggregate, from banks
or financial institutions on the basis of security of current assets;
• Whether the quarterly returns or statements filed by the company with such banks or financial
institutions are in agreement with the books of account of the Company, if not, give details;
• In the instant case: - Sinwar Ltd. has been sanctioned a cash credit limit of 5.5 crore by DMC
Bank during the year under consideration, which is exceeding the prescribed limit of 5 crore based
on the security of current assets. Further, quarterly returns have also been filed by the company
with the DMC bank in this connection which is in agreement with Books of Accounts.
• Conclusion - In view of the above, the auditor is required to report the same in accordance with
Clause (ii) (b) of Para 3 of CARO, 2020

130 Ab Audit Hoga Sabse Scoring - By CA. Sarthak Niraj Jain


Company Audit

41. Statutory Dues


XYZ Pvt. Ltd. has submitted the financial statements for the year ended 31-03-22 for audit.
The audit assistant observes and brings to your notice that the company’s records show following dues:
 Income Tax relating to Assessment Year 2019-20 `125 lacs – Appeal is pending before Hon’ble ITAT
since 30-09-20.
 Customs duty ` 85 lakhs – Demand notice received on 15-09-21 but no action has been taken to pay or
appeal. As an auditor, how would you bring this fact to the members?
(RTP-Nov-2016, MTP-Nov-2018, RTP-May-2019)
Ans. (i) As per Clause (vii) of Para-3 of CARO-2020:
 Whether the company is regular in depositing undisputed statutory dues including Goods and
Services Tax, provident fund, employees' state insurance, income-tax, sales-tax, service tax, duty
of customs, duty of excise, value added tax, cess and any other statutory dues to the appropriate
authorities and if not, the extent of the arrears of outstanding statutory dues as on the last day
of the financial year concerned for a period of more than six months from the date they became
payable, shall be indicated;
 Where statutory dues referred to in sub-clause (a) have not been deposited on account of any
dispute, then the amounts involved and the forum where dispute is pending shall be
mentioned (a mere representation to the concerned Department shall not be treated as a dispute);
(ii) Responsibility of auditor:
 The auditor should also obtain a management representation about the disputed dues, the
amounts involved and the forum where the dispute is pending.
 The auditor should carry out necessary audit procedures to verify the information provided by

FAST
the management.
(iii) Reporting :- In the present case, there is Income Tax demand of ` 125 Lacs and the company has
gone for an appeal, it should be brought to notice of members by reporting under Para 3(vii)(b) of

CA.Sarthak Niraj Jain


CARO, 2020 as below:
S.
No.
Name of the
Statute
Nature of
Dues
Amount (in
Lacs)
Period to which
amount relates
Forum where
dispute is pending
1 Income Tax Income Tax 125.00 AY 2016-17 ITAT
Act, 1961
 In reference to demand notice received for Custom Duty of ` 85 Lacs on 15.09.2019 for which
company has not taken any action and is outstanding for more than 6 months, it leads to the
irregularity which should be brought to notice of members by reporting under Para 3(vii)(b) of
CARO, 2020.
42. Statutory Dues
As an auditor, how will you report under CARO in each of the following situation?
(i) Since more than seven months, payment of electricity bills to company established under statute is outstanding.
(ii) The company had imported goods 5 years back and were placed in bonded warehouse till the end of
financial year under Audit. The company has not paid import duty as goods have not been removed from
such warehouse. The company has also not paid rent and interest expenditure payable on the amount of
custom duty
(iii) The company has received income tax assessment order along with demand notice from Assessing Officer.
The company has not paid dues payable as the same is not acceptable to the company. The company has
neither preferred ft men: against the order nor an application for rectification of mistake has heel made.
The company has just merely represented to the Assessing Officer.
(iv) The company in view of voluminous pay-roll data consistently follows the method of making lump sum
deposit of estimated amount of ESC collections and adjust the excess or deficit against next following
months' deposit and the difference of the said amount always remains insignificant. (Jan-2021-Old)

CA Final Audit - By CA. Sarthak Niraj Jain 131


Company Audit

Ans. (a) As per Clause: -Clause (vii) of Para 3 of CARO, 2020


 Whether the company is regular in depositing undisputed statutory dues including provident
fund, employees' state insurance, income-tax, sales-tax, ,Goods and Services Tax, service tax, duty
of customs, duty of excise, value added tax, cess and any other statutory dues to the appropriate
authorities and
 If not, the extent of the arrears of outstanding statutory dues as on the last day of the financial
year concerned for a period of more than six months from the date they became payable, shall be
indicated;
 Where statutory dues referred to in sub-clause (a) have not been deposited on account of any
dispute, then the amounts involved and the forum where dispute is pending shall be mentioned (a
mere representation to the concerned Department shall not be treated as a dispute)
 It is important to mention that any sum, which is to be regularly paid to an appropriate authority
under a statute (whether Central, State or Local or Foreign) applicable to the company, should be
considered as a “statutory due” for the purpose of this clause.
 In other words, obligation to pay a statutory due is created or arises out of a statute, rather than
being based on an independent contractual or legal relationship.
(b) Reporting under CARO-2020
(i) Electricity dues
 Any sum payable to an electricity company as electricity bill would not constitute a
statutory due despite the fact that such a company has been established under a statute.
 This is so because the due has arisen on account of contract of supply of goods or services
between the parties.

FAST
 Thus, reporting under CARO is not required for electricity dues.
(ii) Payment of import duty
 In the given situation, payment of import duty where the goods had been imported five years

CA.Sarthak Niraj Jain


back and were placed in the bonded warehouse and even till the end of the financial year
under audit, the goods have not been removed from such warehouse.
 It may be noted that when the imported goods are lodged in a bonded warehouse, the payment
of import duty is to be made when the goods are removed from the bonded warehouse.
 However, till the time the importer opts to remove the goods from the warehouse, the
importer is required to incur the rent and interest expenditure on the amount of customs
duty payable.
 Since the payment of the custom duty is not due in the current case, the question of regularity
does not arise in respect of custom duty.
 However, it may be noted that the interest and rent that are required to be incurred under
section 61 of the Customs Act, 1962 would come under other statutory dues and the auditor
would have to examine and comment upon the regularity of the company in depositing such
interest and rent.
(iii) Income tax demand
 In the given situation, the company has received income tax assessment order along with
demand notice from Assessing Officer and company has not paid such demand and is also not
in agreement for the same. Further, the company has just merely represented before the
Assessing Officer.
 The auditor is required to check whether time limit for filing the appeal is expired /
application for rectification of mistake or not. In case time limit, for filing the appeal or
application for rectification of mistake, has expired disputed amount will become undisputed
statutory due (as mere representation to the concerned Department shall not be treated as
a dispute).
 Further, the auditor is also required to ascertain whether such dues are outstanding for a
period of more than six months from the date they became payable.

132 Ab Audit Hoga Sabse Scoring - By CA. Sarthak Niraj Jain


Company Audit

 Accordingly, after ensuring the above, if the statutory dues are outstanding for more than six
months the auditor is required to report the same under clause (vii)(a) of CARO, 2016.
 However, in case the statutory dues are not outstanding for a period of more than six
months from the date they became payable the auditor is not required to report the same
under CARO.
(iv) ESI Payable
 It is possible that in a large company where there are a number of departments with
separate payrolls and where payments are spread over a number of days, the collection of data
regarding the provident fund/employees’ state insurance collections and the company’s
contribution thereto may take some time.
 In order to ensure that deposit of the dues is made in time, the company may make lump-
sum deposits of estimated amounts and adjust the excess or deficit against the following
month’s deposit.
 If this method is consistently followed and the difference between the total dues and the
lump-sum deposit is not significant, it need not be considered that dues have not been
regularly deposited and no unfavorable comment is necessary.
 Thus, no reporting is required for the same under CARO.
43. Default in Repayment of Loans or Other Borrowings
OK Ltd. has taken a term loan from a nationalized bank in 2017 for `200 lakhs repayable in five equal
instalments of ` 40 lakhs from 31st March, 2018 onwards. It had repaid the loans due in 2018 & 2019, but
defaulted in 2020, 2021 & 2022. As the auditor of OK Ltd. what is your responsibility assuming that company
has sought re-schedulement of loan? (Study Material, RTP-May-2015)
Ans. (i)
FAST
As per Clause (ix) of Para-3 of CARO-2020:
(a) Whether the company has defaulted in repayment of loans or other borrowings or in the
payment of interest thereon to any lender, if yes, the period and the amount of default to be

CA.Sarthak Niraj Jain


reported as per the prescribed format which includes:


Nature of borrowing, including debt securities
Name of lender (lender wise details to be provided in case of defaults to)
 Amount not paid on due date
 Whether principal or interest
 No. of days delay or unpaid
 Remarks, if any
(b) whether the company is a declared wilful defaulter by any bank or financial institution or other
lender;
(ii) Schedule-III to the companies Act-2013:
 As per the general instructions for preparation of Balance Sheet, provided under Schedule III to the
Companies Act, 2013, terms of repayment of term loans and other loans is required to be
disclosed in the notes to accounts.
 It also requires disclosure of period and amount of continuing default as on the balance sheet
date in repayment of loans and interest, separately in each case.
(iii) As per Guidance Note on CARO-2020:- The submission of application for re-schedulement/
restructuring does not mean that no default has occurred .
(iv) In the given case:- OK Ltd. has defaulted in repayment of dues for three years. Application for
rescheduling will not change the default position.
(v) Conclusion:- Hence the auditor has to report in his audit report that the Company has defaulted
in its repayment of dues to the bank to the extent of ` 120 lakhs.

CA Final Audit - By CA. Sarthak Niraj Jain 133


Company Audit

44. Reporting as to Mis-utilisation of Term Loans


During the financial year ended on 31/03/2022, LM Private Limited had borrowed from a Nationalized Bank,
a term loan of `120 lakhs consisting of `100 lakhs for purchase of a machinery for the new plant and `20
lakhs for erection expenses. As on the date of 31 st March, 2022, the total of capital and free reserves of the
Company was ` 50 lakhs and turnover for the year 2021-22 was ` 750 lakhs. The Bank paid ` 100 lakhs to
the vendor of the Company for the supply of machinery on 31/12/2021. The machinery had reached the yard
of the Company. On 28/02/2022, the Company had drawn the balance of loan viz. `20 lakhs to the credit of
its current account maintained with the Bank and utilized the full amount for renovating its administrative
office building. The machinery had been kept as capital stock under construction. Comment as to reporting
issues, if any, that the Auditor should be concerned with for the financial year ended on 31/03/2022, in this
respect. (Nov-2018-New)
Ans. (i) Applicability:- Reporting under CARO, 2020 is applicable in case of a private limited company if the
total borrowings exceed `1 Cr. from any bank or financial institution at any point of time during the
financial year.
(ii) As per Clause (ix) of Para-3 of CARO-2020:
Whether term loans were applied for the purpose for which the loans were obtained; if not, the
amount of loan so diverted and the purpose for which it is used may be reported;
(iii) In the instant case:- LM Private limited had borrowed from a Nationalized Bank, a term loan of `
120 lakhs consisting of ` 100 lakhs for purchase of a machinery for the new plant and ` 20 lakhs
for erection expenses. The Bank paid ` 100 lakhs to the vendor of the Company for the supply of
machinery on 31/12/2021. The machinery had reached the yard of the Company. On 28/02/2022, the
Company had drawn the balance of loan viz. ` 20 lakhs to the credit of its current account
maintained with the Bank and utilized the full amount forrenovating its administrative office

FAST
building.
(iv) Conclusion:- Auditor of LM Private Limited is under obligation to report on matters covered
under CARO, 2020 as total borrowings exceed RS. 1 Cr. & As per requirement of Para 3(ix) of CARO,
2020, auditor is required to report the fact that out of the term loan obtained for machinery

45.
CA.Sarthak Niraj Jain
purchase and erection, `20 Lacs was not utilized for the purpose of erection of machinery.
Fraud Reporting
What are the reporting requirements in the audit report under the Companies Act, 2013/CARO, 2020 for the
following situations?
(a) A fraud has been committed against the company by an officer of the company.
(b) A fraud has been committed against the company by a vendor of the company.
(c) The company has committed a major fraud on its customer and the case is pending in the court.
(d) A fraud has been reported in the cost audit report but not noticed by statutory auditors in his audit.
(Study Material, Nov-2018-Old)
Ans. (i) As per Clause (xi) of Para-3 of CARO-2020:
(a) Whether any fraud by the company or any fraud on the company has been noticed or
reported during the year, if yes, the nature and the amount involved is to be indicated;
(b) Whether any report under section 143(12) of the Companies Act has been filed by the
auditors in Form ADT-4 as prescribed under rule 13 of Companies (Audit and Auditors) Rules,
2014 with the Central Government;
(c) Whether the auditor has considered whistle-blower complaints, if any, received during the
year by the company;
(ii) As per Section 143(12) of Companies Act 2013:- If an auditor of a company in the course of the
performance of his duties as auditor, has reason to believe that an offence of fraud involving such
amount or amounts as may be prescribed, is being or has been committed in the company by its
officers or employees, the auditor shall report the matter to the Central Government within such
time and in such manner as may be prescribed.

134 Ab Audit Hoga Sabse Scoring - By CA. Sarthak Niraj Jain


Company Audit

(iii) Rule 13of Companies Act 2013:


 For the above purpose, prescribes the amount of ` 1 Cr. or more.
 However, in case of a fraud involving lesser than the specified amount, i.e. below ` 1 Cr., the
auditor shall report the matter to the audit committee constituted u/s 177 or to the Board in
other cases within such time and in such manner as may be prescribed:
(iv) In the present case:-
 Accordingly, reporting requirements will be:
Situation Reporting under Companies Reporting under CARO,
Act, 2013 2020
A fraud has been committed against Reporting required to C.G. if Nature of Fraud and
the company by an officer of the amount of fraud exceeds `1 Cr. amount involved need to be
company reported under Para 3(xi).
A fraud has been committed against No reporting required. No reporting required.
the company by a vendor of the
company.
The company has committed a major Effect of such fraud on financial Nature of Fraud and
fraud on its customer and the case is statements need to be reported. amount involved need to be
pending in the court. reported under Para 3(xi).
A fraud has been reported in the cost Effect of such fraud on financial Para 3(xi) requires
audit report but not noticed by statements need to be reported. reporting of fraud that has
statutory auditors in his audit. been noticed or reported
during the year.
46.
FAST
Compliances by Nidhi Company
CARO 2020 has made several significant changes and has introduced many new reporting requirements vis-
à-vis CARO 2016. In view of the above, describe the relevant clause relating to Nidhi Companies – compliance

CA.Sarthak Niraj Jain


with net owned funds to deposit requirements and the relevant provisions. What audit procedures are to be
adopted for verification and reporting on the same? (Nov-2016)
Ans. (i) As per Clause (xii) of Para-3 of CARO-2020:
(a) Whether the Nidhi Company has complied with the Net Owned Fund to Deposits in the ratio of
1: 20 to meet out the liability and
(b) Whether the Nidhi Company is maintaining 10% unencumbered term deposits as specified in
the Nidhi Rules, 2014 to meet out the liability.
(c) Whether there has been any default in payment of interest on deposits or repayment thereof
for any period and if so, the details thereof;

(ii) Audit Procedures:


 Net Owned Funds:- As per Rule 3(d) Net Owned Funds are defined as the aggregate of paid-up
equity share capital and free reserves as reduced by accumulated losses and intangible
assets appearing in the last audited balance sheet. Provided that, the amount representing the
proceeds of issue of preference shares, shall not be included for calculating Net Owned Funds.
 Deposits:- A Nidhi company can accept fixed deposits, recurring deposits and savings
deposits from its members in accordance with the directions notified by the Central
Government. The aggregate of such deposits is referred to as “deposit liability”.
 Computation & Ratio:- The auditor should ask the management to provide the computation
of the deposit liability and net owned funds on the basis of the requirements mentioned above.
This would enable him to verify that the ratio of deposit liability to net owned funds is in
accordance with the requirements prescribed in this regard

CA Final Audit - By CA. Sarthak Niraj Jain 135


Company Audit

47. Compliance with Sections 177 and 188


Whilst the Audit team has identified various matters, they need your advice to include the same in your audit
report in view of CARO 2020:-
(i) The long term borrowings from the parent has no agreed terms and neither the interest nor the
principal has been repaid so far. (MTP-Dec-2021)
Ans. (i) As per Clause (xiii) of Para-3 of CARO-2020:
Auditor is required to report “whether all transactions with the related parties are in compliance
with sections 177 and 188 of Companies Act, 2013 where applicable and the details have been
disclosed in the Financial Statements etc., as required by the applicable accounting standards”.
(ii) Reporting:- In the present case, the auditor is required to report as per clause xiii of para 3 of CARO
2020 regarding receipt of long term borrowing from Parent Company which qualifies as a
transaction with the related party.
48. Compliance with section 177 and 188
RNT Ltd. has entered into non-cash transactions with Mr. Ram, son of one of the directors of the company,
which is an arrangement by which the RNT Ltd. is in process to acquire assets for consideration other than
cash. Under CARO, 2020, as a statutory auditor, how would you report? (RTP-Nov-2019)
OR
RPS Ltd. has entered into non-cash transactions with Mr. Rahul, son of one of the directors of the company,
which is an arrangement by which the RPS Ltd. is in process to acquire assets for consideration other than
cash. Under CARO, 2020, as a statutory auditor, how would you report?
(RTP-May-2017, MTP-Nov-2017, MTP-May-2018)
Ans. (i) As per Clause (xv) of Para-3 of CARO-2020:

FAST
 The auditor requires to comment “whether the company has entered into any non-cash
transactions with directors or persons connected with him and if so, whether provisions of

(ii) CA.Sarthak Niraj Jain


Section 192 of Companies Act, 2013 have been complied with.
Section 192 of the Companies Act, 2013:- This section deals with restriction on non-cash
transactions involving directors or persons connected with them. The section prohibits the
company from entering into following types of arrangements unless it meets the conditions laid out in
the said section:
 An arrangement by which a director of the company or its holding, subsidiary or associate
company or a person connected with such director acquires or is to acquire assets for
consideration other than cash, from the company.
 An arrangement by which the company acquires or is to acquire assets for consideration
other than cash, from such director or person so connected.
(iii) Reporting under this clause:- The reporting requirements under this clause are in two parts.
 The first part requires the auditor to report on whether the company has entered into any
non-cash transactions with the directors or any persons connected with such director/s.
 The second part of the clause requires the auditor to report whether the provisions of section
192 of the Act have been complied with.
Therefore, the second part of the clause becomes reportable only if the answer to the first part is in
affirmative.
(iv) Suggested paragraph on reporting:- “According to the information and explanations given to us,
the Company has entered into non-cash transactions with one of the directors/person connected
with the director during the year, by the acquisition of assets by assuming directly related
liabilities, which in our opinion is covered under the provisions of Section 192 of the Act, and for
which approval has not yet been obtained in a general meeting of the Company”.

136 Ab Audit Hoga Sabse Scoring - By CA. Sarthak Niraj Jain


Company Audit

49. Registration with Reserve Bank of India


Rishabh Finance Ltd. is a Non-Banking Finance Company and was in the business of accepting public deposits
and giving loans since 2017. The company was having net owned funds of ` 1,50,00,000/-(one crore fifty lakh)
and was not having registration certificate from RBI and applied for it on 30th March 2022. The company
appointed Mr. Gautam as its statutory auditors for the year 2021-22. Advise the auditor with reference to
auditor procedures to be taken and reporting requirements on the same in view of CARO 2020?
(MTP-Dec-2021)
Ans. (i) As per Clause (xvi) of Para-3 of CARO-2020:
(a) Whether the company is required to be registered under section 45-IA of the Reserve Bank
of India Act, 1934 (2 of 1934) and if so, whether the registration has been obtained;
(b) Whether the company has conducted any Non-Banking Financial or Housing Finance activities
without a valid Certificate of Registration (CoR) from the Reserve Bank of India as per the
Reserve Bank of India Act, 1934;
(c) The auditor is required to examine whether the company is engaged in the business which
attract the requirements of the registration. The registration is required where the financing
activity is a principal business of the company. The RBI restrict companies from carrying on the
business of a non-banking financial institution without obtaining the certificate of registration.
(ii) Audit Procedure:
 The auditor should examine the transactions of the company with relation to the activities
covered under the RBI Act and directions related to the NBFC .
 The financial statements should be examined to ascertain whether company’s financial assets
constitute more than 50% of total assets and income from financial assets constitute more



FAST
than 50% of gross income.
Whether the company has net owned funds as required for the registration as NBFC
Whether the company has obtained the registration as NBFC, if not the reason should be sought

CA.Sarthak Niraj Jain



from the management and documented.
The auditor should report incorporating the following:-
(i) Whether the registration is required under section 45-IA of the RBI Act, 1934.
(ii) If so, whether it has obtained the registration.
(iii) If the registration not obtained, reasons thereof.
(iii) In the instant case: - Rishabh Finance Ltd. is a Non Banking Finance Company and was in the
business of accepting public deposits and giving loans since 2015. The company was having net owned
funds of ` 1,50,00,000/-(one crore fifty lakhs) which is less in comparison to the prescribed limit i.e.
2 crore rupees and was also not having registration certificate from RBI (though applied for it on 30th
March 2020).
(iv) Conclusion: -The auditor is required to report on the same as per Clause (xvi) of Paragraph 3 of
CARO 2020
5.19 - AS/Ind-AS & Schedule III Based Questions
50. Schedule III Basded Question
The Balance Sheet of G Ltd. as at 31st March, 2022 is as under. Comment on the presentation in terms of
Schedule III
Heading Note No. 31st March, 22 31st March, 21
Equity & Liabilities
Share Capital 1 XXX XXX
Reserves & Surplus 2 0 0
Employee stock option outstanding 3 XXX XXX
Share application money refundable 4 XXX XXX

CA Final Audit - By CA. Sarthak Niraj Jain 137


Company Audit

Non-Current Liabilities XXX XXX


Deferred tax liability (Arising from Indian Income Tax) 5 XXX XXX
Current Liabilities
Trade Payables 6 XXX XXX
Total Assets XXXX XXXX
Non-Current Assets
Fixed Assets-Tangible 7 XXX XXX
CWIP (including capital advances) 8 XXX XXX
Current Assets
Trade Receivables 9 XXX XXX
Deferred Tax Asset (Arising from Indian Income Tax) 10 XXX XXX
Debit balance of Statement of Profit and Loss XXX XXX
Total XXXX XXXX
(Study Material, RTP-May-2020)
Ans. (a) Following Errors are noticed in presentation as per Schedule III:
(i) Share Capital and Reserve & Surplus are to be reflected under the heading “Shareholders’
funds”, which is not shown while preparing the balance sheet. Although it is a part of Equity
and Liabilities yet it must be shown under head “shareholders’ funds”. The heading
“Shareholders’ funds” is missing in the balance sheet given in the question.
(ii) Reserve & Surplus is showing zero balance, which is not correct in the given case. Debit
balance of statement of Profit & Loss should be shown as a negative figure under the head
‘Surplus’. The balance of ‘Reserves and Surplus’, after adjusting negative balance of surplus

FAST
shall be shown under the head ‘Reserves and Surplus’ even if the resulting figure is in the
negative.
(iii) Schedule III requires that Employee Stock Option outstanding should be disclosed under the

CA.Sarthak Niraj Jain


heading “Reserves and Surplus”.
(iv) Share application money refundable shall be shown under the sub-heading “Other Current
Liabilities”. As this is refundable and not pending for allotment, hence it is not a part of equity.
(v) Deferred Tax Liability has been correctly shown under Non-Current Liabilities. But
Deferred tax assets and deferred tax liabilities, both, cannot be shown in balance sheet
because only the net balance of Deferred Tax Liability or Asset is to be shown if the enterprise
has a legally enforceable right to set off assets against liabilities representing current tax; and
it relates to the same governing tax laws.
(vi) Under the main heading of Non-Current Assets, Property, Plant and Equipment and
Intangible assets are further classified as under:
(a) Property, Plant and Equipment
(b) Intangible assets
(c) Capital work in Progress
(d) Intangible assets under development.
Keeping in view the above, the CWIP shall be shown under Property, Plant and Equipment and
Intangible assets as Capital Work in Progress. The amount of Capital advances included in
CWIP shall be disclosed under the sub-heading “Long term loans and advances” under the
heading Non-Current Assets.
Subsequent to the notification of Ministry of Corporate Affairs dated October 11, 2018 under
Section 467(1) of the Companies Act, 2013, the words “Fixed assets” shall be substituted with
the words “Property, Plant and Equipment”. Further, subsequent to notification dated 24th
March,2021, under the heading ―II Assets, under sub-heading ―Non-current assets, after the
words ―Property, Plant and Equipment, the words ― and Intangible assets are inserted and
for the words ―Tangible Assets, the words ―Property, Plant and Equipment are substituted.

138 Ab Audit Hoga Sabse Scoring - By CA. Sarthak Niraj Jain


Company Audit

(e) Deferred Tax Asset shall be shown under Non-Current Asset. It should be the net balance of
Deferred Tax Asset after adjusting the balance of deferred tax liability if the enterprise has a
legally enforceable right to set off assets against liabilities representing current tax; and it
relates to the same governing tax laws.
(f) Subsequent to the notification of Ministry of Corporate Affairs dated October 11, 2018
under Section 467(1) of the Companies Act, 2013, Trade Payables should be disclosed as
follows:-
(A) Total outstanding dues of micro enterprises and small enterprises; and
(B) Total outstanding dues of creditors other than micro enterprises and small
enterprises.”
(b) Following Errors have been noticed in presentation, as per Division II of Schedule III:
(i) Balance sheet should begin with Assets on top and then, Equity and Liabilities should be
presented.
(ii) Under the main heading of Non-Current Assets, following sub-headings are provided in the
format as per Division II: (a) Property, plant and equipment (b) Capital work-in-progress In
view of the above, the Fixed asset- Tangible should be presented as “Property, Plant and
Equipment”. CWIP should be presented as “Capital Work in Progress”. Under Ind AS Schedule
III, ‘Capital Advances’ are not to be classified under ‘Capital Work in Progress’, since they are
specifically to be disclosed under ‘Other non-current assets’ .
(iii) Deferred Tax Asset should be presented under “Non-Current Asset”. It should be the net
balance of Deferred Tax Asset, after adjusting the balance of deferred tax liability, if the
enterprise has a legally enforceable right to set off assets against liabilities representing
current tax; and it relates to the same governing tax laws.

FAST
(iv) Trade receivables shall be presented under sub-heading “Financial assets” under heading
“Current Assets”.
(v) Share capital and Reserves & Surplus need to be presented under the heading “Equity”. The

CA.Sarthak Niraj Jain


heading Equity is missing in the balance sheet given in the question. Reserves & Surplus would
form part of sub-heading “Other Equity” in the notes to accounts and such balance of “Other
Equity” would be presented on face of balance sheet under the heading “Equity”.
(vi) Debit balance of statement of profit and loss would be presented as negative balance under
“Retained Earnings” in sub-heading “Other Equity” in the notes to accounts. Such balance of
“Other Equity” even if negative, would be presented on face of balance sheet under the heading
“Equity”.
(vii) Division II of Schedule III requires that Employee Stock Option outstanding should be
disclosed under the sub-heading “Other Equity” in the notes to accounts which should be
presented on face of balance sheet under the heading “Equity”.
(viii) Share application money refundable should be presented under the sub-subheading “Other
Financial Liabilities” under the sub-heading “Financial Liability”. As this is refundable and not
pending for allotment, hence, it should not form part of equity.
(ix) Deferred Tax Liability has been correctly presented under “Non-Current Liabilities”. But
Deferred tax assets and deferred tax liabilities, both, cannot be presented in balance sheet
since only the net balance of Deferred Tax Liability or Asset is to be disclosed if the enterprise
has a legally enforceable right to set off assets against liabilities representing current tax; and
it relates to the same governing tax laws.
(x) Trade payables should be presented under sub-heading “Financial liabilities” under the
heading “Current liabilities”. Subsequent to notification by Ministry of Corporate Affairs dated
October 11, 2018 under Section 467(1) of the Companies Act, 2013, Trade Payables should be
disclosed as follows:
(A) Total outstanding dues of micro enterprises and small enterprises; and
(B) Total outstanding dues of creditors other than micro enterprises and small enterprises.

CA Final Audit - By CA. Sarthak Niraj Jain 139


Company Audit

51. Schedule III Basded Question


Comment on the following with reference to Schedule III to the Companies Act, 2013
(i) A company has disclosed performance guarantee and counter guarantees as Contingent Liabilities.
(ii) A company has clubbed all other expenses under the head ‘Other Expenses” on the basis of 1 percent of
total revenue or ` 5,000 whichever is higher.
(iii) A company has shown Deferred Tax Liability under Non-Current Liabilities and Deferred tax assets
under Non-Current Asset in balance sheet. (MTP-May-2020)
Ans. (i) A contingent liability in respect of guarantees arises when a company issues guarantees to
another person on behalf of a third party e.g. when it undertakes to guarantee the loan given to a
subsidiary or to another company or gives a guarantee that another company will perform its
contractual obligations.
However, where a company undertakes to perform its own obligations, and for this purpose issues,
what is called a "guarantee", it does not represent a contingent liability and it is misleading to show
such items as contingent liabilities in the Balance sheet. For various reasons, it is customary for
guarantees to be issued by Bankers e.g. for payment of insurance premia, deferred payments to
foreign suppliers, letters of credit, etc. For this purpose, the company issues a "counter-guarantee" to
its Bankers. Such "counter guarantee" is not really a guarantee at all, but is an undertaking to perform
what is in any event the obligation of the company, namely, to pay the insurance premia when
demanded or to make deferred payments when due. Hence, such performance guarantees and counter-
guarantees should not be disclosed as contingent liabilities.
(ii) All other expenses not classified under other heads will be classified under "Other Expenses". For
this purpose, any item of expenditure which exceeds one percent of the revenue from operations or
` 1,00,000whichever is higher, needs to be disclosed separately. The given treatment in the scenario
is not in order.

FAST
(iii) Deferred Tax Liability should be shown under Non-Current Liabilities. Deferred Tax Asset shall be
shown under Non-Current Asset. But Deferred tax assets and deferred tax liabilities, both, cannot be
shown in balance sheet because only the net balance of Deferred Tax Liability or Asset is to be shown.

52. CA.Sarthak Niraj Jain


Thus, DTA and DTL shown separately in the balance sheet by the company is not correct.
Disclosure Compliances - Division Il of Schedule III
The financial statements of MP Ltd. as on March 31st, 2022 are to be prepared under Division Il of Schedule III
to the Companies Act, 2013. Comment on the disclosure compliances for MP Ltd. from the following
information in the financial statements which are required to be drawn up in compliance with Ind AS.
(i) Property, Plant and Equipment include ` 2.50 crore for a boiler-plant under construction.
(ii) Cash and cash equivalents include ` 1.25 crore deposited with a nationalized bank on 31st March, 2022
for 18 months. It is shown under current assets.
(iii) Non-current assets include under caption "Biological assets other than bearer Plants" a sum of ` 1.50
crore being cost of cultivation for bringing to yield level, the cashewnut trees whose yield period,
according to estimate shall not be less than 10 years. (Study Material)
Ans. (i) Disclosure of Boiler Plant under Construction: Boiler plant under construction should be shown
under the heading ‘Capital Work in Progress’ instead of Property Plan and Equipment. Thus,
inclusion of value of boiler plant under construction in Property Plan and Equipment is not in
order.
(ii) Disclosure of Cash and Cash Equivalents deposited with Nationalised Bank: Bank deposits with
more than 12 months maturity shall be disclosed under 'Other financial assets'. Therefore,
disclosure of deposits rupees 1.25 crores in a nationalised bank for 18 months as Cash and Cash
Equivalents is not in order as per Division II of Schedule III.
(iii) Disclosure of Cost of Cultivation for bringing to yield level the Cashewnut trees: Cost of 1.5 crore
rupees for Cultivation for bringing to yield level, the cashewnut trees whose yield period is more
than one period will form part of ‘Bearer Plant’. Hence it will not be considered as ‘Biological Assets
other than bearer plant’. Therefore, it should be shown under the heading ‘Property Plant and
Equipment’ as Bearer Plant as per Division II of Schedule III.

140 Ab Audit Hoga Sabse Scoring - By CA. Sarthak Niraj Jain


Audit Report

Chapter
6
AUDIT REPORT

Past Exams Weightage


5

0
MAY-19 NOV-19 NOV-20 JAN-21 JUL-21 DEC-21

May-19 Nov-19 Nov-20 Jan-21 Jul-21 Dec-21


Marks 5 0 0 0 5 4

1.
FAST
Unmodified Opinion
6.1 Types of Opinion/Modification of opinion

CA.Sarthak Niraj Jain


CA Sameer is the statutory auditor of Tram Fram Ltd. for the FY 2021-22. While concluding the audit CA
Sameer decided to issue an unmodified opinion, though he also concluded that a material uncertainity exists
with respect to the company’s ability to continue as a going concern on account of a pending litigation related
to labour laws. He is of the view that the company has made appropriate disclosures with respect to such
pending litigation in the notes to accounts annexed to the financial statements of Tram Fram Ltd. for the FY
2020-21. Explain how CA Sameer will deal with the above situation in his auditor’s report (draft the relevant
portion of the auditor’s report.) (Study Material)
Ans. Material Uncertainty Related to Going Concern
We draw attention to Note 10 in the financial statements, which indicates that the outcome of a litigation
on account of labour laws is pending in case of the company during the year 31 March, 2022. As stated in
Note 11, this event or condition, indicate that a material uncertainty exists that may cast significant doubt
on the Company’s ability to continue as a going concern. Our opinion is not modified in respect of this
matter.
2. Qualified Opinion
XYZ Ltd. is a company engaged in the manufacture of cranes. CA Sudhir is the statutory auditor of the
company for the FY 2021-22. The company has taken long term funding for fixed capital requirements and
short-term funding for its working capital requirements. During the course of audit, CA Sudhir found that the
company’s financing arrangements are about to expire and the company is unable to re- negotiate or obtain
the replacement financing. As such the company may be unable to realize its assets and discharge its
liabilities in the normal course of business. Notes to accounts annexed to the financial statements discuss the
magnitude of financing arrangements, the expiration and the total financing arrangements; however, the
financial statements do not include discussion on the impact or the availability of refinancing. Thus, the
financial statements (and notes thereto) do not fully disclose this fact. What kind of opinion should CA Sudhir
issue in case of XYZ Ltd.? (Study Material)

CA Final Audit - By CA. Sarthak Niraj Jain 141


Audit Report

Ans. (i) In the present case: - XYZ Ltd. is unable to re- negotiate or obtain the replacement financing for its
long term and short-term funding requirements. This situation indicates the existence of a material
uncertainty that may cast significant doubt on the Company’s ability to continue as a going concern
and therefore, XYZ Ltd. may be unable to realize its assets and discharge its liabilities in the normal
course of business. Further, the financial statements of XYZ Ltd. do not disclose this fact adequately.
(ii) Conclusion:- Thus, the financial statements of XYZ Ltd. are materially misstated due to the
inadequate disclosure of the material uncertainty. CA Sudhir will express a qualified opinion as the
effects on the financial statements of this inadequate disclosure are material but not pervasive to the
financial statements.
(iii) Reporting - The relevant extract of the Qualified Opinion Paragraph and Basis for Qualified
Opinion paragraph is as under:
 Qualified Opinion
In our opinion and to the best of our information and according to the explanations given to
us, except for the incomplete disclosure of the information referred to in the Basis for Qualified
Opinion section of our report, the aforesaid standalone financial statements give the information
required by the Act in the manner so required and give a true and fair view in conformity with the
accounting principles generally accepted in India, of the state of affairs of XYZ Ltd. as at March 31,
2022, and profit/loss, for the year ended on that date.
 Basis for Qualified Opinion
As discussed in Note 6, the Company’s financing arrangements are about to expire and the
Company has been unable to conclude renegotiations or obtain replacement financing. This
situation indicates that a material uncertainty exists that may cast significant doubt on the

3.
FAST
Company’s ability to continue as a going concern. The financial statements do not adequately
disclose this matter.
Qualified Opinion

CA.Sarthak Niraj Jain


CA Yash is the statutory auditor of Laksmi Vardhan Limited for the FY 2021-22. In respect of loans and
advances of ` 55,00,000/- given to Sarvagya Private Limited, the Company has not furnished any agreement
to CA Yash and in absence of the same, he is unable to verify the terms of repayment, chargeability of interest
and other terms.
What kind of opinion should CA Yash give in such situation? (Study Material)
Ans. (i) In the present case, with respect to loans and advances of ` 55,00,000/- given to Sarvagya Private
Limited, the Company has not furnished any agreement to CA Yash. In absence of such agreement, CA
Yash is unable to verify the terms of repayment, chargeability of interest and other terms. For an
auditor, while verifying any loans and advances, one of the most important audit evidences is the
loan agreement.
(ii) Conclusion: -Therefore, the absence of such document in the present case, tantamount to a material
misstatement in the financial statements of the company. However, the inability of CA Yash to obtain
such audit evidence is though material but not pervasive so as to require him to give a disclaimer of
opinion. Thus, in the present case, CA Yash should give a qualified opinion
(iii) Reporting: - The relevant extract of the Qualified Opinion Paragraph and Basis for Qualified
Opinion paragraph is as under:
 Qualified Opinion
In our opinion and to the best of our information and according to the explanations given to us,
except for the effects of the matter described in the Basis for Qualified Opinion section of our
report, the financial statements of Laksmi Vardhan Limited give a true and fair view in conformity
with the accounting principles generally accepted in India, of the state of affairs of the Company as
on 31.03.2022 and profit/ loss for the year ended on that date.

142 Ab Audit Hoga Sabse Scoring - By CA. Sarthak Niraj Jain


Audit Report

 Basis for Qualified Opinion


The Company is unable to furnish the loan agreement with respect to loans and advances of `
55,00,000/- given to Sarvagya Private Limited. Consequently, in absence of such agreement, we
are unable to verify the terms of repayment, chargeability of interest and other terms.
4. Adverse Opinion
ABC Ltd. is a company engaged in the manufacture of iron and steel bars. PP & Associates are the statutory
auditors of ABC Ltd. for the FY 2021-22. During the course of audit, CA Prakash, the engagement partner,
found that the Company’s financing arrangements have expired and the amount outstanding was payable on
March 31st, 2022. The Company has been unable to re-negotiate or obtain replacement financing and is
considering filing for bankruptcy. These events indicate a material uncertainty that may cast significant
doubt on the Company’s ability to continue as a going concern and therefore it may be unable to realize its
assets and discharge its liabilities in the normal course of business. The financial statements (and notes
thereto) do not disclose this fact. What opinion should CA Prakash express in case of ABC Ltd.?
(Study Material)
Ans. (i) In the present case: - Based on the audit evidence obtained, CA Prakash has concluded that a material
uncertainty exists related to events or conditions that may cast significant doubt on the entity’s
ability to continue as a going concern, and the entity is considering bankruptcy. The financial
statements of ABC Ltd. omit the required disclosures relating to the material uncertainty.
(ii) Conclusion: -In such circumstances, CA Prakash should express an adverse opinion because the
effects on the financial statements of such omission are material and pervasive.
(iii) Reporting: - The relevant extract of the Adverse Opinion Paragraph and Basis for Adverse Opinion
paragraph is as under:

FAST
Adverse Opinion
In our opinion, because of the omission of the information mentioned in the Basis for Adverse

CA.Sarthak Niraj Jain


Opinion section of our report, the accompanying financial statements do not present fairly, the
financial position of the entity as at March 31, 2022, and of its financial performance and its cash
flows for the year then ended in accordance with the Accounting Standards issued by the Institute
of Chartered Accountants of India.
 Basis for Adverse Opinion
The financing arrangements of ABC Ltd. has expired and the amount outstanding was payable
on March 31, 2022. The entity has been unable to conclude re-negotiations or obtain
replacement financing and is considering filing for bankruptcy. This situation indicates that a
material uncertainty exists that may cast significant doubt on the Company’s ability to continue as
a going concern. The financial statements do not adequately disclose this fact.
5. Adverse Opinion
In the financial year 2021-22, MSD Ltd. faced an extraordinary event (earthquake), which destroyed a lot of
business activity of the company. These circumstances indicate material uncertainty on the company’s ability
to continue as going concern. Due to such event it may not be possible for the company to realize its assets or
pay off the liabilities during the regular course of its business. The financial statement and notes to the
financial statements of the company do not disclose this fact. What kind of opinion should the statutory
auditor of MSD Ltd. issue in such circumstances? (Study Material)
Ans. (i) In the present case: - There exists a material uncertainty that cast a significant doubt on the
company’s ability to continue as going concern and the same is not disclosed in the financial
statements of MSD Ltd.
(ii) Conclusion: -As such, the financial statements of MSD Ltd. for the FY 2021-22 are materially
misstated and the effect of the misstatement is so material and pervasive on the financial
statements that giving only a qualified opinion will be insufficient and therefore the statutory auditor
of MSD Ltd. should issue an adverse opinion.

CA Final Audit - By CA. Sarthak Niraj Jain 143


Audit Report

(iii) Reporting: -The relevant extract of the Adverse Opinion Paragraph and Basis for Adverse Opinion
paragraph is as under:
 Adverse Opinion
In our opinion, because of the omission of the information mentioned in the Basis for Adverse
Opinion section of our report, the accompanying financial statements do not present fairly, the
financial position of MSD Ltd. as at March 31, 2022, and of its financial performance and its cash
flows for the year then ended in accordance with the Accounting Standards issued by the Institute
of Chartered Accountants of India.
 Basis for Adverse Opinion
MSD Ltd. has faced an extraordinary event (earthquake), which destroyed a lot of business
activity of the company. Due to such event it may not be possible for the company to realize its
assets or pay off the liabilities during the regular course of its business. This situation indicates
that a material uncertainty exists that may cast significant doubt on the Company’s ability to
continue as a going concern. The financial statement and notes to the financial statements of the
company do not disclose this fact.
6. Disclaimer of Opinion
MNO Ltd. is a power generating company having its plants in the north eastern states of the country. For the
FY 2021-22, M/s PRT & Associates are the statutory auditors of the company. During the course of audit, the
audit team was unable to obtain sufficient appropriate audit evidence about a single element of the
consolidated financial statements. That is, the auditor was also unable to obtain audit evidence about the
financial information of a joint venture investment (in XYZ Ltd.) that represents over 90% of the entity’s net
assets. What kind of opinion should the statutory auditors issue in such case? (Study Material)
Ans. (i)
FAST
In the present case: -M/s PRT & Associates are unable to obtain sufficient appropriate audit
evidence about the financial information of a joint venture investment that represents over 90% of
the entity’s net assets.

CA.Sarthak Niraj Jain


(ii) Conclusion: - The possible effects of this inability to obtain sufficient appropriate audit evidence
are both material and pervasive to the consolidated financial statements. Therefore, the statutory
auditor should issue a disclaimer of opinion.
(iii) Reporting:- The relevant extract of the Disclaimer of Opinion Paragraph and Basis for Disclaimer of
Opinion paragraph is as under:
 Disclaimer of Opinion
We do not express an opinion on the accompanying financial statements of MNO Ltd. Because of
the significance of the matters described in the Basis for Disclaimer of Opinion section of our
report, we have not been able to obtain sufficient appropriate audit evidence to provide a basis
for an audit opinion on these financial statements. Basis for
 Disclaimer of Opinion
The Group’s investment in its joint venture XYZ Company is carried at ` 95 crores on the Group’s
consolidated balance sheet, which represents over 90% of the Group’s net assets as at March
31, 2022. We were not allowed access to the management and the auditors of XYZ Company,
including XYZ Company’s auditors’ audit documentation. As a result, we were unable to
determine whether any adjustments were necessary in respect of the Group’s proportional share
of XYZ Company’s assets that it controls jointly, its proportional share of XYZ Company’s liabilities
for which it is jointly responsible, its proportional share of XYZ’s income and expenses for the year,
(and the elements making up the consolidated statement of changes in equity) and the consolidated
cash flow statement.

144 Ab Audit Hoga Sabse Scoring - By CA. Sarthak Niraj Jain


Audit Report

7. Disclaimer of Opinion
CA Abhimanyu is the statutory auditor of PQR Ltd. for the FY 2021-22. During the course of audit CA
Abhimanyu noticed the following:
1. With respect to the debtors amounting to ` 150 crores, no balance confirmation was received by the
audit team. Further, there have been defaults on the payment obligations by debtors on the due dates
during the year under audit. The Company has created a provision for doubtful debts to the tune of `25
Cr. during the year under audit. The Company has stated that the provision is based on receivables
which are older than 36 months, which according to the audit team is inadequate and as such the audit
team is unable to ascertain the carrying value of trade receivables.
2. Further, in respect of Inventories (which constitutes 40% of the total assets of the company), during the
reporting period, the management has not undertaken physical verification of inventories at periodic
intervals. Also, the Company has not maintained adequate inventory records at the factory. The audit
team was unable to undertake the physical inventory count as such the value of inventory could not be
verified.
Under the above circumstances what kind of opinion should CA Abhimanyu give? (Study Material)
Ans. (i) In the present case: - CA Abhimanyu is unable to obtain sufficient and appropriate audit evidence
with respect to the following:
1. The balance confirmation with respect to debtors amounting to ` 150 crores is not available.
Further there has been default in payment by the debtors and the provision so made is not
adequate. The audit team is also unable ascertain the carrying value of trade receivables.
2. With respect to 40% of the company’s inventory, neither the physical verification has been done

FAST
by the management nor are adequate inventory records maintained. The audit team is also unable
to undertake the physical inventory count as such the value of inventory could not be verified.
In the above two circumstances the auditor is unable to obtain sufficient appropriate audit

CA.Sarthak Niraj Jain


evidence on which to base the opinion, and the possible effects on the financial statements of
undetected misstatements, if any, could be both material and pervasive.
(ii) Conclusion: -Thus, CA Abhimanyu should give a Disclaimer of Opinion.
(iii) Reporting: -The relevant extract of the Disclaimer of Opinion Paragraph and Basis for Disclaimer of
Opinion paragraph is as under:
 Disclaimer of Opinion
We do not express an opinion on the accompanying financial statements of PQR Ltd. Because of
the significance of the matters described in the Basis for Disclaimer of Opinion section of our
report, we have not been able to obtain sufficient appropriate audit evidence to provide a basis for
an audit opinion on these financial statements.
 Basis for Disclaimer of Opinion
We are unable to obtain balance confirmation with respect to the debtors amounting to ` 150
crores. Further, there have been defaults on the payment obligations by debtors on the due dates
during the year under audit. The Company has created a provision for doubtful debts to the tune
of `25 Cr. during the year under audit which is inadequate in the circumstances of the company.
The carrying value of trade receivables could not be ascertained.
Further, in respect of Inventories (which constitutes 40% of the total assets of the company),
during the reporting period, the management has not undertaken physical verification of
inventories at periodic intervals. Also, the Company has not maintained adequate inventory
records at the factory. We were unable to undertake the physical inventory count and as such the
value of inventory could not be verified.

CA Final Audit - By CA. Sarthak Niraj Jain 145


Audit Report

8. Disclaimer of Opinion
You have been appointed as an auditor of Dharmnath & Sons for FY 2021-22, as entity other than a company
incorporated under the Companies Act, 2013, using a fair presentation framework. Appointment had been
made in the month of April, 2022. The financial statements have been prepared by the management in
accordance with the Accounting Standards. The management had introduced the new computerized accounts
receivable system from November 2021 and still in the implementation phase and thus management is in the
process of rectifying system deficiencies and correcting the errors. At the time of implementation of a new
system, the earlier system of accounting of receivables had been discarded. The auditor was unable to obtain
sufficient appropriate audit evidence about the entity’s accounts receivable and inventories. The possible
effects of the inability to obtain sufficient appropriate audit evidence are deemed to be both material and
pervasive to the financial statements. Write the opinion paragraph and basis of opinion paragraph to be
included in the Independent Auditor’s Report. (RTP-May-2022)
Ans. Opinion Paragraph
Disclaimer of Opinion
We were engaged to audit the financial statements of Dharmnath & Sons (“the entity”), which comprise
the balance sheet as at March 31, 2022, the statement of Profit and Loss, (the statement of changes in
equity) and the statement of cash flows for the year then ended, and notes to the financial statements,
including a summary of significant accounting policies.

We do not express an opinion on the accompanying financial statements of the entity. Because of the
significance of the matters described in the Basis for Disclaimer of Opinion section of our report, we have
not been able to obtain sufficient appropriate audit evidence to provide a basis for an audit opinion on

FAST
these financial statements.

Basis for Disclaimer of Opinion

CA.Sarthak Niraj Jain


We were not appointed as auditors of the Company until after March 31, 2022, and thus did not observe
the counting of physical inventories at the beginning and end of the year. We were unable to satisfy
ourselves by alternative means concerning the inventory quantities held at March 31, 2021, and 2022,
which are stated in the Balance Sheets at ` xxx and ` xxx, respectively. In addition, the introduction of a new
computerized accounts receivable system in November 2021 resulted in numerous errors in accounts
receivable. As of the date of our report, management was still in the process of rectifying the system
deficiencies and correcting the errors. We were unable to confirm or verify by alternative means accounts
receivable included in the Balance Sheet at a total amount of ` xxx as at March 31, 2022. As a result of
these matters, we were unable to determine whether any adjustments might have been found necessary in
respect of recorded or unrecorded inventories and accounts receivable, and the elements making up the
statement of Profit and Loss (and statement of cash flows)
9. Modification to the Auditor’s Opinion
CA Omkar is the statutory auditor of Sabhyata Ltd. for the FY 2021-22. The company is engaged in the
business of manufacture of floor tiles. During the course of audit, CA Omkar obtained certain audit evidence
which were not consistent with the affirmation made in the financial statements. Discuss as to how CA Omkar
should deal with the situation in the auditor’s report. (Study Material)
Ans. (i) Provision: -SA 705 deals with the auditor’s responsibility to issue an appropriate report in
circumstances when, in forming an opinion in accordance with SA 700 (Revised), the auditor
concludes that a modification to the auditor’s opinion on the financial statements is necessary.
 The decision regarding which type of modified opinion is appropriate depends upon:
(a) The nature of the matter giving rise to the modification, that is, whether the financial
statements are materially misstated or, in the case of an inability to obtain sufficient
appropriate audit evidence, may be materially misstated; and

146 Ab Audit Hoga Sabse Scoring - By CA. Sarthak Niraj Jain


Audit Report

(b) The auditor’s judgment about the pervasiveness of the effects or possible effects of the
matter on the financial statements.
Further, the auditor shall modify the opinion in the auditor’s report when the auditor concludes
that based on the audit evidence obtained, the financial statements as a whole are not free from
material misstatement.
(ii) In the present case: - During the course of audit, CA Omkar obtained certain audit evidence which
were not consistent with the affirmation made in the financial statements.
(iii) Conclusion: -Therefore, CA Omkar should modify his report in accordance with SA 705-
“Modifications to The Opinion in the Independent Auditor’s Report. CA Omkar should issue either a
qualified opinion or an adverse opinion depending upon the circumstances of the case:
(a) CA Omkar shall express a qualified opinion when, having obtained sufficient appropriate audit
evidence, he concludes that misstatements, individually or in the aggregate, are material, but not
pervasive, to the financial statements
(b) CA Omkar shall express an adverse opinion, when the auditor, having obtained sufficient
appropriate audit evidence, concludes that misstatements, individually or in the aggregate, are
both material and pervasive to the financial statements.
Thus, since CA Omkar has obtained audit evidence which are inconsistent with the affirmations made
in the financial statement, CA Omkar should modify his opinion as per the circumstances of the case.
6.2 Emphasis of Matter & Other Matter Paragraph
10. EOM & OM Paragraph
In respect of the audit of BDS Ltd., the statutory auditor of the company noticed some matters. The statutory
auditor wants to draw the user’s attention towards such matters, though his opinion is not modified in
respect of such matters. Draft the relevant paragraphs of the audit report for the following matters:
i.
FAST
The company has a plan to resume its construction activities with respect to one of its thermal power
project, The activity of such power plant was suspended in the FY 2021-22. The thermal power project
comprises of the plant and equipment amounting to ` 5.95 crore and capital work in progress of

CA.Sarthak Niraj Jain


` 147.50 crore.
ii. The financial statements of 5 branches are included in the Standalone Financial Statements of BDS Ltd.
whose financial statements reflect total assets of `90 crores as at 31.03.2022 and total revenue from
operations of `40 crores for the year ended on that date. The financial statements of these branches
have been audited by the branch auditors. (Study Material)
Ans. Emphasis of Matter
We draw attention to the following note of the standalone financial statements:
Note 27 regarding the plans of the Company to resume construction/developmental activities of a
thermal power project. The carrying amounts related to the project as at 31st March, 2022 comprise of plant
and equipment of ` 5.95 crore and capital work in progress of ` 147.50 crore. Our opinion is not modified in
respect of this matter.
Other Matter
We did not audit the financial statements of 5 branches included in the Standalone Financial
Statements of the company whose financial statements reflect total assets of ` 90 crores as at 31.03.2021
and total revenue from operations of ` 40 crores for the year ended on that date. The financial statements
of these branches have been audited by the branch auditors whose reports have been furnished to us, and
our opinion in so far as it relates to the amounts and disclosures included in respect of these branches, is
based solely on the report of the branch auditors. Our opinion is not modified in respect of this matter.

CA Final Audit - By CA. Sarthak Niraj Jain 147


Audit Committee & Corporate Governance

Chapter
7
AUDIT COMMITTEE & CORPORATE GOVERNANCE

FAST 7.1 - Audit Committee under LODR Regulations & 177


1.
CA.Sarthak Niraj Jain
Responsibility of Audit Committee vis a vis Review of Financial Statements
Mr. ‘U', a respectable Chartered Accountant of international repute was requested by one of the major
corporates in India to join its Board and also as a Chairman of Audit Committee. He expressed his
apprehensions that he is not having the requisite experience. Mr. 'U' seeks your view on the responsibility of
Audit Committee vis-a-vis the review of Financial Statements.
Ans. Responsibility of Audit Committee vis a vis review of Financial Statements: The responsibility of the
audit committee while reviewing the annual financial statements with management before submission to the
Board, focuses primarily on-
(i) Matters required to be included:- In the Director’s Responsibility Statement to be included in
the Board’s report in terms of section 134(3)(c) of the Companies Act, 2013.
(ii) Changes:- If any, in accounting policies and practices and reasons for the same.
(iii) Major accounting entries:- Involving estimates based on the exercise of judgment by
management.
(iv) Significant adjustments:- Made in the financial statements arising out of audit findings.
(v) Compliance:- Listing and other legal requirements relating to financial statements.
(vi) Disclosure:- Any related party transactions.
(vii) Qualifications:- In the draft audit report.
2. Mandatory Review Areas of Audit Committee
Mr. DG, Partner in M/s. DG and Associates, as part of their audit presentation to the Audit Committee of MABD
Limited, a listed company, highlighted the following:
• Difficulties faced during the audit;
• Disagreements with the management;
• Management Letter Points;
• Draft Management Representation letter to be provided by the Company in connection with the audit.

148 Ab Audit Hoga Sabse Scoring - By CA. Sarthak Niraj Jain


Audit Committee & Corporate Governance

Some of the Audit Committee members were not happy with the above presentation and asked Mr. DG to take
it back and submit directly to the Board. They believe that Audit Committee is not the forum for discussing
such problems and this has to be sorted out between auditors and the management. Please comment on the
above. (MTP-Dec-2021)
Ans. (i) As per the Auditing Standards: - The statutory auditor of the Company is having an obligation to
bring certain matters to the attention of TCWG, which inter alia includes aspects such as -
 Difficulties faced by them during the audit
 Disagreements with the management
 Management Letter Points
 Draft Management Representation letter to be provided by the Company in connection with the audit.
(ii) As per LODR:
 The Audit Committee is also having an obligation to mandatorily review certain areas before
providing their recommendations/inputs to the board.
 Given below are the areas required to be mandatorily reviewed by the AGM in the case of listed
companies.
 The Audit Committee shall mandatorily review among other points the following information as
per LODR Regulations:
(i) Management discussion and analysis of financial condition and results of operations;
(ii) Statement of significant related party transactions (as defined by the Audit Committee),
submitted by management;
(iii) Management letters / letters of internal control weaknesses issued by the statutory
auditors;

FAST
(iv) Internal audit reports relating to internal control weaknesses;
(iii) Auditor’s responsibility:
 The auditor should further ascertain whether the Management Discussion and Analysis report

CA.Sarthak Niraj Jain



includes discussion on the matters stipulated.
Where certain deficiencies or adverse findings are noted by the Audit Committee, the auditor
will be required to see that these have been suitably dealt with by the management in the
report on corporate governance.
(iv) In the instant case:
 Mr. DG, Partner in M/s DG and Associates highlighted the facts such as difficulties faced
during the audit, disagreements with the management, managements letters points and draft
management letters to be provided by the Company in connection with the audit.
 However, some of the audit committee members were not happy and as according to them
audit committee is not the forum for discussing such problems.
(v) Conclusion:
 Contention of those audit committee members regarding problems to be sorted out between
auditors and the management is not in order as Audit Committee is required to mandatorily
review the same in accordance with Schedule II of SEBI (Listing Obligations and Disclosure
Requirements) Regulations, 2015
7.2 - Composition of Board & Obligations of Directors
3. Compliance of Conditions of Corporate Governance
Comment on the following in the light of certificate of compliance of conditions of Corporate Governance to
be issued for a listed company where the Board consists of 10 directors including a non-executive director as
its chairman and further:
(i) There were 5 meetings held during the year as follows 01/04/2021, 01/06/2021, 01/09/2021,
03/01/2022, 25/03/2022
(ii) There are 4 independent directors. One of them resigned on 25/05/2021. A new independent
director was appointed on 01/09/2021.
(iii) The Chairman of Audit Committee did not attend the Annual General meeting held on 14/09/2021.

CA Final Audit - By CA. Sarthak Niraj Jain 149


Audit Committee & Corporate Governance

(iv) The internal audit reports were obtained by Audit Committee on quarterly basis. Quarter 1 internal
audit report commented on certain serious irregularities as regards electronic online auction of
scrap. The agenda of Audit Committee did not deliberate or take note of the issue.
(MTP-May-2019, Nov-2018-Old)
OR
XYZ Limited has conducted 4 meetings in 2021-22. i.e. June 15, 2021, October 18, 2021, February 10, 2022
and June 10, 2022. Does it comply with provisions of conducting meeting? (Study Material)
Ans. As per Listing Obligation and Disclosure Requirements Regulations 2015, depending upon the facts
and circumstances, some situations may require an adverse or qualified statement or a disclosure
without necessarily making it a subject matter of qualification in the Auditors’ Certificate, in respect of
compliance of requirements of corporate governance for example:
(i) As per Regulation-18:- The Audit Committee shall meet at least 4 times in a year and not more
than 120 days shall lapse between two meetings.
Conclusion:- The number of days between the meetings held on 1.9.2017 and 3.01.2018 is more
than 120 days. Hence it is a non-compliance and would require qualification in certificate of
corporate governance (in Second question time between June 18 to Oct 18 is > 120 d)
(ii) As per Regulation-17(1):- Since the Chairman is the non-executive director, there should be
1/3rd of directors (rounded to next integer) to be independent. In this case, 4 directors need to
be independent.
As per Section 149(4):- Any vacancy during shortfall of independent directorship should be
filled:
 Within next 3 months or
 Before the start of next meeting,
 Whichever is later.

FAST
Conclusion:- In the instant case, since the independent director was appointed after lapse of 3
months (i.e. on 1.9.2017) and after next first meeting 1/6/2017, there is default which would
require qualification in certificate on corporate governance.
(iii)
CA.Sarthak Niraj Jain
As per Regulation-18:- Chairman shall be present at Annual General Meeting to answer
shareholder queries.
Conclusion:-In the given scenario, Chairman of Audit Committee did not attend the Annual
General Meeting held on 14/09/2017 which is not in order/compliance.
(iv) As per Regulation-18:- The Audit Committee shall mandatorily review the Internal audit
reports relating to internal control weaknesses as per Part C (B) of Schedule II and the auditor
should ascertain from the minutes book of the Audit Committee and other sources like agenda
papers, etc. whether the Audit Committee has reviewed the above-mentioned information.
Conclusion:- In the given situation, the agenda of Audit Committee did not deliberate or take
note of serious irregularity mention in Internal Audit Report which is again not in compliance
of conditions of Corporate Governance and warrant audit qualification in certificate on
corporate governance.
4. Approval of Remuneration of Directors - Regulation 17(6)
You have been appointed as a statutory auditor of ABC Ltd; a listed company As an auditor, state the points to
he considered by you in verification of approval of remuneration to directors of ABC Ltd. under Regulation
17(6) of the Securities and Exchange Board of India (Listing Obligations and Disclosure Requirements)
Regulations, 2015. (July-2021-Old)
Ans. As an auditor of ABC Ltd., a listed company, following points to be considered regarding verification of
approval of remuneration to directors of ABC Ltd., under Regulation 17(6) of Securities and Exchange
Board of India (Listing Obligations and Disclosure Requirements) Regulations, 2015:
1. The auditor should ascertain from the minutes of the Board of Directors’ meetings, shareholders’
meetings, relevant agenda papers, notices, explanatory statements etc., whether the remuneration of
non-executive directors has been decided by the Board of Directors after receiving prior approval of
the shareholders in the general meeting;

150 Ab Audit Hoga Sabse Scoring - By CA. Sarthak Niraj Jain


Audit Committee & Corporate Governance

2. The approval of shareholders by special resolution shall be obtained every year, in case the annual
remuneration payable to a single non-executive director exceeds fifty percent of the total annual
remuneration payable to all non-executive directors, giving details of the remuneration thereof.
3. The auditor should refer to the Articles of Association of the company, wherever applicable;
4. The auditor is required to examine the Report of the Board of Directors on corporate governance to
be included in the annual report of the company and ascertain whether the same contains the
disclosures with respect to remuneration of directors and compensation to non-executive directors. The
auditor should correlate this data with that contained in the financial statements.
Where application of this clause requires the value of ESOP to be determined, the services of expert may have
to be utilized. In this regard, reference may be made to SA 620 dealing with “Using the Work of an Auditor’s
Expert”.
5. Verification of Composition of Board of Director
Kayask Ltd. is a public company which got listed on BSE and NSE in the F.Y. 2015-16 and is amongst the top
500 listed entities on the basis of market capitalization. JP Bhuj & Co., a CA firm, has been appointed as its
statutory auditor for the F.Y. 2021-22.
Mr. Pankaj Bhuj was assigned its audit as an engagement partner and he was verifying the composition of the
Board of Director because of some changes in the same. The present composition of the Board of Kayask Ltd.
is as follows: -
(1) There are 9 directors out of which there are 4 non-executive directors and 3 independent directors. The
board has only one woman director and she is an executive director.
(2) Mr. Madhusudan Mehra has been appointed as the non-executive chairperson of the Board. He is
brother in law of the Managing Director of Kayask Ltd.
Whether present composition of the board of Kayask Ltd. complies with the requirement of the provisions of

Ans. (i)
FAST
SEBI LODR Regulations?
Provision: - As per Regulation 17 and 17A of the SEBI LODR Regulations, -
(RTP-Dec-2021)

CA.Sarthak Niraj Jain


✓ The auditor should ascertain whether, throughout the reporting period, the Board of Directors
comprises an optimum combination of executive and non-executive directors, with at least
one woman director and not less than 50% of the Board of Directors comprising non-
executive directors. It may be noted that the Board of directors of the top 1000 listed entities
shall have at least one independent woman director.
✓ The auditor should also verify that where the Chairperson of the Board is a non-executive
director, at least one-third of the Board should comprise of independent directors.
✓ The auditor shall ensure that the Chairperson of the board of the top 500 listed entities is –
(a) a non-executive director;
(b) not related to the Managing Director or the Chief Executive Officer as per the definition of
the term “relative” defined under the Companies Act, 2013.
As per the term “relative” defined under the Companies Act, 2013 – Brother-in-law i.e. sister’s
husband is not included.
(ii) In the given case:
 Kayask Ltd. is a public company which got listed on BSE and NSE in the F.Y. 2015-16 and is
amongst the top 500 listed entities on the basis of market capitalization.
 The present composition of the board of Kayask Ltd includes 9 directors out of which there
are 4 non-executive directors and 3 independent directors.
 The board has only one woman director and she is an executive director.
 In addition, Chairperson of the Board Mr. Madhusudan Mehra is brother in law of the
Managing Director of Kayask Ltd. and has been appointed as the non-executive Chairperson.
(iii) Conclusion:
 In view of Regulation 17 and 17A of the SEBI LODR Regulations, there should at least 5 non-
executive directors and 3 Independent directors as its Chairperson is a non-executive director.

CA Final Audit - By CA. Sarthak Niraj Jain 151


Audit Committee & Corporate Governance

 Further as the company is amongst the top 500 listed entities, at least one independent woman
director should be there in its board.
 Thus, it can be concluded that the present composition of the board of Kayask Ltd. does not
comply with the requirement of the provisions of SEBI LODR Regulations as the woman
director should be an independent director and there should be 5 non-executive directors.
7.3 - Code of Conduct
6. Code of Conduct
The Board of Directors of PQR Ltd. have laid down the code of conduct for all Board members and senior
management. The auditor is provided with the annual compliance affirmations received from the Board
members and explained that since there has been no change in the composition of the senior management,
the previous year’s affirmations may be considered valid. Is the contention of the Company valid?
(Study Material)
Ans. (i) As per Regulation 26(3) of LODR:- All Board members and senior management personnel have to
affirm compliance with the code on an annual basis.
(ii) Conclusion:- The decision to consider the previous year’s affirmations from the senior management
personnel as valid is not in line with the LODR Regulations.

7.4 - Vigil Mechanism


7. Vigil Mechanism
RST Ltd. has established a vigil mechanism/whistle blower policy to enable its directors and employees to
report genuine concerns and seek protection against victimization. The details of the mechanism are
available on the company intranet which is accessible by the directors and employees. Are the measures

Ans. (i) FAST


taken by the Company in line with the LODR Regulations? (Study Material)
As per Regulation 22 of the LODR -The vigil mechanism/whistle blower policy can be used by
directors, employees and any other person. To that effect, Regulation 46 of the LODR requires the

(ii)
CA.Sarthak Niraj Jain
details of establishment of such mechanism to be disclosed by the Company on its website and in the
Board Report.
Conclusion:- By only providing the details in the intranet, the Company has failed to meet the LODR
Regulations.

7.5 - Sexual Harassment of Women at Workplace


8. Sexual Harassment of Women at Workplace - Disclosure
Genuine Ltd. has established the Internal Complaints Committee under the Sexual Harassment of Women at
Workplace (Prevention, Prohibition and Redressal) Act, 2013 (‘POSH Act’). The details (names, email
addresses and contact numbers) of the Committee members are available on the company intranet which is
accessible by all employees. However, no disclosure regarding number of complaints pertaining to sexual
harassment of women at workplace is being made. Are the measures taken by the Company adequate?
(Study Material)
Ans. (i) As per Schedule V Disclosures:- In relation to the Sexual Harassment of Women at Workplace
(Prevention, Prohibition and Redressal) Act, 2013, amongst other matters, following should be
disclosed in the section on Corporate Governance of the Annual Report:
 number of complaints filed during the financial year
 number of complaints disposed of during the financial year
 number of complaints pending as on end of the financial year
(ii) The POSH Act:- It offers protection to all women, be it employees or contract staff or any other
women who are associated with the Company in any other capacity (including service providers,
vendors, professionals, etc.)
(iii) Conclusion:- By only providing the details in the intranet, the Company has failed to meet the
requirements under the POSH Act. In view of above, Genuine Ltd. is required to make necessary
disclosures in accordance with Schedule V of SEBI (LODR) Regulation 2015.

152 Ab Audit Hoga Sabse Scoring - By CA. Sarthak Niraj Jain


Audit Committee & Corporate Governance

7.6 - Material Subsidiary Identification & Compliance


9. Material Subsidiary Identification & Compliance
RAO & Co., a Chartered Accountant Firm, is appointed as the principal auditor of a listed company, Triumph
Ltd. Figures of income and net-worth of five out of seven components of Triumph Ltd., which are its unlisted
subsidiaries, is tabulated below for the immediate preceding financial year along with the consolidated
amount:-
(` in crores)
Component Component Component Component Component
Particulars Consolidated
‘A ’ ‘B’ ‘C’ ‘D’ ‘E’
Income 300 35 10 70 65 20
Net Worth 800 40 20 140 180 50
The remaining two components i.e., Component ‘F’ & Component ‘G’ of Triumph Ltd. were unaudited.
According to Mr. RAO, the engagement partner, Component ‘F’ is material to the consolidated financial
statements whereas Component ‘G’ is not material to consolidated financial statements and this fact has also
been discussed in writing with those charged with governance of Triumph Ltd. and it will also form part of
report as a ‘Key audit matter’ in accordance with SA 701.
Which of the components of Triumph Ltd. can be termed as “material subsidiary” and in the board of which
of the unlisted subsidiaries at least one independent director of Triumph Ltd. needs to be appointed or
would be appointed? (MTP-May-2021)
Ans. (1) Definition of Material Subsidiary-
(A) (a) As per Regulation 16(c) of the SEBI (LODR) Regulations, 2015: - “material subsidiary” shall
mean a subsidiary, whose income or net worth exceeds ten percent of the consolidated

FAST
income or net worth respectively, of the listed entity and its subsidiaries in the immediately
preceding accounting year.
(b) Compliance- The listed entity shall formulate a policy for determining ‘material’ subsidiary.

CA.Sarthak Niraj Jain


(B) (a) As per Regulation 24(1) of the SEBI (LODR) Regulations, 2015: - Notwithstanding anything
to the contrary contained in regulation 16, the term “material subsidiary” shall mean a
subsidiary, whose income or net worth exceeds twenty percent of the consolidated income
or net worth respectively, of the listed entity and its subsidiaries in the immediately
preceding accounting year
(b) Compliance - Regulation 24(1) of the SEBI (LODR) Regulations, 2015: - Provides that at
least one independent director on the board of directors of the listed entity shall be a
director on the board of directors of an unlisted material subsidiary, whether incorporated
in India or not.
(2) Identification: -On the basis of above provisions, following information is tabulated as below:
Particulars Share in Consolidated Income Share in Consolidated Net Worth
Component ‘A’ 11.67% 5%
Component ‘B’ 3.33% 2.5%
Component ‘C’ 23.33% 17.5%
Component ‘D’ 21.67% 22.5%
Component ‘E’ 6.67% 6.25%
(3) Conclusion: -It can be observed that Component ‘A’, Component ‘C’ and Component ‘D’,
respectively, can be termed as “material subsidiary” as their shares in either consolidated Income
or net worth exceeds 10%.
Further, at least one independent director from the board of directors of Triumph Ltd. shall be
appointed or would have been appointed on the board of Component ‘C’ and Component ‘D’,
respectively, as their shares in either consolidated income or net worth exceeds 20%.

CA Final Audit - By CA. Sarthak Niraj Jain 153


Audit Committee & Corporate Governance

10. Material Subsidiary


BN Limited is a listed entity having two subsidiaries namely BUS Limited and ROBUS Limited. Both the
subsidiaries are unlisted and are incorporated in Australia. The consolidated net worth of BN Limited and its
subsidiaries is ` 350 crore (including net worth of BUS Limited and ROBUS Limited ` 36 crore & ` 80 crore
respectively) in the immediately preceding year 2020-21. On observing this fact, your senior manager advises
you to inform the management of BN limited to make certain changes in the board of directors of both the
subsidiaries, in accordance with the LODR regulation 24(1). Comment. (MTP-May-2021)
Ans. (i) Material Subsidiary:
 As per Regulation 24(1) of the SEBI (LODR) Regulations, 2015: - Notwithstanding anything to the
contrary contained in regulation 16, the term “material subsidiary” shall mean a subsidiary, whose
income or net worth exceeds twenty percent of the consolidated income or net worth
respectively, of the listed entity and its subsidiaries in the immediately preceding accounting year
 Compliance -Regulation 24(1) of the SEBI (LODR) Regulations, 2015: - Provides that at least one
independent director on the board of directors of the listed entity shall be a director on the board
of directors of an unlisted material subsidiary, whether incorporated in India or not.

(ii) In the given case: -The fact that both the subsidiaries are unlisted and incorporated outside India is
irrelevant. From the above case we have the following details:
 20% of consolidated net worth of BN Limited = 350*20% = ` 70 crore.
 Net worth of BUS Limited = ` 36 crore.
 Net worth of ROBUS Limited = ` 80 crore.

FAST
(iii) Conclusion:
 Accordingly, it is clear that out of the two subsidiaries, the net worth of only one subsidiary (ie.
ROBUS Limited) exceeds 20% of the consolidated net worth of BN Limited and all its subsidiaries.

CA.Sarthak Niraj Jain



Therefore, the change in the composition of board of directors needs to be made only for ROBUS
Limited and not both the subsidiaries.
Thus, contention of senior manager regarding change in composition of board of directors in BUS
Limited is not in order as per Regulation 24(1).
 However, change in composition of board of director is required for ROBUS Limited. Further, as
per Regulation 24 (1) of LODR Regulation, 2015 one of the independent directors present in the
board of director of BN Limited should also be made as a director in the board of directors of
ROBUS Limited.
7.7 - Risk Management Committee
11. Constitution of Risk Management Committee
M/s AIl-in-One Limited is a large-sized listed Indian Company with focus on design and delivery of custom
made Information Technology applications for various business entities in India and abroad. The
Management wants to know whether they are required to constitute Risk Management Committee as per
LODR, 2015 and if so, required, what should be its composition? Advise (May-2018-New, Study Material)
Ans. (i) Regulation:- As per regulation 21 of LODR 2015, provision relating to constitution of risk
management committee is applicable to top 1000 listed entities, determined on the basis of market
capitalisation, as at the end of the immediate previous financial year.
(ii) In the instant case:- All-in-One Limited, is a large sized listed Indian Company with focus on design
and delivery of custom made IT applications for various business entities in India and abroad.
(iii) Conclusion:- As per fact of the case it is a large sized listed Indian company, assuming that it is
included in top 500/1000 listed entities, All – in – One Limited is required to constitute risk
management committee.

154 Ab Audit Hoga Sabse Scoring - By CA. Sarthak Niraj Jain


Audit Committee & Corporate Governance

Composition of Risk Management Committee:


(i) The Board of Directors shall constitute a Risk Management Committee.
(ii) The Risk Management Committee shall have minimum 3 members with majority of them being
members of the board of directors, including at least 1 independent director and in case of a
listed entity having outstanding SR equity shares, at least 2/3rd of the Risk Management
Committee shall comprise independent directors
(iii) The Chairperson of the Risk Management Committee shall be a member of the Board of Directors
and senior executives of the listed entity may be members of the committee.
(iv) The Risk Management committee shall meet atleast twice in a year
(v) The quorum for a meeting shall be either 2 members or 1/3rd of the members of the committee,
whichever is higher, including at least 1 member of the board of directors in attendance.
(vi) The meetings of the RCM shall be conducted in such a manner that on a continuous basis not more
than 180 days shall elapse between any two consecutive meetings.
(vii) The Board of Directors shall define the role and responsibility of the Risk Management
Committee and may delegate monitoring and reviewing of the risk management plan to the
committee and such other functions as it may deem fit.
(viii) It may be noted that the role and responsibilities of the Risk Management Committee shall
mandatorily include the performance of functions specified in Part D of Schedule II.
(ix) The Risk Management Committee shall have powers to seek information from any employee,
obtain outside legal or other professional advice and secure attendance of outsiders with relevant
expertise, if it considers necessary.
7.8 - Resignation of Auditor in case of Listed Entity
12.
FAST
Resignation of the Statutory Auditor of a Listed Entity/Material Subsidiary
Mr. Ibrahim was appointed as statutory auditor of New Limited and Old Limited. Both the Companies were
having their base in Chennai they had recently listed their shares on the Stock Exchange. For the financial

CA.Sarthak Niraj Jain


year 2021-22, Mr. Ibrahim had signed limited review reports for each quarter, till the quarter ended 31st
December 2021 for both the companies. Owing to his personal commitments and increased workload, he
tendered his resignation to M/s New Limited on 30th January 2022 and asked the Company to appoint
another auditor to issue audit report for the remaining quarter and the FY 2021-22 as a whole. But the
management of the Company did not accept the same.
Mr. Ibrahim continued to as act as auditor for M/s Old limited. During the 1st week of March 2022, Mrs. W
(wife of Mr. Ibrahim) had borrowed a sum of ` 6 lakh from the Company for her personal use. Having come to
know about this, Mr. Ibrahim immediately informed the management that he had been disqualified to act as
auditor and told them that he won’t issue audit report for last quarter. But management of the Company
argued that it’s the legal responsibility of Mr. Ibrahim to do the same.
Whether contention of management of New Limited and Old Limited is justified in asking Mr. Ibrahim to
issue audit report for the last quarter and the FY 2021-22 as a whole, despite his resignation? Discuss.
(RTP-May-2021)
Ans. (i) As per SEBI LODR Regulations: - If the auditor has signed the limited review/ audit report for the
first three quarters of a financial year, then the auditor shall, before such resignation, issue the
limited review/ audit report for the last quarter of such financial year as well as the audit report
for such financial year. This provision will not apply if the auditor is disqualified due to Section
141 of the Companies Act, 2013.
(ii) In the given scenario: - Mr. Ibrahim was appointed as statutory auditor of two listed entities i.e.,
New Limited and Old Limited. For the financial year 2021-22, Mr. Ibrahim had signed limited
review reports for first three quarter i.e., till the quarter ended 31st December 2021 for both the
companies. Owing to his personal commitments and increased workload, he resigned from New
Limited and asked the Company to appoint another auditor to issue audit report for the remaining
quarter and audit report for the FY 2021-22.

CA Final Audit - By CA. Sarthak Niraj Jain 155


Audit Committee & Corporate Governance

(iii) Disqualification: - Further, Mr. Ibrahim immediately informed the management of Old Limited
that he had been disqualified to act as auditor and told them that he won’t issue audit report for
last quarter as Mrs. W (wife of Mr. Ibrahim) had borrowed a sum of ` 6 lakh from the Company for
her personal use.
(iv) Analaysis of the case: - In view of above conditions to be complied with upon resignation of the
statutory auditor of a listed entity/material subsidiary with respect to limited review / audit
report as per SEBI LODR Regulations, Mr. Ibrahim is required to issue the audit report for the last
quarter and audit report for the year 2021-22 for New Limited as he has issued audit report for
the first three quarters whereas Mr. Ibrahim is not required to issue the audit report for remaining
quarter and audit report for the year 2021- 22 as a whole for Old Limited as he is disqualified under
section 141 of Companies Act.
(v) Conclusion: -Accordingly, contention of Management of New Limited is correct and tenable for
issuing the audit report for remaining quarter and audit report for financial year 2021-22 however,
contention of management of Old Limited is not correct regarding the legal responsibility of Mr.
Ibrahim to issue audit report for remaining quarter and for the whole year.
7.9 - Related Party Transactions
13. Related Party Transactions in case of Listed Entity
LDH Ltd. a company incorporated in India and Listed on a recognized Stock Exchange in India has entered into
various related parties transactions during the financial year. You are required to answer the following keeping
in mind the Listing Obligations and Disclosure Requirements (LODR) on corporate Governance.
(i) Who should sign the report of material transactions with related parties ?
(ii) What type of transactions and policy are required to be disclosed in relation to related party transactions?
(iii) Whether disclosures of related party transactions on consolidated financial statements are required to be
made ? If yes, what are the guidelines? (Jan-2021-New)
Ans.
(i)
FAST
An Indian company, LDH Ltd., listed on stock exchange entered into various related party transactions.
The report shall be signed either by the compliance officer or the chief executive officer of the
listed entity.
(ii) (a) The company shall disclose the policy on dealing with related party transactions on its website

CA.Sarthak Niraj Jain


and a web link thereto shall be provided in the Annual Report.
(b) The listed entity shall disclose the transactions with any person or entity belonging to the
promoter/ promoter group which hold(s) 10% or more shareholding in the listed entity, in the
format prescribed in the relevant accounting standards for annual results.
(iii) (a) Yes, disclosures of related party transactions on consolidated financial statements are required
to be made by the listed entity within 30 days from the date of publication of its standalone
and consolidated financial results for the half year.
(b) The listed entity shall disclose related party transactions on a consolidated basis, in the format
specified in the relevant accounting standards for annual results to the stock exchanges and
publish the same on its website.
Provided that a ‘high value debt listed entity’ shall submit such disclosures along with its
standalone financial results for the half year
7.10 - Compliance Conditions of Corporate Governance
14. Compliance Conditions of Corporate Governance
M/s FCA & Associates, Chartered Accountants is one of the leading auditing firms in Guwahati. The firm
received an assignment to examine the compliance conditions [as stated in SEBI (LODR) Regulations] of
corporate governance by ABC Ltd., a listed entity with no outstanding SR equity shares. The firm had made
the following observations:
Observation No. 1: Mr. Fine, one of the Director of the Company, also the Chairman of the Stakeholder
Relationship Committee, was acting as the audit committee Chairman in 4 other listed companies as well & 1
private company, simultaneously.
Observation No. 2: The Nomination & Remuneration Committee consisted of 6 members, which regularly
met biannually.
Observation No. 3: The Risk Management Committee consisted of 9 directors, out of which, the number of
independent directors is the majority, but it was less than two thirds of the total strength.
Which among the above three observations made by the auditor of ABC Ltd. should be reported by M/s. FCA
& Associates? (RTP-Nov-2020)

156 Ab Audit Hoga Sabse Scoring - By CA. Sarthak Niraj Jain


Audit Committee & Corporate Governance

Ans. (i) Observation No. 1 -Mr. Fine, one of the Director of the Company, also the Chairman of the
Stakeholder Relationship Committee, was acting as the Audit committee Chairman as well in 4 other
listed companies & 1 private Company, simultaneously:
 As per Regulation 26 of SEBI (LODR) Regulations:- A Director cannot be a Chairman in more
than 5 committees across all listed entities. However, for the purpose of reckoning the limit under
this Regulation, chairmanship of committees in a private company shall be excluded.

 Conclusion:- In this case, since Mr. Fine is the Chairman of audit committee in ABC Ltd. and
Chairman in 4 other listed companies, there is no violation of the limit specified under the
Regulation 26. Accordingly, this observation need not be reported by the auditor.
(ii) Observation No. 2:- The Nomination & Remuneration Committee consisted of 6 members, who
regularly met biannually:
 As per Regulation 19, Part D of Schedule II of SEBI (LODR) Regulations:- Every listed company
should have a Nomination & Remuneration Committee, which shall meet at least once in a year.
 Conclusion:- Since, in the given case the committee met biannually (i.e. once in 2 years), the said
observation needs to be reported by the auditor.
(iii) Observation No. 3: The Risk Management committee consisted of 9 directors, out of which the
number of independent directors is the majority, but it was less than two thirds of the total strength:
 As per Regulation 21 of SEBI (LODR) Regulations:- Only in case of a listed entity having
outstanding SR equity shares, at least two thirds of Risk Management Committee shall
comprise of independent directors.
 Conclusion:- In the given case, ABC Ltd. does not have outstanding SR equity shares. Accordingly,
this observation need not be reported by the auditor.
 Thus, in the given scenario, only observation 2 will be reported
15. Adverse or Qualified Statement or Disclosure in respect of Non-Compliance
A listed entity has to obtain a compliance certificate from either the statutory auditors or practicing company

certificate.
FAST
secretaries regarding compliances of conditions of corporate governance and annex it to the Director’s
Report. Discuss some situations which may require an adverse or qualified statement in respect of the above

OR
(Nov-2019-Old)

CA.Sarthak Niraj Jain


Discuss any eight (8) adverse or qualified statement or disclosure, which you would like to make in respect of
non-compliance with requirements of Corporate Governance of a company.
OR
(May-2018-Old)

Some situations may require an adverse or qualified statement or a disclosure without necessarily making it
a subject matter of qualification in the auditor’s certificate in respect of compliance of requirement of
corporate governance. Give four examples of such situations. (MTP-Nov-2018)
OR
The auditor of Mould Limited made an adverse statement in his certificate as the Audit Committee of the
company did not meet four times a year. Discuss few circumstances which require an adverse or qualified
statement in the auditor’s certificate in respect of compliance of the requirements of Corporate Government
(Nov-2016)
Ans. (i) Number of Non-Executive Directors:- The number of non-executive directors is less than 50%
of the strength of Board of directors.
(ii) Audit Committee:- A qualified and independent audit committee is not set up.
(iii) Chairman:- The Chairman of the audit committee is not an independent director.
(iv) Meeting:- The Audit Committee does not meet 4 times a year.
(v) Powers:- The necessary powers in terms of Part C of Schedule II have not been vested by the
Board in the Audit Committee.
(vi) Time gap:- The time gap between 2 Board meetings is more than 120 days.
(vii) Directorships:- A director is a member of more than 10 committees or acts as Chairman of more
than 5 committees across all companies in which he is a director.
(viii) Quarterly Results:- The information of quarterly results is neither put on the listed entity’s
website nor sent in a form so as to enable the stock exchange on which the entity’s securities are
listed to enable such stock exchange to put it on its own website.
(ix) Transfer of share:- The power of share transfer is not delegated to an officer or a committee or
to the registrar and share transfer agents.

CA Final Audit - By CA. Sarthak Niraj Jain 157


Audit of Consolidated Financial Statements

Chapter
8
AUDIT OF CONSOLIDATED FINANCIAL STATEMENTS

Past Exams Weightage


5

0
MAY-19 NOV-19 NOV-20 JAN-21 JUL-21 DEC-21

May-19 Nov-19 Nov-20 Jan-21 Jul-21 Dec-21


Marks 4 0 5 5 0 0

1.
FAST 8.1- CFS- Applicability & Section 129(3)
Consolidated Financial Statements : Applicability (FR Based Question)

CA.Sarthak Niraj Jain


R Ltd. owns 51% voting power in S Ltd. It however, holds and discloses all the shares as "Stock-in-trade" in its
financial statements since the shares are held exclusively with a view to their subsequent disposal in the near
future. R Ltd. represents that while preparing Consolidated Financial Statements, S Ltd. can be excluded from
the consolidation. As the Statutory Auditor of R Ltd, how would you deal when the consolidated financial
statements are to be drawn up in compliance with Ind AS?
(Study Material, RTP-May-2015, May-2015, May-2019-Old)
OR
M Ltd. acquired 51% shares of S Ltd. on 01.04.2021 and sold 25% of these shares during the financial year
2021-22. M Ltd. did not prepare Consolidated Financial Statements for the financial year 2021-22 on the plea
that the control was only temporary. Do you agree with the view of M Ltd.? Decide, assuming that M Ltd. is
required to prepare its financial statements under Ind AS.
(Study Material, May-2017, May-2018, Nov-2019-Old)
Ans. (i) Provision:- As per Section 129(3) of the Companies Act, 2013,
 Where a company has one or more subsidiaries, including associate company and joint
venture,
 It shall, in addition to its own financial statements prepare a consolidated financial statement of
the company and of all the subsidiaries in the same form and manner as that of its own.
(ii) As per Companies (Accounts) Rules, 2014:
 The consolidation of financial statements of the company shall be made in accordance with the
provisions of Schedule III to the Act and the applicable accounting standards.
 However, a company which is not required to prepare consolidated financial statements under
the Accounting Standards, it shall be sufficient if the company complies with provisions on
consolidated financial statements provided in Schedule III of the Act.

158 Ab Audit Hoga Sabse Scoring - By CA. Sarthak Niraj Jain


Audit of Consolidated Financial Statements
(iii) In case of Investment Company:- An investment entity need not present consolidated financial
statements if it is required, in accordance with Ind AS 110 ‘Consolidated Financial Statements’, to
measure all of its subsidiaries at fair value through profit or loss. A parent shall determine
whether it is an investment entity.
An investment entity is an entity that:
(a) obtains funds from one or more investors for the purpose of providing those investor(s) with
investment management services;
(b) commits to its investor(s) that its business purpose is to invest funds solely for returns from
capital appreciation, investment income, or both; and
(c) measures and evaluates the performance of substantially all of its investments on a fair value
basis.
(iv) In the given case:- R Ltd. owns 51% voting power in S Ltd. It however, holds and discloses all the
shares as "Stock-in-trade" in its financial statements since the shares are held exclusively with a view to
their subsequent disposal in the near future. R Ltd. represents that while preparing Consolidated
Financial Statements, S Ltd. can be excluded from the consolidation.
(v) Conclusion:- As per Ind AS 110, there is no exemption for temporary control where the shares are
held as stock in trade, even though there is an exemption under AS 21. Hence consolidation is
mandatory for Ind AS compliant financial statement in this case.
Therefore, R Ltd. is required to prepare Consolidated Financial Statements including S Ltd. as a
subsidiary company in accordance with Ind AS 110. The auditor is required to ensure the compliance of the
same. In case the management does not agreed for the same the auditor may modify the report accordingly.
2. Consolidated Financial Statements: Applicability (FR Based Question)
H Limited is an Investment Company preparing its Financial Statements in accordance with Ind AS. The
FAST
Company obtains funds from various investors and commits its performance for fair return and capital
appreciation to its investors. During the year under audit, it had been observed that the Company had

CA.Sarthak Niraj Jain


invested 25% in S1 Ltd., 50% in S2 Ltd. and 60% in S3 Ltd. of the respective share capitals of the Investee
Companies. When checking the investment schedule of the Company, an issue cropped as to whether there
would arise any need to consolidate accounts of any such investee companies with those of H Limited in
accordance with section 129(3) of the Companies Act, 2013 which contains no exclusion from consolidation.
Analyse the issues involved and give your views. (RTP-Nov-2018, Nov-2018-New, Study Material)
Ans. (i) Provision:- As per Section 129(3) of the Companies Act, 2013,
 Where a company has one or more subsidiaries, including associate company and joint
venture,
 It shall, in addition to its own financial statements prepare a consolidated financial statement of
the company and of all the subsidiaries in the same form and manner as that of its own.
(ii) As per Companies (Accounts) Rules, 2014:
 The consolidation of financial statements of the company shall be made in accordance with the
provisions of Schedule III to the Act and the applicable accounting standards.
 However, a company which is not required to prepare consolidated financial statements under
the Accounting Standards, it shall be sufficient if the company complies with provisions on
consolidated financial statements provided in Schedule III of the Act.
(iii) In case of Investment Company:- An investment entity need not present consolidated financial
statements if it is required, in accordance with Ind AS 110‘Consolidated Financial Statements’, to
measure all of its subsidiaries at fair value through profit or loss. A parent shall determine whether
it is an investment entity.
An investment entity is an entity that:
(a) obtains funds from one or more investors for the purpose of providing those investor(s) with
investment management services;

CA Final Audit - By CA. Sarthak Niraj Jain 159


Audit of Consolidated Financial Statements
(b) commits to its investor(s) that its business purpose is to invest funds solely for returns from
capital appreciation, investment income, or both; and
(c) measures and evaluates the performance of substantially all of its investments on a fair
value basis.
(iv) In the given case:- H Limited is an investment company preparing its financial statements in
accordance with Ind AS and the company had invested 25% in SI Ltd., 50% in S2 Ltd. and 60% in S3
Ltd. of the respective share capitals of the investee companies. In view of provisions discussed in Ind
AS 110, the Company is not required to prepare consolidated financial statements however, for the
compliance of Companies (Accounts) Rules, 2014, it shall be sufficient if the company complies with
provisions on consolidated financial statements provided in Schedule III of the Act.
(v) Conclusion:- Thus, it can be concluded that ultimate authority on consolidation is AS / Ind AS as
pre scribed by law and if they give some exemption it should be followed. If out of exemption some
subsidiaries are not consolidated then list should be disclosed in notes to accounts with reason.
8.2 - Responsibility of Auditor in case of Consolidation
3. Identification of Completeness of Information in CFS
CA. Vimal is the auditor of Excellent Ltd., a parent company which presents Consolidated Financial
Statements. The management of Excellent Ltd. has provided the list of the components included in the
Consolidated Financial Statements. As an auditor of Consolidated Financial Statements, CA Vimal has to verify
that all the components have been included in the Consolidated Financial Statements and review the
information provided by the management in identifying the components. State the procedures to be followed
by CA. Vimal in respect of completeness of this information. (Nov-2020-New)
Ans. Procedures to be followed by auditor of Parent Entity: Auditor should review the information provided

FAST
by the management of the parent identifying the subsidiaries, associates and joint ventures.
In respect of completeness of this information,
the auditor should perform the following procedures:

CA.Sarthak Niraj Jain


(a) review his working papers for the prior years for the known subsidiaries, associates & joint ventures;
(b) review the parent’s procedures for identification of subsidiaries and joint ventures;
(c) review the investments to determine the shareholding in other entities;
(d) review the joint venture and other relevant agreements entered into by the parent;
(e) review the statutory' records maintained by the parent, for example register required under section
186 of the Companies Act, 2013.
(f) The auditor should also identify the changes in the shareholding that might have taken place since the
last audit
4. CFS: Audit Procedures
A Ltd. holds the ownership of 10% of voting power and control over the composition of Board of Directors of
B Ltd. While planning the statutory audit of A Ltd., what factors would be considered by you as the statutory
auditors of A Ltd for the audit of its consolidated financial statements prepared under Ind AS?
(MTP-May-2021, Study Material, May-2015)
OR
Alfa Ltd. holds the ownership of 10% of voting power and control over the composition of Board of Directors
of Gama Ltd. While planning the statutory audit of Alfa Ltd., what factors would be considered by you as the
statutory auditors of Alfa Ltd for the audit of its consolidated financial statements? (RTP-Nov-2017)
Ans. (i) Analysis of the case:
 In this case, A Ltd. holds only 10 percent of the voting power but has control over the
composition of the Board of Directors of B Ltd.
 In such a case, A Ltd shall be considered as a parent of B Ltd and, therefore, it would
consolidate B Ltd in its consolidated financial statements as a subsidiary.

160 Ab Audit Hoga Sabse Scoring - By CA. Sarthak Niraj Jain


Audit of Consolidated Financial Statements
(ii) Audit procedure:
 Obtain understanding:- The auditor should make plan, among other things, for the
understanding of accounting policies of the A Ltd and B Ltd and determining and
programming the nature, timing, and extent of the audit procedures to be performed etc.
 As per Ind-as-110/AS-21:- The auditor should verify A Ltd’s management’s assessment of
having control in B Ltd despite having only 10% voting power as per the requirements of Ind AS
110/AS-21.
 Control over Board:- Auditor would need to verify as to how A Ltd controls the composition of
the Board of Directors or corresponding governing body of B Ltd.
 There can be various means by which such kind of control can be established.
 In this regard, the auditor may verify: the minutes of Board meetings, shareholder agreement
entered into by the parent, agreements with B Ltd to which the parent might have provided any
technology or know how, enforcement of statute, etc.
 Adjustments as per Ind-AS/AS-The auditor should verify that the adjustments warranted by
Ind AS 110/AS-21 have been made wherever required and have been properly authorised by the
management of the parent. The preparation of consolidated financial statements gives rise to
permanent consolidation adjustments and current period consolidation adjustments.
 IndAS-24/AS-18:- The duties of an auditor with regard to reporting of transactions with any
other related parties as required by IndAS-24/AS-18are given in SA 550 on Related Parties
 As per SA 550 on, “Related Parties”:- The auditor should review information provided by the
management of the entity identifying the names of all known related parties. A person or other
entity that has control or significant influence, directly or indirectly through one or more

FAST
intermediaries, over the reporting entity are considered as Related Party.
(iii) Conclusion:- In forming an opinion on the financial statements:
 The auditor shall evaluate whether the identified related party relationships and transactions have

CA.Sarthak Niraj Jain



been appropriately accounted for and disclosed in accordance with Ind AS 110/AS-21 and
Schedule III and
Whether the effects of the related party relationships and transactions prevent the financial
statements from achieving
 True and fair presentation (for fair presentation frameworks) or
 Cause the financial statements to be misleading (for compliance frameworks).
8.3 - Permanent & Current Period Consolidation adjustment
5. Current Period Consolidation Adjustments
H Limited, a company registered with SEBI, has three subsidiaries and one associate. While doing the audit of
Consolidated Financial Statement (CFS) of H Limited you have come to know that the associate entity had
made a provision dividend in its financial statement. H Limited computed its share of the results of operations
of the associate after taking into account the proposed dividend. Comment. (FR Based Question)(RTP-
Nov-2015)
Ans. (i) As per AS 23 “Accounting for Investments in Associates in Consolidated Financial Statements”
requires that in case an associate has made a provision for proposed dividend in its financial statements,
the investor’s share of the results of operations of the associate is computed without taking in to
consideration the proposed dividend.
(ii) In the present case:- H Limited, a company registered with SEBI, has three subsidiaries and one
associate. While doing the audit of Consolidated Financial Statement (CFS) of H Limited you have come
to know that the associate entity had made a provision dividend in its financial statement. H Limited
computed its share of the results of operations of the associate after taking into account the
proposed dividend.

CA Final Audit - By CA. Sarthak Niraj Jain 161


Audit of Consolidated Financial Statements
(iii) Conclusion:- The consolidated financial statements are prepared in violation of AS-23 to the extent
that share of result of operations of associate is computed after taking into account the proposed
dividend. Auditor is required to state the matter and quality the report accordingly.
SJ Note:- IND-AS-27 deals with equity method of accounting for associate
6. Losses in Investment in Individual Subsidiaries
K Ltd, had 5 subsidiaries as at 31st march, 2022 and the investments in subsidiaries are considered as long
term and valued at cost. Two of the subsidiaries net worth recorded as at 31 st march, 2022 and the prospects
of their recovery are very bleak and the other three subsidiaries are doing exceptionally well. The company
did not provide for the decline in the value of the investment in two subsidiaries because the overall
investment portfolio in subsidiaries did not suffer any decline as the other 3 subsidiaries are doing
exceptionally well. Comment. (FR Based Question)(MTP-May-2016, Study Material)
Ans. (i) As per AS-13 Accounting for Investments:
 Issued by ICAI, long term investments are usually of individual importance to the investing
enterprise. The carrying amount of long term investment in therefore determined on an individual
investment basis.
 Investments classified as long term investments should be carried in the financial statements at
cost .
 Provision for diminution shall be made to recognize a decline, other than temporary, in the
value of the investments, such reduction being determined and made for each investment
individually.
(ii) In the given case:- K Ltd, had 5 subsidiaries as at 31st march, 2015 and the investments in subsidiaries
are considered as long term and valued at cost. Two of the subsidiaries net worth recorded as at 31st

FAST
march, 2015 and the prospects of their recovery are very bleak and the other three subsidiaries are
doing exceptionally well. The company did not provide for the decline in the value of the investment
in two subsidiaries because the overall investment portfolio in subsidiaries did not suffer any decline as

CA.Sarthak Niraj Jain


the other 3 subsidiaries are doing exceptionally well.
(iii) Conclusion:- As per the facts given above, K Ltd. should provide for the decline in the value of
investment in the subsidiaries despite the fact that the overall investment portfolio in subsidiaries did
not suffer any decline.
7. CFS: Harmonizing Accounting Policies
H Co. Ltd., is a holding company with two subsidiaries R Co. Ltd., and S Co. Ltd. The H Co. Ltd., adopts straight
line method of depreciation for its assets whereas S Co. Ltd., follows written down value or diminishing value
method. Though R Co. Ltd., follows straight line method of depreciation, it does not give effect to component
accounting of depreciation in respect of high value assets. While consolidating the financials of the R Co. Ltd.,
and S Co. Ltd., with those of H Co. Ltd., determine the possible issues that you have to ensure for compliance in
the light of above facts. (FR Based Question)(May-2018-New)
Ans. (i) When the Components Auditor Reports on FS under an Accounting Framework is Different than
that of the Parent:
 A component may alternatively prepare financial statements on the basis of the parent’s
accounting policies, as outlined in the group accounting manual, to facilitate the preparation of
the group’s consolidated financial statements.
 The group accounting manual would normally contain all accounting policies, including
relevant disclosure requirements, which are consistent with the requirements of the financial
reporting framework under which the group’s consolidated financial statements are prepared.
 Thus, using group accounting policies as the financial accounting framework for components to
report under, the principal/parent auditors should perform procedures necessary to
determine compliance of the group accounting policies with the GAAP applicable to the
parent’s financial statements.

162 Ab Audit Hoga Sabse Scoring - By CA. Sarthak Niraj Jain


Audit of Consolidated Financial Statements
(ii) As Per Ind-AS-8:-
 It may be noted that change in the selection of the method of depreciation is an accounting
estimate and not an accounting policy as per Ind-AS 8.
 Accordingly, the entity should select the method that most closely reflects the expected pattern of
consumption of the future economic benefits embodied in the asset.
 That method should be applied consistently from period to period unless there is a change in the
expected pattern of consumption of those future economic benefits in separate financial statements
as well as consolidated financial statements.
 Therefore, there can be different methods for calculation of depreciation for its assets, if their
expected pattern of consumption is different. The method once selected in the stand- alone financial
statements of the subsidiary should not be changed while preparing the consolidated financial
statements.
(iii) In the given case:-
 Assets of R Co. Ltd. (subsidiary company) is depreciated using straight line method, assets of S
Co. Ltd. (subsidiary company) are depreciated using written down value method and assets of
parent company (H Co. Ltd.) are depreciated using straight line method, is in order.
 However, each part of an item of Property Plant and Equipment with a cost that is significant in
relation to the total cost of the item should be depreciated separately under Component
Method of Depreciation as per AS 10 on Property, Plant and Equipment.
(iv) Conclusion:- Thus, R Co. Ltd., though adopting straight line method but does not giving effect to
component accounting of depreciation in respect of high value assets, is not in compliance with
Ind AS 16/ Accounting Standard 10 Property Plan and Equipment.
8.
FAST
Consolidation of Adjustments for Current Period
The adjustment required for preparation of consolidated financial statements are made in memorandum
records kept for the purpose, by the Parent. The auditor should review the memorandum records to verify

CA.Sarthak Niraj Jain


the adjustment entries made in the preparation of consolidated financial statements. Elucidate the other
points, apart from reviewing the memorandum records, the auditor should verify while consolidation of
adjustments for current period. (July-2021-Old)
Ans. The adjustments required for preparation of consolidated financial statements are made in memorandum
records kept for the purpose by the parent. The auditor should review the memorandum records to verify
the adjustment entries made in the preparation of consolidated financial statements. Apart from
reviewing the memorandum records, the auditor should inter alia:
(i) verify that the intra group transactions and account balances have been eliminated;
(ii) verify that the consolidated financial statements have been prepared using uniform accounting
policies for like transactions and other events in similar circumstances;
(iii) verify that adequate disclosures have been made in accordance with AS 21 in the consolidated
financial statements of application of different accounting policies in case, it was impracticable to
harmonize them. Applying a requirement is impracticable when the entity cannot apply it after making
every reasonable effort to do so but while preparing CFS under Ind AS, auditors should ensure that
appropriate adjustments are made to that group member’s financial statements in preparing the
consolidated financial statements to ensure conformity with the group’s accounting policies in
accordance with Ind AS 110;
(iv) verify the adjustments made to harmonise the different accounting policies including adjustments
made by management to convert a component’s financial statements prepared under the component’s
GAAP to the GAAP under which the consolidated financial statements are prepared;
(v) verify the calculation of minorities/non-controlling interest;
(vi) verify adjustments relating to deferred tax on account of temporary differences arising out of
elimination of profit and losses resulting from intergroup transactions (where the parent’s accounts
are maintained in Ind AS);

CA Final Audit - By CA. Sarthak Niraj Jain 163


Audit of Consolidated Financial Statements
(vii) verify that income and expenses of the subsidiary are included in consolidated financial
statements from the date it gains control until the date when the entity ceases to control the subsidiary
and further such income and expenses are based on the amounts of the assets and liabilities recognised
in consolidated financial statements at the acquisition date (where the consolidated financial
statements are prepared under Indian Accounting Standards.)
The auditor should gain an understanding of the procedures adopted by the management of the
enterprise to make the abovementioned adjustments. This helps the auditor in reducing the audit risk to
an acceptably low level.
8.4 - Reporting in case of CFS
9. Reporting : Comprehensive
You are appointed as an auditor of Nawab Limited, a listed company which is a main supplier to the UK
building and construction market. With a turnover of ` 2.9 billion, the company operates through 11 business
units and has nearly 180 branches across the countries.
As an auditor, how will you draft the report in case:
(a) When the Parent’s Auditor is also the Auditor of all its Components?
(b) When the Parent’s Auditor is not the Auditor of all its Components?
(c) When the Component(s) Auditor Reports on Financial Statements under an Accounting Framework
Different than that of the Parent?
(d) When the Component(s) Auditor Reports under an Auditing Framework Different than that of the
Parent?
(e) Where the financial statements of one or more components are not audited?
(RTP-May-2022)(Study Material, MTP-May-2018, RTP-May-2017)

FAST OR
You are appointed as an auditor of Azad Limited, a listed company which is a main supplier to the USA
building and construction market. With a turnover of `1.6 billion, the company operates through 9 business

CA.Sarthak Niraj Jain


units and has nearly 135 branches across the countries. As an auditor, how will you draft the report in case
(i) When the Parent’s Auditor is also the Auditor of all its Components? and
(ii) When the Parent’s Auditor is not the Auditor of all its Components? (MTP-May-2020)
OR
You are appointed as an auditor of Najib Limited, a listed company which is a main supplier to the USA
building and construction market. With a turnover of ` 1.9 billion, the company operates through 11 business
units and has nearly 170 branches across the countries. As an auditor, how will you draft the report in case
(I) When the Component(s) Auditor Reports on Financial Statements under an Accounting Framework
Different than that of the Parent?
(II) When the Component(s) Auditor Reports under an Auditing Framework Different than that of the
Parent?
(RTP-Nov-2018, MTP-Nov-2018)
Ans. (a) Parent Auditor is also the auditor of all of its components
 Auditor should issue an audit report expressing opinion whether the consolidated financial
statements give a true and fair view of the state of affairs of the Group as on balance sheet date
and as to whether consolidated profit and loss statement gives true and fair view of the results
of consolidated profit or losses of the Group for the period under audit.
 Where the consolidated financial statements also include a cash flow statement, the auditor should
also give his opinion on the true and fair view of the cash flows presented by the consolidated
cash flow statements.
 Auditor of Parent should report whether principles and procedures for preparation and
presentation of consolidated F.S. as laid down in the relevant AS(s) have been followed.
 In case of any deviation, the auditor should make adequate disclosure in the audit report so that
users of the consolidated F.S. are aware of such deviation.

164 Ab Audit Hoga Sabse Scoring - By CA. Sarthak Niraj Jain


Audit of Consolidated Financial Statements

(b) Parent’s Auditor is not the Auditor of all of its components:-


 If the parent’s auditor is not the auditor of the components included in the consolidated F.S., the
auditor of the consolidated F.S. should also consider the requirement of SA 600.
 If the parent’s auditor decides that he will make reference to the audit of the other auditors in the
report as required by SA 706, he should disclose clearly the portion of the F.S. audited by the
other auditor(s). This may be done by stating:
 the amount or %age of total assets and
 total revenue of subsidiary(s) included in consolidated F.S. not audited by him.
 It is to be noted that reference in the report of the auditor of consolidated F.S. to the fact that part
of the audit of the group was made by other auditor(s) is not to be construed as a qualification of
the opinion but rather as an indication of the divided responsibility between the auditors of the
parent and its subsidiaries.
(c) Component Auditor Reports on F.S. under an Accounting Framework different than that of the
Parent
 When a component’s F.S. are prepared under an accounting framework that is different than that of
the framework used by the parent in preparing group’s consolidated F.S., the parent’s management
perform a conversion of the components’ audited F.S. from the framework used by the
component to the framework under which the consolidated F.S. are prepared.
 The conversion adjustments are audited by the principal auditor to ensure that the financial
information of the component(s) is suitable and appropriate for the purposes of consolidation.
 Alternatively, component may prepare financial statements on the basis of the parent’s

FAST
accounting policies, as outlined in the group accounting manual. The local component auditor can
then audit and issue an audit report on the components F.S. prepared in accordance with “group
accounting policies”.

CA.Sarthak Niraj Jain


 The Principal auditor can then decide whether or not to rely on the components’ audit report
and make reference to it in the auditor’s report on the consolidated financial statements.
(d) Component Auditor Reports under an Auditing Framework Different than that of the Parent
 Audits of F.S., including consolidated F.S., are performed under auditing standards generally
accepted in India.
 In order to maintain consistency of the auditing framework and to enable the parent auditor to
rely and refer to the other auditor’s audit report in their audit report on the consolidated F.S., the
components’ F.S. should also be audited under a framework that corresponds to Indian Auditing
Standards.
(e) Components Not Audited
 F.S. of all components included in consolidated F.S. should be audited or subjected to audit
procedures. Such audits and audit procedures can be performed by the auditor reporting on the
C F.S. or by the components’ auditor.
 Where the F.S. of one or more components continues to remain unaudited, the auditor reporting
on the consolidated F.S. should consider unaudited components in evaluating a possible
modification to his report on the consolidated F.S.
 The evaluation is necessary because the auditor has not been able to obtain sufficient
appropriate audit evidence in relation to such consolidated amounts/balances.
 Auditor should evaluate both qualitative and quantitative factors on the possible effect of such
amounts remaining unaudited when reporting on the consolidated F.S. using the guidance
provided in SA 705, “Modifications to the Opinion in the Independent Auditor’s Report”

CA Final Audit - By CA. Sarthak Niraj Jain 165


Audit of Consolidated Financial Statements

10. Reporting: Components Not Audited in case of CFS


RAO & Co., a Chartered Accountant Firm, is appointed as the principal auditor of a listed company, Triumph
Ltd. Figures of income and net-worth of five out of seven components of Triumph Ltd., which are its unlisted
subsidiaries, is tabulated below for the immediate preceding financial year along with the consolidated
amount:-
(` in crores)
Particulars Consolidated Component Component Component Component Component
‘A ’ ‘B’ ‘C’ ‘D’ ‘E’
Income 300 35 10 70 65 20
Net Worth 800 40 20 140 180 50

The remaining two components i.e., Component ‘F’ & Component ‘G’ of Triumph Ltd. were unaudited.
According to Mr. RAO, the engagement partner, Component ‘F’ is material to the consolidated financial
statements whereas Component ‘G’ is not material to consolidated financial statements and this fact has also
been discussed in writing with those charged with governance of Triumph Ltd. and it will also form part of
report as a ‘Key audit matter’ in accordance with SA 701.
What shall be the audit consideration in relation to reporting in case of unaudited components of Triumph
Ltd. by RAO & Co. and how RAO & Co. as a principal auditor shall report in case of Component ‘F’ &
Component ‘G’, respectively? (MTP-May-2021)
Ans. (i) Provisions related to audit of CFS
 Generally, the financial statements of all components included in consolidated financial
statements should be audited or subjected to audit procedures in the context of a multilocation


FAST
group audit.
Such audits and audit procedures can be performed by the auditor reporting on the consolidated

CA.Sarthak Niraj Jain



financial statements or by the components’ auditor.
Where the financial statements of one or more components continue to remain unaudited, the
auditor reporting on the consolidated financial statements should consider unaudited components
in evaluating a possible modification to his report on the consolidated financial statements.
 The evaluation is necessary because the auditor (or other auditors, as the case may be) has not been
able to obtain sufficient appropriate audit evidence in relation to such consolidated
amounts/balances.
 In such cases, the auditor should evaluate both qualitative and quantitative factors on the
possible effect of such amounts remaining unaudited when reporting on the consolidated financial
statements using the guidance provided in SA 705, “Modifications to the Opinion in the
Independent Auditor’s Report”.
(ii) In the given situation: - Two out of seven components of Triumph Ltd. have remained unaudited
where Component ‘F’ is material and Component ‘G’ is not material to the consolidated financial
statements. Since Component ‘F’ is material, therefore, it may be assumed that reporting of Key Audit
Matter in accordance with SA 701 is being done for Component ‘F’ and not for Component ‘G’.
(iii) Reporting:
 In case of Component ‘F’: - The Principal Auditor needs to consider its impact on the auditor’s
opinion on the consolidated financial statements of the group, in terms of the principles laid down
in SA 705, Modifications to the Opinion in the Independent Auditor’s Report.
 In case of Component ‘G’: - The principal auditor should make appropriate reporting under the
“Other Matters” paragraph, pursuant to SA 706, Emphasis of Matter Paragraphs and Other
Matter Paragraphs, in the Independent Auditor’s Report.

166 Ab Audit Hoga Sabse Scoring - By CA. Sarthak Niraj Jain


Audit of Consolidated Financial Statements

11. Reporting: Opinion Paragraph


ALM Associates has been appointed as auditor of M/s Hary Ltd. which acquired 55% shares-in M/s Sam Ltd.
on 15th October, 2021. During audit of Harry Ltd., the auditors found that the company has not prepared
consolidated financial statements because on the date of acquisition the fair value of certain assets & liabilities
has not been ascertained which is significant and are accounted for on estimated basis only. Help ALM
Associates in framing opinion paragraph of audit report. (May-2019-New)
Ans. (i) As per SA-705:- The auditor shall express an adverse opinion when the auditor, having obtained
sufficient appropriate audit evidences, concludes that misstatements, individually or in the
aggregate, are both material and pervasive to the financial statements.
(ii) Facts of the case:- M/s Hary Ltd. acquired 55% shares in M/s Sam Ltd. and the company did not prepare
the consolidated financial statements because on the date of acquisition the fair value of certain assets
and liabilities has not been ascertained and accounting is done on estimate basis only
(iii) Conclusion:- Therefore, The auditor shall express an adverse opinion because accounting is done
on estimate basis only which is not correct as the financial statements are materially misstated
due to non-consolidation of subsidiary. The material misstatement is deemed to be pervasive to
the consolidated financial statements.
(iv) Forming of opinion
 Adverse Opinion:
 In our opinion and to the best of our information and according to the explanations
given to us, because of the significance of the matter discussed in the Basis for Adverse
Opinion section of our report,
 The accompanying consolidated financial statements do not give a true and fair view in

FAST
conformity with the accounting principles generally accepted in India, of their consolidated
state of affairs of the Group, its associates and jointly controlled entities, as at March 31, 2019,
of its consolidated profit/loss, (consolidated position of changes in equity) and the

CA.Sarthak Niraj Jain


consolidated cash flows for the year then ended.
 Basis for Adverse Opinion is given below:
 As explained in Note X, the M/s Hary Ltd. has not consolidated subsidiary M/s Sam Ltd.
that the M/s Hary Ltd acquired during 2018 because it has not yet been able to determine
the fair values of certain of the subsidiary’s material assets and liabilities at the acquisition
date. This investment is therefore accounted for on an estimate basis.
 Under the accounting principles generally accepted in India, the Group should have
consolidated this subsidiary and accounted for the acquisition based on provisional
amounts. Had M/s Sam Ltd. been consolidated, many elements in the accompanying
consolidated financial statements would have been materially affected. The effects on the
consolidated financial statements of the failure to consolidate have not been determined.

Notes

CA Final Audit - By CA. Sarthak Niraj Jain 167


Audit of Banks

Chapter
9
AUDIT OF BANKS

Past Exams Weightage


5

0
MAY-19 NOV-19 NOV-20 JAN-21 JUL-21 DEC-21

May-19 Nov-19 Nov-20 Jan-21 Jul-21 Dec-21

FAST
Marks 4 5 5 5 5 5

1.
CA.Sarthak Niraj Jain 9.1 - Audit of Banks - Basic & Legal Framework
Bank Audit – Audit Report & Reporting under Co. Act, 2013
M/s GH & Associates have been appointed as Central Statutory Auditors of BNB Bank a nationalized bank,
headquartered in New Delhi for the F.Y. 2021-2022. Bank functions in automated environment using “FLC
Software”. While preparing audit report, one of the partner highlighted that some matters covered by
Companies Act, 2013 and the requirements of Companies (Auditor's Report), Order, 2020 reporting. You are
required to answer the following:
(i) To which authority auditors should submit their audit report
(ii) List the matters covered under Companies Act, 2013 and
(iii) Reporting under Companies (Auditor's Report), Order, 2020. (July-2021-New)
Ans. (i) Authority to whom Auditors to submit their Audit Report - In the case of a nationalised bank, the
auditor is required to make a report to the Central Government. So, GH & Associates, Central Statutory
Auditors of BNB Bank- a nationalized bank, would be required to submit their report to Central Govt.
(ii) The auditor of a banking company is also required to state in the report the matters covered by Section
143 of the Companies Act, 2013.
1. Report on adequacy and operating effectiveness of Internal Controls over Financial
Reporting in case of banks which are registered as companies under the Companies Act in terms of
Section 143(3)(i) of the Companies Act, 2013 which is normally to be given as an Annexure to
the main audit report as per the Guidance Note on Audit of Internal Financial Controls over
Financial Reporting issued by the ICAI.
2. Report on whether any serious irregularity was noticed in the working of the bank which
requires immediate attention (in accordance with sec 143(12) of the Companies Act, 2013.)

168 Ab Audit Hoga Sabse Scoring - By CA. Sarthak Niraj Jain


Audit of Banks
3. As per reporting requirements cast through Rule 11 of the Companies (Audit and Auditors)
Rules, 2014 the auditor’s report shall also include their views and comments on the following
matters, namely:
(a) Whether the bank has disclosed the impact, if any, of the pending litigations on its financial
position in its financial statements.
(b) Whether the bank has made provision, as required under the law or accounting standards,
for material foreseeable losses, if any, on long term contracts including derivative contracts.
(c) Whether there has been any delay in transferring amounts, required to be transferred to
the Investor Education and Protection Fund by the bank.
(d) (i) Whether the management has represented that, to the best of it’s knowledge and belief,
other than as disclosed in the notes to the accounts, no funds have been advanced or
loaned or invested (either from borrowed funds or share premium or any other sources
or kind of funds) by the company to or in any other person(s) or entity(ies), including
foreign entities (“Intermediaries”), with the understanding, whether recorded in writing
or otherwise, that the Intermediary shall, whether, directly or indirectly lend or invest in
other persons or entities identified in any manner whatsoever by or on behalf of the
company (“Ultimate Beneficiaries”) or provide any guarantee, security or the like on
behalf of the Ultimate Beneficiaries;
(ii) Whether the management has represented, that, to the best of it’s knowledge and belief,
other than as disclosed in the notes to the accounts, no funds have been received by the
company from any person(s) or entity(ies), including foreign entities (“Funding Parties”),
with the understanding, whether recorded in writing or otherwise, that the company shall,
whether, directly or indirectly, lend or invest in other persons or entities identified in any

FAST manner whatsoever by or on behalf of the Funding Party (“Ultimate Beneficiaries”) or


provide any guarantee, security or the like on behalf of the Ultimate Beneficiaries; and
(iii) Based on such audit procedures that the auditor has considered reasonable and
CA.Sarthak Niraj Jain
appropriate in the circumstances, nothing has come to their notice that has caused
them to believe that the representations under sub-clause (i) and (ii) contain any material
mis-statement.
(e) Whether the dividend declared or paid during the year by the company is in compliance
with section 123 of the Companies Act, 2013.
(f) [Whether the company, in respect of financial years commencing on or after the 1 st April, 2022,]
has used such accounting software for maintaining its books of account which has a feature
of recording audit trail (edit log) facility and the same has been operated throughout the
year for all transactions recorded in the software and the audit trail feature has not been
tampered with and the audit trail has been preserved by the company as per the statutory
requirements for record retention.
(iii) Reporting requirements relating to the Companies (Auditor’s Report) Order, 2020 are not applicable
to a banking company as defined in clause (c) of section 5 of the Banking Regulation Act, 1949.
9.2 - Internal Control in Certain Selected Areas
2. Credit Card Operations
You have been appointed as an auditor of OM Bank, a nationalized bank. OM Bank also deals in providing credit
card facilities to its account holder. The bank is aware of the fact that there should be strict control over storage
and issue of credit cards. How will you evaluate the Internal Control System in the area of Credit Card operations
of a Bank? (RTP-May-2022)(Study Material)(MTP-Nov-2020)(Nov-2019-New)
OR
How will you evaluate the Internal Control system in the area of Credit Card operations of a Bank?

CA Final Audit - By CA. Sarthak Niraj Jain 169


Audit of Banks

Ans. The following points should be considered so as to evaluate the internal control system in the area of
credit card operations:
(i) There should be effective screening of applications with reasonably good credit assessments.
(ii) There should be strict control over storage and issue of cards.
(iii) There should be a system whereby a merchant confirms the status of un-utilized limit of a credit-card
holder from the bank before accepting the settlement in case the amount to be settled exceeds a specified
percentage of the total limit of the card holder.
(iv) There should be system of prompt reporting by the merchants of all settlements accepted by them
through credit cards.
(v) Reimbursement to merchants should be made only after verification of the validity of merchant’s
acceptance of cards.
(vi) All the reimbursements (gross of commission) should be immediately charged to the customer’s
account.
(vii) There should be a system to ensure that statements are sent regularly and promptly to the customer.
(viii) There should be a system of monitor and follow-up of customers’ payments.
(ix) Items overdue beyond a reasonable period should be identified and attended to carefully. Credit
should be stopped by informing the merchants through periodic bulletins, as early as possible, to avoid
increased losses.
(x) There should be a system of periodic review of credit card holders’ accounts. On this basis, the limits of
customers may be revised, if necessary. The review should also include determination of doubtful
amounts and the provisioning in respect thereof.
3. System of Internal Controls in Banks

FAST
You are auditing a small bank branch with staff strength of the manager, cashier and three other staff S1, S2 and
S3. Among allocation of work for other areas, S1 who is a peon also opens all the mail and forwards it to the
concerned person. He does not have a signature book so as to check the signatures on important

CA.Sarthak Niraj Jain


communications. S2 has possession of all bank forms (e.g. Cheque books, demand draft/pay order books,
travelers’ cheques, foreign currency cards etc.). He maintains a record meticulously which you have test checked
also. However, no one among staff regularly checks that. You are informed that being a small branch with
shortage of manpower, it is not possible to always check the work and records. Give your comments.
(MTP-Dec-2021)(Study Material)(RTP-May-2021)
Ans. (i) Banks are required to implement and maintain a system of internal controls for mitigating risks,
maintain good governance and to meet the regulatory requirements.
(ii) Given below are examples of internal controls that are violated in the given situation:
 Staff-S1: - Who is a peon opens all the mail and forwards it to the concerned person. Further, he
does not have a signature book so as to check the signatures on important communications is not
in accordance with implementation and maintenance of general internal control.
Comment: - The mail should be opened by a responsible officer. Signatures on all the letters and
advices received from other branches of the bank or its correspondence should be checked by an
officer with the signature book.
 Staff-S2: - Has possession of all bank forms (e.g. cheque books, demand draft/pay order books,
travelers’ cheques, foreign currency cards etc.). He maintains a record meticulously which were also
verified on test check basis.
Comment: -All bank forms (e.g. Cheque books, demand draft/pay order books, travelers’ cheques,
foreign currency cards etc.) should be kept in the possession of an officer, and another responsible
officer should verify the issuance and stock of such stationery.
(iii) Conclusion: -Contention of bank that being a small branch with shortage of manpower they are not able
to check the work and records on regular basis, is not tenable as such lapses in internal control pose
risk of fraud. The auditor should report the same in his report accordingly

170 Ab Audit Hoga Sabse Scoring - By CA. Sarthak Niraj Jain


Audit of Banks

9.3 - Verification of Assets & Balance


4. Money at Call and Short Notice & Non Banking Assets acquired in Satisfaction of Claims
As a statutory auditor of a bank, how would you verify the following:
(a) Money at call and Short Notice.
(b) Non-Banking Assets acquired in satisfaction of claims
Ans. Verification of Money at Call and Short Notice:
(a) Examine whether there is a proper system of authorisation general or specific, for lending of the money
at call or short -notice.
(b) Compliance with the instructions or guidelines laid down in this behalf by the head office or
controlling office of the branch, including the limits on lending’s in inter-bank call money market should
also be examined.
(c) Verify the call loans with the certificates of the borrowers and the call loan receipts held by the bank.
(d) Check whether the aggregate balances of money at call and short notice as shown in the relevant
register agree with the control accounts as per the general ledger.
(e) Examine subsequent repayments received from borrowing banks to verify the amounts shown under
this head as at the year-end.
(f) Verify that the interest has been properly accrued and accounted for on year-end outstanding
balances of call/ short notice money.
Verification of Non-Banking Assets acquired in satisfaction of claims:
(a) Ensure that the heading includes those immovable properties/tangible assets which the bank has
acquired in satisfaction of debts due or its other claims and these are being held with intention of being
disposed off.
(b) Verify such assets with reference to the relevant documentary evidence, e.g., terms of settlement with
FAST
the party, order of the Court or the award of arbitration, etc.
(c) Ensure the ownership of the assets so acquired is legally vested in the bank. If there is any dispute over

CA.Sarthak Niraj Jain


the title of the property, the auditor should examine whether the recording of the asset is appropriate or
not.
(d) Ensure compliance with Section 9 of Banking Regulation Act, on holding period of such assets.
(e) In case the dispute arises subsequently, the auditor should examine whether a provision for liability
or disclosure of a contingent liability is appropriate, keeping in view the requirements of AS 29
"Provisions, Contingent Liabilities and Contingent Assets".
(f) Ensure that as at date of acquisition, the assets should be recorded at lower of net book value of advance
or net realizable value of asset acquired.
5. Bank situated in Foreign Country – Balance Verification
M/s CAS & Associates have been appointed as one of the statutory central auditors of FDMH Bank., for the
Financial Year 2021-22. During the course of the audit, the auditor found that the bank has a balance with a
Zurich based bank. The auditor understands that such balance is a matter of important consideration in the
audit of the bank. Being head-in-charge of the assignment, while planning, you distributed the work among your
team members and assigned Mr. Ansh for verification of Balances in account of the bank situated in foreign
country. However, Mr. Ansh, being fresh to the bank audits, needs your guidance. Kindly guide.
(MTP-May-2020)
OR
As a branch auditor of a nationalized bank, how would you verify the Balances in account of a bank situated in a
foreign country.
Ans. (i) Examine the ledger balances: -In each account with reference to the confirmation certificates
obtained from the bank and reconciliation statements available.
(ii) Review the reconciliation statements to examine the following:
a. No debit for charges or credit for interest is outstanding and all the items which ought to have
been taken to revenue for the year have been considered.

CA Final Audit - By CA. Sarthak Niraj Jain 171


Audit of Banks
b. No cheque sent or received in clearing is outstanding.
c. All bills or outstanding cheques sent for collection and outstanding as on the closing date have
been credited subsequently.
(iii) Examine the large transactions in the accounts:- Particularly near the year-end, to ensure that there is
no manipulation in the account.
(iv) In respect of year-end balance: -Obtain confirmation certificates from the banks and examine the
original deposit receipts.
(v) Ensure compliance of AS 11: -"Accounting for the Effects of Changes in Foreign Exchange Rates". In
regard to conversion of balance into the Indian currency at the exchange rates prevailing on the balance
sheet date.
6. Verification of PMS Transactions
ABN Bank was engaged in the business of providing Portfolio Management Services to its customers, for which it
took prior approval from RBI. Your firm has been appointed as the statutory auditors of the Bank’s financial
statements for the year 2021-22. Your senior has instructed you to verify the transactions of Portfolio
Management Services (PMS). While verifying the transactions you noticed that the bank has not maintained
separate record for PMS transactions from the Bank’s own investments. As a statutory auditor what methodology
will be adopted by you for verification of PMS transactions? (MTP-May-2021)(RTP-Nov-2020)
Ans. Separation of Investment Functions:
(i) Audit procedure: -The auditor needs to examine whether the bank, as required by the RBI, is
maintaining separate accounts for the investments made by it on their own Investment Account, PMS
clients’ account, and on behalf of other Constituents (including brokers).
(ii) As per the RBI guidelines: - Banks are required to get their investments under PMS separately audited

FAST
by external auditors.
(iii) In the instant case: - ABN Bank is required to prepare separate records for PMS and as per RBI
guidelines PMS investments need to be audited separately by the external auditors and the auditors

CA.Sarthak Niraj Jain


are required to give a certificate separately for the same.
(iv) Conclusion: - In the above case the auditor should not verify the PMS transactions and advise the bank
to segregate the PMS transactions from its own investments and provide the certificate of external
auditor as described above. In case ABN Bank does not provide the same the auditor may report
accordingly.
7. Special Purpose Certificates
Write short note on: Special purpose certificates relating to investments required to be issued by Central
Statutory Auditors of a bank.
Ans. Pursuant to RBI‟s circulars, issued from time to time, banks require their central auditors to issue the following
certificates regarding investments of the bank (in addition to their main audit report and the LFAR:
(i) Investments made by it on their own investment account.
(ii) PMS clients account.
(iii) Investment on behalf of other constituents (including brokers).
(iv) Examination of classification and shifting: - Whether shifting of investment from ‘available for sale’ to
‘held to maturity’ is duly approved by BOD of the Bank.
8. Sale of NPA
CA K have been doing audit of branch of LUD Bank Ltd. The principal business of the branch is lending advances
to large corporates. Since last one year, many large accounts have become Non-Performing Asset (NPA) as per
guidelines. The Management of the Bank decided to sell one of the NPA account and consequently one NPA
namely DEF Ltd. amounting to 10.00 Crores was sold to Asset Reconstruction Company. What audit points CA K
should keep in mind while doing audit of this transaction? (Jan-2021-New)
OR
In the course of audit of Skip Bank Ltd., you found that the bank had sold certain of its non-performing assets.
Draft the points of audit check that are very relevant to this area of checking. (Study Material)(May-2018-New)

172 Ab Audit Hoga Sabse Scoring - By CA. Sarthak Niraj Jain


Audit of Banks

Ans. CA K conducting audit of branch of LUD Bank Ltd. whose principal business is lending money to large
corporates. Many large accounts of this branch have turned NPA category and Management sold DEF
Ltd.’s NPA account amounting to ` 10 Crores to Asset Reconstruction Company.
CA K should proceed as under: In case of Sale of NPA by Bank, the auditor should examine:
(i) the policy laid down by the Board of Directors in this regard relating to procedures, valuation and
delegation of powers including non performing financial assets that may be purchased/sold, norms or
such purchase/sale, valuation procedure and accounting policy.
(ii) only such NPA has been sold which has remained NPA in the books of the bank for at least 2 years.
(iii) the assets have been sold “without recourse’ only. i.e the entire credit risk associated with the non-
performing asset should be transferred to the purchasing bank.
(iv) subsequent to the sale of the NPA, the bank does not assume any legal, operational or any other type
of risk relating to the sold NPAs.
(v) the NPA has been sold at cash basis only. Under no circumstances, NPA can be sold to another bank at a
contingent price .The entire sale consideration has to be received on upfront basis.
(vi) on the sale of the NPA, the same has been removed from the books of the account of selling bank on
transfer;
(vii) If the sale is at a price below the net book value (NBV) (i.e., book value less provisions held), the
shortfall should be debited to the profit and loss account of that year.
(viii) If the sale is for a value higher than the NBV, the excess provision shall not be reversed but will be
utilised to meet the shortfall/ loss on account of sale of other non-performing financial assets.
9. Income Recognition w.r.t. Investments
Your firm has been appointed as central statutory auditors of a nationalised bank. The bank has recognised on

FAST
accrual basis income from dividends on securities and Units of Mutual Funds held by it as at the end of financial
year. The dividends on securities and Units of Mutual Funds were declared after the end of financial year.
Comment. (Study Material)
Ans. (i)
CA.Sarthak Niraj Jain
Prudential Norms on income recognition of investments provides the following:
o Income from dividend on shares of corporate bodies and units of mutual funds should be booked on
cash basis.
o In respect of income from government securities and bonds and debentures of corporate bodies,
where interest rates on these instruments are pre-determined, income could be booked on accrual
basis, provided interest is serviced regularly and as such is not in arrears.
o Banks may book income from dividend on shares of corporate bodies on accrual basis, provided
dividend on the shares has been declared by the corporate body in its AGM and the owner's right
to receive payment is established. This is also in accordance with AS-9 as well.
(ii) In the present case: -Dividends were declared after the end of financial year.
(iii) Conclusion: -Recognition of dividend income on securities may be recognized on accrual basis if the
same has been declared by the Corporate Body in its AGM and the Shareholder’s right to receive
payment is established. In this case dividends were declared after the end of financial year. Hence,
recognition of income from dividends on securities and units of mutual fund on accrual basis is not
in order.
10. NPA Classification - Consortium Advances
Your firm has been appointed as auditors of a branch of a nationalised bank. The bank is a consortium member
of Cash Credit Facilities of ` 50 crores to X Ltd. Bank’s own share is ` 10 crores only. During the last two
quarters against a debit of ` 1.75 crores towards interest, the credits in X Ltd.’s account are to the tune of ` 1.25
crores only. Based on the certificate of lead bank, the bank has classified the account of X Ltd. as performing. The
Bank follows financial year as accounting year. Advise your views on the issue which were brought to your
notice by your Audit Manager. (Study Material)(RTP-Nov-2018)
Ans. (i) NPA Classification in Consortium advances:
Prudential Norms on Asset Classification provides that in case of consortium advances, each bank may
classify the advance given by it according to its own experience of recovery and other factors.

CA Final Audit - By CA. Sarthak Niraj Jain 173


Audit of Banks
(ii) In the present case: - In the last two quarters, the amount remains outstanding and, thus, interest
amount should be reversed. Despite the certificate of lead bank to classify the account as performing,
the advance needs to be classified as nonperforming asset.
(iii) Conclusion: -Advance to be classified as NPA.
11. NPA Classification - Consortium Advances
M/s. S Ltd. is a MSME unit. The company does multiple banking. The company is availing cash credit limit from U
bank of ` 25 crores. The limit availed remained less than ` 5.00 crores during all the days of F.Y. 2021-22. The
company has not done any credit in cash credit account during the year as it is operating current account in
newly opened another bank branch adjoining to company premises. The company is having sufficient security of
stocks and debtors and DP of ` 25,00 crores remain all over the year. The co. is availing term loans from other
bank branches. Now the bank manager is insisting to route the sale proceeds through U bank, otherwise cash
credit limit and term loan accounts with other banks will be treated as Non-Performing Accounts, now company
seeks your opinion. (MTP-Nov-2018)(RTP-Nov-2018)
Ans. (i) NPA Classification in Consortium advances:
 An Advance will be classified as NPA if :
(a) It ceases to generate income for a bank.
(b) Interest and /or installment of principal in respect of such an advance have remain overdue or
out of order for a specified period of time
 An amount is said to be ‘Overdue’:
If it is not paid on the due date fixed by the Bank.
 An account should be treated as ‘Out-of-order:
If the outstanding balance remains continuously in excess of the sanctioned limit/drawing power


FAST
or if there are no credits continuously for 90 days as on the balance sheet date or the credits are
not enough to cover the interest debited during the same period.
Prudential Norms on Asset Classification:

(ii)
CA.Sarthak Niraj Jain
Provides that in case of consortium advances, each bank may classify the advance given by it
according to its own experience of recovery and other factors.
In the present case:
 Company is availing cash credit limit from U Bank of ` 25 crores. The limit availed remained less
than ` 5.00 crores during all the days of F.Y. 2018-19.
 The company has not done any credit in cash credit account during the year as it is operating
current account in newly opened another bank branch adjoining to company premises.
 The company is availing term loans from other bank branches.
 Now the Bank Manager is insisting to route the sale proceeds through U Bank, otherwise cash
credit limit and term loan accounts with other banks will be treated as Non-Performing Accounts.
(iii) Conclusion: - Cash credit facility with U bank need to be classified as NPA as there are no credit in the
account to serve the interest charged in the account. Classification of term loan with other banks
depends upon the payment made to that bank.
12. NPA Norms - Agricultural Advances
Shy & Co. had been allotted the branch audit of a nationalized bank for the year ended 31st March, 2022. In the
audit planning, the partner of Shy & Co. observed that the allotted branches are predominantly based in rural
areas and major portion of the advances were for agricultural purpose. He needs your assistance in
incorporating the criteria prescribed for determination of NPA norms in respect of agricultural advance, in audit
plan.
Ans. Criteria for determination of NPA norms in respect of agricultural advances:
 If government guaranteed advance becomes NPA, then for the purpose of income recognition, interest
on such advance should not to be taken to income unless interest is realized.

174 Ab Audit Hoga Sabse Scoring - By CA. Sarthak Niraj Jain


Audit of Banks
 However, for purpose of asset classification, credit facility backed by Central Government Guarantee
though overdue, can be treated as NPA only when the Central Government repudiates its guarantee,
when invoked.
 This exception is not applicable for State Government Guaranteed advances, where advance is to be
considered NPA if it remains overdue for more than 90 days.
 In case the bank has not invoked the Central Government Guarantee though the amount is overdue for
long, the reasoning for the same should be taken and duly reported in LFAR.
13. Agricultural Advances & Working Capital
M/s Aadi & Co., Chartered Accountants, have been allotted the branch audit of a nationalized bank for the year
ended 31st March, 2022. You are part of audit team and have been instructed by your partner to verify the
following areas:
(i) Fulfilment of the criteria prescribed for NPA norms for government guaranteed advance.
(ii) Fulfilment of the criteria prescribed for NPA norms for the advances given for agricultural purposes.
(iii) Drawing power calculation from stock statements in respect of working capital accounts.
(iv) Accounts where regular/ad hoc limits are not reviewed within 180 days from the due date/date of ad hoc
sanction. What may be your areas of concern as regards matters specified above? (MTP-Dec-2021)
Ans. Area of Focus Suggested Audit Procedures
Government Guaranteed  If government guaranteed advance becomes NPA, then for the
Advances purpose of income recognition, interest on such advance should
not to be taken to income unless interest is realized. However, for
purpose of asset classification, credit facility backed by Central
Government Guarantee, though overdue, can be treated as NPA only

FAST when the Central Government repudiates its guarantee, when


invoked.
This exception is not applicable for State Government Guaranteed

CA.Sarthak Niraj Jain advances, where advance is to be considered NPA if it remains


overdue for more than 90 days.
 In case the bank has not invoked the Central Government
Guarantee though the amount is overdue for long, the reasoning for
the same should be taken and duly reported in LFAR.
Agricultural Advances  Ensure that NPA norms have been applied in accordance with the
crop season determined by the State Level Bankers’ Committee in
each State. Depending upon the duration of crops – short term/ long
term - raised by an agriculturist, the NPA norms would also be made
applicable to agricultural term loans availed of by them. Also ensure
that these norms are made applicable to all direct agricultural
advances listed in Master Circular on lending to priority sector.
 In respect of agricultural loans, other than those specified in the
circular, ensure that identification of NPAs has been done on the
same basis as non-agricultural advances.
Drawing Power  Ensure that the drawing power is calculated as per the extant
Calculation guidelines (i.e. the Credit Policy of the Bank) formulated by the
Board of Directors of the respective bank and agreed upon by the
concerned statutory auditors. Special consideration should be given
to proper reporting of sundry creditors and stocks covered under
LCs/guarantees for the purposes of calculating drawing power.
 The stock audit should be carried out by the bank for all accounts
having funded exposure of more than stipulated limit. The report

CA Final Audit - By CA. Sarthak Niraj Jain 175


Audit of Banks
submitted by the stock auditors should be reviewed during the
course of the audit and special focus should be given to the comments
made by the stock auditors on valuation of security and calculation of
drawing power.
 The drawing power needs to be calculated carefully in case of
working capital advances to companies engaged in construction
business. The valuation of work in progress should be ensured in
consistent and proper manner. It also needs to be ensured that
mobilization advance being received by the contractors is reduced
while calculating drawing power.
Limits not reviewed Accounts where regular/ad hoc limits are not reviewed within 180
days from the due date/date of ad hoc sanction, should be considered
as NPA. Auditors should also ensure that the ad hoc/short reviews
are not done on repetitive basis. In such cases, auditor can consider
the classification of account based on other parameters and
functioning of the account.
14. Account Classification- NPA
ABC Bank had sanctioned credit limits of `100 lakh to M/s Volkart Ltd on 1st September 2020. The renewal of
limits was due on 1st September 2021. While doing the statutory branch audit for the year ended 31st March
2022, you find that the renewal has not been done even though 180 days are over. The bank says that the
renewal process has been initiated on time and most of the document are received. The account is operated
regularly and is in order; balance is maintained within drawing power. It also shows a letter from Volkart stating
that due to a sudden death of their auditor, a new auditor had to be appointed. Procedure for appointment took

FAST
some time and the new auditor was doing the audit all over again. The limit was not renewed till 31/3/2022.
However, the audited financials are received on 10th April 2022 and the renewal letter was issued immediately.
Your assistant is insisting that the account must be classified as NPA since the limit was not renewed as on

Ans. (i)
CA.Sarthak Niraj Jain
31/3/2022. What is your opinion?
As per Guidelines of RBI:
(Study Material)

 The account should be classified as NPA if renewal is not done in 180 days.
(ii) In the present case:
 Operations in the account are excellent. The bank has shown a letter from that company that due to
certain reasons the audited financial statements are delayed.
 Further, the limit has been renewed before signing the audit report.
(iii) Conclusion:
 Thus, even if the sanction was issued after the balance sheet date, it relates to the position as on
the balance sheet date. Therefore, it is an adjusting event under AS 4, Contingencies and Events
Occurring After the Balance Sheet Date.
 It is also a matter of substance over form. The auditor would consider classifying the account as a
standard asset.
9.4 - Some Other Important Topics
15. Government Guaranteed Advances
In course of audit of Good Samaritan Bank as at 31st March 2022 you observed the following: in a particular
account there was no recovery in the past 18 months. The bank has not applied the NPA norms as well as
income recognition norms to this particular account. When queried the bank management replied that this
account was guaranteed by the central government and hence these norms were not applicable. The bank has
not invoked the guarantee. Please respond. Would your answer be different if the advance is guaranteed by a
State Government?
(Study Material) (MTP-May-2021)(RTP-May-2020)(RTP-Nov-2019)(MTP-May-2018)

176 Ab Audit Hoga Sabse Scoring - By CA. Sarthak Niraj Jain


Audit of Banks

Ans. (i) Classification of Government Guaranteed advances:


 The credit facilities backed by Central Government guarantee:-
Though overdue will be classified as NPA only when the govt. repudiates its guarantee when
invoked.
 Credit facilities backed by State Government guarantee:-
Should be classified as NPA in normal way.
 This exemption in case of Central Government:-
Is only for the purpose of asset classification and provisioning and not for the purpose of
recognition of income
 Interest on government guaranteed advances :-
Should not be taken to Income Account unless it has been realized.
(ii) In the present case: Bank has not revoked the guarantee, the question of repudiation does not arise.
Therefore, advance need not be classified as NPA in case of Central Government Guarantee. However, as
this exemption is not available in respect of income recognition norms, the income to the extent not
recovered should be reversed.
(iii) Conclusion:
• Bank is correct to the extent of not applying the NPA norms for provisioning purpose. But this
exemption is not available in respect of income recognition norms.
• The situation would be different if the advance is guaranteed by State Government because this
exception is not applicable for State Government Guaranteed advances.
16. Queries raised by Auditor & Actions taken by Bank
Gupta & Co. has been appointed as a statutory auditor of TCB Bank Ltd., a private sector bank, registered with

FAST
RBI. Mr. Kaival Gupta, the engagement partner, while performing the audit as per the checklist, noted down the
following points, which would be part of the audit queries, as tabulated below:
Sr.

CA.Sarthak Niraj Jain


No.
1
Queries

Interest on State Government Guaranteed advance has been taken to income even though such
advance has remained overdue for more than 90 days.
2 There is an account for which an ad hoc limit has not been reviewed for 180 days from the date of
such ad hoc sanction and such account has been treated as a performing asset in the books.
3 One of the NPAs was sold for a value higher than the net book value. Profit was not recognized but
the excess provision in respect of the same has been reversed.
4 In case of one of the accounts, an additional temporary limit has been sanctioned for 25% of the
existing limit and for 120 days tenure.
5 On verification of outstanding forward exchange contracts, the ‘net position’ in respect of one of the
foreign currencies was not squared and was uncovered by a substantial amount.
You are required to provide the reasons due to which such queries would have been raised by Mr. Kaival and
describe the actions that may be taken by the person responsible on behalf of TCB Bank Ltd. for solving such
queries. (RTP-Dec-2021)
Ans.
Sr. Action that may be taken in response to the query
Reason for such Query
No.
1 A State Government Guaranteed advance has Interest income recognized on such advance would be
to be treated as NPA even if it remains overdue reversed and would be taken to income only when it
for more than 90 days and in case of NPA, for is realized.
the purpose of income recognition, interest on
such advance should not be taken to income
unless interest is realized.

CA Final Audit - By CA. Sarthak Niraj Jain 177


Audit of Banks

2 Accounts for which an ad hoc limit has not been It’s treatment in the books would be changed from
reviewed for 180 days from the date of such ad performing asset to a non-performing asset from
hoc sanction, should be considered as NPA. the date when such change in the treatment was
required.
3 If the sale is for a value higher than the NBV, The entry for reversal of the excess provision
the excess provision shall not be reversed but would be cancelled in the books and such excess
will be utilised to meet the shortfall/ loss on provision would be retained to meet the shortfall/
account of sale of other non-performing financial loss that may arise because of the sale of other non-
assets. (updated) performing financial assets.
4 Additional temporary limit may be sanctioned, The terms of additional temporary limit in case of
for a maximum of 20% of the existing limit and such account would be revised to 20% of the existing
90 days maximum tenure. limit and for 90 days maximum tenure.
5 Net position in respect of each of the foreign The net “position” of the branch in relation to each
currencies should be generally squared and foreign currency should be squared off and get
should not be uncovered by a substantial covered by a substantial amount.
amount.
9.5 - Concurrent Audit -Scope & Procedure
17. Verification of Advances
ABC Chartered Accountants have been appointed as concurrent auditors for the branches of Effective Bank Ltd.
for the year 2021-22. You are part of the audit team for Agra branch of the bank and have been instructed by
your senior to verify the advances of the audit period. You are required to guide your assistant about the areas
to be taken care while doing verification during the concurrent audit.
(Study Material)(RTP-May-2019)

FAST OR
Your firm has been appointed as branch auditor of East West Bank Ltd. In carrying out verification of advances,
what are the primary evidences you will look into? (May-2018-Old)
Ans. a)
b)
c)
CA.Sarthak Niraj Jain
Ensure that loans and advances have been sanctioned properly.
Verify whether the sanctions are in accordance with delegated authority.
Ensure that securities and documents have been received and properly charged/ registered.
d) Ensure that post disbursement supervision and follow-up is proper.
e) Verify whether there is any mis-utilisation of the loans and whether there are instances indicative of
diversion of funds.
f) Check whether the letters of credit issued by the branch are within the delegated power and ensure that
they are for genuine trade transactions.
g) Check bank guarantees issued, whether they have been properly worded and recorded.
h) Ensure proper follow-up of overdue bills of exchange.
i) Verify whether the classifications of advances are as per RBI guidelines.
j) Verify whether the submission of claims to DICGC and ECGC is in time.
k) Verify that instances of exceeding delegated powers have been promptly reported
l) Verify the frequency and genuineness of such exercise of authority beyond the delegated powers by the
concerned officials
18. Foreign Exchange Transactions
You have been appointed as Concurrent Auditor of a nationalized bank branch. The main business at the branch
is dealing in foreign exchange. Suggest the main areas of coverage in regard to foreign exchange transactions of
the said branch under concurrent audit. (Study Material)(Nov-2019-Old)
OR
You are the Concurrent Auditor of a Branch of Nationalized Bank which deals in foreign exchange transactions.
Give focus areas of your checking in this respect. (Nov-2018-New)

178 Ab Audit Hoga Sabse Scoring - By CA. Sarthak Niraj Jain


Audit of Banks

Ans. Foreign exchange transactions to be verified with reference to RBI guidelines.


Important points of verification in this respect are:
(i) Check foreign bills negotiated under letters of credit.
(ii) Check Foreign Currency Non-Resident and other non-resident accounts to ensure that only
permissible transactions route through these accounts.
(iii) Check whether inward/outward remittance have been properly accounted for.
(iv) Examine extension and cancellation of forward contracts for purchase and sale of foreign currency.
Ensure that they are duly authorised and necessary charges have been recovered.
(v) Ensure that the overbought/oversold position maintained in different currencies is reasonable
considering the foreign exchange operations.
(vi) Ensure compliance of the guidelines issued by RBI/HO of the bank.
(vii) Ensure that balances in Nostro accounts in different foreign currencies are within the limit as prescribed
by the bank.
(viii) Verify transactions of Nostro and Vostro account and their reconciliation statements.

Notes

FAST
CA.Sarthak Niraj Jain

CA Final Audit - By CA. Sarthak Niraj Jain 179


Audit of Insurance Companies

Chapter
10
AUDIT OF INSURANCE COMPANIES

Past Exams Weightage


5

0
MAY-19 NOV-19 NOV-20 JAN-21 JUL-21 DEC-21

May-19 Nov-19 Nov-20 Jan-21 Jul-21 Dec-21


Marks 5 0 0 4 4 4

FAST 10.1 - Insurance Company - Legal Framework


1.
CA.Sarthak Niraj Jain
Inception of Risk - Section 64VB
While auditing Suryan kiran Insurance Ltd. as on 31st March, 2022, you observed that there is one policy
which has been issued on 25th March, 2022 towards fire risk favouring one of the leading corporate hoses in
the country without the actual receipt of premium and it was reflected as premium receivable. It is the usual
practice maintained by the company in respect of big customers that they would issue the policy before
receiving the premium. The premium money was collected on 5th of April 2022. It is further noticed that
there was a fire accident in the premises of insured on 31st March, 2022 and a claim was lodged. The
insurance company also provided for the same. How would you respond? (May-2016)

Ans. (i) Provision: - Inception of Risk- As per Section 64VB of the Insurance Act, 1938, no insurer should
assume any risk in India in respect of any insurance business on which premium is ordinarily
payable in India unless and until the premium payable is received or is guaranteed to be paid by
such person in such manner and within such time, as may be prescribed, or unless and until deposit of
such amount, as may be prescribed, is made in advance in the prescribed manner.
For the purposes of this section, in the case of risks for which premium can be ascertained in
advance, the risk may be assumed not earlier than the date on which the premium has been paid in
cash or by cheque to the insurer
(ii) In the present case: - Though policy was issued on 25th March, 2022, but premium was collected
only on 5th April, 2022. Applying the provisions of Sec. 64VB, company cannot be held liable for the
claim arises due to fire accident happened on 31st March, 2022.
(iii) Conclusion: - In view of the above, the insurance company is not liable to pay the claim and hence
no provision for claim is required.”

180 Ab Audit Hoga Sabse Scoring - By CA. Sarthak Niraj Jain


Audit of Insurance Companies

2. Inception of Risk – Section 64VB


You have been appointed as an auditor of ABC Insurance Co. Ltd. and found that M/s PQR Ltd. got their plant
& Machinery insured on 01-10-2021 but the amount of premium has been paid by them on 15-10-2022. In
the meanwhile, on 10-10-2022 a fire has broken out in the factory and the company filed a claim for damages
of plant & machinery with the Insurance company. Advise the insurance company in this regard.
(May-2019-New)
Ans. (i) Provision: - As per Section 64VB of the Insurance Act, 1938, no insurer should assume any risk in
India in respect of any insurance business on which premium is ordinarily payable in India unless
and until the premium payable is received or is guaranteed to be paid by such person in such
manner and within such time, as may be prescribed, or unless and until deposit of such amount, as may
be prescribed, is made in advance in the prescribed manner.
For the purposes of this section, in the case of risks for which premium can be ascertained in
advance, the risk may be assumed not earlier than the date on which the premium has been paid in
cash or by cheque to the insurer
(ii) In the present case:- Plant & Machinery insured on 01.10.2022, but premium was paid only on
15.10.2022. Applying the provisions of Sec. 64VB, company cannot be held liable for the claim arises
due to fire accident happened on 10.10.2022.
(iii) Conclusion: - In view of the above, the insurance company is not liable to pay the claim.
3. Inception of Risk – Section 64VB
As at 31st March 2022 while auditing Safe Insurance Ltd you observed that a policy has been issued on 25th
March 2022 for fire risk favouring one of the leading corporate houses in the country without the actual
receipt of premium and it was reflected as premium receivable. The company maintained that it is a usual
practice in respect of big customers and the money was collected on 5th April 2022. You further noticed that

FAST
there was a fire accident in the premises of the insured on 31st March 2022 and a claim was lodged for the
same. The insurance company also made a provision for claim. Please respond.
(Study Material) (MTP-Dec-2021)(RTP-May-2021)
CA.Sarthak Niraj Jain OR
While auditing Innocent Insurance Ltd, you observed that a policy has been issued on 31 st March, 2020
evening to LMN Company. LMN Company had signed all the papers and taken insurance policy (value insured
= ` 11 lac) for its new godown and premium for the same was paid through cheque subject to realization.
However, on the night of 31st March, a huge fire accident took place in LMN Company and goods worth `15
lac were destroyed. Further, cheque was also dishonoured due to insufficient fund. The Company informed
the incident to Innocent Insurance Ltd and a claim was lodged for the same. The insurance company also
made a provision for claim. Advise the Innocent Insurance Ltd in this regard. (MTP-Nov-2020)

Ans. (i) Provision:- As per Section 64VB of the Insurance Act, 1938, no insurer should assume any risk in
India in respect of any insurance business on which premium is ordinarily payable in India unless
and until the premium payable is received or is guaranteed to be paid by such person in such
manner and within such time, as may be prescribed, or unless and until deposit of such amount, as may
be prescribed, is made in advance in the prescribed manner.
For the purposes of this section, in the case of risks for which premium can be ascertained in
advance, the risk may be assumed not earlier than the date on which the premium has been paid in
cash or by cheque to the insurer
(ii) In the present case: - Though policy was issued on 27th March, 2020, but premium was collected
only on 7th April, 2020. Applying the provisions of Sec. 64VB, company cannot be held liable for the
claim arises due to fire accident happened on 31st March, 2020.
(iii) Conclusion: - In view of the above, the insurance company is not liable to pay the claim and hence
no provision for claim is required.
SJ Note: - Though question is specific to Section 64VB of Insurance Act 1938; the suggested answer is generic
in nature.

CA Final Audit - By CA. Sarthak Niraj Jain 181


Audit of Insurance Companies

10.2 - Audit of Accounts of Insurance Company


4. Appointment of Statutory Auditor
R.O.K. & Co. and TNK & Co. were appointed as the joint statutory auditors at the AGM of Auspic General
Insurance Co. Ltd. Apart from the aforesaid audit, R.O.K. & Co. is also being appointed as a joint statutory
auditor of one another General Insurance Company and TNK & Co. is appointed as a joint statutory auditor of
Life Insurance Company. How many further audits can be accepted by R.O.K. & Co. and TNK & Co.,
respectively, of either general or life insurance companies? (MTP-May-2021)
Ans. (i) Provision:
 The appointment of statutory auditors in the General Insurance Corporation of India, and its
subsidiaries & the divisions as well as other public sector Insurance Companies is made by
the Comptroller and Auditor General of India, as in the case of other public sector undertakings.
 In the case of others: - Auditor is appointed at the AGM after ensuring that the auditor satisfies
the compliance requirements with the relevant sections of the IRDAI Guidelines on Corporate
Governance.
 These guidelines pose certain restrictions on the number of insurance companies a statutory
auditor can audit.
 Currently, an auditor can conduct audit only for three insurance companies and not more
than 2 life or 2 general.
 The Guidelines also mandate a mandatory joint audit for all insurance companies.
(ii) In the given case: - R.O.K. & Co. is joint statutory auditor of Auspic General Insurance Co. Ltd. And of
one another General Insurance Company.
(iii) Conclusion: -Accordingly, it can now, further, accept only one audit and that too of a Life Insurance
Company only. Further, TNK & Co. is joint statutory auditor of Auspic General Insurance Co. Ltd. as

FAST
well as of one Life Insurance Company. Accordingly, it can now, further, accept only one audit of
either a Life Insurance Company or a General Insurance Company.
10.3 - Areas Peculiar to Life Insurance Business & Role of Auditor
5.
CA.Sarthak Niraj Jain
Policy Lapse and Revival & Role of Auditor
Briefly explain the term policy lapse and revival in case of Life Insurance Company and role of auditor in
verifying the same. (Study Material)(RTP-Nov-2018)
Ans. (i) Lapse
• “Lapse” is the discontinuance if the policy owing to non-payment of premium dues.
• If payment is missed, the insurer allows period of 15/30 days from the premium due date for
making the payment, this period is termed as grace period.
• If policy holder does not make the payment within the grace period, the policy gets “lapsed”.
(ii) Revival
• The T&C of the policy stipulate that where the premium is not paid within the grace period, the
policy get lapsed but it may be revived during the life time of the policy assured
• Some insurers do not allow revival, if the policy has remained in lapse condition for more than
5 years.
(iii) Role of Auditor
• To check and confirm the due dates are recorded and monitored properly and policies are
marked as “Lapsed” on non-receipt of renewal premium within due dates/grace period.
• In case of revival request, whether adequate checks are in place for receipt of outstanding
amounts and adequate documents are obtained before reviving the policy.
6. Operating Expenses
TNT Limited is engaged in the Life Insurance business. The company's operations have been considerable in
the Northern India and its Head Office is also based at New Delhi. TNT Ltd. while preparing financial
statements have classified administrative expenses under 14 heads as mentioned in Schedule 3 forming part
of financial statements given under schedule A to IRDA Regulations, 2002. What is your responsibility as an
auditor particularly in relation to administrative/expenses of management (Jan-2021-New)

182 Ab Audit Hoga Sabse Scoring - By CA. Sarthak Niraj Jain


Audit of Insurance Companies

Ans. Operating Expenses related to Insurance Business (Expenses of Management):


(i) Any major expenses (` 5 lacs or in excess of 1% of net premium, whichever is higher) are
required to be shown separately.
(ii) The auditor should ensure that these expenses are first aggregated and then apportioned to the
Revenue Account of each class of business on a reasonable and equitable basis.
(iii) The accounting policy should clearly indicate the basis of apportionment of these expenses to the
respective Revenue Accounts (i.e., Participating and Nonparticipating policies and in between Linked
and Non- Linked business) along with the certificate that all expenses of management, wherever
incurred, directly or indirectly, read with the accounting policy, have been fully debited to the
respective Revenue Account as expenses.
(iv) Any expenses which are not covered under the 14 heads as mentioned in Schedule 3 are required to
be disclosed under the head ‘Others’.
10.4 - Audit Procedure in the Audit of GIC
7. Commission Paid to Agents
You have been appointed to carry out the audit of Blue Insurance Company Ltd. for the year 2021-22. In the
course of your audit, you observed that the commission paid to agents constituted a major expense in
operating expenses of the Company. Enumerate the audit concerns that address to the assertions required
for the Auditor to ensure the continued existence of internal control as well as fairness of the amounts in
accounting of commission paid to agents. (RTP-Dec-2021)(MTP-May-2021)
OR
You have been appointed to carry out the audit of Sky Insurance Company Ltd. for the year 2021-22. In the
course of your audit, you observed that the commission paid to agents constituted a major expense in

FAST
operating expenses of the company. Enumerate the audit concerns that address to the assertions required for
the Auditor to ensure the continued existence of internal control as well as fairness of the amounts in
accounting of commission paid to agents. (Study Material)(Nov-2018-New)

CA.Sarthak Niraj Jain OR


While auditing Secure Insurance Ltd., you observed that the major proportion of expense of the company is
the remuneration/commission paid to its insurance agents. As the auditor of the company, what audit
procedure would you adopt for verification of such expense? (MTP-Nov-2018)
OR
Write Short note on: Verification of payment of remuneration to an Insurance Agent. (May-2017)
Ans. Commission/Brokerage
 The commission is the consideration payable for getting the insurance business. The term
‘commission’ is used for the payment of consideration to get Direct business.
 Commission received on amount of premium paid to a re-insurer is termed ‘Commission on
reinsurance accepted’ and is reduced from the amount of commission expenditure.
 The internal control with regard to commission is aimed at ensuring that commission is paid in
accordance with the rules and regulations of the company and in accordance with the agreement
with the agent, commission is paid to the agent who brought the business and the legal compliances,
for example, tax deduction at sources, GST on reverse charge mechanism and provisions of the
Insurance Act, 1938 have been complied with.
 It is a well-known fact that insurance business is solicited by insurance agents. The remuneration of
an agent is paid by way of commission which is calculated by applying a percentage to the premium
collected by him. Commission is payable to the agents for the business procured through them and is
debited to Commission on Direct Business Account.
 There is a separate head for commission on reinsurance accepted which usually arise in case of
Head Office. It may be noted that under section 40 of Insurance Act, 1938, no commission can be paid
to a person who is not an agent of the insurance company

CA Final Audit - By CA. Sarthak Niraj Jain 183


Audit of Insurance Companies
 Commission cannot be paid in excess of the maximum rates of commission as framed by IRDAI. The
rates of commission/brokerage are agreed and documented with the agent and filed with IRDAI.

Role of Auditor: The auditor should, inter alia, do the following for verification of commission:
(a) Ensure that commission/brokerage is not paid in excess of the limits specified by IRDAI.
(b) Ensure that commission/brokerage is paid as per rates agreed with the agent and filed with IRDAI.
(c) Ensure that commission/brokerage is paid to the agent/broker who has solicited the business.
(d) Ensure that the agent/broker is not blacklisted by IRDAI and is not terminated for fraud etc.
(e) Vouch disbursement entries with reference to the disbursement vouchers with copies of commission
bills and commission statements.
(f) Check whether the vouchers are authorised by the officers-in–charge as per rules in force and
income tax is deducted at source, as applicable.
(g) Test check correctness of amounts of commission allowed
(h) Scrutinise agents’ ledger and the balances, examine accounts having debit balances, if any, and obtain
information on the same. Necessary rectification of accounts and other remedial actions have to be
considered.
(i) Check whether commission outgo for the period under audit been duly accounted.
8. Unexpired Risks Reserve
You have been appointed as an auditor of a General Insurance Company. In this context, explain Unexpired
Risks Reserve and audit procedures for the same. (Nov-2019-Old)
OR

FAST
AX Insurance Limited has made a provision of 75% of net premium in case of marine hull insurance and 50%
in case of marine cargo and miscellaneous business of net premium for unexpired risks reserve in its books.
Comment (Nov-2015)

Ans. (i) CA.Sarthak Niraj Jain


Reserve for Unexpired Risks:
 The need for unexpired risks reserve arises from the fact that all policies are renewed
annually except in specific cases where short period policies are issued. Since the insurers
close their accounts on a particular date, not all risks under policies expire on that date.
 In other words at the closing date, there is an unexpired liability under various policies which
may occur during the remaining term of the policy beyond the year end.
 There are two methods of creating this reserve:-
 One is based on the proportionate number of days of risk remaining to risk expired, which
is called 1/365 method.
 The other method is by taking a URR directly on 50% of the premium amount.
(ii) IRDA (General Insurance-Claim Reserving) Regulations, 2013
Requires creation of a minimum amount of unexpired risks reserve at a specified percentage of net
premium as under:
 For marine hull insurance – 100% of net premium
 For fire, marine cargo and miscellaneous business – 50% of net premium.
(iii) Conclusion:- Auditor of AX Insurance Ltd should qualify his report as the company has made a
provision of only 75% against the prescribed minimum of 100% (Marine Hull Insurance), thereby
resulting in overstatement of profit.
SJ’s Note: Section 64V of Insurance Act, 1938 also specifies these percentages. Subsequent to Amendment Act of
2015, these percentages are no more specified in amended Section 64V.

184 Ab Audit Hoga Sabse Scoring - By CA. Sarthak Niraj Jain


Audit of Insurance Companies

9. Outstanding Premium & Agents Balances


ARHAM Limited is engaged in the business of Insurance for the last 27 years. KUSHAL & Co., a firm of
Chartered Accountants are the statutory auditors of this company and have been required to perform the
audit of all the divisions and head office for the financial year 2021-2022. At the planning state CA K,
Engagement Partner has identified outstanding premium and agents’ balances as a focus area. Guide CA K by
explaining key audit procedures to be performed for verification of outstanding premium and agent’s
balance. (RTP-May-2022)
OR
State the audit procedures for verification of outstanding premium and agents’ balances of General Insurance
companies. (Study Material)(Nov-2011, Nov-2010)
OR
M/s ABC & Co., a CA firm was appointed as the auditor of ‘Always Safe General Insurance Ltd.’ Advise them
how they will verify outstanding premium and agent’s balances.
Ans. The following are the audit procedures to be followed for verification of outstanding premium and
agents’ balances:
(i) Inquire reasons for long outstanding credit balances in outstanding premium accounts and examine
the reasons for policies not being issued or the outstanding premium not adjusted against
amounts due.
(ii) Scrutinize and review control account debit balances and their nature should be enquired into.
(iii) Examine in-operative balances and treatment given for old balances with reference to company
rules.
(iv) Enquire into the reasons for retaining the old balances.

FAST
(v) Verify old debit balances which may require provision or adjustment. Notes of explanation may be
obtained from the management in this regard.
(vi) Check age-wise, sector-wise analysis of outstanding premium.

10.
CA.Sarthak Niraj Jain
(vii) Verify whether outstanding premiums have since been collected.
(viii) Check the availability of adequate bank guarantee or premium deposit for outstanding premium.
Solvency Margin – Section 64VA
M/s MPS & Associates, Chartered Accountants started the statutory audit of their client Contingencies Ltd., a
General Insurance company, which has a paid-up capital of `16,800/- lac. During the course of the audit, it
was found that the Company was not maintaining the required solvency margin as per the provisions of
Insurance Act, 1938. When the issue was escalated to the management, they replied that solvency margin
needs to be maintained as per limits prescribed only on last day of the financial year. Comment whether
reply of management is tenable or not. (RTP-Nov-2020) (MTP-Dec-2021)
OR
Discuss Solvency Margin in case of an Insurer carrying on General Insurance Business. (May-2019-Old)
OR
Write a short note on Solvency Margin.
Ans. Solvency Margin
(i) Provision: - Section 64VA of Insurance Act, 1938 as amended by Insurance Laws (Amendment)
Act, 2015):
 Requirement of solvency margin: Sec. 64VA of Insurance Act 1938 requires that every insurer
and re-insurer shall at all times maintain an excess of value of assets over the amount of
liabilities of, not less than 50% of the amount of minimum capital as stated u/s 6 and arrived
at in the manner specified by the regulations.
 Non-compliance of solvency margin: An insurer or reinsurer, as the case may be, who does not
comply with the requirement of solvency margin shall be deemed to be insolvent and may be
wound-up by the court on an application made by the Authority.

CA Final Audit - By CA. Sarthak Niraj Jain 185


Audit of Insurance Companies
 Power of authority to prescribe level of solvency: The Authority shall by way of regulation
made for the purpose, specify a level of solvency margin known as control level of solvency on
the breach of which the Authority shall act in accordance with without prejudice to taking of any
other remedial measures as deemed fit.
 Submission of Financial Plan: If, at any time, an insurer or reinsurer does not maintain the
required control level of solvency margin, he shall, in accordance with the directions issued by
the Authority, submit a financial plan to the Authority, indicating a plan of action to correct the
deficiency within a specified period not exceeding six months.
 Modifications to Financial Plan: If the authority considers the financial plan submitted by an
insurer inadequate, it shall propose modifications to the plan and shall give directions, as may
be deemed necessary, including direction in regard to transacting any new business, or,
appointment of an administrator or both.
 Non-submission of financial plan: An insurer or re-insurer, as the case may be, who does not
submit financial plan shall be deemed to have made default in complying with the
requirements of this section.
(ii) Conclusion: - Therefore, in the said case Contingencies Ltd has not maintained the Solvency
Margin throughout the year. Accordingly, contention of Contingencies Ltd. that solvency margin is
required to be maintained as per limits prescribed only on last day of the financial year is not
tenable.

Notes

FAST
CA.Sarthak Niraj Jain

186 Ab Audit Hoga Sabse Scoring - By CA. Sarthak Niraj Jain


Audit of Non Banking Financial Companies

Chapter
11
AUDIT OF NON BANKING FINANCIAL COMPANIES

Past Exams Weightage


5

0
MAY-19 NOV-19 NOV-20 JAN-21 JUL-21 DEC-21

May-19 Nov-19 Nov-20 Jan-21 Jul-21 Dec-21


Marks 0 5 5 0 0 4

11.1- NBFC : Definition, Registration & Classification


1. Deemed NBFC

FAST
Satyam Pvt Ltd is a company engaged in trading activities, it also has made investments in shares of other
Companies and advanced loans to group companies amounting to more than 50% of its total assets.
However, trading income constitutes majority of its total income. Whether the Company is an NBFC?

Ans. CA.Sarthak Niraj Jain


(i) Provision: -NBFC Company
(Study Material)

1. In order to identify a particular company as Non-Banking Financial Company (NBFC), it will


consider both assets and income pattern as evidenced from the last audited balance sheet of
the company to decide its principal business.
2. The company will be treated as NBFC when: a company's financial assets constitute more than
50 per cent of the total assets (netted off by intangible assets) and income from financial
assets constitute more than 50 per cent of the gross income.
A company which fulfils both these criteria shall qualify as an NBFC and would require to be
registered as NBFC by Reserve Bank of India.
(ii) In the present case: - Satyam Pvt Ltd is a company engaged in trading activities, it also has made
investments in shares of other Companies and advanced loans to group companies amounting to
more than 50% of its total assets. However, trading income constitutes majority of its total income.
(iii) Conclusion: -In the given case, though Satyam Pvt Ltd is fulfilling the criteria on the asset side, but
however is not fulfilling the criteria on the income side, the company cannot be classified as a
deemed NBFC.
11.2- Public Deposits & Prudential Norms
2. Mobilization of Public Deposits (Downgrading of credit facilities)
CA Nadar is conducting the statutory audit of RHL Ltd., a non-banking financial company. It has branches in
various parts of India. The company with a focus on housing finance, has outstanding non-convertible
debentures worth ` 150 Crores. The company reportedly missed interest payments of INR 15 Crores on its
debts because of inadequate liquidity. As a result, RHL Ltd. faced a series of downgrades by rating agencies
on its debts over the past two months. Rating was cut to D from A4 implying that the company was in default
or expected to be in default soon. What aspects CA Nadar should look into in relation to the activity of
mobilization of public deposits (particularly in relation to downgrading of credit facilities) by RHL Ltd?
(Nov-2020-New)

CA Final Audit - By CA. Sarthak Niraj Jain 187


Audit of Non Banking Financial Companies

Ans. CA Nadar has to ascertain whether the company has complied with the following aspects in relation
to the activity of mobilization of public deposits:-
(i) The ceiling on quantum of public deposits has been linked to its credit rating as given by an
approved credit rating agency. In the event of a upgrading/downgrading of credit rating, the
auditor should bear in mind that the NBFC will have to increase/reduce its public deposits in
accordance with the revised credit rating assigned to it within a specified time frame and should
ensure that the NBF Chas informed about the same to the RBI in writing.
(ii) In the event of downgrading of credit rating below the minimum specified investment grade, a
non-banking financial company, being an investment and credit company or a factor, shall regularise
the excess deposit as provided hereunder:
1) With immediate effect, stop accepting fresh public deposits and renewing existing deposits;
2) All existing deposits shall run off to maturity; and
3) Report the position within 15 working days, to the concerned Regional Office of the RBI where
the NBFC is registered.
4) No matured public deposit shall be renewed without the express and voluntary consent of
the depositor.
11.3- Investment Credit Companies
3. Investment and Credit Companies (ICC)
Shivam & Co LLP are the auditors of NBFC-ICC. Some of the team members (TMs) of the audit team who
audited this NBFC have left the firm and the new Team Members are in discussion with the previous Team
Members who are still continuing with the firm regarding the verification procedures to be performed.
explain what verification procedures should be performed in relation to audit (Study Material)

FAST OR
Ram and Associates, a firm of Chartered Accountants, are the auditors of NBFC (Investment and Credit
Company). Some of the team members of the audit team who audited this NBFC have left the firm and the

CA.Sarthak Niraj Jain


new team members are in discussion with the previous team members who are still continuing with the firm
regarding the verification procedures to be performed. In this context, please explain what verification
procedures should be performed in relation to audit of NBFC - Investment and Credit Company (NBFC-ICC).
(MTP-Nov-2019)(RTP-Nov-2019)
Ans. (A) Points related in Investments:
(i) Physical verification: Auditor should physically verify the securities held by a NBFC. Where
any security is lodged with an institution or a bank, a certificate from the bank/institution to
that effect must be verified.
(ii) Income recognition: Verify that dividend income wherever declared by a company, has been
duly received and accounted for. NBFC Prudential Norms directions require dividend income
on shares of companies and units of mutual funds to be recognised on cash basis.
(iii) Authorisation: Verify the Board Minutes for purchase and sale of investments.
(iv) Classification: Ascertain from the Board resolution or obtain a management certificate to the
effect that the investments so acquired are current investments or Long Term Investments.
(v) Valuation: Check whether the investments have been valued in accordance with the NBFC
Prudential norms directions and adequate provision for fall in the market value of securities,
wherever applicable, have been made there against, as required by the Directions.
(vi) Compliance of AS 13: An auditor will have to ascertain whether the requirements of AS 13
“Accounting for Investments” or other accounting standard, as applicable, (to the extent they
are not inconsistent with the Directions) have been duly complied with by the NBFC.
(vii) External confirmations: In respect of shares/securities held through a depository, obtain a
confirmation from the depository regarding the shares/securities held by it on behalf of the
NBFC. Obtain a confirmation from the approved intermediary regarding securities deposited
with/borrowed from it as at the year end.

188 Ab Audit Hoga Sabse Scoring - By CA. Sarthak Niraj Jain


Audit of Non Banking Financial Companies
(B) Point related to Loan/Advance:
(i) Sanctioning: Auditor should examine whether each loan or advance has been properly
sanctioned. He should verify the conditions attached to the sanction of each loan or advance
i.e. limit on borrowings, nature of security, interest, terms of repayment, etc.
(ii) Security: Auditor should verify the security obtained and the agreements entered into, if any,
with the concerned parties in respect of the advances given. He must ascertain the nature and
value of security and the net worth of the borrower/guarantor to determine the extent to which
an advance could be considered realisable.
(iii) Loan against own shares: Verify whether the NBFC has not advanced any loans against the
security of its own shares.

(iv) Compliance of prudential norms: Check whether the NBFC has not lent/invested in excess of
the specified limits to any single borrower or group of borrowers as per NBFC Prudential Norms
Directions.
(v) Appraisal and follow up system: Auditor should verify whether the NBFC has an adequate
system of proper appraisal and follow up of loans and advances. In addition, he may analyse the
trend of its recovery performance to ascertain that the NBFC does not have an unduly high level
of NPAs.
(vi) Classification: Check the classification of loans and advances (including bills purchased and
discounted) made by a NBFC into Standard Assets, Sub-Standard Assets, Doubtful Assets and Loss
Assets and the adequacy of provision for bad and doubtful debts as required by NBFC
Prudential Norms Directions.
SJ’s Note: The above checklist is not exhaustive. It is only illustrative. There could be various other audit

FAST
procedures which may be performed for audit of an NBFC.
11.4- Auditor’s Reports & Frauds
4.
CA.Sarthak Niraj Jain
Audit Report to BoD
You are appointed as the auditor of a NBFC registered with the RBI and which is accepting and holding public
deposits. You are considering your reporting requirement in addition to your report made under Section 143
of the Companies Act, 2013 on the accounts of this NBFC as per the prescribed Directions. Please explain
what points are required to be known in respect of separate report to be given by you to the Board of
Directors of this NBFC. (Study Material)
Ans. Material to be included in the Auditor’s report to the Board of Directors:
The auditor’s report on the accounts of a non-banking financial company shall include a statement on the
following matters, namely –
(a) Matters to be reported in case of all NBFC
I. Conducting Non-Banking Financial Activity without a valid Certificate of Registration (CoR)
granted by the RBI is an offence under RBI Act, 1934. Auditor shall examine whether the
company has obtained a Certificate of Registration (CoR) from the RBI.
II. In case of a company holding CoR issued by the RBI, whether that company is entitled to
continue to hold such CoR in terms of its Principal Business Criteria (Financial asset/income
pattern) as on March 31 of the applicable year.
III. Whether the non-banking financial company is meeting the required net owned fund
requirement as laid down in Master Direction issued by RBI.
(b) Matters to be reported in case of NBFC accepting/holding public deposits
In the case of a non-banking financial companies accepting/holding public deposits Apart from the
matters enumerated in (A) above, the auditor shall include a statement on the following matters,
namely-
(i) Whether the public deposits accepted by the company together with other borrowings
indicated below viz.

CA Final Audit - By CA. Sarthak Niraj Jain 189


Audit of Non Banking Financial Companies
(a) from public by issue of unsecured non-convertible debentures/bonds;
(b) from its shareholders (if it is a public limited company); and
(c) which are not excluded from the definition of ‘public deposit’ in the Non-Banking
Financial Companies Acceptance of Public Deposits (Reserve Bank) Directions, 2016, are
within the limits admissible to the company as per the provisions of the Non-Banking
Financial Companies Acceptance of Public Deposits (Reserve Bank) Directions, 2016;
(ii) Whether the public deposits held by the company in excess of the quantum of such deposits
permissible to it under the provisions of NBFC Acceptance of Public Deposits (Reserve Bank)
Directions, 2016 are regularised in the manner provided in the said Directions;
(iii) Whether the non-banking financial company is accepting "public deposit” without minimum
investment grade credit rating from an approved credit rating agency as per the provisions
of Non-Banking Financial Companies Acceptance of Public Deposits (Reserve Bank) Directions,
2016;
(iv) Whether the capital adequacy ratio as disclosed in the return submitted to the Bank has been
correctly determined and whether such ratio is in compliance with the minimum CRAR
prescribed therein;
(v) In respect of non-banking financial companies referred to in clause (iii) above,
(a) whether the credit rating, for each of the fixed deposits schemes that has been assigned
by one of the Credit Rating Agencies listed in NBFC Acceptance of Public Deposits
(Reserve Bank) Directions, 2016 is in force; and
(b) whether the aggregate amount of deposits outstanding as at any point during the year
has exceeded the limit specified by the such Credit Rating Agency;
(vi) Whether the company has violated any restriction on acceptance of public deposit as

FAST provided in Non-Banking Financial Companies Acceptance of Public Deposits (Reserve Bank)
Directions, 2016;

CA.Sarthak Niraj Jain


(vii)

(viii)
Whether the company has defaulted in paying to its depositors the interest and /or principal
amount of the deposits after such interest and/or principal became due;
Whether the company has complied with the prudential norms on income recognition,
accounting standards, asset classification, provisioning for bad and doubtful debts, and
concentration of credit/investments as specified in the Directions.
(ix) Whether the company has complied with the liquid assets requirement as prescribed by the
Bank in exercise of powers under section 45-IB of the RBI Act and whether the details of the
designated bank in which the approved securities are held is communicated to the office
concerned of the RBI in terms of NBS 3
(x) Whether the company has furnished to the RBI within the stipulated period the return on
deposits as specified in the NBS 1 to – NBFC Returns (Reserve Bank) Directions, 2016;
(xi) Whether the company has furnished to the RBI within the stipulated period the quarterly
return on prudential norms as specified in Directions.
(xii) Whether, in the case of opening of new branches or offices to collect deposits or in the case
of closure of existing branches/offices or in the case of appointment of agent, the company has
complied with the requirements contained in the Non-Banking Financial Companies Acceptance
of Public Deposits (Reserve Bank) Directions, 2016.
5. Exception Report to the RBI
Kamna & Co LLP, a firm of Chartered Accountants, was appointed as auditor of an NBFC. The audit work has
been completed. The audit team which was involved in the fieldwork came across various observations
during the course of audit of this NBFC and have also an limited understanding about the exceptions which
are required to be reported in the audit report. They would like to understand in detail regarding the
obligations on the part of an auditor in respect of exceptions in his report so that they can conclude their
work. Please explain. (Study Material) (MTP-May-2020)(MTP-Dec-2021)

190 Ab Audit Hoga Sabse Scoring - By CA. Sarthak Niraj Jain


Audit of Non Banking Financial Companies
OR
R and Associates, a firm of chartered accountants, is appointed as auditor of NBFC. During the audit, audit
team comes across various observations / exceptions and Mr. A, a junior member of audit team, due to his
limited understanding about exceptions which are required to be reported in the audit report, would like to
understand in detail, the obligations on the part of an auditor in respect of exceptions in the audit report so
that he can conclude his work. Discuss. (July-2021-Old)
OR
What are the obligations of auditor to submit Exception Report to RBI in case of non-banking financial
companies? (Nov-2016)
Ans. Obligation of auditor to submit an exception report to the RBI
(i) Para 5 of NBFC Auditor’s Report (Reserve Bank) Directions 2016 provides that Where, in the case
of a NBFC, the statement regarding any of the items referred to in para 3, is unfavourable or
qualified, or in the opinion of the auditor the company has not complied with:
(a) The provisions of Chapter III B of Reserve Bank of India Act, 1934; or
(b) The NBFC Acceptance of Public Deposits (Reserve Bank) Directions, 2016; or
(c) NBFC–Non-Systemically Important Non-Deposit taking Company (Reserve Bank)
(ii) Directions, 2016 and NBFC Systemically Important Non-Deposit taking Company and Deposit
taking Company (Reserve Bank) Directions, 2016
1) It shall be the obligation of the auditor to make a report containing the details of such
unfavourable or qualified statements and/or about the non-compliance, as the case may be,
2) In respect of the company to the concerned Regional Office of the Department of Non-
Banking Supervision of the Bank under whose jurisdiction the registered office of the company
is located as per first Schedule to the Non-Banking Financial Companies Acceptance of Public
Deposits (Reserve Bank) Directions, 2016..
(iii) Duty of the Auditor to submit exception report shall be to report only the contraventions of the

6.
FAST
provisions of RBI Act, 1934, and Directions, Guidelines, Instructions and such report shall not contain
any statement with respect to compliance of any of those provisions.
Content of the Auditor’s Report to the Board (MD)

CA.Sarthak Niraj Jain


Karma Pvt Ltd is a Non-Deposit Taking Non-Systemically Important NBFC registered with Reserve Bank of
India. The Statutory Auditor of the company is required to give a report to the Board of Directors. What shall
be the content of the Auditor’s Report to the Board? (Study Material)
Ans. The statutory auditor of Karma Pvt Ltd, being a Non-Deposit Taking Non-Systemically Important NBFC
is required to submit separate report to the Board of Directors on the matters as specified as below:
I. Company not holding CoR issued by the RBI: -Conducting Non-Banking Financial Activity without a
valid Certificate of Registration (CoR) granted by the RBI is an offence under chapter V of the RBI
Act, 1934. Therefore, if the company is engaged in the business of non-banking financial institution as
defined in section 45-I (a) of the RBI Act and meeting the Principal Business Criteria (Financial
asset/income pattern) and directions issued by DNBR, auditor shall examine whether the company
has obtained a Certificate of Registration (CoR) from the RBI.
II. In case of a company holding CoR issued by the RBI: - Whether that company is entitled to continue
to hold such CoR in terms of its Principal Business Criteria (Financial asset/income pattern) as on
March 31 of the applicable year.
III. Net owned fund: -Whether the non-banking financial company is meeting the required net owned
fund requirement as laid down in Master Direction.
Apart from the aspects enumerated above, the auditor shall include a statement on the following
matters, namely: -
(i) Whether the Board of Directors has passed a resolution for non- acceptance of any public deposits;
(ii) Whether the company has accepted any public deposits during the relevant period/year;
(iii) Whether the company has complied with the prudential norms relating to income recognition,
accounting standards, asset classification and provisioning for bad and doubtful debts as applicable to it
in terms of NBFC – Non-Systemically Important Non-Deposit taking Company (Reserve Bank)
Directions, 2016;
CA Final Audit - By CA. Sarthak Niraj Jain 191
Audit of Non Banking Financial Companies
Reporting
(i) Where, in the auditor’s report, the statement regarding any of the items referred to matters specified
above is unfavourable or qualified, the auditor’s report shall also state the reasons for such
unfavourable or qualified statement, as the case may be.
(ii) Where the auditor is unable to express any opinion on any of the items referred above, his report
shall indicate such fact together with reasons thereof.
11.5 Companies Act Related
7. CARO, 2020- Audit Procedure
Abhimanyu Finance Ltd. is a Non Banking Finance Company and was in the business of accepting public
deposits and giving loans. The company was having net owned funds of `1,50,00,000/-(one crore fifty lakhs)
and was not having registration certificate from RBI and applied for it on 30th March 2022. The company
appointed Mr. Kabra as its statutory auditors for the year 2021-22. Advise the auditor with reference to
auditor procedures to be taken and reporting requirements on the same in view of CARO 2020?
(Study Material)(MTP-May-2019)
Ans. Audit Procedure for reporting under CARO, 2020 w.r.t. registration u/s 45-IA of RBI Act, 1934:
(i) Clause (xvi) of Paragraph 3 of CARO 2020
Requires the company auditor to report:-
1) Whether the company is required to be registered under Section 45-IA of the RBI Act, 1934
and if so,
2) Whether the registration has been obtained.
(ii) The auditor is required to examine whether the company is engaged in the business which attract
the requirements of the registration. The registration is required where the financing activity is a

(iii)
FAST
principal business of the company. The RBI restrict companies from carrying on the business of
a non-banking financial institution without obtaining the certificate of registration.
Audit Procedure and Reporting:-

CA.Sarthak Niraj Jain


1) The auditor should examine the transactions of the company with relation to the activities
covered under the RBI Act and directions related to the Non-Banking Financial Companies.
2) Auditor should examine the financial statements to ascertain whether company’s financial
assets constitute more than 50 per cent of the total assets and income from financial assets
constitute more than 50 per cent of the gross income.
3) Ascertain whether the net owned funds of the company exceeds such amount so as to require
the company to get itself registered as NBFC with RBI.
4) Ascertain whether the company has obtained the registration as NBFC, if not, the reasons
should be sought from the management and documented.
5) Auditor’s Report under CARO, 2020 shall incorporate the following:
 Whether the registration is required under section 45-IA of the RBI Act, 1934.
 If so, whether it has obtained the registration.
 If the registration not obtained, reasons thereof.
(iv) Conclusion: -In the instant case Abhimanyu Finance Ltd. is a Non Banking Finance Company and
was in the business of accepting public deposits and giving loans since 2015. The company was having
net owned funds of ` 1,50,00,000/-(one crore fifty lakhs) which is less in comparison to the
prescribed limit i.e. 2 crore rupees and was also not having registration certificate from RBI (though
applied for it on 30th March 2022). The auditor is required to report on the same as per Clause
(xvi) of Paragraph 3 of CARO 2020.

192 Ab Audit Hoga Sabse Scoring - By CA. Sarthak Niraj Jain


Audit Under Fiscal Laws

Chapter
12
AUDIT UNDER FISCAL LAWS

Past Exams Weightage


8
7
6
5
4
3
2
1
0
MAY-19 NOV-19 NOV-20 JAN-21 JUL-21 DEC-21

May-19 Nov-19 Nov-20 Jan-21 Jul-21 Dec-21


Marks 4 4 0 8 8 5

AUDIT UNDER DIRECT TAX LAWS (TAX AUDIT)

12.1 - Tax Audit - Computation of Turnover & Applicability


1.
FAST
Computation of Turnover & Applicability
Mr. A engaged in business as a sole proprietor presented the following information to you for the FY 2021-

CA.Sarthak Niraj Jain


22.


Turnover expected to be made during the year ` 1124 crore.
Goods returned in respect of sales made during FY 2020-21 is ` 20 lac not included in the above.
 Cash discount allowed to his customers ` 1 lac for prompt payment.
 Special rebate allowed to customer in the nature of trade discount ` 5 lac.
Kindly advise him whether he has to get his accounts audited u/s 44AB of the Income Tax Act, 1961.
(Study Material)
Ans. Turnover limit for the purpose of Tax Audit:
The following points merit consideration as stated in the Guidance note on Tax Audit issued by the Institute
of Chartered Accountants of India-
(i) Price of goods returned should be deducted from the figure of turnover even if the return are from
the sales made in the earlier years.
(ii) Cash discount otherwise than that allowed in a cash memo/sales invoice is in the nature of a
financing charge and is not related to turnover. The same should not be deducted from the figure of
turnover.
(iii) Special rebate allowed to a customer can be deducted from the sales if it is in the nature of trade
discount.
Applying the above stated points to the given problem,
1. Total Turnover 1124 Lac
2. Less – (i) Goods Returned 20 Lac
(ii) Special rebate allowed to customer in the nature
of trade discount would be deducted 5 Lac
Balance 1099 Lac

CA Final Audit - By CA. Sarthak Niraj Jain 193


Audit Under Fiscal Laws
As the limit for tax audit is ` ten crore (assuming the aggregate of all amounts received including amount
received for sales, turnover or gross receipts during the previous year, in cash, does not exceed five per
cent of the said amount and aggregate of all payments made including amount incurred for expenditure, in
cash, during the previous year does not exceed five per cent of the said payment), he would be required to
get his accounts audited under section 44AB of the Income Tax Act, 1961.
SJ Note -: Assumed condition related to banking transactions fulfilled
2. Computation of Turnover & Applicability
Mr. Abhinandan engaged in business as a sole proprietor presented the following information to you for the
FY 2021-22.
 Turnover expected to be made during the year ` 1024 lacs.
 Goods returned in respect of sales made during FY 2020-21 is ` 20 lacs not included in the above.
 Cash discount allowed to his customers ` 1 lac for prompt payment.
 Special rebate allowed to customer in the nature of trade discount ` 5 lacs.
Further, the aggregate of all amounts received including amount received for sales, turnover or gross receipts
during the previous year, in cash, does not exceed five per cent of the said amount and aggregate of all
payments made including amount incurred for expenditure, in cash, during the previous year does not exceed
five per cent of the said payment. Kindly advise him whether he has to get his accounts audited u/s 44AB of
the Income tax Act, 1961. (MTP-May-2021)
Ans. Turnover limit for the purpose of Tax Audit:
(i) Provision:- As per section 44AB of the Income tax Act, 1961, audit is required in case of
 Every person carrying on business, if his total sales, turnover or gross receipts in business
exceed ` 10 crore (assuming the aggregate of all amounts received including amount received for

FAST
sales, turnover or gross receipts during the previous year, in cash, does not exceed five per cent
of the said amount and aggregate of all payments made including amount incurred for
expenditure, in cash, during the previous year does not exceed five percent of the said payment)

CA.Sarthak Niraj Jain


and

(ii) As per Guidance Note on Tax Audit issued by the ICAI,


The following points merit consideration for the purpose of computing turnover:
1) Price of goods returned should be deducted from the figure of turnover even if the return are
from the sales made in the earlier years.
2) Cash discount otherwise than that allowed in a cash memo/sales invoice is in the nature of a
financing charge and is not related to turnover. The same should not be deducted from the
figure of turnover.
3) Special rebate allowed to a customer can be deducted from the sales if it is in the nature of
trade discount.

(iii) Computation of Turnover


Accordingly, the turnover of Mr. Abhinandan may be computed as under:
Recorded turnover during the year ` 10,24,00,000
Less: Goods Returned ` (20,00,000)
Special Rebate ` (5,00,000)
Effective turnover ` 9,99,00,000
(iv) Conclusion:- In the given situation, Abhinandan would not be required to get his accounts audited
under section 44AB of the Income tax Act, 1961 as ` 999 lac is below prescribed tax audit limit i.e.
ten crore rupees.

194 Ab Audit Hoga Sabse Scoring - By CA. Sarthak Niraj Jain


Audit Under Fiscal Laws

3. Computation of Turnover & Applicability


Concession Ltd. is engaged in the business of manufacturing of threads. The company recorded the turnover
of ` 1.13 crore during the financial year 2021-22 before adjusting the following:
Discount allowed in the Sales Invoice ` 8,20,000
Cash discount (other than allowed in Cash memo/sales invoice) ` 9,20,000
Trade discount ` 2,90,000
Commission on Sales ` 6,00,000
Sales Return (F.Y. 2020-21) ` 1,60,000
Sale of Investment ` 6,60,000
You are required to ascertain the effective turnover to be considered for the prescribed limit of tax audit and
guide the company whether the provisions relating to tax audit applies. (MTP-Dec-2021)
Ans. Computation of Turnover for the purpose of determining requirement of Tax Audit:
(i) Provision:- As per section 44AB of the Income tax Act, 1961, audit is required in case of
 Every person carrying on business, if his total sales, turnover or gross receipts in business
exceed ` 1 crore and
 In case of every person carrying on a profession, if his gross receipts from profession exceed `
50 lakhs in any previous year.

(ii) As per Guidance Note on Tax Audit issued by the ICAI

FAST
The following points merit consideration for the purpose of computing turnover:
1) Discount allowed in the sales being in the nature of trade discount will be deducted from the
turnover.

CA.Sarthak Niraj Jain


2) Cash discount otherwise than that allowed in a cash memo/sales invoice is in the nature of a
financing charge and hence should not be deducted from the turnover.
3) Special rebate allowed to a customer can be deducted from the sales if it is in the nature of trade
discount. If it is in the nature of commission on sales, the same cannot be deducted from the
figure of turnover.
4) Price of goods returned should be deducted from the turnover even if the returns are from the
sales made in the earlier year/s.
5) Sale proceeds of any shares, securities, debentures, etc., held as investment will not form part
of turnover. However, if the shares, securities, debentures etc., are held as stock-in-trade, the sale
proceeds thereof will form part of turnover.

(iii) Calculation of turnover of concession limited as per above guidance note:


Recorded turnover during the year ` 1,13,00,000
Less: Discount allowed in the Sales Invoice ` (8,20,000)
Trade discount ` (2,90,000)
Sales Return `(1,60,000)
Effective turnover ` 1,00,30,000

(v) Conclusion: - As the effective turnover of Concession Ltd. is more than ` 1 Crore, the provisions
related to tax audit are applicable to the company.
SJ Note: -Assumed condition related to banking transactions not fulfilled

CA Final Audit - By CA. Sarthak Niraj Jain 195


Audit Under Fiscal Laws

4. Computation of Sales, Turnover or Gross Receipts


Comment with respect to computation of total sales, turnover or gross receipts in business exceeding the
prescribed limit under Section 44 AB of Income tax Act, 1961
(i) Discount allowed in the sales invoice
(ii) Cash discount
(iii) Price of goods returned related to earlier year
(iv) Sale proceeds of fixed assets. (Study Material)(May-2015)
Ans. In the context of section 44AB of the Income tax Act, 1961, following considerations are required with
regard to computation of sales, turnover or gross receipts in business exceeding the prescribed limit
under section 44AB of the Income tax Act, 1961-
(i) Discount allowed in the sales invoice it will reduce the sale price and, therefore, the same can be
deducted from the turnover.
(ii) Cash discount otherwise than that allowed in a cash memo/sales invoice is in the nature of a
financing charge and is not related to turnover. Therefore, should not be deducted from the turnover
(iii) Price of goods returned related to earlier year should be deducted from the turnover even if the
returns are from the sales made in the earlier year/s.
(iv) Sale proceeds of fixed assets would not form part of turnover as these are not held for resale.
5. Applicability of Tax Audit in Various Cases
UT & Co. is a Chartered Accountant Firm, that provides consultancy services. Recently, it got queries from
different clients with respect to applicability of tax audit provisions to their businesses.
In response to such queries, UT & Co., asked from them details such as turnover, total receipts and total
payments made during the year respectively along with mode of receipt/payment, whether filing return of
Income under normal tax provisions or presumptive tax provisions such as section 44AD, 44AE, etc. So, in the

Client Sr.
FAST
trailing mail, UT & Co., got the aforesaid details from different clients, which it classified into following
categories for ease of framing an opinion, as follows:
Turnover % of Cash % of Cash

CA.Sarthak Niraj Jain


No. (` in crore) Receipts in
Total
Receipts
Payments in
Total Payments
Remarks

1 8.5 5% 5% Has been filing return as per the regular


provisions of income tax.
2 1.8 7% 4% Has declared business income as per
presumptive taxation under section 44AD
of the Income-tax Act, 1961.
3 0.85 6% 4% Has declared business income as per
presumptive taxation under section 44AD
of the Income-tax Act, 1961 during last 2
previous years but during current
previous year has declared income lower
than as per section 44AD and the total
income is less than basic exemption limit.
4 3.2 8% 6% Has declared business income as per
presumptive taxation under section 44AE
of the Income-tax Act, 1961 during last 4
previous years but during current
previous year has declared income lower
than as per section 44AE and the total
income is less than basic exemption limit.

On behalf of UT & Co., please provide your opinion, along with reasons, as a consultant in case of aforesaid
clients that whether tax audit is applicable to them or not?
(figures re-stated) (RTP-Dec-2021)

196 Ab Audit Hoga Sabse Scoring - By CA. Sarthak Niraj Jain


Audit Under Fiscal Laws

Ans. Applicability of Tax Audit:


Opinion
Client (Tax Audit Reason
Sr. No. applicable or
not)
1 No As the turnover is up to ` 10 crore, Cash Receipts and Cash Payments are
upto 5% of total receipts & total payments, respectively, and has been
filing return as per the regular provisions of income tax, so tax audit is not
applicable.
2 No Even though turnover exceeds ` 1 crore and Cash Receipts are greater
than 5% of Total Receipts but as the business income has been declared as
per presumptive taxation under section 44AD of the Income-tax Act, 1961,
so tax audit is not applicable.
3 No Even though business income as per presumptive taxation under section
44AD of the Income-tax Act, 1961 has been declared during last 2 previous
years but has not been declared during the current previous year but as
the total income is less than basic exemption limit, so tax audit is not
applicable.
4 Yes Has been declaring total income as per presumptive taxation under
section 44AE of the Income-tax Act, 1961 during last 4 previous years but
during current previous year has declared income lower than as per
section 44AE, so tax audit is applicable.
12.2 - Tax audit - Special Issues

6.
FAST
Tax Audit - Co-operative Society
A Co-operative society having receipts over ` 2 crores has appointed Mr. D as the statutory auditor –Mr. D is
eligible to do the same under the state Co-operative Societies Act, Mr. D is not a chartered accountant, Mr. D is

Ans.
CA.Sarthak Niraj Jain
also appointed to conduct the tax audit of the society under section 44 AB of the Income tax Act, 1961,
Comment
Tax Audit Report in case of Co-operative society
(Nov-2017)

(i) Provision:- Section 44AB of Income tax Act, 1961 lays down that where the accounts of an
assessee are required to be audited by or under any other law, it shall be sufficient compliance with
the provisions of this section, if such person get the accounts of such organisation audited under such
other law before the specified date and furnishes by that date, the report of the audit as required
under such other law and a further report by an Accountant in the form prescribed under this
section.
(ii) The term “accountant” as defined under section 288 under the Income tax Act, 1961 -means a
chartered accountant within the meaning of the Chartered Accountants Act, 1949, who holds a valid
certificate of practice.
(iii) Analysis: -The person who is not a Chartered Accountant as mentioned in the question, though is
eligible to act as auditor of Cooperative Society under the Cooperative Society Act, 1912, but is not
eligible to carry out tax audit under Section 44AB of the Income tax Act, 1961.
(iv) Conclusion: -Audit report by a person other than Chartered Accountant cannot be furnished as tax
audit report under Section 44AB of the Income-tax Act, 1961.
7. Furnishing Particulars in Form 3CD - Considerations
Mr. PK is conducting the tax audit under section 44 AB of the Income tax Act, 1961 of MG Ltd. for the year
ended 31st March, 2022. There is a difference of opinion between Mr. PK and the Management in respect of
certain information to be furnished in Form No. 3CD. As a tax auditor, Mr. PK has to report whether the
statement of particulars in Form 3CD are true and correct and the same is to be annexed to the report in Form
No. 3CA. Advise on the matters to be considered by Mr. PK while furnishing the particulars in Form No. 3CD.
(Study Material)(Nov-2019-New)

CA Final Audit - By CA. Sarthak Niraj Jain 197


Audit Under Fiscal Laws

Ans. While furnishing the particulars in Form No. 3CD it would be advisable for the tax auditor to consider the
following:
1. If a particular item of income/expenditure is covered in more than one of the specified clauses, care
should be taken to make a suitable cross reference to such items at the appropriate places.
2. If there is any difference in the opinion of the tax auditor and that of the assessee in respect of any
information furnished in Form No. 3CD, the tax auditor should state both the view points and also
the relevant information in order to enable the tax authority to take a decision in the matter.
3. If any particular clause in Form No. 3CD is not applicable, he should state that the same is not
applicable.
4. In computing the allowance or disallowance, he should keep in view the law applicable in the
relevant year, even though the form of audit report may not have been amended to bring it in
conformity with the amended law.
5. In case the prescribed particulars are given in part to the tax auditor or relevant form is
incomplete and the assessee does not give the information against all or any of the clauses, the
auditor should not withhold the entire audit report. In such a case, he can qualify his report on
matters in respect of which information is not furnished to him.
6. The information in Form No. 3CD should be based on the books of accounts, records, documents,
information and explanations made available to the tax auditor for his examination.
7. In case the auditor relies on a judicial pronouncement, he may mention the fact as his observations
in Form No. 3CA or Form No. 3CB, as the case may be.
8. Liability for Tax Audit - Speculative Transactions
Mr. X deals in a commodity and purchase and sales of that commodity is ultimately settled otherwise than by
the actual delivery. During the financial year 2021-22 he purchased the commodity worth ` 95 Lakhs and sold
FAST
the same commodity for ` 104 Lakhs and the contract was settled otherwise than by the actual delivery. X
seeks your advice whether he is liable for tax audit u/s 44AB of the Income-tax Act.
Ans. (i)
CA.Sarthak Niraj Jain
Speculative Transactions meaning: -A speculative transaction means a transaction in which a
contract for the purchase or sale of any commodity, including stocks and shares, is periodically or
ultimately settled otherwise than by the actual delivery.
(ii) Analysis of the question:
• As such, in such transaction the difference amount is ‘turnover’.
• In the given case the difference of ` 104 lacs and ` 95 lakhs i.e., ` 9 Lakhs is the turnover.
• In such transactions though the contract notes are issued for full value of the purchases or sales,
but the entries in the books of account are made only for the differences.
(iii) Conclusion: -Mr. X deals in commodity as a speculator. Hence Mr. X is not liable for Tax audit u/s
44AB of the Income tax Act, 1961.
9. Revision of Tax Audit Report (TAR)
State whether a Tax audit report can be revised and if so, state those circumstances.
(Study Material) (RTP-Nov-2019)
OR
You are doing the tax audit of a Limited Company. After submission of Tax Audit Report, management notices
that there was apparent mistake of law and due to this mistake, revised the final accounts. As a tax auditor,
company seeks your opinion whether the tax audit can also be revised or not.
(RTP-Nov-2018)
Ans.  Section 44AB lays obligation on certain persons mentioned thereunder carrying on business or
profession, to get their accounts audited before the “specified date” by a Chartered Accountant, if their
turnover exceeds the stipulated threshold or in cases where they are eligible to declare income on
presumptive basis, if they claim that their income is lower than the income so computed.

198 Ab Audit Hoga Sabse Scoring - By CA. Sarthak Niraj Jain


Audit Under Fiscal Laws
 These persons have to furnish by the specified date, a report of the audit in the prescribed form. For this
purpose, the CBDT has prescribed under Rule 6G, Forms 3CA/ 3CB and Form 3CD.
 The CBDT has, vide this notification, amended Rule 6G to provide that the audit report furnished may
be revised by the person by getting revised report of audit from a chartered accountant, duly signed and
verified by such chartered accountant, if there is payment by such person after furnishing of report
which necessitates recalculation of disallowance under section 40 or section 43B.
 The said revised audit report has to be furnished before the end of the relevant assessment year for
which the report pertains.
 Further when the accounts are revised in the following circumstances, the tax auditor may have to
revise his Tax audit report also.
 Revision of accounts of a company after its adoption in the annual general meeting.
 Change in law with retrospective effect.
 Change in interpretation of law (e.g.) CBDT Circular, Notifications, Judgments, etc.
The Tax Auditor should state that it is a revised report, clearly specifying the reasons for such
revision with a reference to the earlier report.
10. Circumstances –CAiP cannot conduct Tax audit
Write short note on: Circumstances in which Chartered Accountant in practice or firm of Chartered
Accountants cannot conduct tax audit u/s 44AB of the Income tax Act, 1961 of the concern.
Ans. (i) Provision: -As per Explanation to Sec. 288, the following persons cannot conduct tax audit:
(A) In case of a company
The person who is not eligible for appointment as an auditor of as per provisions of Sec. 41(3)
the Companies Act, 2013 cannot conduct tax audit.

FAST
(B) In case of assessee other than company
Following persons cannot conduct tax audit:
1. the assessee himself or in case of the firm or AOP or HUF, any partner of the firm, or

CA.Sarthak Niraj Jain


2.
3.
member of the AOP or the HUF;
in case of the assessee, being a trust or institution, any person referred to in Sec. 13(3);
the person who is competent to verify the return in accordance with the provisions of
section 140;
4. any relative of any of the persons referred above.;
5. an officer or employee of the assessee;
6. an individual who is a partner, or who is in the employment, of an officer or employee of
the assessee;
7. an individual who, or his relative or partner
(a) is holding any security of, or interest in, the assessee:
Provided that the relative may hold security or interest in the assessee of the face
value not exceeding ` 1,00,000;
(b) is indebted to the assessee:
Provided that the relative may be indebted to the assessee for an amount not
exceeding ` 1,00,000;
(c) has given a guarantee or provided any security in connection with the
indebtedness of any third person to the assessee:
Provided that the relative may give guarantee or provide any security in
connection with the indebtedness of any third person to the assessee for an amount
not exceeding ` 1,00,000;
8. a person who, whether directly or indirectly, has business relationship with the
assessee of such nature as may be prescribed;
9. a person who has been convicted by a court of an offence involving fraud and a period
of ten years has not elapsed from the date of such conviction.
CA Final Audit - By CA. Sarthak Niraj Jain 199
Audit Under Fiscal Laws

12.3 - Reporting in Form 3CD & Audit Procedure

11. Clause 4 - Indirect Tax Registration


You are appointed as tax auditor of Mr. X, a practicing advocate in Agra. During the previous year he collected
Service Tax of ` 7 lakhs but utilized for personal use. The Commissioner of Central Excise issued a show cause
notice to him why the tax collected by him is not deposited to the Government account. He appeared before
the Commissioner and stated his inability to pay the sum due to financial crisis. The proceedings are still
pending. Mr. X requests you not to disclose his Service Tax registration details while filling particulars to be
furnished in Form No. 3CD. As a tax auditor how would you deal with this (May-2016)
OR
Mr. A, is a renowned lawyer. During the previous year, he collected GST of ` 25 lakhs but utilized it for his
personal use. The department issued a show cause notice to him as to why the tax, collected by him, is not
deposited to the government account. He appeared before the department and stated his inability to pay the
sum due to financial crisis. The proceedings are still pending. Mr. A instructed his tax auditor not to disclose
his GST registration details, while filling particulars to be furnished in Form No. 3CD, believing that the
income tax department might trace his scrutiny proceedings details pending before department which would
bring disrepute to his profession.
Ans. Reporting Requirement of Form 3CD:
(i) Clause of Form 3CD: -Clause (4) of Form 3CD, requires tax auditor to mention the registration
number or any other identification number, if any, allotted, in case the assessee is liable to pay
indirect taxes like excise duty, service tax, sales tax, GST, customs duty, GST etc.
Auditor is required to furnish the details of registration numbers as provided to him by the assessee.

FAST
Sr.
The reporting is however, to be done in the manner or format specified by the e-filing utility
in this context. The information may be obtained and maintained in the following format:
Relevant Indirect Tax Place of Business/Profession / Service Registration/

(ii)
CA.Sarthak Niraj Jain
No Law which requires
registration
unit for which registration is in place/
or has been applied for
Identification number

In the present case: - Mr. X has defaulted in payment of GST for the previous year. Consequently, the
department issued a show cause notice for such non-payment of tax. The arguments are still going
on between the department and assessee. He also restrained his tax auditor from disclosing GST
registration details in tax audit report.
(iii) Conclusion:- Instruction of Mr. X is not acceptable as clause 4 of Form 3CD requires tax auditor to
furnish the details of registration number or other identification number of assessee, if assessee is
required to pay indirect taxes like excise duty, service tax, sales tax, goods and service tax, custom
duty, etc..
12. Clause 4 - Indirect Tax Registration
Arihant Pvt Ltd is engaged in the business of providing corporate/professional training programs. It has an
annual turnover of INR 74 crore. The Company is subject to tax audit for which the work has been started by
the tax auditor. For the financial year ending 31st March 2022, the Company applied for GST registration for 5
new locations for which registration certificates have not yet been received by the Company. However, the
registration number is available on the portal of relevant authority which can be verified by checking the
details of the Company. In this case what should be the audit procedures to verify this registration number?
(RTP-May-2021)
Ans. (i) As per Clause: -Clause (4) of Part A of Form No. 3CD generally requires the auditor to ensure:
 whether the assessee is liable to pay indirect tax like excise duty, service tax, sales tax, goods
and service tax, custom duty, etc.
 If yes, please furnish the registration number or GST number or any other identification number
allotted for the same.

200 Ab Audit Hoga Sabse Scoring - By CA. Sarthak Niraj Jain


Audit Under Fiscal Laws
Thus, the auditor is primarily required to furnish the details of registration numbers as provided
to him by the assessee. The reporting is required to be done in the manner or format specified by
the e-filing utility in this context.
(ii) In the given situation:
 Arihant Pvt Ltd is engaged in the business of providing corporate/professional training programs.
The Company is subject to tax audit.
 For the financial year ending 31st March 2022, the Company applied for GST registration for 5
new locations for which registration certificates have not yet been received by the Company.
 However, the registration number is available on the portal of relevant authority.
(iii) Audit Procedure to verify registration number:
a) The tax auditor of Arihant Private Limited should verify the registration number for the
locations for which registration certificates have not been received from online portal of the
relevant authority.
b) The auditor should also ensure that the details furnished while checking the registration number
pertains to the company only. If the company has filed any returns for these locations,
c) The auditor should enquire for the same from the management and should check those returns
to verify the correctness of the registration numbers.
d) In addition, the auditor should also obtain specific representation in respect of this point from
the management.
13. Clause- 8 Relevant Sections & Clause-12 Presumptive Income
BB Ltd., a non-resident company, is engaged in the business of extraction of mineral oils, having turnover of

FAST
` 20 lakhs during the financial year 2021-22. The company claims that its profits and gains chargeable to tax
under the head “Profits and gains of business or profession” is lower than the deemed income chargeable
under section 44BB of the Income-tax Act, 1961. Therefore, it decided to get its accounts audited under

Ans. (i) CA.Sarthak Niraj Jain


section 44AB of the Income-tax Act, 1961. Discuss reporting requirement of Form 3CD in this behalf.
Clause of Form 3CD:-
 Clause (8) of Form 3CD, requires tax auditor to mention the relevant clause of section 44AB
under which the audit has been conducted. Accordingly, auditor is required to mention clause (c)
of Section 44AB which requires tax audit.
 Further, as per Clause (12) of Form 3CD, if the profit and loss account of the assessee includes
any profits and gains assessable on presumptive basis, the tax auditor has to indicate the
amount and the relevant sections (44AD, 44ADA, 44AE, 44AF, 44B, 44BB, 44BBA, 44BBB Chapter
XII-G, First Schedule or any other relevant section)

(ii) In the given case:


 BB Ltd., is a non-resident company which is engaged in the business of extraction of mineral
oils, hence, its income is chargeable in accordance with the provisions of section 44BB of the
Income tax Act, 1961.
 But as the company is claiming lower income in comparison to deemed income u/s 44BB,
provisions of Section 44AB in relation to audit has to be complied with.

(iii) Conclusion:
 Under Clause 8, auditor is required to indicate the relevant clause of Section 44AB under which
audit is to be conducted and
 In addition under clause 12 auditor is required to indicate the amount of profits of business
covered u/s 44BB and the relevant section.

CA Final Audit - By CA. Sarthak Niraj Jain 201


Audit Under Fiscal Laws

14. Clause 11(b) & Clause 35(b) - Non-Maintenance of Stock Register


ABC Printing Press, a proprietary concern, is expecting to make a turnover more than ` 7.03 crore for the year
ending 31.03.2022. The Management explained its auditor Mr. Z, that it undertakes different job work orders
from various customers. The raw materials required for each job are dissimilar. It purchases the raw
materials as per specification/ requirements of each customer and there is hardly any balance of raw
materials remaining in the stock except pending work-in-progress at the year end. Because of variety and
complexity of materials, it is impossible to maintain a stock-register. Give your comments.
(Study Material)(RTP-May-2020)
Ans. (i) Clause of Form 3CD: -As per requirement of Para 35(b) and Para 11(b) of Form 3CD, auditor is
required to report on the details of stock and account books (including stock register) maintained.

(ii) Reporting:
1) Auditor need to verify the closing stock of raw materials and finished products and by-
products of the entity.
2) In case the details are not properly maintained: - he has to specifically mention the same, with
reasons for non-maintenance of stock register by the entity.
3) The explanation of the entity for the use of varieties of raw materials for different jobs
undertaken may be valid. But the auditor needs to verify the specified job-orders received and
the different raw materials purchased for each job separately. The use of different papers
(quality, quantity and size) ink, colour etc. may be examined.
4) The auditor may also enquire with the other similar printers in the locality to ensure the
prevailing custom.

FAST
5) At the same time, he has to report and certify under clause 35(b) and clause 11(b) of Form
3CD read with the Rule 6G(2) of the Income-tax Act, 1961, about the details of stock and
account books (including stock register) maintained.

15.
CA.Sarthak Niraj Jain
(iii) Conclusion: -Auditor is required to report under Para 35(b) and 11(b) about non-maintenance of
stock register.
Applicability of ICDS – Section 145
CA Sumati has been appointed as a tax auditor under section 44 AB of the Income-tax Act, 1961, of M/s Pal &
Company, a partnership firm, following cash basis of accounting. CA Sumati made the qualification that ICDS
were not followed by the entity while maintaining books of accounts. Whether qualification made by CA
Sumati is correct? Explain the provisions of the applicability of ICDS with reference to Sec 145(2) of the
Income Tax Act, 1961. (RTP-May-2022)
Ans. (i) Provision: -Section 145(2) empowers the Central Government to notify in the Official Gazette from
time to time, income computation and disclosure standards are to be followed by any class of
assessees or in respect of any class of income.
 Accordingly, the Central Government had, in the exercise of the powers conferred under section
145(2), notified ten income computation and disclosure standards (ICDSs) to be followed by
all assesses (other than an individual or a HUF who is not required to get his accounts of one
previous year audited in accordance with the provisions of section 44AB), following the
mercantile system of accounting, for the purposes of computation of income chargeable to
income-tax under the head “Profit and gains of business or profession” or “ Income from other
sources”. from the A.Y. 2017-18.
 All the notified ICDSs are applicable for computation of income chargeable under the head
“Profits and gains of business or profession” or “Income from other sources” and not for the
purpose of maintenance of books of accounts.
 In the case of conflict between the provisions of the Income‐tax Act, 1961 and the notified ICDSs,
the provisions of the Act shall prevail to that extent.

202 Ab Audit Hoga Sabse Scoring - By CA. Sarthak Niraj Jain


Audit Under Fiscal Laws
(ii) In the given situation: - Sumati has been appointed as a tax auditor of M/s Pal & Company, a
partnership firm, following the cash basis of accounting. CA Sumati made the qualification that ICDS
were not followed by the entity while maintaining books of accounts.
(iii) Conclusion: -In view of the above provisions, it is clear the ICDS is applicable on a mercantile
system of accounting, and it is only for the purpose of computation of income chargeable to income
tax under the head “Profits and Gains of business or profession” or “Income from other sources”
and not for the maintenance of books of accounts. Thus, qualification made by CA. Sumati is not
correct.
16. Clause 14 - Changes in Valuation of Inventory
A leading jewellery merchant used to value his inventory at cost on LIFO basis. However, for the current year,
in view of requirements of AS-2, he changed over to FIFO method of valuation. The difference in value of stock
amounted to `55 lakhs which is higher than that under the previous method. In such a situation, what are the
reporting responsibilities of a Tax Auditor u/s 44AB of Income-tax Act, 1961
Ans. Reporting of Changes in Valuation of Inventory:
(i) Provision:- As per the provisions of Income tax Act, 1961, if the change in method of valuation is
bonafide, and is regularly and consistently adopted in the subsequent years as well, such change
would be permitted to be made for tax purposes.
(ii) In the given case:- the change in the valuation of stock is pursuant to mandatory requirements of
the AS-2 ‘Valuation of Inventories’ and therefore should be viewed as bonafide change and allowed.
(iii) Clause 14 of Form 3CD:- also requires in this regard reporting over the following:
a) Method of valuation of closing stock employed in the previous year.
b) In case of deviation from the method of valuation prescribed under section 145A, and the effect

FAST
thereof on the profit or loss, please furnish:
Serial Number Particulars Increase in Profit (`) Decrease in Profit (`)

CA.Sarthak Niraj Jain


(iv) Conclusion:- In reference to Section 145A, auditor is not required to change the method of
valuation of purchases, sales and inventories which is regularly employed by the assessee. Auditor is
required to adjust the valuation for any tax, duty, cess or fee actually paid or incurred by the
assessee, if the same had not already been adjusted.
17. Clause 16(e) - Capital receipt & Audit Procedure
In the course of your tax audit assignment u/s 44AB of the Income tax Act, 1961 of Dream Bank Ltd. You have
instructed your assistant to find out receipt of capital nature which might not have been credited to Profit &
Loss Account and needs to be reported in Para 16(e) of 3CD. Your audit assistant seeks your guidance in
reporting the same. Specify any four illustrative examples of such receipt. (May-2019–New)
OR
What can be the possible instances of capital receipt which, if not credited to the P & L Account, needs to be
reported in Form 3CD? (Nov-2015)
OR
While writing the audit program for tax audit in respect of A Ltd you wish to include possible instances of
capital receipt if not credited to Profit & Loss Account which needs to be reported under clause 16(e) of Form
3CD. Please elucidate possible instance.
Ans. (i) Instances of Capital receipt:
1. Capital subsidy received in the form of Government grants, which are in the nature of
promoters’ contribution i.e., they are given with reference to the total investment of the
undertaking or by way of contribution to its total capital outlay.
For example: -Capital Investment Subsidy Scheme.

CA Final Audit - By CA. Sarthak Niraj Jain 203


Audit Under Fiscal Laws
2. Government grant in relation to a specific fixed asset where such grant is shown as a deduction
from the gross value of the asset by the concern in arriving at its book value.
3. Compensation for surrendering certain rights.
4. Profit on sale of fixed assets/investments to the extent not credited to the profit and loss
account

(ii) Audit procedures:


Capital receipts are not generally credited to profit and loss account hence the auditor should take
enough care to check out any transaction generating the capital receipts by –
a) Enquiring whether the assessee is in receipt of any amount of capital nature during the previous
year.
b) Go through the financial statements in particular reserve account, to ascertain whether the
assessee has received any such receipts and credited them directly to reserve account.
c) Enquiring whether the assessee has credited such receipts to profit and loss account.
d) Checking that any such receipt is accounted for in terms of method of accounting followed by the
assessee.
18. Clause 17 - Sale of Property at Lower Price than SDV
ABC Ltd., a manufacturing concern, sold a house property in Mumbai for a consideration of ` 48 lakh, to Mr. X
on 1-8-2021. ABC Ltd. had purchased the house property in the year 2017 for ` 30 lakh. The stamp duty value
on the date of transfer, i.e., 1-8-2021, is ` 65 lakh for the house property. How would you deal this matter in
tax audit report?
(1) Details of property
(2)
(3)
FAST
Consideration received or accrued
Value adopted or assesses or assessable

Ans.
(4)

(i)
CA.Sarthak Niraj Jain
Whether provisions of section 43CA (1) 2nd proviso or section 56(2) (x) 4th proviso applicable?
[Yes/No]
Clause of Form 3CD:
 Clause 17 requires tax auditor to furnish certain information if land or building or both is
transferred during the previous year for a consideration less than value adopted by any
authority of a State Government referred to in section 43CAor 50C as under:
Details of Consideration Value adopted or Whether provisions of section 43CA (1)
Property Received or assesses or 2nd proviso or section 56(2) (x) 4th
Accrued assessable proviso applicable? [Yes/No]

(ii) Audit Procedure:-


 For this purpose, auditor should obtain a list of all properties transferred by the assessee during
the previous year and
 Furnish the amount of consideration received or accrued, as disclosed in the books of account of
the assessee.
 For reporting the value adopted or assessed or assessable, the auditor should obtain from the
assessee a copy of the registered sale deed.
 In case the property is not registered, the auditor may verify relevant documents from
relevant authorities or
 Obtain third party expert like lawyer, solicitor representation to satisfy the compliance of section
43CA / section 50C of the Act.
(iii) Conclusion: - ABC ltd. has sold the house property to Mr. X at a price lower than value adopted for
stamp duty purpose, tax auditor is required to report on the same under Clause 17 of Form 3CD.

204 Ab Audit Hoga Sabse Scoring - By CA. Sarthak Niraj Jain


Audit Under Fiscal Laws

19. Clause (17) & 29B of Form 3CD


A is the proprietor of a firm M/s ABC & Co. The firm is expecting a turnover of ` 900 lakhs during the financial
year ending 31/03/2022. The firm sold land and building during the year for a consideration of ` 15 lakhs,
whose value for stamp duty purposes was ` 17 lakhs. As’ the Tax Auditor of the said firm, is the above
required to be reported? If yes, how will you report the same?(Figures Restated)
(Study Material)(Nov-2019–Old)
Ans. (i) Clause of Form 3CD:- As per Clause 17 of Form 3CD, the tax auditor is required to furnish detailed
information in case if any land or building or both is transferred during the previous year for a
consideration less than value adopted or assessed or assessable by any authority of a State
Government referred to in section 43CA or 50C, as under:
Details of Consideration Value adopted or Whether provisions of section 43CA(1)
property received or assessed or 2nd proviso or section 56(2)(x) 4th
accrued assessable proviso applicable? [Yes/No]

(ii) Audit Procedure:-


a) The auditor should obtain a list of all properties transferred by the assessee during the previous
year.
b) He may also verify the same from the statement of profit and loss or balance sheet, as the case
may be.
c) Further, the auditor has to furnish the amount of consideration received or accrued, during the
relevant previous year of audit, in respect of land/building transferred during the year as
disclosed in the books of account of the assessee.

FAST
d) For reporting the value adopted or assessed or assessable, the auditor should obtain from the
assessee a copy of the registered sale deed in case, the property is registered.
e) In case the property is not registered, the auditor may verify relevant documents from
relevant authorities or obtain third party expert like lawyer, solicitor representation to satisfy

CA.Sarthak Niraj Jain


the compliance of section 43CA / section 50C of the Act.
f) In exceptional cases where the auditor is not able to obtain relevant documents, he may state
the same through an observation in his report 3CA/CB.
g) In addition, as per clause 29(B) (Finance Act,2021),
in case of an immovable property, where the stamp duty value exceeds the consideration by
less than the higher of
 Rupees 50,000 or
 10% of the consideration, the difference is not chargeable to tax.
Therefore, for any immovable property, where the stamp duty value is up to 110% of the sale
consideration, no addition can be made under section 56(2)(x).
(iii) In the given case:- M/s. ABC & Co., has sold land and building during the year for a consideration of
rupees 15 lakhs which is less than stamp duty value i.e. rupees 17 lakh. Further, it is also more
than the higher of 10% of consideration 15 lakh i.e. 1,50,000 rupees or 50,000 rupees.
(iv) Conclusion:- Hence, tax auditor is required to report on the same under Clause 17 and clause 29(B)
of Form 3CD.
20. Clause 18 & Clause 13(f)
As an auditor of a partnership firm under section 44 AB of the Income Tax Act, 1961, how would you report
on the following :
(i) M/s. WAR wants to purchase a new machine which is available for ` 45,000, if purchased on 45 days'
credit and for ` 40,000, if purchased by cash. The machine is purchased by paying cash on 01/08/2021
and depreciation on it charged to profit and loss account is ` 6000.
(ii) Technical support is received from Mr. DND, a non-resident for installation of a new machinery. He has
raised invoice dated 01/05/2021 in USD equivalent to ` 5,00,000. The entire dues are cleared through
appropriate banking channel on 25/06/2021. (July-2021-Old)

CA Final Audit - By CA. Sarthak Niraj Jain 205


Audit Under Fiscal Laws

Ans. (i) Clause 18 - Depreciation


(a) Provision:- As per section 43(1) to the Income-tax Act, 1961,
 The expression "actual cost" means the actual cost of the asset to the assessee as reduced by that
portion of the cost thereof, if any, as has been met directly or indirectly by any other person or
authority.
 However, where an assessee incurs any expenditure for acquisition of any asset or part thereof in
respect of which a payment or aggregate of payments made to a person in a day, otherwise than by
an account payee cheque drawn on a bank or account payee bank draft or use of electronic
clearing system through a bank account or through such other prescribed electronic mode, exceeds
`10,000, such expenditure shall not form part of actual cost of such asset.

(b) In the given case: - M/s. WAR purchased a new machine by paying cash amounting ` 45,000 and
charged the depreciation of ` 6,000 to profit and loss account on the same.

(c) Conclusion & Reporting: -


 According to section 43(1), ` 45,000 paid in cash will not form part of actual cost of such asset.
Therefore, depreciation on the same is also not allowable.
 The tax auditor is required to report on the same in accordance with Clause 18.
As per Clause 18 of Form 3CD, tax auditor is required to report the particulars of depreciation
allowable as per the Income-tax Act, 1961 in respect of each asset or block of assets, as the case
may be, in the following form:

FASTClause 18 (d) Additions/deductions during the year with dates;


in the case of any addition of an asset, date put to
use; including adjustments on account of –

CA.Sarthak Niraj Jain


Clause 18 (e)
Clause 18 (f)
Depreciation allowable.
Written down value at the end of the year.
(ii) Clasue 13f –Capitalization of technical support fees.
(a) In the given case: - Technical support received from Mr. DND, a non-resident, is for
installation of new machinery and therefore, it would have been capitalized as the cost of
machinery. It is assumed that amount paid through bank was after complying with TDS
requirements.
(b) Reporting: -Therefore, the tax auditor will report the same in accordance with Clause 13 (f) of
Form 3CD i.e., Disclosure as per ICDS.
Clause 13 (f): Disclosure as per ICDS:
S. No. ICDS Disclosure
ICDS V Tangible Fixed Assets Addition of ` 5,00,000 technical support fee to Non-resident
being capitalized as the cost of machinery

21. Clause 21(d) - Payment made in Cash >10000


XYZ Ltd. pays ` 90,000 for its 10 employees to a Hotel as boarding and lodging expenses of such employees
for a conference. The Company pays the amount in cash to the Hotel. The Hotel gives 9 bills each amounting to
` 9,000. The Company contends that each bill is within the limit; so, there is no violation of the provisions of
the Income-tax Act, 1961. As the tax auditor, how would you deal with the matter in your tax audit report for
the Assessment Year 2022-23?

206 Ab Audit Hoga Sabse Scoring - By CA. Sarthak Niraj Jain


Audit Under Fiscal Laws

Ans. Reporting for Cash Payments above ` 10,000:


(i) Clause of Form 3CD:
• Clause 21(d) of Form 3CD requires tax auditor to report on disallowance under section
40A(3).
• Disallowance u/s 40A(3) of the Income tax Act, 1961 is attracted if the assessee incurs any
expenses in respect of which payment or aggregate of payments made to a person in a day,
otherwise than by an account payee cheque drawn on bank or account payee draft, exceeds `
10,000.
(ii) In the given case: -The tax auditor found that a hotel issued 6 bills to XYZ Ltd. Each amounting to
` 9,000 for boarding & lodging expenses of 6 employees. XYZ Ltd. In aggregate has paid ` 90,000 to
the hotel in cash.
(iii) Conclusion: -Contention of the company that each bill is within the limit is not tenable since aggregate
of payments need to be considered. Hence payments made by the XYZ Ltd. are inadmissible u/s
40A(3) of the Income tax Act, 1961 and hence, needs to be reported under clause 21(d) of Form 3CD.
22. Clause 21(d) - Payment made in Cash >10000
You are the Tax auditor of BL & Co., a partnership firm engaged in the business of plying of Goods Carriages
for the financial year 2021-22 having a turnover of ` 20 crores. How would you deal and report on the
following:
(i) Payment of ` 50,000 in cash to Mr. R on 10th September, 2021 towards settlement of invoice for
expenses accounted in financial year 2020-21.
(ii) Payments of 3 invoices of ` 15,000 each made in cash to Mr. Y on 8th, 9th, 10th, July, 2020 respectively.
(Nov-2018-Old)
Ans.
(i) FAST
Reporting of payments exceeding ` 10,000 in cash:
Clause of Form 3CD
• Clause 21(d)(A) and 21(d)(B) of Form 3CD, requires tax auditor to scrutinize on the basis of

CA.Sarthak Niraj Jain


the examination of books of account and other relevant documents/evidence, whether the
expenditure covered under section 40A(3) and 40A(3A) respectively read with rule 6DD were
made by account payee cheque drawn on a bank or account payee bank draft. If not, the same
has to be reported under abovementioned clauses.
(ii) Provisions
• As per section 40A(3) of the Income tax Act, 1961, an expenditure is disallowed if the assessee
incurs any expenses in respect of which payment or aggregate of payments made
• To a person in a day, otherwise than by an account payee cheque drawn on bank or account
payee draft, exceeds ` 10,000.
(iii) Exception: -In case of payment made for plying, hiring or leasing of goods carriage, limit is ` 35,000
instead of ` 10,000.
• As per section 40A(3A) of the Income tax Act, 1961:-
Where an allowance has been made in the assessment for any year in respect of any liability
incurred by the assessee for any expenditure and subsequently during any previous year the
assessee makes payment in respect thereof, otherwise than by an account payee cheque
drawn on a bank or account payee bank draft, the payment so made shall be deemed to be the
profits and gains of business or profession and accordingly chargeable to income-tax as income of
the subsequent year if the payments made to a person in a day, exceeds ` 10,000 (` 35,000 in
case of plying, hiring or leasing of goods carriages).
(iii) Conclusions
• Based on the above-mentioned provisions, following conclusion may be drawn:
(i) Reporting required under Clause 21(d)(B) w.r.t. payment of ` 50,000.

CA Final Audit - By CA. Sarthak Niraj Jain 207


Audit Under Fiscal Laws
(ii) Reporting required under Clause 21(d)(A) w.r.t. each payment as individual payment
made on a day exceeds ` 10,000.
SJ’s Note:- Limit of ` 35,000 is not applicable as this limit is applicable when the payee is engaged in the
business of plying of goods carriages. In this case, payer is engaged in the business of plying of goods
carriages hence, limit of ` 10,000 will be applicable assuming that payee is not engaged in business of plying
of goods carriages.
23. Clause 21(d) - Payment made in Cash >10000
Mr. R, the tax auditor finds that some payments inadmissible under Section 40A(3) were made, and advised
the client to report the same in form 3CD. The client contends that cash payments were made since the other
parties insisted upon the same and did not have Bank Accounts. Comment.
Ans. (i) Clause of Form 3CD:
 Clause 21(d) of Form 3CD requires tax auditor to report on disallowance under section 40A(3).
 Disallowance u/s 40A(3) of the Income tax Act, 1961 is attracted if the assessee incurs any
expenses in respect of which payment or aggregate of payments made to a person in a day,
otherwise than by an account payee cheque drawn on bank or account payee draft, exceeds `
10,000.
(ii) Rule 6DD:
 However, there are certain cases as specified in Rule 6DD, in which, disallowance under section
40A(3) would not be attracted.
 Cash payment made on insistence of other parties on the contention that they do not have bank
accounts is not covered under the list of exceptions provided under Rule 6DD.
(iii) Audit Procedure:-
 In the present case, tax auditor is required to scrutinize on the basis of the examination of

FAST
books of account and other relevant documents/evidence,
 Whether the expenditure covered under section 40A(3) read with rule 6DD were made by
account payee cheque drawn on a bank or account payee bank draft. If not, the same has to be

CA.Sarthak Niraj Jain


reported under abovementioned clause.
(iv) Conclusion: -Payments made by the XYZ Ltd. are inadmissible u/s 40A(3) of the Income tax Act,
1961 and hence, needs to be reported under clause 21(d) of Form 3CD.
24. Clause 21(d)(A) & 21(d)(B) - Payment made in Cash >35000
Gambit Ltd., engaged in the leasing of goods carriage, appointed you as the tax auditor for the financial year
2021-22. How would you deal with the following payments to Mr. X, Mr. Y and Mr. Z (engaged in leasing of
goods carriage) relating to the leasing transactions in your tax audit report:
(i) Payments of 6 invoices of ` 5,000 each made in cash to Mr. X on 4th July, 2021.
(ii) Payments of 2 invoices of ` 18,000 each made in cash to Mr. Y on 5th July, 2021 and 6th July, 2021
respectively.
(iii) Payment of ` 40,000 made in cash to Mr. Z on 7th July, 2021 against an invoice for expenses booked in
2020-21 (MTP-Dec-2021)
OR
ABC Ltd., engaged in the manufacturing of goods carriage, appointed you as the tax auditor for the financial
year 2021-22. How would you deal with the following matters in your tax audit report?
(i) Payments of 6 invoices of ` 5,000 each made in cash to Mr. X, engaged in leasing of goods carriages on
4th July, 2021.
(ii) Payments of 2 invoices of ` 18,000 each made in cash to Mr. Y, engaged in leasing of goods carriages on
5th July, 2021 and 6th July, 2021 respectively.
(iii) Payment of ` 40,000 made in cash to Mr. Z, engaged in leasing of goods carriages on 7th July, 2021
against an invoice for expenses booked in 2020-21.
Ans. (i) Clause of Form 3CD:
 Clause 21(d)(A) and 21(d)(B) of Form 3CD, requires tax auditor to scrutinize on the basis of the
examination of books of account and other relevant documents/evidence, whether the

208 Ab Audit Hoga Sabse Scoring - By CA. Sarthak Niraj Jain


Audit Under Fiscal Laws
expenditure covered under section 40A(3) and 40A(3A) respectively read with rule 6DD were
made by account payee cheque drawn on a bank or account payee bank draft. If not, the same
has to be reported under abovementioned clauses.
(ii) Provision:
 As per section 40A(3) of the Income tax Act, 1961:- an expenditure is disallowed if the
assessee incurs any expenses in respect of which payment or aggregate of payments made to a
person in a day, otherwise than by an account payee cheque drawn on bank or account payee
draft, exceeds ` 10,000.
 However, in case of payment made for plying, hiring or leasing of goods carriage, limit is
` 35,000 instead of ` 10,000.
 As per section 40A(3A) of the Income tax Act, 1961:- where an allowance has been made in the
assessment for any year in respect of any liability incurred by the assessee for any expenditure
and subsequently during any previous year the assessee makes payment in respect thereof,
otherwise than by an account payee cheque drawn on a bank or account payee bank draft,
 The payment so made shall be deemed to be the profits and gains of business or profession and
accordingly chargeable to income-tax as income of the subsequent year if the payments made to
a person in a day, exceeds ` 10,000 (` 35,000 in case of plying, hiring or leasing of goods
carriages).
(iii) Conclusion:- Based on the above-mentioned provisions, following conclusion may be drawn:
 Payments of 6 invoices of ` 5,000 each aggregating ` 30,000 made in cash on 4th July, 2021 need
not be reported as the aggregate of payments do not exceed ` 35,000.
 Payments of 2 invoices of ` 18,000 each made in cash on 5th July, 2021 and 6th July, 2021


FAST
respectively aggregating ` 36,000 need not be reported as the payment do not exceed ` 35,000
in a day.
Payment of ` 40,000 made in cash against an invoice for expenses booked in 2020-21 is likely to be
deemed to be the profits and gains of business or profession under section 40A(3A) of the Income

25.
CA.Sarthak Niraj Jain
tax Act, 1961. Thus, the details of such amount need to be furnished under clause 21(d)(B) of
Form 3CD.
Clause 25 - Profit Chargeable to tax u/s 41 & Computation
M/s RB Ltd. is engaged in the trading of engineering goods. Turnover of the Company for the year ended
31.03.2022 was ` 150.00 crores. During the F.Y 2019-2020, the Company claimed deduction of bad debts
amounting to ` 100.00 lakhs while filing income tax return (Out of total debts of ` 150.00 lakhs due from Mr.
X). However, during the F.Y. 2021-2022, the Company was able to recover ` 75.00 lakhs from Mr. X through
legal means and correctly credited to its profit and loss account. As a tax auditor, is it required to report such
transactions in Tax Audit report under Income Tax Act, 1961? (July-2021-New)
Ans. Profit chargeable to tax u/s 41 & Computation
(i) In the given case: - RB Ltd, the company had claimed deduction on account of bad debts amounting
to ` 100 Lakhs in the F.Y. 2018-19. However, in F.Y 2020-21 company recovered ` 75 Lakhs which was
correctly credited to its profit and loss account. This amount of ` 75 Lakhs would be taxed under
section 41(4) of the Income Tax Act, 1961.
(ii) Reporting: - Tax Auditor shall report this under clause 25 as under- Any amount of profit chargeable
to tax under section 41 and computation thereof.
 The tax auditor should obtain a list containing all the amounts chargeable under section 41 with
the accompanying evidence, correspondence, etc.
 He should in all relevant cases examine the past records to satisfy himself about the correctness of
the information provided by the assessee.
 The tax auditor has to state the profit chargeable to tax under this section. This information has
to be given irrespective of the fact whether the relevant amount has been credited to the profit
and loss account or not.

CA Final Audit - By CA. Sarthak Niraj Jain 209


Audit Under Fiscal Laws
 The computation of the profit chargeable under this clause is also to be stated.
 The tax auditor should maintain the following in his working papers for the purpose of
furnishing details required in the format provided in the e-filing utility:
Sr. Name of Amount of Section Description of Computation if
No. person income transaction any
1 2 3 4 5 6
26. Clause 26 (A) & (B)- Tax Liability Pre-existed/Incurred in the PY
You are doing Tax Audit of Private Limited Company for the financial year ending 31st March, 2022. During
audit, you notice that the company is not regular in deposit of VAT/GST and there remains pendency every
year. The details of VAT/GST payable are:
(i) GST payable as on 31/03/2021 of FY 2020-21 was ` 200 Lakh and out of which ` 100 Lakh was paid on
15/09/2021 and ` 50 Lakh on 30/03/2022 and balance of ` 50 Lakh paid on 16/09/2022.
(ii) GST payable of current financial year 2021-22 was ` 100 lakh and out of this, ` 40 Lakh was paid on
25/05/2021 and balance of ` 60 Lakh remained unpaid till the due date of return. The date of Tax Audit
report and due date of return was 30th September. Now as a Tax Auditor, how/where the said
transaction will be reflected in Tax Audit Report under Section 43B(a)?
(RTP-Nov-2019)(MTP-Nov-2019)
Ans. Reporting in Tax Audit Report:
(i) Clause of Form 3CD:- Clause 26(A) and 26(B)
• Any amount of GST/Tax payable on the last day of previous year (opening balance) as well as
on the last day of current year has to be reported in Tax Audit Report under clause 26(A) and
26(B) in reference of section 43 B.

FAST
Clause 26 (A) dealt GST/VAT payable on the pre-existed of the first day of the previous year
but was not allowed in the assessment of any preceding previous year and was either paid
{clause 26(A) (a)}/ or/ and/ not paid during the previous year {clause 26(A)(b)}
(ii)
CA.Sarthak Niraj Jain
Reporting:- The details will be as under in regard to opening balances:
Liability Pre-existed on the previous year.
Outstanding Amount Amount
Amount paid /
Sr. Nature of Opening balance written unpaid at
Sec. set-off during
No. Liability not allowed in back to P&L the end of
the year
previous year Account the year
01 43B(a) VAT / GST 100 lakh 50 lakh 0 50 lakh
It has been assumed that 100 lakh was allowed in last year as it was paid before the due date of return.
Liability incurred during the previous year
Amount incurred in Amount paid/set-off Amount unpaid on
previous year but before the due date of the due of filing of
Sr. Nature of remaining filing return/date upto return/date upto
Sec.
No. Liability outstanding on last which reported in the which reported in
day of previous tax audit report, the tax audit report,
year. whichever is earlier whichever is earlier
01 43B(a) VAT / GST 100 lakh 40 lakh 60 lakh
27. Clause 27(b)- Expenditure/Income of earlier years Dr./ Cr. in current year
How will you verify the Income & Expenditure of earlier years credited/debited in current year to reporting
under clause 27(b) of 3CD while carrying out Tax Audit u/s 44AB of the Income tax Act, 1961?
(Study Material)(Nov-2019-Old)

210 Ab Audit Hoga Sabse Scoring - By CA. Sarthak Niraj Jain


Audit Under Fiscal Laws

Ans. Reporting under Clause 27(b) of Form 3CD:


(i) Clause of Form 3CD:- Clause 27(b) requires the tax auditor to report particulars of income or
expenditure of prior period credited or debited to the profit and loss account. Information under
clause 27(b) would be relevant only in those cases where the assessee follows mercantile system
of accounting.
(ii) Audit Procedure:
The tax auditor may perform following procedures for the purpose of reporting under clause 27(b):
 Ask the assesse to furnish a schedule indicating particulars of expenditure/ income of any
earlier year debited/credited to the Profit & Loss account of the relevant previous year.
 Verify various expenses account to see whether any expenditure pertaining to any earlier
year has been debited to the profit and loss account.
For example, an assessee might have paid some certification fee etc., for four years in the year
under audit. In that case payment of expenses for previous three years would be prior period
payment.
 Also see that all such items are properly disclosed.
 Check while conducting the routine audit that there is no expenditure/ income relating to
earlier years that has not been mentioned in the particulars furnished by the assessee.
 In case of cash system of accounting there will be no amount to be disclosed under this head.
28. Clause 28 & Section 56(2)(viia)
ABC Pvt. Ltd. and XYZ Pvt. Ltd. are the Companies in which public are not substantially interested. During the
previous year 2021-22, ABC Pvt. Ltd. received some property being shares of XYZ Pvt. Ltd. The details of
which are provided below:
(a)
(b)
(c)
FAST
No. of Shares = 1000
Face Value = ` 10 per share
Aggregate Fair Market Value = ` 1,00,000
(d)
CA.Sarthak Niraj Jain
Consideration Value =Nil
As the tax auditor how would you deal with the situation? (MTP-Nov-2018)(May-2016)
Ans. Reporting requirement in Form 3CD:
(i) Clause of Form 3CD :- Clause 28 of Form 3CD requires the auditor to report “Whether during the
previous year the assessee has received any property, being share of a company not being a
company in which the public are substantially interested, without consideration or for inadequate
consideration as referred to in section 56(2)(viia), if yes, please furnish the details of the same”
(ii) Provision:- Section 56(2)(viia) provides that where a firm or a company not being a company in
which the public are substantially interested, receives, in any previous year any property being
shares of a company (not being a company in which the public is substantially interested, without
consideration, the aggregate fair value of which exceeds ` 50,000, the whole of the aggregate fair
market value of such property shall be chargeable to income-tax under the head “Income from
other sources”.
(iii) Finance Act, 2017 amends Sec. 56(2): -In accordance with which provisions of Section 56(2)(viia) are
not applicable for transactions taken place on or after 01.04.2017.
(iv) Reporting:- This transaction now, falls under clause (x) of Sec. 56(2). For transactions falling u/s
56(2)(x), reporting is required as per Clause No. 29B, which provides as follow:
(a) Whether any amount is to be included as income chargeable under the head ‘income from other
sources’ as referred to in clause (x) of sub-section (2) of section 56? (Yes/No)
(b) If yes, please furnish the following details:
 Nature of income;
 Amount (in `) thereof.

CA Final Audit - By CA. Sarthak Niraj Jain 211


Audit Under Fiscal Laws

29. Clause 28,29(B) & Section 56(2)(viia)


While doing Tax Audit, under section 44AB of the Income tax Act, 1961, of the accounts of Glue Private
Limited for the Assessment Year 2021-22, it was found that during the Financial Year 2020-21, Glue Private
Limited had received 9,000 shares, the market value of which was ` 90,000 on the date of transfer, at a price
of ` 45,000 from stick Private Limited. The Management of Glue Private Limited maintained that the
transaction was as per the terms of negotiations and there would be no cause for the Auditor to bring this
matter in his Tax Audit Report-Comment. (Nov-2018-New)
Ans. Reporting requirement in Form 3CD:
(i) Clause of Form 3CD :- Clause 28 of Form 3CD requires the auditor to report “Whether during the
previous year the assessee has received any property, being share of a company not being a
company in which the public are substantially interested, without consideration or for inadequate
consideration as referred to in section 56(2)(viia), if yes, please furnish the details of the same”
(ii) Provision:- Section 56(2)(viia) provides that where a firm or a company not being a company in
which the public are substantially interested, receives, in any previous year any property being
shares of a company (not being a company in which the public is substantially interested, without
consideration, the aggregate fair value of which exceeds ` 50,000, the whole of the aggregate fair
market value of such property shall be chargeable to income-tax under the head “Income from
other sources”.
(iii) Finance Act, 2017 amends Sec. 56(2):- In accordance with which provisions of Section 56(2)(viia) are
not applicable for transactions taken place on or after 01.04.2017.
(iv) Reporting: - This transaction now, falls under clause (x) of Sec. 56(2). For transactions falling u/s
56(2)(x), reporting is required as per Clause No. 29B, which provides as follow:
 Whether any amount is to be included as income chargeable under the head ‘income from other
sources’ as referred to in clause (x) of sub-section (2) of section 56? (Yes/No)
 If yes, please furnish the following details:

FAST
 Nature of income;
 Amount (in `) thereof.
(v) As per Sec. 56(2)(x):- Where any person receives, in any previous year, from any person or persons

CA.Sarthak Niraj Jain


on or after the 1st day of April, 2017, any property, other than immovable property, for a
consideration which is less than the aggregate FMV of the property by an amount exceeding `
50,000, the aggregate FMV of such property as exceeds such consideration, shall be chargeable to
income-tax under the head "Income from other sources".
(vi) Conclusion: -In the present case, difference of Fair Market Value and consideration paid is lower
than ` 50,000, auditor is required to state the answer with “No”.
30. Clause 29 & Section 56(2)(viia) - Issue of shares consideration exceeds FMV –
AB Ltd. is a company in which public are not substantially interested. During the previous year 2021-22, the
company issued shares to residents of India and provides you the following data related to such issue:
 No. of shares issued 1,00,000
 Face Value ` 10 per share
 Fair Market Value (FMV) ` 60 per share
 Consideration received ` 80 per share
The management of the company contends that, it is a normal issue of shares, thus, needs not to be reported.
As the tax auditor of AB Ltd., how would you deal with the matter in your tax audit report? (MTP-Nov-2018)
Ans. Reporting requirement in Form 3CD:
(i) Clause of Form 3CD :- Clause 29 of Form 3CD requires tax auditor to report whether, the assessee
received any consideration for issue of shares which exceeds the fair market value of the shares
as referred to in section 56(2)(viib) of the Income tax Act, 1961.
(ii) Provision:
 Section 56(2)(viia) provides that where a firm or a company not being a company in which the
public are substantially interested, receives, in any previous year any property being shares of a
company (not being a company in which the public is substantially interested, without
consideration,

212 Ab Audit Hoga Sabse Scoring - By CA. Sarthak Niraj Jain


Audit Under Fiscal Laws
 The aggregate fair value of which exceeds ` 50,000, the whole of the aggregate fair market
value of such property shall be chargeable to income-tax under the head “Income from other
sources”.
(iii) In the given case: - AB Ltd. is a company, in which public is not substantially interested. Company
has received consideration for issue of shares of ` 80 per share (Face value ` 10 per share + Premium
` 70 per share) which exceeds the face value of ` 10 per share and fair market value of the shares of
` 60 per share, and hence the case covered u/s 56(2)(viib).
(iv) Conclusion:- Tax auditor of AB Ltd. is required to furnish the details of shares issued under clause
29 of Form 3CD.
31. Clause 31 (ba) & section 269ST - Amount received in Cash
Mr. KK is a contractor dealing in Food catering, flower decorating and light decorating activities. He has
received contract in respect of food catering and flower decorating from one NGO for holding Annual Talent
evening event to celebrate completion of 25 years of their establishment. For the said event Mr. KK has
received in cash ` 1,85,000 for food catering and ` 1,25,000 for flower decoration. As a tax auditor how would
you deal and report on the above? (Jan-2021-Old)
Ans. Amount received in cash - More than ` 2 lakh or more
(i) Provision: - Section 269ST provides that no person shall receive sum of ` 2 lakh or more
 in aggregate from a person in a day; or
 in respect of a single transaction; or
 in respect of transactions relating to one event or occasion from a person otherwise than by:
 an account payee cheque or
 an account payee demand draft or
 by use of electronic clearing system through a bank account
(ii) Audit Procedure:

FAST
 The tax auditor has the responsibility to verify the compliance with the provisions of 269T of
the Income Tax Act.
 The tax auditor is required to report under Clause 31 (ba) particulars of each receipt in an

CA.Sarthak Niraj Jain


amount exceeding the limit specified in section 269ST, in aggregate from a person in a day or in
respect of a single transaction or in respect of transactions relating to one event or occasion
from a person, during the previous year, where such receipt is otherwise than by a cheque or
bank draft or use of electronic clearing system through a bank account:-
(i) Name, address and Permanent Account Number (if available with the assessee) of
the payer;
(ii) Nature of transaction;
(iii) Amount of receipt (in `);
(iv) Date of receipt;
(iii) In the present case: - Mr. KK, contractor dealing in food catering, flower decorating and light
decorating activities, received in cash ` 1,85,000 for food catering and ` 1,25,000 for flower
decoration from one NGO for holding one event, by way of cash which is exceeding prescribed
amount of ` 2,00,000.
(iv) Conclusion: -Thus, tax auditor is required to report the same in compliance with Clause 31 (ba) of
Form 3CD.
32. Section-269SS - Exempted Organisations
The provisions of Sec. 269SS of Income Tax Acts 1961 are not applicable to some organisations. Mention those
organisations. (Jan-2021-Old)
Ans. As per the proviso to section 269SS, the provisions of section 269SS shall not apply to any loan or
deposit taken or accepted from, or any loan or deposit taken or accepted by-
(i) Government;
(ii) Any banking company, post office savings bank or co-operative bank;
(iii) Any corporation established by a Central, State or Provincial Act;

CA Final Audit - By CA. Sarthak Niraj Jain 213


Audit Under Fiscal Laws
(iv) Any Government company as defined in section 2(45) of the Companies Act, 2013
(v) Such other institution, association or body or class of institutions, associations or bodies which the
Central Government may, for reasons to be recorded in writing, notify in this behalf in the Official
Gazette.
Reporting:
 The auditor should maintain the prescribed information in his working papers for the purpose of
reporting against this clause in the format provided in the e-filing utility.
 These particulars need not be given in case of a Government Company, a Banking Company or a
Corporation established by a Central, State or Provincial Act.
33. Clause 34 - TDS Reporting
Discuss the reporting requirement in Form 3CD of Tax Audit Report under Section 44AB of the Income-tax
Act, 1961 for the Tax Deducted at Source.
Ans. Reporting requirement for Tax Deducted at Source in Form 3CD:
Clause 34(a) of Form 3CD:- requires the auditor to furnish below mentioned information if the assessee
is required to deduct or collect tax as per the provisions of Chapter XVII-B or Chapter XVII-BB:
1 Tax deduction and collection Account Number (TAN)
2 Section
3 Nature of payment
4 Total amount of payment or receipt of the nature specified in (3)
5 Total amount on which tax was required to be deducted or collected out of (4)
6
7
FAST
Total amount on which tax was deducted or collected at specified rate out of (5)
Amount of tax deducted or collected out of (6)
8
9
CA.Sarthak Niraj Jain
Total amount on which tax was deducted or collected at less than specified rate out of (7)
Amount of tax deducted or collected on (8)
10 Amount of tax deducted or collected not deposited to the credit of the Central Government out of
(6) and (8)
34. Clause 40 - Accounting Ratios
Write a short note on - Accounting ratios in Form 3CD of Tax Audit.
OR
As a tax auditor, which are the accounting ratios required to be mentioned in the report in case of
manufacturing entities? Explain in detail any one of the above ratios and how does it help the tax auditor in
his analytical review
Ans. Accounting Ratios in Form 3CD:
(i) Clause 40 of Form 3CD: -requires the details regarding turnover, gross profit, etc., for the previous
year and preceding previous year as mentioned below:
Serial Preceding
Particulars Previous year
number previous year
1 Total turnover of the assesse
2 Gross profit/turnover
3 Net profit/turnover
4 Stock-in-trade/turnover
5 Material consumed/finished goods produced

214 Ab Audit Hoga Sabse Scoring - By CA. Sarthak Niraj Jain


Audit Under Fiscal Laws

(ii) Reporting
The details required to be furnished for principal items of goods traded or manufactured or
services rendered. These ratios have to be given for the business as a whole and not product wise.
(iii) Details of Profitability Ratio:
• By relating sales with the Gross profit or net profit, auditor may ascertain the operating
efficiency of an enterprise.
• Auditor is required to inquire any variations in any of these ratios.
• The fall in the gross profit ratio and net profit ration alert the auditor who, in turn should ask
the management for the reasons thereof and which should be carefully examined by him.
(iv) Details of Stock in trade/Turnover Ratio:
• The relationship of stock-in-trade to turnover over a period of time would reveal whether the
entity has been accumulating stocks or there is a decline in the same.
• The auditor may obtain data for about 4-5 years, compute ratio of stock-in-trade/turnover and
compare the change made.
• A study of this relationship would reveal whether stocks are being accumulated or they are
dwindling over a period.
• Such information would provide an input to tax auditor as to whether figures of either stock or
turnover are being manipulated.
35. Clause 41 - Demand Raised or Refund Issued in Other Laws
ABC Ltd. is engaged in providing certain services on which it did not pay any service tax. As per company, said
services were not liable to service tax. However, Department issued a show cause notice to company
demanding service tax along with interest worth ` 5,45,000 on the same and such demand was also
confirmed. An appeal was filed to the Commissioner of Central Excise (Appeals) which passed an order which

FAST
upheld the demand on company. Company, being aggrieved by the order of the Commissioner of Central
Excise (Appeals), decided to file an appeal to the CESTAT against such order. ABC Ltd. has also requested the
tax auditor not to report as those services were not liable for service tax and it has also filed an appeal for the

Ans.
CA.Sarthak Niraj Jain
same
Reporting of taxation demands in Form 3CD:
(i) Clause of Form 3CD:- Clause 41 of Form 3CD requires auditor to furnish the details of demand
raised or refund issued during the previous year under any tax laws other than Income tax Act, 1961
and Wealth tax Act, 1957 along with details of relevant proceedings.
(ii) In the given case:- ABC Ltd. is engaged in providing certain services on which it did not paid any
service tax. Therefore, Department issued a show cause notice and demand for Service Tax along with
interest thereon. ABC Ltd. has also filed an appeal mentioning that said services are not liable to
service tax, but Central Excise (Appeals) has passed an order confirming the demand and ABC Ltd.
being aggrieved by the order of Commissioner of Central Excise (Appeals) decided to file an appeal
against the same. ABC Ltd. also requested the tax auditor not to report on the same as the concerned
services were not liable for any service tax and they have also decided to file an appeal to CESTAT
against the order of Commissioner of Central Excise (Appeals).
(iii) Audit Procedure:
• Tax auditor should obtain a copy of all the demand/ refund orders issued by the Excise
Authorities.
• It may be noted that even though the demand/refund order is issued during the previous year,
it may pertain to a period other than the relevant previous year.
(iv) Conclusion:- Request of ABC Ltd. is not acceptable as clause 41 of Form 3CD requires tax auditor to
furnish the details of demand raised during the previous year under any tax law other than Income
tax Act, 1961.

CA Final Audit - By CA. Sarthak Niraj Jain 215


Audit of Public Sector Undertakings

Chapter
13
AUDIT OF PUBLIC SECTOR UNDERTAKINGS

Past Exams Weightage


5

0
MAY-19 NOV-19 NOV-20 JAN-21 JUL-21 DEC-21

May-19 Nov-19 Nov-20 Jan-21 Jul-21 Dec-21


Marks 5 5 0 5 0 5

13.1 - Basics : PSU Audit & C&AG


1. C&AG Role - Financial Committees
The Comptroller & Auditor General of India plays a key role in the functioning of the financial committees of

Ans.
FAST
Parliament and the State Legislatures. He has come to be recognised as a 'friend, philosopher and guide' of
the Committees. In view of above, you are required to list down role of C&AG.
Role of C&AG in the Financial Committees
(MTP-Nov-2018)

CA.Sarthak Niraj Jain


The Comptroller & Auditor General of India plays a key role in the functioning of the financial committees of
Parliament and the State Legislatures. He has come to be recognised as a 'friend, philosopher and guide' of
the Committees.
(i) His Reports generally form the basis of the Committees' working, although they are not precluded
from examining issues not brought out in his Reports;
(ii) He scrutinises the notes which the Ministries submit to the Committees and helps the Committees to
check the correctness of submissions to the Committees and facts and figures in their draft reports;

(iii) The Financial Committees present their Report to the Parliament/ State Legislature with their
observations and recommendations. The various Ministries / Department of the Government are
required to inform the Committees of the action taken by them on the recommendations of the
Committees (which are generally accepted) and the Committees present Action Taken Reports to
Parliament / Legislature;
(iv) In respect of those Audit Reports, which could not be discussed in detail by the Committees, written
answers are obtained from the Department / Ministry concerned and are sometimes incorporated in
the Reports presented to the Parliament / State Legislature.
This ensures that the Audit Reports are not taken lightly by the Government, even if the entire report is
not deliberated upon by the Committee.
13.2 - Government Company Audit
2. Propriety Audit - Areas
Write short note on areas of proprietary audit under section 143(1) of the Companies Act, 2013
(Study Material)(May-2018-New)

216 Ab Audit Hoga Sabse Scoring - By CA. Sarthak Niraj Jain


Audit of Public Sector Undertakings

Ans. Section 143(1) of the Companies Act, 2013 requires the auditor to make an enquiry into certain specific
areas. In some of the areas, the auditor has to examine the same from propriety angle as to:
(a) Whether loans and advances made by the company on the basis of security have been properly
secured and whether the terms on which they have been made are prejudicial to the interests of the
company or its members;
(b) Whether transactions of the company which are represented merely by book entries are
prejudicial to the interests of the company;
(c) Where the company not being an investment company or a banking company, whether so much of the
assets of the company as consist of shares, debentures and other securities have been sold at a
price less than that at which they were purchased by the company;
(d) Whether loans and advances made by the company have been shown as deposits;
(e) Whether personal expenses have been charged to revenue account;
(f) Where it is stated in the books and documents of the company that any shares have been allotted for
cash, whether cash has actually been received in respect of such allotment, and if no cash has
actually been so received, whether the position as stated in the account books and the balance sheet is
correct, regular and not misleading. A control has been set up to verify the receipt of cash in case of
allotment of shares for cash. Further, if cash is not received, whether the books of accounts and
statement of affairs shows the true picture.
13.3 - Performance Audit
3. Performance Audit & Issues addressed
“A performance audit is an objective and systematic examination of evidence for the purpose of providing an
independent assessment of the performance of a government organization, program, activity, or function in

FAST
order to provide information to improve public accountability and facilitate decision-making by parties with
responsibility to oversee or initiate corrective action.” Briefly discuss the issues addressed by Performance
Audits conducted in accordance with the guidelines issued by C&AG.

CA.Sarthak Niraj Jain OR


(Study Material)(RTP-Nov-2018) (RTP-May-2020)

Write short note on: Issues addressed in Performance Audit of PSUs. (Nov-2018-Old)
Ans. (1) Meaning:
 A performance audit is an objective and systematic examination of evidence for the purpose of
providing an independent assessment of the performance of a government organization,
program, activity, or function
 In order to provide information to improve public accountability and facilitate decision-making
by parties with responsibility to oversee or initiate corrective action.

(2) Performance audit usually addresses the issues of :


Performance audits include evaluation of economy, efficiency and effectiveness.
 Economy: Economy stands for minimising the cost of resources used for an activity, having
regard to appropriate quantity, quality and at the best price. Evaluating economy implies
forming an opinion whether resources have been used economically and acquired in due time, in
appropriate quantity and quality at the best price.
 Efficiency Audit: Efficiency is the measurement of input-output. It is said to be achieved when
the output is maximised at the minimum of inputs, or input is minimised for any given quantity and
quality of output.
Examining efficiency embraces aspects such as whether:
i) procurement practices followed are sound;
ii) resources are properly protected and maintained
iii) human, financial and other resources are efficiently used;

CA Final Audit - By CA. Sarthak Niraj Jain 217


Audit of Public Sector Undertakings
iv) optimum amount of resources (staff, equipment, and facilities) are used in producing or
delivering the appropriate quantity and quality of goods or services in a timely manner;
v) public sector programmes, entities and activities are efficiently managed, regulated,
organised and executed;
vi) efficient operating procedures are used; and
vii) the objectives of public sector programmes are met cost effectively.
 Effectiveness: Effectiveness is the measurement of the extent to which objectives are achieved
and the relationship between the intended impact and the actual impact of an activity.
Examining effectiveness will cover the following:
i) assess whether the objectives of and the means provided (legal, financial, etc.) for a new or
ongoing public sector programme are proper, consistent, suitable or relevant to the policy;
ii) determine the extent to which a program achieves a desired level of results;
iii) assess and establish with evidence whether the observed direct or indirect social and
economic impacts of a policy are due to implementation of the policy or to other causes;
iv) identify factors inhibiting satisfactory performance or goal-fulfillment;
v) assess whether the programme complements, duplicates, overlaps or counteracts other
related programmes;
vi) assess the effectiveness of the program and/or of individual program components;
vii) determine whether management has considered alternatives for carrying out the program
that might yield desired results more effectively or at a lower cost;
viii) assess the adequacy of the management control system for measuring, monitoring and
reporting a programme's effectiveness;
ix) ensure compliance with laws and regulations applicable to the program
x) identify ways of making programmes work more effectively.
4.
FAST
Efficiency of PSU - Aspects
In carrying out efficiency audit of a Public-Sector Undertaking (PSU), what important aspects are required to
be looked into, to assess the efficiency? (May-2018-Old)
Ans.
CA.Sarthak Niraj Jain
Following important aspects are required to be looked into, to assess the efficiency
Efficiency is the measurement of input-output. It is said to be achieved when the output is maximized at
the minimum of inputs, or input is minimized for any given quantity and quality of output.
Examining efficiency embraces aspects such as whether:
(a) sound procurement practices are followed;
(b) resources are properly protected and maintained;
(c) human, financial and other resources are efficiently used;
(d) optimum amount of resources (staff, equipment, and facilities) are used in producing or delivering the
appropriate quantity and quality of goods or services in a timely manner;
(e) public sector programmes, entities and activities are efficiently managed, regulated, organised and
executed;
(f) efficient operating procedures are used; and
(g) the objectives of public sector programmes are met cost-effectively.
5. Audit Criteria Draft & Sources
You have been appointed as auditor of a AKY Ltd. After having determined the audit objectives, now you have
been requested to draft audit criteria. What are the sources that you will use while doing the task?
(MTP-Dec-2021)(Study Material)
Ans. Determining Audit Criteria –
 Audit criteria are the standards used to determine whether a program meets or exceeds
expectations. It provides a context for understanding the results of the audit.
 Audit criteria are reasonable and attainable standards of performance against which economy,
efficiency and effectiveness of programmes and activities can be assessed.
The audit criteria may be sought to be obtained from the following sources:
(i) Procedure manuals of the entity.

218 Ab Audit Hoga Sabse Scoring - By CA. Sarthak Niraj Jain


Audit of Public Sector Undertakings
(ii) Policies, standards, directives and guidelines.
(iii) Criteria used by the same entity or other entities in similar activities or programmes.
(iv) Independent expert opinion and know how
(v) New or established scientific knowledge and other reliable information.
(vi) General management and subject matter literature and research papers.
6. Audit Identification & Sample Observations
The objectives of audit in connection with a State Electricity Distribution Company were to ascertain whether
the:
(i) total cost of providing electricity is being recovered by timely submissions to the State Electricity
Regulatory Comission;
(ii) tariff orders, sales circulars and sales instructions were issued timely, without any ambiguity. They
were implemented in time;
(iii) metering, billing and collection was managed efficiently and effectively;
(iv) monitoring and internal controls were efficient.
What kind of audit is this?
Prepare two sample observations which could be part of the audit report. (Study Material)
Ans. This is a performance audit. Sample observations could be:
(i) Non replacement of defective/ burnt meters: Large number of meters, much in excess of the
permitted limit of 1% of the total meters were defective and their replacement was not completed
within the stipulated time of 1 month. This resulted in billing on average basis for a continuous period
of several months. This could result in losses as well as administrative hassles and disputes with
consumers.
(ii) Under charging of meter rent: As per Schedule of Charges, the Company is required to charge meter
rent of `30 per month for a single phase meter and ` 40 per month for three phase meter. It was
observed that the Company had short charged meter rent of ` 60 lakh from 3 lakh consumers in 5

7.
FAST
lakh bills during the period.
13.4 - Comprehensive Audit
Comprehensive Audit - Matters to be taken by the CA firm

CA.Sarthak Niraj Jain


The Comptroller and Auditor General of India has appointed a chartered accountant firm to conduct the
comprehensive audit of Metro Company Limited (a listed government company) which is handling the Metro
project of the metropolitan city for the period ending 31-03-2022. The work to be conducted under Project A
handled by the Metro Company Limited was of laying down railway line of 124 kilometers. [The chartered
accountant firm reviewed the internal audit report and observed the shortcoming reported about the
performance of Project A regarding the understatement of the Current liabilities and Capital work in
progress by ` 84.68 crore.] Explain some of the matters to be undertaken by the chartered accountant firm
while conducting the comprehensive audit of Metro Company Limited. (Jan-2021-New)
OR
Sun Light Limited is a public sector undertaking engaged in production of electricity from solar power. It had
commissioned a new project near Goa with a new technology for a cost of ` 5,750 crore. The project had seen
delay in commencement and cost overrun. State the maters that a Comprehensive Audit by C&AG may cover
in reporting on the performance and efficiency of this project.
(Study Material)(MTP-Dec-2021)(MTP-Nov-2020)(MTP-May-2021)(Nov-2018-New)
OR
XYZ & Co., a CA. firm was appointed by C&AG to conduct comprehensive audit of ABC Public undertaking.
C&AG advised to cover areas such as investment decisions, project formulation, organisational effectiveness,
capacity utilisation, management of equipment, plant and machinery, production performance, use of
materials, productivity of labour, idle capacity, costs and prices, materials management, sales and credit
control, budgetary and internal control systems, etc. Discuss stating the issues examined in comprehensive
audit. (RTP-May-2020)
OR
The Comptroller and Auditor General assists the legislature in reviewing the performance of public
undertakings. He conducts an efficiency-cum-performance audit other than the field which has already been
covered either by the internal audit of the individual concerns or by the professional auditors. He locates the
area of weakness and extravagance for managements’ information. Explain stating clearly the issues
examined in comprehensive audit. (MTP-Nov-2019)
CA Final Audit - By CA. Sarthak Niraj Jain 219
Audit of Public Sector Undertakings

Ans. (i) A CA Firm has been appointed to conduct comprehensive audit of Metro Company Limited, which is
a listed Govt. Company handling the Metro project. CA firm has observed the shortcomings as stated
in internal audit report regarding understatement of Current liabilities and CWIP by ` 84.68 crore.
(ii) The matters to be undertaken by the CA firm while conducting the comprehensive audit of Metro
Company Limited: Some of the broad areas are listed below:
a) How does the overall capital cost of the project compare with the approved planned costs? Were
there any substantial increases and, if so, what are these and whether there is evidence of
extravagance or unnecessary expenditure?
b) Have the accepted production or operational outputs been achieved? Has there been under-
utilisation of installed capacity or shortfall in performance and, if so, what has caused it?
c) Has the planned rate of return been achieved?
d) Are the systems of project formulation and execution sound? Are there inadequacies? What has
been the effect on the gestation period and capital cost?
e) Are cost control measures adequate and are there inefficiencies, wastages in raw materials
consumption, etc.?
f) Are the purchase policies adequate? Or have they led to piling up of inventory resulting in
redundancy in stores and spares?
g) Does the enterprise have research and development programmes? What has been the
performance in adopting new processes, technologies, improving profits and in reducing costs
through technological progress?
h) If the enterprise has an adequate system of repairs and maintenance?
i) Are procedures effective and economical?
j) Is there any poor or insufficient or inefficient project planning?
SJ’s Comment:-
Though question is specific to solar power manufacturer, the suggested answer is generic in nature.

8. FAST
Audit Report of C&AG
13.5 - Audit Report

The reports of the Comptroller and Auditor General of India on the audit of PSUs are presented to the

CA.Sarthak Niraj Jain


Parliament and to various state legislatures to facilitate a proper consideration. Enumerate the contents of
Audit Report presented by C & AG.
OR
(May-2019–Old)

Write short note on: Contents of Audit report given by Comptroller & Auditor General of India. (Nov-2017)
OR
For facility of consideration, the reports of the Comptroller and Auditor General on the public sector
undertakings of the Central Government are presented to the Parliament in several parts. Briefly describe
those parts.
Ans. For facility of consideration, the reports of the Comptroller and Auditor General on the public sector
undertakings of the Central Government are presented to the Parliament in several parts consisting of the
following:
(a) Introduction containing a general review of the working results of Government companies, Deemed
Government companies and Corporations.
(b) Results of comprehensive appraisals of selected undertakings conducted by the Audit Board.
(c) Resume of the company auditor’s reports submitted by them under the directions issued by the
C&AG and that of comments on the accounts of the Government companies.
(d) Significant results of audit of the undertakings not taken up for appraisal by the Audit Board.
For certain specified states, the Comptroller and Auditor General submit a separate audit report
(commercial) to the legislature, while for other States/Union Territories with legislature, there is a
commercial chapter in the main audit report. The State audit reports, contains both the results of audit
appraisal of performance of selected companies/corporations as well as important individual instances of
financial irregularities, wasteful expenditure, system deficiencies noticed by the statutory auditors,
comments noticed in Government audit in the audit functions of certification of accounts and a general
review of the working results of Government companies and corporations.

220 Ab Audit Hoga Sabse Scoring - By CA. Sarthak Niraj Jain


Liabilities of Auditors

Chapter
14
LIABILITIES OF AUDITORS

Past Exams Weightage


1

0
MAY-19 NOV-19 NOV-20 JAN-21 JUL-21 DEC-21

May-19 Nov-19 Nov-20 Jan-21 Jul-21 Dec-21


Marks 0 0 0 0 0 0

14.1 - Professional Negligence


1. Auditor’s Negligence in Performance of Duties
A Chartered Accountant in practice has been appointed as an auditor of a company which raised finance from
the capital market on the basis of a prospectus issued a few years back. The main object for raising the finance
FAST
was specified to be setting up a project on information technology. The company advanced the sum so raised to
various firms and private companies in which the directors of the company were a partner or a director

CA.Sarthak Niraj Jain


respectively. These parties had no standing whatsoever with information technology. In the Balance Sheet, these
advances appeared as a current asset under the head “Short-term Loans and Advances - unsecured, considered
good”. There was no mention to the notes to accounts about nature and purpose of such advances; and the
auditor has issued routine audit report without any qualifications. On the very next day to the issuance of audit
report, the directors and their related parties gone disappeared. The company, in which the auditor was
conducting audit, has just vanished. You are required to state whether the auditor will be held guilty for
professional misconduct? Is there any liability subsists under any law?
Ans. (i) Provisions
 Clause 7 of Part I of Second Schedule to the CA Act, 1949 provides that a CA in practice will be
deemed to be guilty of professional misconduct if he does not exercise due diligence in
performance of his duties or is grossly negligent while performing his duties.
 Schedule III to the Companies Act, 2013 requires specific disclosure of loans and advances due
by directors or other officers of the company or any of them either severally or jointly with any
other person or amounts due by firms or private companies respectively in which any director is a
partner or a director or a member.
 As per Section 188 of the Companies Act, 2013 no company shall enter into any contract or
arrangement with a related party with respect to sale, purchase or supply of any goods or materials,
except with the consent of the Board of Directors given by a resolution at a Board Meeting.
 Section 184 requires disclosure of interest by director and also lays down the procedure to be
followed in this regard.
 Section 189 of the Companies Act, 2013 requires that every company shall keep one or more
registers in which particulars of all contracts or arrangements, to which Section 184 or Section 188
applies, shall be entered separately.

CA Final Audit - By CA. Sarthak Niraj Jain 221


Liabilities of Auditors
(ii) In the present case
 The company has advanced the sum to the parties that are related to the Directors of the company
and showed the same under the head “Short-term Loans and Advances – unsecured, considered
good” rather than specific disclosure under the notes to accounts.
 The auditor of the company also issued clean audit report without any qualifications. It appears
that the auditor did not perform his duties properly.
(iii) Conclusion: -Auditor is guilty of professional misconduct under Clause 7 of Part I of Second
Schedule to CA Act, 1949 and is also liable to be punished u/s 147 of Companies Act 2013, due to non-
observance of compliance of Schedule III and Section 184, 188 and 189 of Companies Act 2013.
2. Precise Nature of Auditor’s Liability
Certain weaknesses in the internal control procedure in the payment of wages in a large construction company
were noticed by the statutory auditor who in turn brought the same to the knowledge of the Managing Director
of the company. In the subsequent year huge defalcation came to the notice of the management. The origin of the
same was traced to the earlier year. The management wants to sue the auditor for negligence and also plans to
file a complaint with the Institute? (Study Material)(RTP-May-2019)
Ans. Liabilities of auditor:
(i) Provision: -SA 265 on “Communicating Deficiencies in Internal Control to TCWG and Management”
requires the auditor to determine whether, on the basis of the audit work performed, he has identified
one or more deficiencies in internal control. If one or more deficiencies in internal control has been
identified, he shall determine whether, such deficiencies individually or in combination constitute
significant deficiencies. Significant deficiencies are required to be communicated in writing to TCWG
and management on a timely basis.

FAST
(ii) Facts of the case: - Certain weaknesses in the internal control procedure in the payment of wages in a
large construction company were noticed by the statutory auditor and brought the same to the
knowledge of the Managing Director of the company. In the subsequent year, a huge defalcation took

CA.Sarthak Niraj Jain


place, the ramification of which stretched to the earlier year. The management of the company desires to
sue the statutory auditor for negligence.
(iii) Analysis of the case:- The precise nature of auditor's liability in the case can be ascertained on the basis
of the following considerations:
(a) Whether the defalcation emanated from the weaknesses noticed by the statutory auditor, the
information regarding which was passed on to the management; and
(b) Whether the statutory auditor properly and adequately extended the audit programme of the
previous year having regard to the weaknesses noticed.
• If circumstances indicate the possible existence of fraud or error, the auditor should
consider the potential effect of the suspected fraud or error on the financial information. If
the auditor believes the suspected fraud or error could have a material effect on the
financial information, he should perform such modified or additional procedures as he
determines to be appropriate. Thus, normally speaking, as long as the auditor took due
care in performing the audit work, he cannot be held liable.
(iv) Conclusion: -The fact that the matter was brought to the notice of the managing director may be a good
defence for the auditor as well. In Kingston Cotton Mills Ltd., it was held that it is the duty of the auditor
to probe into the depth only when his suspicion is aroused. The statutory auditor, by bringing the
weakness to the notice of the managing director had alerted the management which is judicially held
to be primarily responsible for protection of the assets of the company and can put forth this as defence
against any claim arising subsequent to passing of the information to the management.
3. Auditor’s Liabilities in case of Short Provisions
Indicate the precise nature of auditor’s duties in the following situation; Based upon the legal opinion of a
leading advocate, X Ltd. made a provision of ` 5 crores towards Income tax liability. The assessing authority has
worked out the liability at ` 15 crores. It is observed that the opinion of the advocate was inconsistent with legal
position with regard to certain revenue items. (Study Material) (RTP-May-2019)

222 Ab Audit Hoga Sabse Scoring - By CA. Sarthak Niraj Jain


Liabilities of Auditors

Ans. (i) Provision: -SA 500 on "Audit Evidence” requires that auditor to perform appropriate procedures
while using the work of management expert as audit evidence. During the audit, the auditor may seek to
obtain, in conjunction with the client or independently, audit evidence in the form of reports,
opinions, valuations and statements of an expert, e.g., legal opinions concerning interpretations of
agreements, statutes, regulations, notifications, circulars, etc. Before relying on expert’s opinion, the
auditor should have seen that opinion given by the expert is prima facie dependable.
(ii) Facts of the case: - The question states very clearly that the opinion of the management expert was
inconsistent with legal position with regard to certain items. It is, perhaps, quite possible that auditor
did not seek reasonable assurance as to the appropriateness of the source data, assumptions and
methods used by the expert properly.
(iii) Analysis of the case:
 SA 500 requires that auditor to resolve the inconsistency by discussion with the management
and the expert.
 In case, the expert's' work does not support the related representation in the financial information,
the inconsistency in legal opinions could have been detected by the auditor if he had gone through
the same.
This seems apparent having regard to wide difference in the liability worked out by the assessing
authority.
(iv) Conclusion: -Auditor should reject the opinion and insisted upon making proper provision.
S14.2 - Liabilities under Companies Act, 2013– Civil & Criminal
4. Civil Liability for Mis-statement in Prospectus
As a part of the listing process, M/s FRAD Limited had prepared and issued its prospectus to the public. The top

FAST
executives thought that a pending litigation against the company (which would cause a cash outflow of `1 crore)
may affect the demand for share application. Due to this, they had omitted the fact, for the well-being of the
company. Mr. K, who was well aware of this matter, had authorized himself to be named in the prospectus as a

CA.Sarthak Niraj Jain


director. However, Mr. K was little reluctant, so he informed and agreed that he shall become such director after
an interval of some time. Unfortunately, after few days, this matter got leaked and several subscribers sustained
huge loss. Mr. K is now defending himself stating that he is currently not holding the director post hence no
action can be taken against him. Analyse. (MTP-Dec-2021)
Ans. Damages for negligence:
(i) Provision: -Section 35 of Companies Act 2013 provides that where:
 a person has subscribed for securities of a company acting on any statement included, or the
inclusion or omission of any matter, in the prospectus which is misleading and has sustained
any loss or damage as a consequence thereof,
 the company and every person who has authorised himself to be named and is named in the
prospectus as a director of the company, or has agreed to become such director, either immediately
or after an interval of time;
 shall, without prejudice to any punishment to which any person may be liable under section 36, be
liable to pay compensation to every person who has sustained such loss or damage.
(ii) Punishment for Fraud - Section 447 of Companies Act 2013
(a) Without prejudice to any liability including repayment of any debt under this Act or any other
law for the time being in force, any person who is found to be guilty of fraud
(a) Involving an amount of at least ` 10 lakh or
(b) 1% of the turnover of the company, whichever is lower
Punishment:
(i) Shall be punishable with imprisonment for a term which shall not be less than 6 months but
which may extend to 10 years and

CA Final Audit - By CA. Sarthak Niraj Jain 223


Liabilities of Auditors
(ii) Shall also be liable to fine which shall not be less than the amount involved in the fraud, but
which may extend to 3 times the amount involved in the fraud.
(iii) Involve public interest where the fraud in question involves public interest, the term of
imprisonment shall not be less than 3 years.
(iii) Conclusion:
 Hence, in this case, Mr. K is liable for punishment even though he is currently not a director in the
company as per section 35 of the Companies Act, 2013.
 He shall be liable to punishment as per section 447 discussed above as he was aware of the
litigation against the company which may cause outflow of ` 1 crore which may affect the demand
for share application and had also authorized himself to be named in the prospectus as director.
5. Criminal Liabilities under the Companies Act, 2013
Mr. Fresh, a newly qualified Chartered Accountant started his practice wants to specialize in Audits of corporate
and required your advice on criminal liabilities of an auditor under the Companies Act, 2013. Kindly guide him.
(Study Material)(May-2016)
OR
Mr. X, a young chartered accountant, wants to start practice and he required your advice, among other things, on
criminal liabilities of an auditor under the Companies Act, 2013. Kindly guide him.
Ans. (i) Provision:
 Criminal Liability for misstatement in prospectus
Section 34 of Companies Act, 2013 provides that where any prospectus is issued or circulated or
distributed, which includes any statement which is untrue or misleading in form or context in
which it is included or where any inclusion or omission of any matter is likely to mislead, then

 FAST
every person who authorises the issue of such prospectus shall be liable u/s 447 (fraud).
Punishment for false statement
Section 448 of Companies Act 2013 provides that if in any return, report, certificate, financial

CA.Sarthak Niraj Jain


statement, prospectus, statement or other document required by, or for, the purposes of any of
the provisions of this Act or the rules made thereunder, any person makes a statement -
(a) which is false in any material particulars, knowing it to be false; or
(b) which omits any material fact, knowing it to be material, he shall be liable under section
447
(ii) Punishment for Fraud - Section 447 of Companies Act 2013
(a) Provides that, any person who is found to be guilty of fraud,
(a) Involving an amount of at least ` 10 lakh or
(b) 1% of the turnover of the company, whichever is lower
Punishment:
 Shall be punishable with imprisonment for a term which shall not be less than 6 months but
which may extend to 10 years and
 Shall also be liable to fine which shall not be less than the amount involved in the fraud, but
which may extend to 3 times the amount involved in the fraud.
(b) Where the fraud
(a) Involves an amount less than ` 10 lakh or
(b) 1% of the turnover of the company, whichever is lower, and
Punishment:
 Does not involve public interest, any person guilty of such fraud shall be punishable with
imprisonment for a term which may extend to 5 years or with fine which may extend to `
50 lakh or with both.
 Involve public interest where the fraud in question involves public interest, the term of
imprisonment shall not be less than 3 years.

224 Ab Audit Hoga Sabse Scoring - By CA. Sarthak Niraj Jain


Liabilities of Auditors

6. Power of Tribunal - Auditor acted in a Fraudulent Manner


Direction by Tribunal in case auditor acted in a fraudulent manner. (RTP-Dec-2021) (RTP-May-2021)
OR
On the advice of Management of Quick Ltd., the auditor of the Company overlooked and did not report on
shifting of certain current year's sales transactions to the next year. The National Company Law Tribunal (NCLT)
wants to take action against the auditor. Describe the powers of the NCLT under Section 140(5) of the
Companies Act, 2013 for such action and consequences for the auditor. (Study Material)(Nov-2019-old)
OR
The Auditor of M/s Quick Limited succumbed to the pressure of the management in certifying the financials
with an over stated figure of turnover by not adhering to the cut-off principles of the time scale for the
transactions of the year. On taking cognizance of this act of the auditor, the Tribunal under the Companies Act,
2013 initiated the proceedings against him. Briefly list the powers of the Tribunal in this respect including those
relating to making orders against the Auditor found to be guilty) (Study Material)(May-2018 New)
OR
Anvisha Ltd. is a company engaged in the business of software development. It is one of the largest companies in
this sector with a turnover of ` 25,000 crores. The operations of the company are increasing constantly,
however, the focus of the management is more on cost cutting in the coming years to improve its profitability. In
respect of the financial statements of the company which are used by various stakeholders, some fraud was
observed in respect of assets reported therein due to which those stakeholders suffered damages. As a result,
those stakeholders applied to Tribunal for change of auditor on the basis that auditor is colluded in the fraud.
Elucidate the power of tribunal to change the auditor of a company if found acted in a fraudulent manner as
provided under sub-section (5) of section 140 of the Companies Act, 2013. (RTP-Nov-2019)
Ans. (i) Provision:- Section 140(5) of the Companies Act, 2013, the Tribunal either suo motu or on an
FAST
application made to it by the Central Government or by any person concerned, if it is satisfied that
the auditor of a company has, whether directly or indirectly, acted in a fraudulent manner or abetted

CA.Sarthak Niraj Jain


or colluded in any fraud by, or in relation to, the company or its directors or officers, it may, by order,
direct the company to change its auditors.
(ii) When application made by CG:- if the application is made by the Central Government and the Tribunal
is satisfied that any change of the auditor is required, it shall within fifteen days of receipt of such
application, make an order that he shall not function as an auditor and the Central Government may
appoint another auditor in his place.
(iii) When final order passed: - It may be noted that an auditor, whether individual or firm, against whom
final order has been passed by the Tribunal under this section shall not be eligible to be appointed as an
auditor of any company for a period of five years from the date of passing of the order and the auditor
shall also be liable for action under section 447 of the said Act.
(iv) In case of Firm :- It is hereby clarified that in the case of a firm, the liability shall be of the firm and that
of every partner or partners who acted in a fraudulent manner or abetted or colluded in any fraud by,
or in relation to, the company or its director or officers.
SJ’s Comment:-Though question is specific to Auditor acted in a Fraudulent Manner, the suggested answer is
generic in nature.
7. Punishment for False Statement
State the nature of liability as provided in the Companies Act, 2013 of an auditor for not appropriately dealing
with a misstatement appearing in audited financial statements or a false statement in audit report.
(Study Material)(Nov-2018-Old)
Ans. Auditor’s Liability for not appropriately dealing with a misstatement appearing in audited financial
statements or a false statement in Audit Report:
(i) Provision:- Punishment for false statement:
Section 448 of Companies Act 2013 provides that if in any return, report, certificate, financial
statement, prospectus, statement or other document required by, or for, the purposes of any of the
provisions of this Act or the rules made thereunder, any person makes a statement,

CA Final Audit - By CA. Sarthak Niraj Jain 225


Liabilities of Auditors

 which is false in any material particulars, knowing it to be false; or


 which omits any material fact, knowing it to be material, he shall be liable under section 447.
(ii) Punishment for Fraud - Section 447 of Companies Act 2013
(a) Provides that, any person who is found to be guilty of fraud,
(a) Involving an amount of at least ` 10 lakh or
(b) 1% of the turnover of the company, whichever is lower
Punishment:
(i) Shall be punishable with imprisonment for a term which shall not be less than 6 months but
which may extend to 10 years and
(ii) Shall also be liable to fine which shall not be less than the amount involved in the fraud, but
which may extend to 3 times the amount involved in the fraud.
(b) Where the fraud
(a) Involves an amount less than ` 10 lakh or
(b) 1% of the turnover of the company, whichever is lower, and
Punishment:
(i) Does not involve public interest, any person guilty of such fraud shall be punishable with
imprisonment for a term which may extend to 5 years or with fine which may extend to ` 50
lakh or with both.
(ii) Involve public interest where the fraud in question involves public interest, the term of
imprisonment shall not be less than 3 years.
(iii) Conclusion: - Thus, in view of above, an auditor will be held liable for criminal prosecution for not
appropriately dealing with a misstatement appearing in audited financial statements or a false

8.
FAST
statement in Audit Report assuming that it was known to auditor.
(14.3 - Liabilities under Income tax Act, 1961
Liability for Submission of False Information - Section 278

CA.Sarthak Niraj Jain


CA Prince, a Chartered Accountant has appeared before the Income tax Authorities as the authorized
representative of his client and delivers to the Income tax Authorities a false declaration. What are the liabilities
of CA. Prince under Income tax Act, 1961? (MTP-Nov-2018)(May-2017)(Study Material)
Ans. False Declaration as Authorized Representative
(i) Provision:- Section 278 of Income tax Act, 1961- any person who acts or induces, in any manner
another person to make and deliver to the Income Tax Authorities a false account, statement, or
declaration relating to any income chargeable to tax which he knows to be false or does not believe to
be true is punishable
In connection with proceedings under the Income Tax Act 1961, a Chartered Accountant often acts as the
authorised representative of his clients and attends before an Income Tax Authority or the appellate
tribunal.
(ii) Punishment:
 Amount more than 25 Lacs:- In a case where the amount of tax, penalty or interest which would
have been evaded, if the declaration, account or statement had been accepted as true, or which is
wilfully attempted to be evaded, exceeds ` 25 Lacs, with rigorous imprisonment for a term which
shall not be less than 6 months but which may extend to 7 years and with fine;
 In any other case:- With rigorous imprisonment for a term which shall not be less than 3 months
but which may extend to 2 years and with fine

(iii) Conclusion: - In the instant case, Mr. Prince, a chartered accountant has appeared before the Income
Tax Authorities as the authorized representative of his client and delivered a false declaration, thus,
he would be liable under section 278 of the Income Tax Act, 1961.

226 Ab Audit Hoga Sabse Scoring - By CA. Sarthak Niraj Jain


Liabilities of Auditors

9. Liabilities of Auditor – Income tax Frauds


In assessment procedure of M/s Cloud Ltd., Income tax Officer observed some irregularities. Therefore he
started investigation of Books of Account audited and signed by Mr. Old, a practicing Chartered Accountant.
While going through books he found that M/s Cloud Ltd. used to maintain two sets of Books of Account one is
the official set and other is covering all the transactions. Income tax Department filed a complaint with the
Institute of Chartered Accountants of India saying Mr. Old had negligently performed his duties. Comment.
(Study Material)
Ans. (i) Provision: -It is the auditor’s responsibility to audit the statement of accounts and prepare tax
returns on the basis of books of accounts produced before him. After being satisfied with the books
and documents produced to him, he can give his opinion on the basis of those documents only by
exercising requisite skill and care.
(ii) Facts of the case: -Income tax Officer observed some irregularities during the assessment proceeding
of M/s Cloud Ltd. Therefore, he started investigation of books of accounts audited and signed by Mr. Old,
a practicing Chartered Accountant. While going through the books, he found that M/s Cloud Ltd. Used to
maintain two sets of Books of Accounts, one is the official set and other is covering all the transactions.
Income Tax Department filed a complaint with the ICAI saying Mr. Old had negligently performed his
duties.
Mr. Old, the auditor was not under a duty to prepare books of accounts of assessee and he should, of
course, neither suggest nor assist in the preparations of false accounts. He is responsible for the
books produced before him for audit. He completed his audit work with official set of books only.
(iii) Conclusion: -As Mr. Old, performed the auditing with due skill and diligence; and, therefore, no
question of negligence arises. It is the duty of the Department to himself investigate the truth and
correctness of the accounts of the assessee.

Notes
FAST
CA.Sarthak Niraj Jain

CA Final Audit - By CA. Sarthak Niraj Jain 227


Internal Audit, Management and Operational Audit

Chapter INTERNAL AUDIT, MANAGEMENT AND


15 OPERATIONAL AUDIT

Past Exams Weightage


5

0
MAY-19 NOV-19 NOV-20 JAN-21 JUL-21 DEC-21

May-19 Nov-19 Nov-20 Jan-21 Jul-21 Dec-21


Marks 0 0 5 0 5 0

UNIT -1
INTERNAL AUDIT

1. FAST 15.1 - Internal Audit Basics


Appointment of internal Auditors for the Group Companies
One of the independent directors sought information regarding the appointment of internal auditors for

CA.Sarthak Niraj Jain


following Group Companies in accordance with the Companies Act, 2013 of which certain Financial Information
are given below:
Figures are in crore and correspond to the previous year
Name Nature Equity Share Turnover Loan from Public
Capital Bank and PFI Deposits
AADI Ltd. Listed 100 190 50 24
AJIT Ltd. Unlisted Public 60 190 50 24
NEMI Ltd. Unlisted Private 60 190 50 -
You are required to evaluate the requirements of the Companies Act, 2013 regarding the appointment of
internal Auditors for the Group Companies. Discuss. (RTP-May-2022)
Ans. Applicability of Provisions of Internal Audit:
(i) Provision: -As per section 138 of the Companies Act, 2013, following class of companies (prescribed in
Rule 13 of Companies (Accounts) Rules, 2014) shall be required to appoint an internal auditor or a firm
of internal auditors, namely:-
(A) every listed company;
(B) every unlisted public company having-
(1) paid up share capital of fifty crore rupees or more during the preceding financial year; or
(2) turnover of two hundred crore rupees or more during the preceding financial year; or
(3) outstanding loans or borrowings from banks or public financial institutions exceeding one
hundred crore rupees or more at any point of time during the preceding financial year; or
(4) outstanding deposits of twenty five crore rupees or more at any point of time during the
preceding financial year; and

228 Ab Audit Hoga Sabse Scoring - By CA. Sarthak Niraj Jain


Internal Audit, Management and Operational Audit
(C) every private company having-
(1) turnover of two hundred crore rupees or more during the preceding financial year; or
(2) outstanding loans or borrowings from banks or public financial institutions exceeding one
hundred crore rupees or more at any point of time during the preceding financial year.
(ii) Analysis of the cases:
 AADI Ltd. is a listed company. As per section 138 of the Companies Act, 2013, every listed company
is required to appoint an internal auditor or a firm of internal auditors. Thus, in view of the above,
AADI Ltd. is required to appoint an internal auditor.
 AJIT Ltd. is unlisted public company. The company is having 60 crore as equity share capital
which is exceeding the prescribed limit of rupees fifty crore as per section 138. Thus, AJIT Ltd. is
required to appoint an internal auditor as per section 138 of the Companies Act, 2013.
 NEMI Ltd. is unlisted private company and having 60 crore as equity share capital, 190 crore
as turnover and 50 crore loan from Bank and PFI. In view of provisions of section 138 of the
Companies Act, 2013 discussed above, all the limits are below the prescribed limit for a private
company. Therefore, NEMI Ltd. is not required to appoint an internal auditor.
(iii) Conclusion: - It can be concluded that AADI Ltd. and AJIT Ltd. is required to appoint the internal
auditor as per the provisions of the Companies Act, 2013 whereas NEMI Ltd. is not required to do the
same.
15.2 - Scope of internal Audit
2. Appraisal of Organisational Structure
Internal auditor makes an appraisal of organization structure to ensure that it is in harmony with the objectives
of the entity, besides checking of financial transactions and operational activities of the entity. Elaborate.

FAST OR
(Nov-2018-New)

ABC Ltd. is engaged in manufacturing of Yarns and Towels. It sells its product in both domestic as well as in

CA.Sarthak Niraj Jain


International Market. It has achieved turnover of 200 crores in the F.Y. 2016-17. Directors of the company
realized that they are not managing the company professionally and thereby request your firm of Internal
Auditors for appraisal of its organizational structure to ascertain whether it is in harmony with the objectives of
ABC (P) Ltd. Comment. (MTP-Nov-2018)
Ans. The internal auditor should conduct an appraisal of the organisation structure to ascertain:
(i) Whether it is in harmony with the objectives of the enterprise and
(ii) Whether the assignment of responsibilities is in consonance therewith.
For this purpose, internal auditor should:
(a) Review the manner in which the activities of the enterprise are grouped for managerial control so
as to find out whether responsibility and authority are in harmony with the grouping pattern.
(b) Examine the organisation chart to find out whether the structure is simple and economical and that
no function enjoys an undue dominance over the others.
(c) Ensure that the responsibilities of managerial staff at headquarters do not overlap with those of chief
executives at operating units
(d) Examine the reasonableness of the span of control of each executive (the number of sub-ordinates
that an executive controls). He should examine whether there is a unity of command i.e., whether each
person reports only to one superior.
(e) Evaluate the process of managerial development in the enterprise.
15.3 - Relationship between Internal Auditor, Internal Audit Function (IAF) & External Auditor
3. Direct Assistance from IA & External Confirmation Procedure
Mr. A is appointed as a statutory auditor of XYZ Ltd. XYZ Ltd is required to appoint an internal auditor as per
statutory provisions given in the Companies Act, 2013 and appointed Mr. B as its internal auditor. The external
auditor Mr. A asked internal auditor to provide direct assistance to him regarding evaluating significant
accounting estimates by the management and assessing the risk of material misstatements.

CA Final Audit - By CA. Sarthak Niraj Jain 229


Internal Audit, Management and Operational Audit
(a) Discuss whether Mr. A, statutory auditor, can ask direct assistance from Mr. B, internal auditor as stated
above in view of auditing standards.
(b) Will your answer be different if Mr. A asks direct assistance from Mr. B, internal auditor with respect to
external confirmation requests and evaluation of the results of external confirmation procedures?
(Study Material)
Ans. (a) Direct Assistance from Internal Auditor
As per SA 610 “Using the Work of Internal Auditor: -
 The external auditor shall not use internal auditors to provide direct assistance to perform
procedures that Involve making significant judgments in the audit.
 Since the external auditor has sole responsibility for the audit opinion expressed, the external
auditor needs to make the significant judgments in the audit engagement.
 Significant judgments include the following:
 Assessing the risks of material misstatement;
 Evaluating the sufficiency of tests performed;
 Evaluating the appropriateness of management’s use of the going concern assumption;
 Evaluating significant accounting estimates; and
 Evaluating the adequacy of disclosures in the financial statements, and other matters
affecting the auditor’s report. In view of above, Mr. A cannot ask direct assistance from internal
auditors regarding evaluating significant accounting estimates and assessing the risk of material
misstatements.
(b) Direct Assistance from Internal Auditor in case of External Confirmation Procedures:
 SA 610 “Using the Work of Internal Auditor:
 Provide relevant guidance in determining the nature and extent of work that may be assigned
to internal auditors.

FAST
 In determining the nature of work that may be assigned to internal auditors, the external
auditor is careful to limit such work to those areas that would be appropriate to be assigned.
 Further as per SA 505, “External Confirmation:

CA.Sarthak Niraj Jain


 The external auditor is required to maintain control over external confirmation requests
and
 Evaluate the results of external confirmation procedures, it would not be appropriate to
assign these responsibilities to internal auditors.
 However, internal auditors may assist in assembling information necessary for the external
auditor to resolve exceptions in confirmation responses.

UNIT -2
MANAGEMENT AUDIT
15.4 - Management Audit – Programme/Procedure
4. Company Facing Financial Crunch
You have been appointed Management Auditor of Zoya Ltd. a large manufacturing company suffering from
working capital crunch. Enlist and discuss the related areas which you would probe into to overcome the
company’s problem
Ans. Action plan for Management Audit of a company facing financial crunch
(i) Working capital estimation: Prepare a statement of the projected working capital requirements. It
should be based on the functional budgets in sales, production, expenses, capital expenditure and the
master budget consisting of projected profit and loss and the balance sheet.
(ii) Cash flow statement / Cash Budget: Month-wise cash budgets showing inflows and outflows of cash
heading-wise should be prepared to analyse the major inflows and outflows affecting the entity.
(iii) Inventory/Stock management: Raw materials and inventories should be classified properly to
determine the level of stock of materials. The system of inventory management needs to be looked at so
as to check the avoidable wastes/scraps generated during storage and handling.

230 Ab Audit Hoga Sabse Scoring - By CA. Sarthak Niraj Jain


Internal Audit, Management and Operational Audit
(iv) Credit management: The company should lay down a proper policy for evaluating customers,
determining the credit period and offering discounts for early payment. An age-wise analysis of debtors
should also be prepared so as to avoid credit to defaulters.
(v) Funds flow analysis: The company should prepare a funds flow analysis, distinguishing between long-
term and short-term sources and applications.
(vi) Investment management: The idle funds of the company, if any, should be invested in short-term
securities to augment the income
(vii) WIP analysis: WIP should be monitored and for the purpose it is necessary to ensure that no
bottlenecks develop at any stage during the production process.
15.5 - Behavioural Aspects – Problem & Solution
5. Management Audit - Behavioral Aspects
The management auditor has to analyse the nature and causes of behavioural problems in the discharge of the
review function and to arrive at possible solutions to overcome these problems. As a management auditor of
Real Limited, how will you demonstrate in arriving at a solution to behavioural problems and create an
atmosphere of trust and friendliness, so that audit reports will be understood in their proper perspective?
(Nov-2020-New)
OR
Explain in brief the behavioural aspects encountered in the management audit and state the ways to solve them.
Ans. (i) Staff / Line conflict:
Management auditors are staff people while the members of other departments are line people.
Management auditors has superiority complex and they may think their approach and solutions are the
only answers. Staff does not consider the line before advising; as a result Line may not use advice of

FAST
staff properly.
(ii) Control: The management auditor is expected to evaluate the effectiveness of controls; there is an
instinctive reaction from the auditee that the report of the auditor may affect them. Causes of

CA.Sarthak Niraj Jain


antagonism are:- Fear of criticism /changes, - Punitive action, - Insensitive audit practices
(iii) Resistance to change : There is a certain built in resistance to change the auditor is looked upon as a
likely instrument for recommending changes & auditee’s do not welcome the visits of auditors.
Solution to behavioral problems
Auditee may improve if auditor’s acts as a professional advisor & consultant
(i) Demonstrate that audit is part of an overall programme of review for protective and constructive
benefit.
(ii) Demonstrate the objective of review is to provide maximum service in all feasible managerial
dimensions.
(iii) Perform the review to ensure minimum interference with regular operation.
(iv) Involve responsible officers in the process of review of the findings and recommendations before the
audit report is formally released.
(v) Create an atmosphere of trust and friendliness.
15.6 - Management Audit – Reports
6. Report
Write a short note on-Summary written report. (RTP-Dec-2021)(Study Material)
Ans. (i) Management by exception
Summary Reports are also known as Flash Reports. These reports summarize various individual reports
on integrated approach and facilitates management by exception.
(ii) For immediate attention
They are significant highlights for immediate attention of top management. Generally suspected
defalcations are reported briefly to the appropriate management official on the 'flash' basis, often ending
up in referral for criminal investigation and legal action.

CA Final Audit - By CA. Sarthak Niraj Jain 231


Internal Audit, Management and Operational Audit
(iii) Frequently reporting
It is common practice in number of companies of issuing a report quite frequently summarising the
various individual reports issued and describing the range of their contents in a very brief and
comprehensive manner where only important points are highlighted.
(ii) For immediate action
Such reports are primarily issued for audit committees and other top-level managers who do not
have sufficient time to go through the elaborate reports and matters which are required to be brought
to their notice for immediate action.
7. Reports
Write a short note on – Management audit reports.
Ans. Management Audit Reports are of following types:
(i) Oral reports: Oral reports are required in case of emergency. But they not considered reliable as no
permanent record. Oral Reports helps in taking corrective steps timely.
(ii) Interim written report: These reports are issued in case of matters requiring early consideration so
as to inform management before regular report.
(iii) Regular written reports: These are the formal report submitted after completion of work.
(iv) Summary written report: Summary reports are also known as “Flash’ reports. These reports
summarize various individual reports on integrated approach and facilitate management by exception.

UNIT -3
OPERATIONAL AUDIT
15.7 - Operational Audit – Meaning, Scope & Objective
8. Need of Operational Audit

FAST
Perfect Steel Ltd. has reported a higher turnover of ` 560 crores in the year 2021-22 as compared to earlier
years but its sales return has also increased to 10% from only 4% up to the last year. The management is
concerned about the high sales returns and feels a need to get the operational audit done for sales and
CA.Sarthak Niraj Jain
production department of the company. The company is also having an internal audit system in the company.
Elaborate the possible reasons, why management is getting operational audit done when internal audit has
already been done for both the departments by stating the shortcomings of conventional information sources.
(Study Material)(RTP-Nov-2019)
Ans. The need for operational auditing has arisen due to the inadequacy of traditional sources of information for
effective management of the company where the management is at a distance from actual operations due to
layers of the delegation of responsibility, separating it from actualities in the organisation.
An operational audit is considered as a specialized management information tool to fill the void that
conventional information sources fail to fill. Conventional sources of management information are
departmental managers, routine performance reports, internal audit reports, and periodic special
investigation and survey.
These conventional sources fail to provide information for the best direction of the departments all of
whose activities do not come under direct observation of managers. The shortcomings of these sources can be
stated as under:
(i) Executives and managers are too preoccupied with the implementation of plans and achieving of
targets and may be out of touch with the bigger picture including the interconnectedness of their tasks
with the overall objectives of the organisation. Further, they may be left with very little time to collect
information and locate problems. They may even come across problems that have come to surface but
hardly aware of the problems that are brewing and potential while actually performing their day-to-day
activities.
(ii) Managers or their aides are generally relied upon for transmitting information rather than for booking
for information or for analyzing the situations.
(iii) The information that is transmitted by managers is not necessarily objective - often it may be biased

232 Ab Audit Hoga Sabse Scoring - By CA. Sarthak Niraj Jain


Internal Audit, Management and Operational Audit
for various reasons.
(iv) Conventional internal audit reports are often routine and mechanical in character and have a definite
leaning towards accounting and financial information. They are also historical in nature.
(v) Other performance reports contained in the annual audited accounts and the routine reports
prepared by the operating departments have their own limitations. The annual audited accounts are
good as far as an overall true and fair evaluation of the financial statements are concerned.
(vi) Operations of controls in a satisfactory manner cannot be relied upon to bring to light the
environmental conditions. Controls are specific and their satisfactory operation is related to the specific
situation under control. Also, monitoring the breakdown or non-operation of controls is a periodic
activity.
(vii) Surveys and special investigations, no doubt, are very useful but these are occasional in character.
Also, they are costly, time consuming and keep the departmental key personnel busy during the period the
operations are on. These are basically an attempt to carry out a post-mortem rather than continuous
improvement efforts or a system to give signals for potential risks (dangers and disasters to come)
9. Scope of Operational Audit
Mr. ‘P’ have been appointed as operational auditor of M/s Books & Magazine Ltd. and observed a totalling error
in invoice of ` 1,000. He has not taken care of the same saying that this is out of scope of his work. Comment.
Ans. (i) Operational Audit is the review and appraisal of operations of an organization conducted by
competent independent person.
(ii) It analyse the regular operations like production, purchase, sales etc. to check whether they are in
tune with company’s policies, objectives and goals or not. It focuses more on Qualitative aspects of
operations rather than regular accounting aspects.
(iii)
FAST
In carrying out his work, operational auditor will have to exercise judgement to evaluate evidence in
connection with the situations and issues; he will have to get the assistance of norms and standards
in every operating field to be able to objectively judge a situation.
(iv)
CA.Sarthak Niraj Jain
In the given situation: -to a financial auditor, a loss of ` 1,000 caused by a wrong totaling of invoice is
important, but for an auditor engaged in the review of operations, it does not assume so much
importance. But simultaneously, it should not be assumed, that, an operational auditor is not
concerned with the financial aspects of transaction and controls.
(v) Conclusion:
 Contention of operational auditor that totaling error in invoice of ` 1,000 is out of scope is not
correct as operational audit is being carried out to ensure that all the management functions like
planning, organizing, staffing, directing and controlling are working effectively and efficiently.
 Such kind of error is very much in scope because such an existence of error indicates that control
system (controlling function) is not sound.
15.8 - Comparison Internal/Management/Operational Audit
10. Financial Auditing Vs. Operational Auditing:
The main objective of operational auditing is to verify the fulfillment of plans, and sound business requirements.
While in financial auditing, the concentration is more in the financial and accounting areas to ensure that
possibilities of loss, wastage and fraud are minimized or removed. In the light of above, state the major
differences between financial and operational auditing. (Study Material)
OR
Financial vs. Operational Audit (Nov-2017)
Ans. Basis Financial Auditing Operational Auditing
(1) Purpose It concerned with the opinion that It emphasizes on effectiveness and
whether the historical information efficiency of operations for future
recorded is correct or not performance.
(2) Area It is restricted to the matters directly It covers all the activities that are related

CA Final Audit - By CA. Sarthak Niraj Jain 233


Internal Audit, Management and Operational Audit
affecting the appropriateness of the to efficiency and effectiveness of business
presented financial statements operations.
(3) Reporting Financial audit report is sent to all Operational audit report is primarily for
stock holders, bankers and other the management
persons having interest in the
organization.
(4) End task Financial audit reports the findings to Operational audit is not limited to the
the persons as its end objectives. reporting only, but includes suggestions for
improvements also.
11. Management Audit & Operational Audit
M/s ABC & Co., Chartered Accountants have been approached by PQR Ltd., a company engaged in iron and steel
manufacturing industry. The company has been facing following operational issues:
(a) Penal interest for delayed payments to the overseas vendors despite having enough cash flows; and
(b) Despite having regular production and enough inventory, delays in shipping the final goods to the
customers leading to its deteriorating vendor rating.
As a partner of M/s ABC & Co., through detailed discussion with the Senior Manager of PQR Ltd., you have
concluded that all these delays are because of long decision-making cycles in the company. As a consultant to the
Company, would you recommend Management Audit or Operational Audit? (Study Material)
OR
M/s Raka & Co., Chartered Accountants have been approached by Abhinandan Ltd., a company engaged in iron
and steel manufacturing industry. The Company has been facing following operational issues:
(a) Penal interest for delayed payments to the overseas vendors despite having enough cash flows; and

FAST
(b) Despite having regular production and enough inventory, delays in shipping the final goods to the
customers leading to its deteriorating vendor rating.
As a partner of M/s Raka & Co., through detailed discussion with the Senior Manager of Abhinandan Ltd., you

Ans. (i)
CA.Sarthak Niraj Jain
have concluded that all these delays are because of long decision making cycles in the Company. As a consultant
to the Company, would you recommend Management Audit or Operational Audit?
A comparison between the Management Audit & the Operational Audit is as follows:
(RTP-May-2021)

 Management audit is concerned with the “Quality of managing”, whereas operational audit focuses
on the “Quality of operations”.
 Management audit is the “Audit of management” while the operational audit is the “Audit for the
management.
 The focus of Management Audit is on “Quality of Decision Making” rather than the effectiveness
or efficiency of operations.
 The basic difference between the two audits, then, is not in method, but in the level of appraisal.
 In a management audit, the auditor is to make his tests to the level of top management, its
formulation of objectives, plans and policies and its decision making.
 It is not that he just verifies the operations of control and procedures and fulfillment of plans in
conformity with the prescribed policies.
(ii) In the given case: - Since, the delays in payments and consequent penal interest payments and the
delays in shipping and the consequent deteriorating vendor ratings are happening because of the delays
in decision-making process of the management.
(iii) Conclusion: -Therefore, it appears that this is not just an internal control or operational issue but an
issue of management process. Therefore, management audit would be recommended in this case.
12. Management Audit & Operational Audit
The PQR Ltd. has come across many instances where it could buy products at lesser cost than the actual
procurement price it paid. The management believes that the adequate purchase policy is in place including the

234 Ab Audit Hoga Sabse Scoring - By CA. Sarthak Niraj Jain


Internal Audit, Management and Operational Audit
requirements of three quotations from registered vendors, appropriate vendor vetting and rating mechanism,
however, the on-ground implementation of the purchase policy might be defective. Further, it has observed that
there might be some employees involved in choosing the higher cost vendors as well. The company approaches
you to advise the type of audit it should get done: Management or Operational. Please advise through a
comparison between both the audits. (MTP-Dec-2021)(Study Material)
Ans. (i) A comparison between the Management Audit & the Operational Audit is as follows:
 Management audit is concerned with the “Quality of managing”, whereas operational audit focuses
on the “Quality of operations”.
 Management audit is the “Audit of management” while the operational audit is the “Audit for the
management.
 The focus of Management Audit is on “Quality of decision making” rather than the effectiveness or
efficiency of operations.
 The basic difference between the two audits, then, is not in method, but in the level of appraisal.
 In a management audit, the auditor is to make his tests to the level of top management, its
formulation of objectives, plans and policies and its decision making.
 It is not that he just verifies the operations of control and procedures and fulfillment of plans in
conformity with the prescribed policies.
(ii) In the given case: - Company has come across many instances where it could buy products at lesser cost
than the actual price. Adequate purchase policy is in place, however, the on-ground implementation
of the purchase policy might be defective and also it has observed that there might be some
employees involved in choosing the higher cost vendors as well.
(iii) Conclusion: -Since it is not the management’s decisions that are creating the operational

FAST
bottlenecks. The purchase policy and procedure seem to be in place, the missing part is the
operational implementation by the process employees. Therefore, the Operational Audit is
recommended in this case.

13. CA.Sarthak Niraj Jain


High Employee Attrition Rate
15.9 - Operational Audit – Problems & Solution

The Managing Director of X Ltd is concerned about high employee attrition rate in his company. As the internal
auditor of the company he requests you to analyse the cause for the same. What factors would you consider in
such analysis? (Study Material)
OR
You have been appointed as an internal auditor of a company RSM Ltd. The Managing Director Rakesh is
worried about employee attrition in large number. Rakesh requests you to analyse the causes for high employee
attrition rate in his company. What factors would you consider in such analysis? (MTP-May-2020)
Ans. The factors responsible for high employee attrition rate are as under:
(i) Job Stress & work life imbalance
(ii) Wrong policies of the Management
(iii) Unbearable behaviour of Senior Staff
(iv) Safety factors
(v) Limited opportunities for promotion
(vi) Low monetary benefits
(vii) Lack of labour welfare schemes
(viii) Whether the organization has properly qualified and experienced personnel for the various levels of
works?
(ix) Is the number of people employed at various work centres excessive or inadequate?
(x) Does the organization provide facilities for staff training so that employees and workers keep
themselves abreast of current techniques and practices?

CA Final Audit - By CA. Sarthak Niraj Jain 235


Internal Audit, Management and Operational Audit

14. Operational Audit - Continuous Improvement Cycle


The Marketing Department of ISHITA Ltd. has been consistently showing a lower performance whereas the cost
of the department is increasing in spurts over the years. The management believes that since the marketing
department is under a regular radar of the CFO, an audit might result in the employee hostility. Also, an
operational audit of Marketing Department was done two years back however, the recommendations of the
previous audit were not followed by the concerned employees. Please advise the management if another audit is
the solution and whether only one-time operational audit is enough? Further, advise on the ways to deal with
the employee hostility. (Study Material)(MTP-May-2021)
Ans. The Operational Audit is not one-time activity. It should be viewed as a continuous improvement cycle:
Plan > Do> Check >Act
(i) All the significant operations must be subjected to the scrutiny of operational audit, at least, once in
three years. Therefore, the operational audit should be done in the current scenario. However, to deal
with the employee hostility the participative approach of the audit should be adopted:
(ii) The participative approach:
 In this approach the auditor discusses the ideas for improvements with those managers that
have to implement them and make them feel that they have participated in the recommendations
made for improvements.
 By soliciting the views of the operating personnel, the operational audit becomes a co-operative
enterprise.
 This participative approach encourages the auditee to develop a friendly attitude towards the
auditors and look forward to their guidance in a more receptive fashion.
 When the participative method is adopted then the resistance to change becomes minimal,

FAST
feelings of hostility disappear and gives room for feelings of mutual trust. Team spirit is
developed.
 The auditors and the auditee together try to achieve the common goal. The proposed
recommendations are discussed with the auditee and modifications as may be agreed upon are

(iii) CA.Sarthak Niraj Jain


incorporated in the operational audit report.
Advice: With this attitude of the auditor, it becomes absolutely easy to implement the proposed
suggestions as the auditee themselves take initiative for implementing and the auditor does not
have to force any change on the auditee.
15.10 - Qualities of Operational Auditor
15. Operational Auditor –Positive Traits/Qualities
The Operational audit is carried out effectively when the operational auditor responds with positive traits in a
scenario which is blended with behavioural issues. Explain few positive traits that help to conclude an
Operational Audit, a success. (Study Material)(May-2018-New)
OR
Employees of GIG Ltd. have to travel frequently for business purposes, so the company entered into a contract
with a Simony Travels Ltd. for managing booking, cancellation and other services required by their employees.
As per contract terms, Simony travels has to raise its monthly bills for the tickets booked or cancelled during the
period and the same are paid by GIG Ltd. within 15 days of the bill date. The bills raised by Simony travels were
of huge amount, so the management of GIG Ltd. decided to get an audit conducted of the process followed for
booking/ cancellation of tickets and verify the accuracy of bills raised by the travel agency. Which audit do you
feel the management should opt for? Also briefly discuss the qualities the auditor should possess for such audit.
(MTP-Dec-2021)(RTP-May-2019)
Ans. (a) In the case mentioned in the question: - management should opt for operational audit. Operational
auditor will ensure verification of effectiveness, efficiency and economy of operations done by the
Simony travels for the organisation.
(b) Positive traits/qualities an operational auditor should possess:
The operational auditor should possess some very essential personal attributes to be effective in his
work. Such personal attributes comprise of:

236 Ab Audit Hoga Sabse Scoring - By CA. Sarthak Niraj Jain


Internal Audit, Management and Operational Audit
 Operational auditor is required to visualise whether simpler alternative means are available to
do a particular work.
 He should ask the who, why, how of everything.
 Operational Auditor should possess an attitude of skepticism.
 He should try to see everything as to whether particular operational aspects properly fit in the
business frame and organisational policy.
 He should imbibe a constructive approach rather than a faultfinding approach and should give a
feeling that his efforts are to help attaining an improved operation and not merely fault finding.
 If the auditor succeeds in giving a feeling of help and assistance through constructive criticism, he
will be able to obtain co-operation of the persons who are involved in the operations.
 He should try to develop a team comprised of people of different backgrounds.
15.11 - Operational Audit – Misc. Case Studies
16. Decreasing Operating Efficiency in Material Consumption
The Board of Directors of XYZ Ltd. is concerned with decreasing operating efficiency in material consumption.
As an Auditor entrusted with investigating the causes for this poor state, what may be the areas of your focus in
this respect (Nov-2018-New)
Ans. Focus area while investigating the causes of decreasing operating efficiency in material consumption:
To determine the reasons for decreasing efficiency in material consumption, the auditor is required to
understand the manufacturing operations from the initial stage of purchase of materials, issue of material,
consumption of material and conversions into finished goods.
For these purposes, auditor may focus on following aspects:
1. Obtain a list of raw materials used in the production process mentioning therein the names and

2.
FAST
detailed characteristics of each raw material.
Ascertain the basis of computation of normal wastage figures and its consistency as compared to
previous months.
3.
CA.Sarthak Niraj Jain
Obtain internal control reports, in respect of manufacturing costs incurred with reference to
predetermined standards or budgets.
4. Examine the stock records so as to determine the raw material purchased, material issued to
production.
5. Examine the production records so as to determine the material received from the stores and actual
material consumed in the production and waste produced in the production process.
6. Study the maintenance programme of machinery to ensure that the machinery does not consume more
raw material and quality of goods manufacture is not of sub-standard nature.
7. Ascertain whether employees engaged in production are properly trained and working efficiently.
8. Determine the existence and effectiveness of quality control techniques employed in the entity.
9. Examine inventory controls in respect of storage, pilferage, issues etc.
10. Obtain a statement showing break up of wastage figures in storage, handling and production for the
period under reference and compare the results of the analysis for each of the month.
11. Consider the existence of following situations that may also cause the abnormal wastage:
(a) Purchase of poor quality of raw materials.
(b) Machinery breakdown or power failure.
(c) High rate of rejections of finished goods
(d) Deterioration of raw material lying in godowns
(e) Abnormal wastages in storage and handling.
(f) Commencing new production line.

CA Final Audit - By CA. Sarthak Niraj Jain 237


Investigation, Due Diligence and Forensic Audit

Chapter INVESTIGATION, DUE DILIGENCE AND


16 FORENSIC AUDIT

Past Exams Weightage


9
8
7
6
5
4
3
2
1
0
MAY-19 NOV-19 NOV-20 JAN-21 JUL-21 DEC-21

May-19 Nov-19 Nov-20 Jan-21 Jul-21 Dec-21


Marks 5 4 4 5 9 5

FAST UNIT -1
INVESTIGATION

1. CA.Sarthak Niraj Jain


Assistance of Expert
16.1- Investigation Basics

XYZ Ltd. has bought a land in Nagpur for setting up a manufacturing unit in the year 2021 at a price of ` 10
crores. In the year 2022, one of the directors of company raised suspicion on the price and transactions
related to purchase of land. Therefore, an investigation was ordered by the management and PV Associates
were appointed to investigate the matter and submit their report accordingly. PV Associates were of the view
that they need to take an expert’s opinion on the price of land. Whether PV Associates is authorized to take
assistance of expert? If yes, what is the process they need to follow? (Study Material)
Ans.  In the case of many enterprises, the valuation is mainly based on the value of net worth only. For
valuation of immovable properties and plant, if required, the assistance of expert valuers could also
to be taken.
 If PV Associates feels the necessity of obtaining views and opinions of experts in various fields to
properly conduct the investigation, they are allowed to do so.
 It would be therefore, proper for the investigator to get the written general consent of his client, to
refer special matters for views of different experts at the beginning of investigation and
 He should settle the question of costs for obtaining the views and other related implications.
16.2- Investigation under Companies Act, 2013
2. Investigation u/s 210 of Companies Act, 2013
Briefly explain the investigation into the affairs of a company as envisaged under section 210 of the
Companies Act, 2013
Ans. Section 210 of Companies Act 2013 provides the following:
(i) Central Government may order an investigation into the affairs of the company if it considers it
in the following circumstances:

238 Ab Audit Hoga Sabse Scoring - By CA. Sarthak Niraj Jain


Investigation, Due Diligence and Forensic Audit

 on the receipt of a report of the Registrar or Inspector;


 on intimation of a special resolution passed by a company that the affairs of the company ought
to be investigated; or
 in public interest,
(ii) Central Government shall order an investigation into the affairs of that company, if an order is
passed by a Court or the Tribunal in any proceedings before it that the affairs of a company ought
to be investigated
(iii) For the purposes of investigation, the Central Government may appoint one or more persons as
inspectors to investigate into the affairs of the company and to report thereon in such manner as
the Central Government may direct.
3. General Approach for Investigation u/s- 210 & 213
What will be your approach in investigation under Section 210 and 213 of Companies Act, 2013 into the
affairs of the company registered under Companies Act, 2013?
Ans. A general approach for investigation under section 210 & 213 can be formulated considering these
steps
(i) Clarity of terms of reference
a. Approach to any investigation is determined on nature of investigation & terms of reference
b. Inspector to ensure terms are clear, unambiguous & in writing obtain clarification in writing
in case of any doubt
c. Inspector should ask for reframing of order specifying the exact matters to be investigated
and take into consideration possible effect of limitations
d. He should keep CG informed in writing about their effect on investigation
(ii)
FAST
Scope of investigation
a. Inspector should determine scope of the investigation on the basis of the terms of reference.
b. At this stage, it may be useful for the inspector to go into the history of the company and its

CA.Sarthak Niraj Jain


affiliates or associates.
c. He must evaluate the terms of reference which will enable him to locate the limitation. For
purposeful investigation, he may need to stretch his inquiry into the books & records of allied &
associated persons & concerns
(iii) Period for investigation
Inspector should also have regard to the period over which the investigation should stretch. The
evaluation of terms of reference and the consequential determination of the scope of
investigation are the twin props on which the entire investigation would rest.
(iv) Framing of programme
a. The next step is the investigator/inspector should frame his programme for investigation in
a systematic manner.
b. He should keep adequate working notes and papers with references and cross references in a
proper and methodical way to aid him in the preparation of the report.
(v) Using the work of experts
He should also consider whether assistance of other experts like engineers, lawyers, etc., is
necessary in the interest of a comprehensive and full proof examination of the documents and
information.
(vi) Legal requirements and investigation report:
a. After completion of steps in the investigation programme and collection of all the information
that he needed, report should be prepared.
b. However, an interim report may be made if required. The findings should be completed and
exhaustive.

CA Final Audit - By CA. Sarthak Niraj Jain 239


Investigation, Due Diligence and Forensic Audit
c. Before he makes his final report, he should obtain and keep on record the evidences relied
upon by him. By the nature of things, such evidence should be as conclusive as possible
depending on circumstances of the case.
d. He should make his report in accordance with the provisions of the Companies Act, 2013.
The general approach for investigations under Sections 210 and 213 should, therefore, be formulated
having regard to the terms of reference, scope, the period, the programme and procedure of the
investigation and the attending legal requirements specified above.
16.3 - Investigation on behalf of Bank
4. Investigation - Bank Proposing to Advance Loan
A nationalized bank received an application from a Limited company seeking sanction of a term loan to
expand its existing business. In this connection, the Loan Manager of the Bank approaches you to conduct a
thorough investigation of the items of the Balance Sheet of this Limited company and submit a confidential
report based on which he will decide whether to sanction this loan or not. List out the major steps an
investigating accountant
would keep in mind while verifying assets and liabilities included in the Balance Sheet of the borrower
company which has been furnished to the Bank. (MTP-Nov-2020)
Ans. Steps involved in the verification of Assets and Liabilities –
The investigating accountant should prepare schedules of assets and liabilities of the borrower and
include in the particulars stated below:
(i) Fixed assets –
a. A full description of each item, its gross value/ rate at which depreciation has been charged
and the total depreciation written off.

FAST
b. In case any asset is encumbered, the amount of the charge and its nature should be disclosed.
c. In case an asset has been revalued recently, the amount by which the value of the asset has
been decreased or increased on revaluation should be stated along with the date of

CA.Sarthak Niraj Jain


(ii)
revaluation.
Inventory –
a. The value of different types of inventories held and the basis on which these have been valued.
b. Details as regards the nature and composition of finished goods should be disclosed.
c. Slow moving or obsolete items should be separately stated along with the amounts of
allowances.
(iii) Trade receivables, including bills receivable –
a. Their composition should be disclosed to indicate the nature of different types of debts that
are outstanding for recovery; also
b. Whether the debts were being collected within the period of credit as well as the fact
c. Whether any debts are considered bad or doubtful and the provision if any, that has been made
against them.
Further, the total amount outstanding at the close of the period should be segregated as
follows:
1. Debts due in respect of which the period of credit has not expired;
2. Debts due within six months; and
3. Debts due but not recovered for over six months.
If any debts are due from directors or other officers or employees of the company, the particulars
thereof should be stated. Amounts due from subsidiary and affiliated concerns, as well as those
considered abnormal should be disclosed. The recoveries out of various debts subsequent to the
date of the Balance sheet should be stated.
(iv) Investments –
a. The schedule of investments should be prepared.

240 Ab Audit Hoga Sabse Scoring - By CA. Sarthak Niraj Jain


Investigation, Due Diligence and Forensic Audit
b. It should disclose the date of purchase, cost and the nominal and market value of each
investment.
c. If any investment is pledged as security for a loan, full particulars of the loan should be given.
(v) Secured loans –
a. Debentures and other loans should be included together in a separate schedule.
b. Against the debentures and each secured loan, the amounts outstanding for payments along
with due dates of payment should be shown.
c. In case any debentures have been issued as a collateral security, the fact should be stated.
(vi) Provision of taxation –
a. The previous year’s up to which taxes have been assessed should be ascertained.
b. If provision for taxes not assessed appears in be inadequate, the fact should be stated along
with the extent of the shortfall.
(vii) Other liabilities –
 It should be stated whether all the liabilities, actual and contingent, are correctly disclosed.
(viii) Insurance –
 A schedule of insurance policies giving details of risks covered, the date of payment of last
premiums and their value should be attached as an annexure.
(ix) Contingent liabilities –
a. By making direct enquiries from the borrower company, from members of its staff, perusal of
the files of parties to whom any loan has been advanced those of machinery suppliers and the
legal adviser,
b. For example: - The investigating accountant should ascertain particulars of any contingent

FASTliabilities which have not been disclosed. In case, there are any, these should be included in a
schedule and attached to the report.
Finally, the investigating accountant should ascertain whether any application for loan to another bank or

5.
CA.Sarthak Niraj Jain
any other party has been made. If so, the result thereof should be examined.

Embezzlement of Cash Receipts


16.4 - Investigation of Frauds

In a company, it is suspected that there has been embezzlement in cash receipts. As an investigator, guide the
company the areas where embezzlement is possible? (Study Material)
OR
In cases like holding back cash sales, collections by travelling salesmen, V.P.P receipts, or casual receipts, e.g.,
sales of scrap, recoveries out of debts written off earlier, etc., the amount or amounts of receipts embezzled
may be subsequently covered up by the perpetrator adopting certain methods. In a company, it is suspected
that there has been embezzlement in cash receipts. The company appoints you as an investigator. What are
the areas you would verify?
Ans. Following areas auditor need to verify:
In cases like holding back cash sales, collections by travelling salesmen, V.P.P receipts, or casual receipts, e.g.,
sales of scrap, recoveries out of debts written off earlier, etc., the amount or amounts of receipts embezzled
may be subsequently covered up by the perpetrator adopting one or other of the under-mentioned
devices:
(i) Issuing a receipt for full amount collected, entering lesser amount on the counterfoil.
(ii) Showing a larger cash discount than actually allowed.
(iii) Adjusting a fictitious credit in the account of a customer for goods returned.
(iv) Cash sales entered as credit sales with debit to customer.
(v) Writing off a good debt as bad & irrecoverable to cover up misappropriation of amount collected.

CA Final Audit - By CA. Sarthak Niraj Jain 241


Investigation, Due Diligence and Forensic Audit
(vi) Short-debiting customer’s ledger account and withdrawing the difference on collection of full
amount.
(vii) Under-casting the receipts side of cash book & over-casting the payment side of the cash book
(viii) Carrying over a shorter total of the receipts from one page of the Cash Book to the next or over-
carrying the total of the payment from one page of the Cash Book to the next with a view to covering up
misappropriation; either short banking of cash collection or a part of the amount of withdrawal from
the bank.
6. Investigation of Frauds through Supplier’s Ledger
In a Public Limited Company, it is suspected by the Management that there has been embezzlement in
supplier’s ledger. As an auditor of the Company, you have been asked to investigate the matter. What are the
major areas that you would verify in this regard? (Study Material)(Nov-2019-Old)
OR
State the points in an investigation of frauds through suppliers’ ledger.
Ans. Areas to be verified in case of embezzlement in supplier’s ledger:
While investigating the matters regarding embezzlement in supplier’s ledger, below mentioned areas
should be verified.
(ii) Verify the adjustments in fictitious or duplicate invoices as purchases in the accounts of suppliers
and subsequently misappropriating the amounts when payments are made to the suppliers in respect
of these invoices.
(iii) Check whether any credit notes issued by suppliers are being suppressed and the corresponding
amounts not claimed by them are subsequently withdrawn.
(iv) Check whether any amount unclaimed by suppliers, for one reason or another is being withdrawn by

FAST
showing that the same have been paid to suppliers.
(v) Verify whether purchase invoices are accepted at prices considerably higher than their market
prices.

CA.Sarthak Niraj Jain


(vi) Verify the bought journal with reference to entries in the goods inward book and the suppliers’
invoices to confirm that amounts credited to the accounts of suppliers were in respect of goods, which
were duly received, and the suppliers’ accounts had been credited correctly.
(vii) Request all the suppliers to furnish statements of their accounts to see whether or not any balance
is outstanding or due so as to confirm that allowances and rebates given by them have been
correctly adjusted.
7. Ways of Committing Inventory Frauds
MF Ltd., engaged in the manufacturing of various products in its factory, is concerned with shortage in
production and there arose suspicion of inventory fraud. You are appointed by MF Ltd. to evaluate the
options for verifying the process to reveal fraud and the corrective action to be taken. As an investigating
accountant what will be your areas of verification and the procedure to be followed for verification of
defalcation of inventory? (Nov-2019-New)
OR
Mr. Sharma is reviewing the anti-fraud controls for a construction company. The company has witnessed a
few frauds in the past mainly in the nature of material stolen from the sites and fake expense vouchers. Mr.
Sharma is evaluating options for verifying the process to reveal fraud and the corrective action to be taken in
such cases. As an expert, you are required to brief Mr. Sharma about the inventory fraud and verification
procedure with respect to defalcation of inventory? (RTP-May-2019)(MTP-Nov-2019)
Ans. Areas of verification and the procedure to be followed for verification of defalcation of inventory
Inventory frauds are many and varied but here we are concerned with misappropriation of goods and
their concealment.
(i) Employees may simply remove goods from the premises.
(ii) Theft of goods may be concealed by writing them off as damaged goods, etc.

242 Ab Audit Hoga Sabse Scoring - By CA. Sarthak Niraj Jain


Investigation, Due Diligence and Forensic Audit
(iii) Stock records may be manipulated by employees who have committed theft so that book quantities
tally with the actual quantities of stocks in hand.
(iv) Inflating the quantities issued for production is another way of defalcating raw materials and store
items.
(v) Stocks actually dispatched but not entered in sales/ debtor’s account.

Verification procedure:
(i) The first step towards verification is to establish the different items of inventory defalcated and
their quantities by checking physically the quantities in inventory held and those shown by the
Inventory Book.
(ii) Verify all the receipts and issues of inventory recorded in the Inventory Book by reference to
entries in the Goods Inward and Outward Registers and the documentary evidence as regards
purchases and sales. This would reveal the particulars of inventory not received but paid for as well as
that issued but not charged to customers.
(iii) Afterwards, the shortages observed on physical verification of inventory should be reconciled with
the discrepancies observed on checking the books in the manner mentioned above.
(iv) Defalcations of inventory, sometimes, also are committed by the management, by diverting a part of
production and the consequent shortages in production being adjusted by inflating the wastage
in production; similar defalcations of inventories and stores are covered up by inflating quantities
issued for production. For detecting such shortages, the investigating accountant should take
assistance of an engineer. For that he will be more conversant with factors which are responsible for
shortage in production and thus will be able to correctly determine the extent to which the shortage in
production has been inflated.

FAST
In this regard, guidance can also be taken from past records showing the extent of wastage in production
in the past. The per hour capacity of the machine and the time that it took to complete one cycle of
production, also would show whether the issues have been larger than those required.

CA.Sarthak Niraj Jain UNIT -2


DUE DILIGENCE

16.5 - Financial Due Diligence


8. Financial Due Diligence
Sri Rajan is above 80 years old and wishes to sell his proprietary business of manufacture of specialty
chemicals. Ceta Ltd. wants to buy the business and appoints you to carry out a due diligence audit to decide
whether it would be worthwhile to acquire the business. What procedures you would adopt before you could
render any advice to Ceta Ltd.? (Study Material)
OR
A German Company engaged in the business of manufacturing and distribution of industrial gases, is
interested in acquiring a listed Indian Company having a market share of more than 65% of the industrial gas
business in India, request you to conduct a “Due Diligence” of this Indian Company and submit your report, as
due Diligence Auditor, discuss the key areas you will cover in your review. (MTP-Nov-2018)
OR
Mr. Q is the proprietor of a very profitable business dealing in specialty chemicals. Due to his old age, Mr. Q
wants to sell his business and has approached XYZ Pvt. Ltd., a competitor, for the same. As an advisor to XYZ
Pvt. Ltd., you are appointed to do a “Due Diligence’ of the business. Enumerate the points which you would
look into as part of the Due Diligence exercise. (Nov-2017)
Ans. A due diligence audit on behalf of Ceta Ltd. with a view to acquiring the business shall involve
following steps before rendering any advice:
(i) Brief history of the target company and background of its promoters –
The accountant should begin the financial due diligence review by looking into the history of the

CA Final Audit - By CA. Sarthak Niraj Jain 243


Investigation, Due Diligence and Forensic Audit
company and the background of the promoters. The details of how the company was set up and who
were the original promoters has to be gone into, before verification of financial data in detail. An eye
into the history of the target company may reveal its turning points, survival strategies adopted by the
target company from time to time, the market share enjoyed by the target company and changes
therein, product life cycle and adequacy of resources.
(ii) Accounting policies - The accountant should study the accounting policies being followed by the
target company and ascertain whether any accounting policy is inappropriate. The accountant
should also see the effects of the recent changes in the accounting policies.
(iii) Review of Financial Statements - Before commencing the review of each of the aspect covered by
the financial statements, the accountant should examine whether the financial statements of the
target company have been prepared in accordance with the Statute governing the target
company, Framework for Preparation and Presentation of the Financial Statements and the relevant
Accounting Standards. If not, the accountant should record the deviations from the above and
consider whether it warrant an inclusion in the final report on due diligence.
After having an overall view of the financial statements, as mentioned in the above paragraphs,
the accountant should review the operating results of the target company in great detail. It is
important to make an evaluation of the profit reported by the target company. The reason being that
the price of the target company would be largely based upon its operating results.
(iv) Taxation - Tax due diligence is a separate due diligence exercise but since it is an integral
component of the financial status of a company, it is generally included in the financial due diligence.
It is important to check if the company is regular in paying various taxes to the Government. The
accountant has to also look at the tax effects of the merger or acquisition.
(v) Cash Flow - A review of historical cash flows and their pattern would reflect the cash generating

FAST
abilities of the target company and should highlight the major trends. It is important to know if
the company is able to meet its cash requirements through internal accruals or does it have to seek
external help from time to time.

CA.Sarthak Niraj Jain


It is necessary to check that:
(a) Is the company able to honor its commitments to its trade payables, to the banks, to
government and other stakeholders?
(b) How well is the company able to turn its trade receivables and inventories?
(c) How well does it deploy its funds?
(d) Are there any funds lying idle or is the company able to reap maximum benefits out of the
available funds?
(e) What is the investment pattern of the company and are they easily reali sable?
(vi) Financial Projections - The accountant should obtain from the target company the projections for
the next five years with detailed assumptions and workings. He should ask the target company to
give projections on optimistic, pessimistic and most likely bases.
(vii) Management and Employees - In most of the companies which are available for take over the
problem of excess work force is often witnessed. It is important to work out how much of the
labour force has to be retained. It is also important to judge the job profile of the administrative
and managerial staff to gauge which of these matches the requirements of the new incumbents. Due to
complex set of labour laws applicable to them, companies often have to face protracted litigation
from its workforce and it is important to gauge the likely impact of such litigation.
(viii) Statutory Compliance - During a due diligence this is one aspect that has to be investigated in detail.
It is important therefore, to make a list of laws/ statues that are applicable to the entity as well as
to make a checklist of compliance required from the company under those laws. If the company has
not been regular in its legal compliance it could lead to punitive charges under the law. These may
have to be quantified and factored into the financial results of the company

244 Ab Audit Hoga Sabse Scoring - By CA. Sarthak Niraj Jain


Investigation, Due Diligence and Forensic Audit

9. Hidden Liabilities & Overvalued Assets


KDK Bank Ltd., received an application from a pharmaceutical company for takeover of their outstanding
term loans secured on its assets, availed from and outstanding with a nationalized bank. KDK Bank Ltd.,
requires you to make a due diligence audit in the areas of assets of pharmaceutical company especially with
reference to valuation aspect of assets. State what may be your areas of analysis in order to ensure that the
assets are not stated at overvalued amounts.
(MTP-Dec-2021)(Study Material)(May-2018-New)(RTP-Nov-2020)
OR
A European company engaged in the business of manufacturing and distribution of industrial gases, is
interested in acquiring a listed Indian Company having a market share of 51 % and assets over ` 1000
Crores. It requests you to conduct "Due Diligence" of assets of this Indian Company to find out, if any of the
assets is overvalued. List down the areas of due diligence exercise to find out overvalued assets.
(Jan-2021-New)
OR
K Ltd. is intending to acquire M Ltd. Your firm of Chartered Accountants is appointed to conduct due
diligence. While reviewing hidden liabilities list out any five areas which will be specifically examined by you
in your due diligence exercise (Jan-2021-Old)
OR
Beta Ltd. is anticipating taking over a manufacturing concern and appoints you for due diligence review.
While reviewing, it requests you to look specifically for any hidden liabilities and overvalued assets. State in
brief the major areas you would examine for hidden liabilities and overvalued assets. (RTP-Nov-2019)
OR

FAST
M Limited is going to acquire S Limited. The purchase consideration has been decided at ` 4000 Crores. M
Limited is worried about hidden liabilities or overvalued assets of S Limited and approached you to examine
the same. List out eight important transactions/items which you would like to investigate in the Due
Diligence exercise.
Ans.
CA.Sarthak Niraj Jain
Investigation of hidden liabilities and overvalued assets
(a) In order to investigate hidden liabilities, the auditor should pay his attention to the following
areas:
1. The company may not show any show cause notices which have not matured into demands, as
contingent liabilities. These may be material and important
2. The company may have given “Letters of Comfort” to banks and Financial Institutions. Since
these are not “guarantees”, these may not be disclosed in the Balance sheet of the target
company
3. The Company may have sold some subsidiaries/businesses and may have agreed to take over
and indemnify all liabilities and contingent liabilities of the same prior to the date of transfer.
These may not be reflected in the books of accounts of the company
4. Product and other liability claims; warranty liabilities; product returns/discounts; liquidated
damages for late deliveries etc. and all litigation.
5. Tax liabilities under direct and indirect taxes
6. Long pending sales tax assessments
7. Pending final assessments of customs duty where provisional assessment only has been
completed
8. Agreement to buy back shares sold at a stated price
9. Future lease liabilities
10. Environmental problems/claims/third party claims
11. Unfunded gratuity/superannuation/leave salary liabilities; incorrect gratuity valuations
12. Huge labour claims under negotiation when the labour wage agreement has already expired
13. Contingent liabilities not shown in books.

CA Final Audit - By CA. Sarthak Niraj Jain 245


Investigation, Due Diligence and Forensic Audit
(b) Overvalued assets: The auditor shall have to specifically examine the following areas:
a. Uncollected / uncollectable receivables.
b. Obsolete, slow non-moving inventories or inventories valued above NRV; huge inventories of
packing materials etc. with name of company.
c. Underused or obsolete Plant and Machinery and their spares; asset values which have been
impaired due to sudden fall in market value etc.
d. Assets carried at much more than current market value due to capitalization of
expenditure/foreign exchange fluctuation, or capitalization of expenditure mainly in the
nature of revenue.
e. Litigated asset & property.
f. Investment carried at cost though realisable value much lower.
g. Investments carrying a very low rate of income / return.
h. Infructuous project expenditure / deferred revenue expenditure.
i. Group company balance under reconciliation etc.
j. Intangibles of no value.
SJ’S ANALYSIS
 This is a repetitive question in exams
 Many times, Institute has asked either about Hidden Liabilities or Overvalued Assets
 Therefore, students have to focus on learning these points as the chances of asking question in the
exams is very high
 Hidden liabilities are generally the liabilities which are intentionally hidden by the target company
in order to decive the acquirer

FAST
Over valuation of the asset on the other hand depict the good financial position of the entity where
actually it is worse than those that are shown.
10.
CA.Sarthak Niraj Jain
Due Diligence Review – Cash Flow
LMN Ltd. entered into a deal with SP Ltd. for buying its business of manufacturing wooden products/ goods.
LMN Ltd. has appointed your firm for conducting due diligence review and they want to know the cash
generating abilities of SP Ltd. What points will you check in order to ensure that the manufacturing unit of SP
Ltd. will be able to meet the cash requirements internally? (MTP-May-2021)
OR
PB Ltd. entered into a deal with SV Ltd. for buying its business of manufacturing wooden products/ goods. PB
Ltd. has appointed your firm for conducting due diligence review and they want to know the cash generating
abilities of SV Ltd. What points will you check in order to ensure that the manufacturing unit of SV Ltd. will be
able to meet the cash requirements internally? (Study Material)
OR
What are the important aspects to be looked into a due diligence review of cash flow?
Ans. Important aspects to be looked into a due diligence review of cash flow
In order to ensure that the manufacturing unit of SP Ltd. will be able to meet the cash requirements
internally, one is required to verify:
a. Is the company able to honor its commitments to its trade payables, to the banks, to the
government and other stakeholders?
b. How well is the company able to convert its trade receivables and inventories?
c. How well the Company deploys its funds?
d. Are there any funds lying idle or is the company able to reap maximum benefits out of the
available funds?
e. What is the investment pattern of the company and are they easily realizable?

246 Ab Audit Hoga Sabse Scoring - By CA. Sarthak Niraj Jain


Investigation, Due Diligence and Forensic Audit

16.6 - Due Diligence Report


11. Due Diligence Report – Contents
Write a short note on the following: Example of Headings of a Due Diligence Report. (RTP-Dec-2021)
OR
An American Company engaged in the business of manufacturing and distribution of industrial gases, is
interested in acquiring a listed Indian Company having a market share of more than 65% of the industrial gas
business in India. It requests you to conduct a “Due Diligence” of this Indian Company and submit your
Report. List out the contents of your Due Diligence Review Report that you will submit to your USA based
Client. (Study Material)
OR
XYZ Limited, a company engaged in the business of manufacturing and distribution of copper rods and
copper wire is interested in acquiring a listed company having a market share of 38% of Insulated Copper
Wires. You were appointed to conduct a “Due Diligence” of the target company and you have completed
review of a few key areas. List out the contents of the Due Diligence Report which you will submit to XYZ
Limited. (RTP-Nov-2019)(Nov-2015)
OR
Ekbote Co. is currently a large organisation trading in items of office furniture. The entity wants to expand
and hence are looking at acquisition of Rawat Co which deals in items of household furniture. Ekbote Co.
hires a Chartered accountant to conduct a due diligence to consider whether there is the potential for
additional value to be brought out of the target company by improving its operational function and also
whether there are serious operational risks about which the potential buyer should be concerned (thereby
allowing the buyer to consider aborting the deal or renegotiating the price). Which of the due diligence
review would be helpful to achieve the above objective? You are also required to briefly discuss the contents

Ans.
FAST
of a due diligence report.
Contents of the Due Diligence Report
(MTP-May-2019)

CA.Sarthak Niraj Jain


(i) Due Diligence: In the instant case Operational Due Diligence is required to confirm that the
business plan provided is achievable with the existing facilities plus the capital expenditure
outlined in the business plan. The contents of due diligence report will always vary with individual
circumstances.
(ii) Examples of Heading of a Due Diligence Report –
1. Executive summary.
2. Introduction.
3. Background of target.
4. Objective of due diligence.
5. Terms of reference and scope of verification.
6. Brief history of the company.
7. Share holding pattern.
8. Observation on the review.
9. Assessment of management structure.
10. Assessment of financial liabilities.
11. Assessment of valuation of assets.
12. Comments on properties, terms of lease, lien & encumbrances.
13. Assessment of operating results.
14. Assessment of taxation & statutory liabilities.
15. Assessment of possible liabilities. On account of litigation & legal proceedings against the
company.
16. Assessment of net worth.
17. SWOT analysis.

CA Final Audit - By CA. Sarthak Niraj Jain 247


Investigation, Due Diligence and Forensic Audit
18. Comments on future projections.
19. Status of charges, liens, mortgages, assets and properties of the company.
20. Interlocking investments and financial obligations with group / associates companies,
amount receivables subject to litigation, any other likely liability which is not provided for in
the Books of accounts.
21. Suggestions on ways and means including affidavits, indemnities, to be executed to cover
unforeseen and undetected contingent liabilities.
SJ’s ANALYSIS
 Generally, students gets confused about how to answer such question just by looking at the size of the
question and data relating to acquision;
 Whereas question is only focused on the content of the due diligence report.
 Remember-due diligence report generally includes objectives of due diligence and the list of area
assessed and auditors comment on above so try to recall the purpose and area of due diligence in exam
in order answer on such type of question
16.7- Due Diligence – Special Areas
12. Future Maintainable Turnover
J Ltd. is interested in acquiring S Ltd. The valuation of S Ltd. is dependent on future maintainable sales. As the
person entrusted to value S Ltd, what factors would you consider in assessing the future maintainable
turnover? (Study Material)
OR
A Ltd who is one of the leading manufacturer of kids clothing is interested to acquire B Ltd. B Ltd is currently
a manufacturer of women’ clothing. As a professional consultant in due diligence and valuation, A Ltd

Ans.
FAST
entrusted you to value B Ltd. The valuation of B Ltd is dependent on future maintainable sales. Discuss the
factors you would consider in assessing the future maintainable turnover of B Ltd? (MTP-May-2019)
In assessing the turnover which the business would be able to maintain in the future, the following

CA.Sarthak Niraj Jain


factors should be taken into account:
(i) Trend: Whether in the past, sales have been increasing consistently or they have been fluctuating. A
proper study of this phenomenon should be made.
(ii) Marketability: Is it possible to extend the sales into new markets or that these have been fully
exploited? Product wise estimation should be made.
(iii) Political and economic considerations: Are the policies pursued by the Government likely to
promote the extension of the market for goods to other countries? Whether the sales in the home
market are likely to increase or decrease as a result of various emerging economic trends?
(iv) Competition: What is the likely effect on the business if other manufacturers enter the same field
or if products which would sell in competition are placed on the market at cheaper price? Is the
demand for competing products increasing? Is the company’s share in the total trade constant or
has it been fluctuating?

UNIT -3
FORENSIC AUDIT
16.8 - Forensic Audit-Basics
13. Forensic Audit VS Financial Audit/Assurance Engagement/Other Audit
Shipra recently qualified as a Chartered Accountant and started her own practice. One of her friends told her
that Forensic Audit is a new area and has a lot of potential in terms of professional opportunities and
remuneration. Seema said that there is nothing new in this as ultimately forensic audit is also like other audits.
Do you agree with the views of Seema? Support your answer with relevant explanation. (RTP-May-2022)
OR

248 Ab Audit Hoga Sabse Scoring - By CA. Sarthak Niraj Jain


Investigation, Due Diligence and Forensic Audit
ABC Ltd. is a listed company having turnover of ` 50 crores & plans expansion by installation of new machines at
new building-having total additional project cost of ` 20 crore.
Rupees (In crore) Purpose
10.0 - for Building
8.5 - for Machinery
1.5 - for Working Capital
20 Crore Total
Project gets implemented in 2021-22 and one of the accountants report to the Managing Director that some
suspicious transactions are noticed in the purchase of building material. But the Management is confused as to
whether they should get an audit or Forensic Audit done for the same. Advise Management about the difference
in forensic accounting and audit. (Study Material)
OR
Explain how a Forensic Audit differs from an Assurance Engagement. (May-2018-New)
OR
Forensic Audit is unlike other audits explain.
Ans. A forensic accountant will often look for indications of fraud that are not subject to the scope of a financial
statement audit. Forensic Accounting has an Investigative mentality" however auditing is done with
"professional scepticism". A forensic accountant will often require more extensive corroboration. A forensic
accountant may focus more on seemingly immaterial transactions. Therefore, the contention of Seema is not
correct that ultimately forensic audit is also like other audits.
Difference between forensic audit and other audits
S. Financial Audit/Other
Particulars Forensic Audit
No.
1 FAST
Objective
Audits/Assurance Engagement
To express an opinion on true and
fair view.
Whether fraud has actually taken
place in books
2

3
CA.Sarthak Niraj Jain
Techniques

Period
Substantive & Compliance. Sample
based
Normally for a particulars accounting
Investigative, substantive or in-depth
checking
No such limitations
period.
4 Verification of Relies on the management Independent/verification of
assets, provisions, certificate/Management suspected/selected items where
liability etc. Representation misappropriation in suspected
5 Off balance sheet Used to vouch the arithmetic Regulatory & propriety of these
items (like accuracy & compliance with transactions/contracts are examined.
contracts etc.) procedures.
6 Adverse findings Negative opinion or qualified Legal determination of fraud impact
if any opinion expressed with/without and identification of perpetrators
quantification depending on scope.
16.9 - Forensic Auditor
14. Forensic Auditor - Necessary Skills and Experience
BR Construction was into the business of building roads and other infrastructure facilities for government
contracts. Mr. Tiwari, one of the senior official, was looking after the procurement of cement required at the
construction sites. There was a substantial increase in the price of cement bags bought as compared to those
bought prior to the appointment of Mr. Tiwari. The management of the company decides to get a forensic audit
done for the transactions handled by Mr. Tiwari. What points should be kept in mind by the management while
appointing a forensic auditor? (Study Material)
Ans. While appointing a Forensic Auditor: - The Management of BR Construction must initially consider whether

CA Final Audit - By CA. Sarthak Niraj Jain 249


Investigation, Due Diligence and Forensic Audit
the firm has the necessary skills and experience to accept the work.

A Forensic Auditor should necessarily possess the following characteristics and skills:
Characteristics- Forensic Auditor Skills - Forensic Auditor should possess
 Out of the box thinking  Auditing standards, procedures and related
 Strong visualization and imagination methodologies
curiosity  Accounting & Business reporting systems
 Persistence  Information technology
 Detail-oriented  Data analytics
 Inquisitiveness  Criminology
 Creativity  Legal framework
 Discretion  Litigation processes & procedures
 Skepticism  Investigative techniques
 Confidence and  Evidence gathering
 Sound professional judgement  Network of professional contacts in related fields'
 Objectivity and credibility viz. enforcement, regulatory bodies, law, industry,
peers etc.
16.10 - Forensic Audit Process
15. Forensic Audit - Process
Enumerate the steps to be undertaken in case of forensic audit process. (Study Material)
OR

FAST
You have been appointed as a forensic accountant in M/s Secure Ltd. to carryout various analysis as part of your
assignment to arrive at a particular result. Specify the various analysis which might have to be carried out by you
to arrive at your result. (May-2019 -New)

CA.Sarthak Niraj Jain OR


PQR Ltd. is a listed company having turnover of ` 50 crores & plans expansion by installation of new machines
at new building-having total additional project cost of ` 20 crore.
Rupees (In crore) Purpose
10.00 for Building
8.50 for Machinery
1.50 for Working Capital
20.00 Total

Project gets implemented in 2021-22 and one of the accountants points out to Managing Director that something
wrong has happened in the purchase of building material. On hearing this, the management is planning to
appoint Forensic Auditor. Advise the Forensic Auditor about the steps to be undertaken in case of forensic audit
process. (MTP-Nov-2018)
Ans. Step 1 - Initialisation
 It is vital to clarify and remove all doubts as to the real motive, purpose and utility of the
assignment
 Meeting with the client and accepting the engagement: In order to understand important facts,
players and issues etc., the investigator must meet the client. It is to be considered initially that
whether his firm has the necessary skills and experience to accept the work.

Step 2 – Develop the plan


 This is to be developed based on the meeting with the client and carrying out the initial

250 Ab Audit Hoga Sabse Scoring - By CA. Sarthak Niraj Jain


Investigation, Due Diligence and Forensic Audit
investigation scanner on the subjects to be investigated.
 This action plan will set out the objectives to be achieved and the methodologies to be adopted. The
investigation team must carefully take into consideration the objectives to be achieved and plan their
work accordingly.
Step 3 – Obtain relevant evidence
 The evidences gathered should be sufficient to ultimately identify and prove the fraudster(s) and
the mechanism adopted for such frauds.
 It involves obtaining relevant documents, economic information, tracing different assets/
persons/unaccounted records, meeting with other experts, statutory and internal auditors of the client.
Step 4 – Performing analysis of finding
The actual analysis to be performed will solely depend upon the nature of the assignment. This may
include:
 Calculating the economic damages and if required, the loss of goodwill.
 Summarisation of a large number of transactions.
 Performing robust procedures to trace unidentified assets.
 Estimating the present value of the financial losses or frauds involved in case such irregularities or
frauds took place for a long period of time.
 Performing the statistical regression or sensitivity analysis of the frauds etc.
 Using various computerized application software and graphs etc. to explain and analyse the frauds.
Step 5 – Reporting
 The client will expect a report containing the findings of the investigation, including summary of
evidence and a conclusion as to the amount of loss suffered

FAST
Section on the nature of the assignment, scope, approach utilized, limitation of scope and opinions.
Report is generally submitted to the appointing authority.
Step 6 – Court proceedings

CA.Sarthak Niraj Jain


The evidence gathered during the investigation will need to be presented at court, and team members
may be called to court to describe the evidence they have gathered and to explain how the suspect was
identified.
SJ Note- In case of Secure Ltd. only Step 4-Performing analysis of finding -need to write in the exam
16.11 - Forensic Audit Techniques
16. Technology Based /Digital Forensics Techniques
Write a short note on: Technology based /Digital Forensics Techniques (RTP-May-2021)
Ans. Technology based/Digital Forensics Techniques:
 Every transaction leaves a digital footprint in today's computer-driven society. Close scrutiny of relevant
emails, accounting records, phone logs and target company hard drives is a requisite facet of any modern
forensic audit.
 Before taking steps such as obtaining data from email etc. the forensic auditor should take appropriate
legal advice so that it doesn’t amount to invasion of privacy.
 Digital investigations can become quite complex and require support from trained digital
investigators.
 However, many open-source digital forensics tools are now available to assist in this phase of the
investigation:
Cross Drive Analysis EnCase
Live Analysis MD5
Deleted Files Tracking Log Files
Stochastic Forensics PC System Log

CA Final Audit - By CA. Sarthak Niraj Jain 251


Investigation, Due Diligence and Forensic Audit

Steganography Free Log Tools


17. Data Mining Techniques
CA Robo has been appointed as Forensic Auditor by BMY Bank Limited for one of its borrowal accounts WRONG
Ltd. CA Robo started the audit by first reviewing the transactions of the borrower its Bank statement as Per
Bank records to identify any hidden patterns in that information. She had to review huge volume of data, as the
number of transactions per day were to hundreds and the data was to be re-viewed for the last three years. So,
she was stuck up as to how to proceed further to identify any hidden patterns in information, if any. Guide CA
Robo, suggesting which technique to be used for identifying any hidden patterns in the information.
(Nov-2020-New)
Ans. (i) Data Mining Techniques:
 It is a set of assisted techniques designed to automatically mine large volumes of data for new,
hidden or unexpected information or patterns.
 Data mining techniques are categorized in three ways: -Discovery, Predictive modeling and
Deviation and Link analysis.
 It discovers the usual knowledge or patterns in data, without a predefined idea or hypothesis about
what the pattern may be, i.e. without any prior knowledge of fraud.
 It explains various affinities, association, trends and variations in the form of conditional logic.
(ii) In the given situation: -CA Robo has been appointed as Forensic Auditor by BMY Bank Limited for
one of its borrowal accounts WRONG Ltd. She had to review huge volume of data. She was stuck up as to
how to proceed further to identify any hidden patterns in information, if any
(iii) Advice: -As per given definition and facts of question CA Robo should use data mining techniques for
identify any hidden pattern in information.

Notes
FAST
CA.Sarthak Niraj Jain

252 Ab Audit Hoga Sabse Scoring - By CA. Sarthak Niraj Jain


Peer Review and Quality Review

Chapter
17
PEER REVIEW AND QUALITY REVIEW

Past Exams Weightage


5

0
MAY-19 NOV-19 NOV-20 JAN-21 JUL-21 DEC-21

May-19 Nov-19 Nov-20 Jan-21 Jul-21 Dec-21


Marks 0 0 0 5 0 4

UNIT -1
PEER REVIEW

FAST (Answers updated as per Peer Review Statement-2021)


17.1- Objective, Scope & Definition
1.
CA.Sarthak Niraj Jain
Scope of Peer Review – Exclusions
What are the areas excluded from the scope of Peer Review.
OR
(Study Material)

Areas not to be examined in Peer Review. (Nov-2017)


Ans. Areas excluded from the scope of Peer Review
(i) As per Statement of Peer Review by ICAI, the following services are excluded from the scope of
peer review:
1) Management Consultancy Engagements;
2) Representation before various Authorities;
3) Engagements to prepare tax returns or advising clients in taxation matters;
4) Engagements for the compilation of financial statements;
5) Engagements solely to assist the client in preparing, compiling or collating information other
than financial statements;
6) Testifying as an expert witness;
7) Providing expert opinion on points of principle, such as Accounting Standards or the
applicability of certain laws, on the basis of facts provided by the client; and
8) Engagement for Due diligence
SJ’s Note: -The phrase 'Assurance Services' is used interchangeably with Audit Services, Attestation
Functions, and Audit Functions.
2. Classification of Entity as per Statement of Peer Review
You are required to classify the following practice units into Level I entity or Level II entity for the purpose of
peer review along with providing the reason for such classification, assuming the services have been
undertaken in the period under review by such CA firms:

CA Final Audit - By CA. Sarthak Niraj Jain 253


Peer Review and Quality Review

Name of the Data of assurance services provided by such firms


Firm
MT & Co. Conducted statutory audit of a private company having the net worth of which is ` 300
crore.
GBL & Co. Conducted statutory audit of a Mutual Fund Company.
IML & Conducted statutory audit of a company registered U/S 8 of the Companies Act, 2013 but is
Associates not covered as a public interested entity. However, it has raised a contribution of ` 60 crore.
BTS & Co. Conducted statutory audit of an unlisted public company having net worth of ` 4 crore and
turnover of ` 55 crore. The net worth of its parent company is ` 325 crore.
TJK & Conducted statutory audit of LLP which has raised has a loan of `35 crore from a bank and a
Associates loan of `10 crore from an NBFC, respectively.
(RTP-Dec-2021)
Ans. Classification of Entity & Reason for Classification
Type of Reason for such classification based on the Statement of Peer Review
Name of Entity
Entity
MT & Co. Level I A Practice Unit which has undertaken Statutory Audit of a entities
entity having Net Worth of more than ` 100 Crores or having Turnover of `
250 Crore or above during the period under Review, shall be treated as a
Level I entity.
GBL & Co. Level I A Practice Unit which has undertaken Statutory Audit of a Mutual Fund
entity shall be treated as a Level I entity.

FAST
IML & Associates Level I
entity
A Practice Unit which has undertaken Statutory Audit of an Entity which
has raised funds from public or banks or financial institution or by
way of donations/contributions over ` 50 crore during the period

CA.Sarthak Niraj Jain


BTS & Co. Level I
entity by
under Review, shall be treated as a Level I entity.
A Practice Unit which has undertaken Statutory Audit of a entity:
(a) having Net Worth of more than ` 100 Crores or having Turnover of
virtue of ` 250 Crore or above during the period under Review,
FS (b) preparing FS as per IndAS
prepared shall be treated as a Level I entity.
as per As per Companies (Indian AS) Rules, 2015 any company having net
IndAS worth of 250 crores or more as on the end of the preceeding financial
year is required to prepare its FS as per IndAS. Further its subsidiary,
associate and parent company are also required to prepare IndAS.
The unlisted public company is not covered in the first limit above but
by virtue of Companies (Indian AS) Rules, 2015 rferred above, it will be
preparing FS as per IndAS and hence its auditor will be considered as
Level entity
TJK & Associates Level II A Practice Unit which has undertaken Statutory Audit of a entity that has
raised funds from bank, FI, >50 Crores is considered as Level I unit.
A Practice Unit which is not a Level I unit will be regarded as Level II
Unit. Since the LLP has not raised funds exceeding 50 crores its auditor
by virtue of these provisions shall not be classified as Level I practice
unit
3. Classification of Entity as per Statement of Peer Review
You are required to classify the following practice units into Level I entity or Level II entity for the purpose of
peer review along with providing the reason for such classification, assuming the services have been
undertaken in the period under review by such CA firms:

254 Ab Audit Hoga Sabse Scoring - By CA. Sarthak Niraj Jain


Peer Review and Quality Review

Name of the Data of assurance services provided by such firms


Firm
Abhinanadan Conducted statutory audit of a private company having net worth of which is ` 279 crore.
& Co.
Sambhav & Co. Conducted statutory audit of a mutual fund company and of a branch of a regional rural bank,
respectively.
Kunthu & Conducted statutory audit of LLP which has raised has a loan of ` 29 crore from a bank and a
Associates loan of ` 18 crore from an NBFC, respectively.
Dharam & Co. Conducted statutory audit of an unlisted public company having net worth of ` 3.79 crore and
turnover of ` 63 crore. The net worth of its parent company is ` 295 crore.
(MTP-Dec-2021)
Ans. Classification of Entity & Reason for Classification
Type of Reason for such classification based on the Statement of Peer Review
Name of Entity
Entity
Abhinanadan & Co. Level I A Practice Unit which has undertaken Statutory Audit of entitites having
entity Net Worth of more than ` 100 Crores or having Turnover of Rs. 250
Crore or above during the period under Review, shall be treated as a
Level I entity.
Sambhav & Co. Level I A Practice Unit which has undertaken Statutory Audit of a Mutual Fund
entity shall be treated as a Level I entity.
Kunthu & Associates Level II A Practice Unit which has undertaken Statutory Audit of a entity that has
entity raised funds from bank, FI, >50 Crores is considered as Level I unit.

FAST A Practice Unit which is not a Level I unit will be regarded as Level II Unit.
Since the LLP has not raised funds exceeding 50 crores its auditor by
virtue of these provisions shall not be classified as Level I practice unit.

CA.Sarthak Niraj Jain


Dharam & Co. Level I
entity by
virtue of
A Practice Unit which has undertaken Statutory Audit of a entity:
(a) having Net Worth of more than ` 100 Crores or having Turnover of
` 250 Crore or above during the period under Review,
FS (b) preparing FS as per IndAS
prepared shall be treated as a Level I entity.
as per As per Companies (Indian AS) Rules, 2015 any company having net
IndAS worth of 250 crores or more as on the end of the preceeding financial
year is required to prepare its FS as per IndAS. Further its subsidiary,
associate and parent company are also required to prepare IndAS.
The unlisted public company is not covered in the first limit above but
by virtue of Companies (Indian AS) Rules, 2015 rferred above, it will be
preparing FS as per IndAS and hence its auditor will be considered as
Level I entity
4. Assurance Services Engagement – Review
CA S has been appointed as peer reviewer of Shivam & Co. LLP. Shivam & Co. LLP submitted a list of its
assurance and due diligence services for the peer review. CA S is in the process of deciding as to how many
assurance services should be reviewed. Guide CA S in deciding the number of assurance services engagement
to be reviewed. (Jan-2021-New)
Ans. (i) In the given case: - CA S appointed as peer reviewer of Shivam & Co. LLP, is in the process of
deciding as to how many assurance services need to be reviewed.
(ii) The number of assurance service engagements to be reviewed shall depend upon:
 The standard of quality controls generally prevailing
 The size and nature of assurance service engagements undertaken by the Practice Unit

CA Final Audit - By CA. Sarthak Niraj Jain 255


Peer Review and Quality Review

 The methodology generally adopted by the Practice Unit in providing assurance services
 The number of partners / members involved in assurance service engagements in the Practice
Unit
 The number of locations / branch offices of the practice Unit
 The Fees charged / received / GST paid by the Practice unit.
From the initial sample selected at the planning stage, the Reviewer, in consultation with the Peer Review
Board, may reduce or enlarge the initial sample size of assurance service engagements for review.
17.2-Peer Review-Process
5. Assessment of Independence – Review & Control
Roshan, a practicing Chartered Accountant is appointed to conduct the peer review of another practicing
unit. What areas Roshan should review in the assessment of independence of the practicing unit?
(RTP-Dec-2021)(Study Material)(Jan-2021-Old)(MTP-Nov-2018)(MTP-Nov-2019)
OR
Write short note on: Area to be reviewed in the assessment of independence of the practicing unit while
conducting peer review. (May-2016)
Ans. (i) Review in the Assessment of Independence of the Practicing Unit –
The reviewer should carry out the compliance review of the five general controls, i.e.,
1. Independence,
2. Maintenance of professional skills and standards,
3. Outside consultation,
4. Staff supervision and development and
5. Office administration and evaluate the degree of reliance to be placed upon them.
FAST
 The degree of reliance will, ultimately, affect the attestation service engagements to be
reviewed. Independence is the main quality expected of an auditor.

CA.Sarthak Niraj Jain


 That is the very basis for the existence of the profession of auditing. Independence is a
condition of mind as well a personal character of a person. It is difficult to define but
very easy to perceive.
 Guidance Note on Independence of Auditors clarifies that independence is of two types, viz.
independence of mind and independence of appearance. The Guidance Note further
states that there are certain threats to independence which are classified as self interest
threats, self review threats, advocacy threats, familiarity threats and intimidation threats.
 The responsibility of the Peer Reviewer, therefore, is to ascertain the existence of
independence and the absence of threats to independence.

(ii) The reviewer should , therefore, check the following aspects in respect of assessment of
independence of the practicing unit:
a) Does the practice unit have a policy to ensure independence, objectivity and integrity, on the
part of partners and staff? Who is responsible for this policy?
b) Does the practice unit communicate these policies and the expected standards of professional
behaviour to all staff?
c) Does the practice unit monitor compliance with policies and procedures relating to
independence?
d) Does the practice unit periodically review the practice unit's association with clients to ensure
objectivity and independence?
e) How does the practice unit deal with the threats to independence?

256 Ab Audit Hoga Sabse Scoring - By CA. Sarthak Niraj Jain


Peer Review and Quality Review

17.3 - Reporting
6. Peer Review Report
ABC & Co LLP is a large firm of Chartered Accountants based out of Chennai. ABC & Co. LLP is subject to peer
review which was last conducted 3 years back. For the peer review of the financial year ended 31 March
2022, the firm got an intimation on 31 May 2022. The process of peer review got started and was completed
on 29 September 2022. In view of peer reviewer, the systems and procedures of ABC & Co. LLP are deficient /
non-compliant. The peer reviewer did not share any of his observations with ABC & Co LLP as draft and final
report was submitted to the Board. Comment. (MTP-Nov-2020)
Ans. Peer Review Report of Reviewer
(i) Discussion/Communication of Findings
 After completing the on-site Review, the Peer Reviewer, before making his Report to the Board,
shall communicate his findings to the Practice Unit if in his opinion, the systems and procedures
are deficient or non-compliant with reference to any matter that has been noticed by him or if there
are other matters where he wants to seek clarification.
The Practice Unit shall, within 5 days of the date of receipt of the findings, make submissions or
representations, in writing to the Reviewer.
(ii) Peer Review Report of Reviewer
 At the end of an on-site Review if the Reviewer is satisfied with the reply received from the
Practice Unit, he shall submit a Peer Review Report to the Board along with his initial findings,
response by the Practice Unit and the manner in which the responses have been dealt with. A copy of
the report shall also be forwarded to the Practice Unit.
 In case the Reviewer is of the opinion that the response by the Practice Unit is not satisfactory, the


FAST
Reviewer shall accordingly submit a modified Report to the Board incorporating his reasons for
the same.
The Reviewer shall also submit initial findings (i.e. Preliminary Report), response by the Practice

CA.Sarthak Niraj Jain



Unit (Response to Preliminary Report) and the manner in which the responses have been dealt with.
A copy of the report shall also be forwarded to the Practice Unit.
In case of a modified report, The Board shall order for a “Follow on” Review after a period of one
year from the date of issue of report as mentioned above. If the Board so decides, the period of one
year may be reduced but shall not be less than six months from the date of issue of the report.
(ii) In the instant case: - In view of peer reviewer systems and procedures in ABC & Co. LLP are
deficient, therefore, peer reviewer should not submit the report directly to the Board
(iii) Conclusion: -Thus, contention of ABC & Co. LLP is correct.
UNIT -2
QUALITY REVIEW
17.4-QR : Scope, Objectives, Areas & Process
7. Quality Reviews in terms of SQC -1
What are the important areas for evaluation while conducting quality reviews in terms of SQC -1 Standard on
Quality Control? (Study Material)
Ans. The important areas for evaluation while conducting quality reviews in terms of SQC -1 Standard on
Quality Control are
(i) Whether the audit firm establishes and implements policies and procedure on all the element of
system of quality control
(ii) Whether the engagement quality control reviewer review at an appropriate time for the planning
of an audit, significant audit judgement, and expressions of an audit opinion.
(iii) Whether the audit firm assigns as the person responsible for the monitoring of the system of
quality control a person with appropriate experience for the role, vest the assigned person with
sufficient and appropriate authority.

CA Final Audit - By CA. Sarthak Niraj Jain 257


Peer Review and Quality Review
(iv) Whether the audit firm obtains, at least annually, a confirmation letter concerning compliance
with policies and procedure for the maintenance of independence from all person required to
maintain independence.
(v) Whether the audit firm performs the independence confirmation procedure set forth in its internal
rules before acceptance and continuance of an audit engagement, and when issuing the auditor’s
report appropriately confirms that there was no change in the status of independence.
(vi) Whether the audit firm develop and provides education/ training program that fully take into
account the knowledge, experience, competence and capabilities of the professional staff.
8. Peer Review & Quality Review
Briefly explain the difference between Peer Review and Quality Review (MTP-May-2021)
Ans. Difference between Peer Review and Quality Review
 Peer Review
a) Peer review is a review of the systems and procedures of an audit firm.
b) Although sample audit files are inspected by the peer reviewer, it is done for the purpose of
testing the effectiveness of the systems and procedures.
c) The intention is to not to find faults but to help the firm develop effective systems.
d) It is a kind of mentoring process.
e) Peer review is a part of the activities of ICAI aimed at improving the quality of service.
 Quality Review
a) In contrast, a quality review is supposed to act as a deterrent.
b) Quality Review Board (QRB) is constituted by the Central Government and is independent
of ICAI.

FAST
c) As per Section 28A of the Chartered Accountant’s Act, the Central Government has the
authority to constitute a Quality Review Board.
d) QRB carries out supervisory and disciplinary functions.

CA.Sarthak Niraj Jain


e) A quality review normally pertains to one particular audit conducted by an audit firm.
f) The main objective quality review is to find errors or inadequacies, if any, committed by the
auditor while conducting the audit. Serious errors detected in quality review lead to
disciplinary action against the member.
17.5 - QR : Report, Qualifications & Actions
9. Actions that may be Recommended by the QRB
What are the consequences if the Quality Review Board notices major non-compliances with the
requirements of the Standards on quality control or standards on auditing or accounting standards?
(Study Material)
Ans. The actions that the Board may take, based upon consideration of recommendations of the QRG,
include one or more of the following:
(i) Make recommendations to the Council of ICAI for referring the case to the Director (Discipline) of
the Institute for consideration and necessary action under the CA Act.
(ii) Issue advisory and guidance to the AFUR for improvement in the quality of services and
adherence to various statutory and other regulatory requirements. A copy of such advisory may
also be sent to the ICAI for information.
(iii) Inform the details of the non-compliance to the regulatory bod(y)/ies relevant to the entity as
may be decided by the Board.
(iv) Intimate the AFUR as to the findings of the Report as well as action initiated as above.
(v) In case of review arising out of a reference received from a regulatory body, inform the results of
review and the details of action taken to the concerned regulatory body
(vi) Consider the matter complete and inform the AFUR accordingly.

258 Ab Audit Hoga Sabse Scoring - By CA. Sarthak Niraj Jain


Professional Ethics

Chapter
18
PROFESSIONAL ETHICS

Past Exams Weightage


17
16
15
14
13
12
11
10
9
8
7
6
5
4
3
2
1
0
MAY-19 NOV-19 NOV-20 JAN-21 JUL-21 DEC-21

May-19 Nov-19 Nov-20 Jan-21 Jul-21 Dec-21


Marks 14 14 12 17 12 12

1.
FAST 18.1 - Introduction (The IESBA Code)
Engagement specific Safeguards

CA.Sarthak Niraj Jain


A professional accountant in public practice is always subject to various threats in compliance with
fundamental principles of his profession and you, as a professional accountant, is worried about engagement
specific threat in your audit assignment of M/s Soft Ltd. and want to implement some measures to eliminate
and reduce the same. Enumerate some engagement specific safeguards which you may introduce in your
work environment to ward off such threats. (May-2019-New)
Ans. Engagement-specific safeguards in the work environment may include:
(i) Involving an additional professional accountant to review the work done or otherwise advise as
necessary.
(ii) Consulting an independent third party, such as a committee of independent directors, a professional
regulatory body or another professional accountant.
(iii) Discussing ethical issues with those charged with governance of the client.
(iv) Disclosing to TCWG of the client the nature of services provided and extent of fees charged.
(v) Involving another firm to perform or re-perform part of the engagement.
(vi) Rotating senior assurance team personnel.
18.2 - Important Provisions of Chartered Accountant,1949
Section-2(2) -Members Deemed to be in Practice
2. Assignment as Advisor and Consultant
CA Natraj, in practice, accepted an assignment as advisor and consultant to the public issue of shares by his
client M/s Super Ltd. Besides helping the company as an advisor, he also underwrote the public issue of the
company to the extent of 25% at a commission of 1%. Remaining shares were underwritten by banks and
other financial institutions at the same rate of commission. He contends that above assignments are part of
management consultancy work permitted by the council of the Institute. Do you agree with the view of CA
Natraj? Decide in the light of applicable code of conduct. (May-2019-New)

CA Final Audit - By CA. Sarthak Niraj Jain 259


Professional Ethics

Ans. (i) As per Council guidelines:-


• The Council of the Institute of Chartered Accountants of India (ICAI) pursuant to Section
2(2)(iv) of the Chartered Accountants Act, 1949 has passed a resolution permitting
“Management Consultancy and other Services” by a Chartered Accountant in practice.
• A clause of the aforesaid resolution allows Chartered Accountants in practice to act as
advisor or consultant to an issue of securities including such matters as drafting of
prospectus, filing of documents with SEBI, preparation of publicity budgets, advice regarding
selection of brokers, etc.
• It is, however, specifically stated that Chartered Accountants in practice are not permitted to
undertake the activities of broking, underwriting and portfolio management services.
(ii) In the instant case:- CA Natraj accepted an assignment as advisor and consultant to the public
issue of shares by his client M/s Super Ltd. In addition, he also under wrote the public issue of the
company to the extent of 25% at a commission of 1%. Contention of CA. Natraj that advisor,
consultant and underwriting work is part of management consultancy work and permitted by the
council is not correct as Chartered Accountants in practice are not permitted to undertake the
activities of broking, underwriting and portfolio management services.
(iii) Conclusion:- In view of the above, CA. Natraj would be guilty of misconduct under the Chartered
Accountants Act, 1949.
Section-6 - Certificate of Practice
3. Undertaking Tax Representation Work
A Chartered Accountant in practice has been suspended from practice for a period of 6 months. During the
said period, though he did not undertake the audit assignment since he had surrendered certificate of

Ans. (i)
FAST
practice, he had appeared before Income Tax authorities in his capacity as a Chartered Accountant.
(Study Material)
Provision:- Section 6 of CA Act, 1949, requires that no member of the Institute shall be entitled to

CA.Sarthak Niraj Jain


practice unless he has obtained from the Council a certificate of practice.
(ii) In the present case:- A chartered accountant not holding certificate of practice cannot take up any
other work because it would amount to violation of the relevant provisions of the Chartered
Accountants Act, 1949.
• In case a member is suspended and is not holding Certificate of Practice, he cannot in any
other capacity take up any practice separable from his capacity to practices as a member of
the Institute.
• This is because once a member becomes a member of the Institute; he is bound by the
provisions of the Chartered Accountants Act, 1949 and its Regulations. If he appears before
the income tax authorities, he is only doing so in his capacity as a chartered accountant and a
member of the Institute.
(iii) Conclusion:- Thus in the instant case, a chartered accountant would not be allowed to represent
before the income tax authorities for the period he remains suspended.
Section-21 - Disciplinary Action: Other Misconduct
4. Bringing Disrepute to the Profession
Mr. A, a practicing Chartered Accountant, failed to return the books of account and other documents of a
client despite many reminders from the client. The client had settled his entire fees dues also.
(MTP-Nov-2018)
Ans. (i) Provision:- A member is liable to disciplinary action under Section 21 of the Chartered Accountants
Act, 1949, if he is found guilty of any professional or “Other Misconduct”.
(ii) As per part:- IV of the First Schedule to the Chartered Accountants Act,
A member of the Institute, whether in practice or not, shall be deemed to be guilty of other
misconduct, if he –
(1) is held guilty by any civil or criminal court for an offence which is punishable with
imprisonment for a term not exceeding six months;

260 Ab Audit Hoga Sabse Scoring - By CA. Sarthak Niraj Jain


Professional Ethics
(2) In the opinion of the Council, brings disrepute to the profession or the Institute as a result of
his action whether or not related to his professional work.
A member may be found guilty of “Other Misconduct”, as per Clause (2), under the aforesaid
provisions rendering, himself unfit to be member if he retains the books of account and documents of
the client and fails to return these to the client on request without a reasonable cause.
(iii) In the given case:- Mr. A failed to return the books of accounts and other documents of his client
without any reasonable cause,
(iv) Conclusion:- He would be guilty of other misconduct under the aforesaid provisions.
Section-27 - Maintenance of Branch Offices
5. Maintenance of Branch Offices
Mr. G, a Chartered Accountant in practice as a sole proprietor has an office in Mumbai near Church Gate. Due
to increase in professional work, he opens another office in a suburb of Mumbai which is approximately 80
kilometers away from his existing office. For running the new office he employs three retired Income-tax
Officers. Is Mr. G guilty of professional misconduct? (Study Material)
Ans. (i) Provision:- As per section 27 of the Chartered Accountants Act, 1949, if a chartered accountant in
practice has more than one office in India; each one of these offices should be in the separate
charge of a member of the Institute.
(ii) 2nd office Exemption:- There is however an exemption for the above
• If the second office is located in the same premises, in which the first office is located; or
• The second office is located in the same city, in which the first office is located; or
• The second office is located within a distance of 50 kms from the municipal limits of a city,
in which the first office is located.

FAST
(iii) Conclusion:- Since the second office is situated beyond 50 kms of municipal limits of Mumbai
city. Thus, he would be liable for committing a professional mis-conduct. (Answer given assuming
distance >50 KM from municipal limits of Mumbai city as well)
6.
CA.Sarthak Niraj Jain
Maintenance of Branch Office
Mr. Z, a newly qualified chartered accountant started his practice in February 2018 by setting up an office in
the hill station Kodaikanal. Initially, since he was getting very less assignments, he decided to set up a
temporary office in the nearby city Marudai, situated at about 100 kms from the main office. As planned, he
took an office space on rent for the months of April, May & June. During these months, his regular office was
not closed and Mr. Z was in-charge for both the offices. Mrs. A, another newly qualified chartered accountant
who is also in practice in Marudai came to know about the new office of Mr. Z. Thinking that he could be a
potential competitor, she informed the institute stating that Mr. Z had violated the provisions of the
Chartered Accountant Act. As a member of the Board of Discipline of ICAI, you are requested to analyse this
complaint. (MTP-May-2021)
Ans. (i) Provision: -As per section 27 of the Chartered Accountants Act, 1949, if a chartered accountant in
practice has more than one office in India; each one of these offices should be in the separate
charge of a member of the Institute.
(ii) 2nd office Exemption:- There is however an exemption for the above
• If the second office is located in the same premises, in which the first office is located; or
• The second office is located in the same city, in which the first office is located; or
• The second office is located within a distance of 50 kms from the municipal limits of a city, in
which the first office is located.
(iii) Hill area Exemption: - Members in practicing in hill areas subject to certain conditions such as:
• Such member/ firm be allowed to open temporary offices in a city in the plains for a limited
period not exceeding 3 months in a year.
• The regular office need not be closed during this period and all correspondence can continue to
be made at the regular office.

CA Final Audit - By CA. Sarthak Niraj Jain 261


Professional Ethics
• The name board of the firm in temporary office should not be displayed at times other than the
period such office is permitted to function.
• The temporary office should not be mentioned in letter head, visiting card, any other documents
as a place of business of the member/ firm.
• Before commencement of every winter, it shall be obligatory on the member/firm to inform the
Institute that he/it is opening the temporary office from a particular date and after the office is
closed at the expiry of the period of permission, an intimation to that effect should also be sent to
the office of the Institute by registered post.
(iv) In the given case:- Mr. Z has set up his regular office in the hill area of Kodaikanal , he decided to set
up a temporary office in the nearby city Marudai, situated at about 100 kms from the main office.
As planned, he took an office space on rent for the months of April, May & June. During these months,
his regular office was not closed. Further he was in-charge for both the offices.
(v) Conclusion: -In view of abovementioned criteria’s, he is eligible to avail the benefits of the above
exemptions. Also, it is given that the temporary office was open in Madurai for only 3 months and
not beyond that. The fact that Mr. Z is in-charge for both the offices, the temporary office being set-
up in the plains which is 100 kms away and the regular office kept open during the 3 months does
not constitute any violation of the provisions of the Chartered Accountant Act. Assuming Mr. Z has
informed the Institute regarding such temporary office in the prescribed manner. Therefore, in the
given case, no penal action needs to be taken on the basis of complaint registered by Mrs. A, as
Mr. Z is not guilty of professional misconduct.
18.3 - Schedules to the CA Act, 1949 (Professional & Other Misconduct)
First Schedule Part I – Professional Misconduct in Relation to Members in Practice
Schedule – 1, Part – 1, Clause – 1
7. FAST
Allowing Others to use Firm’s Name
CA Sant, a newly qualified professional with certificate of practice, approached CA Pant, the auditor of his

CA.Sarthak Niraj Jain


father's company M/s Max Ltd. to allow him to have some practical and professional knowledge and
experience in his firm before he can set up his own professional practice. CA Pant allowed him to sit in his
office for 6 month and allotted a small chamber with other office infrastructure facility in the course of his
association with CA Pant's office, he used to provide tax consultancy independent to the client of the firm and
also filed few and GST return and represented himself before various tax authorities on behalf of the firm
although no documents were signed by him. During his association in CA Pant's office he did not get any
share of profit or commission but only re-imbursement of usual expenses like conveyance, telephone etc. was
made to him. After the end of the agreed period, he was given a lump sum amount of 3,00.000 by CA Pant for
his association out of gratitude.
Examine the case in the light of code of professional misconduct. (May-2019-New)
Ans. (i) As per Clause:- Clause (1) of Part I of the First Schedule to the Chartered Accountants Act, 1949
states that a chartered accountant in practice shall be deemed to be guilty of professional
misconduct:
• If he allows any person to practice in his name as a chartered accountant
• Unless such person is also a chartered accountant in practice and
• Is in partnership with or employed by him.
(ii) Purpose of Clause:-
• The above clause is intended to safeguard the public against unqualified accountant
practicing under the cover of qualified accountants.
• It ensures that the work of the accountant will be carried out by a Chartered Accountant
who may be his partner, or his employee and would work under his control and supervision.
(iii) In the instant case:- CA Pant allowed CA Sant (who is a newly qualified CA professional with COP) to
sit in his office for 6 months, and allowed him to provide tax consultancy independently to his firm’s
clients, filing of some IT and GST Returns. He also allowed him to appear before various tax authorities

262 Ab Audit Hoga Sabse Scoring - By CA. Sarthak Niraj Jain


Professional Ethics
on behalf of his firm. CA Sant was only reimbursed with his usual expenses and was not paid any salary
or share of profit for the same. However, after the end of agreed period he was given a lump-sums of
rupees 3,00,000 for his association out of gratitude.
(iv) Conclusion:- Thus, in the present case CA. Pant will be held guilty of professional misconduct as
per Clause (1) of Part I of First Schedule to the Chartered Accountants Act, 1949 as he allowed CA
Sant to practice in his name as Chartered accountant and CA Sant is neither in partnership nor in
employment with CA. Pant.
Schedule – 1, Part – 1, Clause – 2
8. Sharing Fees with an Articled Clerk
Mr. Ankit, who passed his CA Examination of ICAI on 18 th July 2022, had started his practice from 15th
August 2022. On 16th August 2022 one candidate approached him for Articleship. Mr. Ankit decided to give
her 1% profits of his CA Firm in addition to monthly stipend. She agreed to take both 1% of Profits and
prescribed stipend. The ICAI had sent a letter to Mr. Ankit objecting the payment of 1% Profits.
Mr. Ankit replied stating that sharing 1% Profits is over and above the Stipend, to help the Articled Clerk to
overcome her Financial crisis. Is Mr. Ankit liable for professional misconduct? (Study Material, May-2016)
Ans. (i) As per Clause:- Clause (2) of Part I of First Schedule to the Chartered Accountants Act, 1949
A member shall be held guilty if a Chartered Accountant in practice pays or allows or agrees to pay
or allow, directly or indirectly, any share, commission or brokerage in the fees or profits of his
professional business, to any person other than
• a member of the Institute or
• a partner or
• a retired partner or



FAST the legal representative of a deceased partner, or
a member of any other professional body or
with such other persons having such qualification as may be prescribed, for the purpose of

CA.Sarthak Niraj Jain


rendering such professional services from time to time in or outside India.
(ii) Analysis of case:- In view of the above, the objections of the Institute of Chartered Accountants of India,
as given in the case, are correct and reply of Mr. X, stating that he is paying 1 % profits of his firm over
and above the stipend to help the articled clerk as the position of the articled clerk is weak is not tenable.
(iii) Conclusion:- Hence, Mr. X is guilty of professional misconduct in terms of Clause (2) of Part I of
First Schedule to the Chartered Accountants Act 1949.
9. Sharing Professional Receipts with Others
A CA obtained a loan from a finance company for purchase of office building agreeing to pay interest at 6%
p.a. and 10% of his gross professional receipts till the loan is repaid.
Ans. (i) As per Clause:- Clause (2) of Part I of First Schedule to the Chartered Accountants Act, 1949
A member shall be held guilty if a Chartered Accountant in practice pays or allows or agrees to pay
or allow, directly or indirectly, any share, commission or brokerage in the fees or profits of his
professional business, to any person other than
• a member of the Institute or
• a partner or
• a retired partner or
• the legal representative of a deceased partner, or
• a member of any other professional body or
• with such other persons having such qualification as may be prescribed, for the purpose of
rendering such professional services from time to time in or outside India.
(ii) In the given case:- In substance the CA has agreed to share his receipt with a finance company from
whom he has taken a loan, which is not permitted.
(iii) Conclusion:- Mr. K will be deemed to be guilty of professional misconduct under Clause 2 of Part I of
First Schedule.

CA Final Audit - By CA. Sarthak Niraj Jain 263


Professional Ethics

Schedule – 1, Part – 1, Clause – 3


10. Sharing Fees of Professional Work of Others
CA. P is a newly qualified Chartered Accountant in practice and in order to increase his professional practice
and client base, entered into an agreement with Mr. A, a qualified and experienced registered valuer to share
20% professional fees for all cases of valuation referred to him by CA. P. Based on this, CA. P received
₹1,20,000 during the year 2021-22 from Mr. A. Is CA. P guilty of misconduct under the Chartered
Accountants’ Act, 1949? (Nov-2019–Old)
Ans. (i) As per Clause:- Clause 3 of Part I of the First Schedule to the Chartered Accountant Act, 1949, A CA
in Practice is deemed to be guilty of professional Misconduct if he:
• Accepts or agrees to accept any part of the profits of the professional work of a person
• Who is not a member of the Institute
(ii) Exception:-
• However, such a restriction does not apply in respect of member of any other professional
bodies or with such other persons having prescribed qualifications.
• Regulation 53A and 53B of CA Regulations 1988 prescribes the qualifications of persons with
whom profits can be shared.
(iii) In the given case:- CA. P a practicing Chartered Accountant entered into an agreement with Mr. A, a
qualified and experienced registered valuer to share 20% professional fees for all cases of
valuation referred to him by CA. P. Based on this, CA. P received ₹1,20,000 during the year 2021-22
from Mr. A.
(iv) Conclusion:- Mr. P will be deemed to be guilty of professional misconduct by virtue of Clause 3, Part
I of First Schedule as he accepts professional fees from a person who is not a member of ICAI. Further,

11.
FAST
registered valuers are not recognised for profit sharing purpose under Regulation 53A and 53B.
Schedule – 1, Part – 1, Clause – 4
Partnership with CPA & ICAEW Members

CA.Sarthak Niraj Jain


A Chartered Accountant practicing in India enters into partnership with
(a) A Certified Public Accountant in New York .
(b) A Chartered Accountant from the Institute of Chartered Accountants in England and Wales in London,
and in each case, the members concerned take the profits earned in their own country.
Will it make any difference, if an Indian Chartered Accountant is practicing outside India and becomes a
partner with the aforesaid accountants?
Ans. (a) Partnership with a CPA in New York:
(i) As per Clause:- Clause (4) of Part I to the First Schedule to the Chartered Accountants Act,
1949 specifies that a chartered accountant in practice shall be deemed to be guilty of
professional misconduct if he enters into partnership:-
• in or outside India, with any person other than a chartered accountant in practice or
• such other person who is a member of any other professional body having such
qualifications as may be prescribed, including a resident who but for his residence
abroad would be entitled to be registered as a member or
• whose qualifications are recognised by the Central Government or
• the Council for the purpose of permitting such partnerships;
(ii) Conclusion:- Thus, chartered accountant would be guilty of professional misconduct since
certified public accountants (CPA) are not eligible to become members of the Institute.
(b) Partnership with a chartered accountant from ICAEW:
(i) As per Clause:- it is important that partnership with a member of the foreign professional
body is permissible provided inter alia such bodies are eligible for the membership of the
Institute. Earlier, the Council had passed a resolution permitting chartered accountants
from ICAEW to become members of the Institute as also fulfillment of certain conditions in
respect of persons not permanently residing in India.

264 Ab Audit Hoga Sabse Scoring - By CA. Sarthak Niraj Jain


Professional Ethics
(ii) Council General Meeting:- However, the Council of the Institute at its meeting held in
December, 1995 decided to withdraw the resolution w.e.f. December 8, 1995.
(iii) Analysis of the case:- In view of this, persons qualified from any of the four Institutes in the
United Kingdom including England and Wales are not entitled to have their names entered in the
Register of Members maintained by the Institute effective from December 8, 1995. Based on this
development, partnership between members of the Institute and members of above foreign
professional bodies will not be permissible from the above date. Even a chartered accountant
from ICAEW who was eligible to become member of the Institute, the profit sharing
arrangement stated in the question goes against the provisions of Clause (4).
(iv) Conclusion:- Hence, it would constitute professional misconduct.

Chartered Accountants practicing outside India:


(i) Provision:- A member of the Institute of Chartered Accountants of India is governed by the Code of
Ethics whether practicing in India or outside India. Accordingly, the question of professional
misconduct would arise if an Indian chartered accountant practicing outside India becomes a
partner with aforesaid accountants and enters into partnership in that country with a member
of the Institute of that country.
(ii) Conclusion:- There would be professional misconduct within the provisions of the Institute of
Chartered Accountants Act, 1949 as the applicability of such provisions extend to members
practicing outside India.
Schedule – 1, Part – 1, Clause – 6
12. Soliciting Clients
A special notice has been issued for a resolution at 3rd annual general meeting of Fiddle Ltd. providing

FAST
expressly that CA. Smart shall not be re-appointed as an auditor of the company. Consequently, CA. Smart
submitted a representation in writing to the company as provided under section 140(4)(iii) of the
Companies Act, 2013. In the representation, CA. Smart incorporated his independent working as a

Ans.
CA.Sarthak Niraj Jain
professional throughout the term of office and also indicated his willingness to continue as an auditor if
reappointed by the shareholders of the Company.
(i)
(Nov-2019-New)(MTP-May-2019)
As per Clause:- (6) of Part I of First Schedule to the Chartered Accountants Act, 1949
A Chartered Accountant in practice is deemed to be guilty of professional misconduct if he solicits
clients or professional work
• either directly or indirectly by circular, advertisement, personal communication or
• interview or by any other means except applying or requesting for or inviting or securing
professional work from another chartered accountant in practice and responding to
tenders.
(ii) As per Provision:- Section 140(4)(iii) of the Companies Act, 2013
• Provides a right, to the retiring auditor, to make representation in writing to the company.
• The retiring auditor has the right for his representation to be circulated among the members
of the company and to be read out at the meeting.
• However, the content of letter should be set out in a dignified manner how he has been acting
independently and conscientiously through the term of his office and
• May, in addition, indicate, if he so chooses, his willingness to continue as auditor, if re-
appointed by the shareholders.
(iii) Interpretation of case:- Thus, the incorporation as an independent professional, made by CA. Smart,
while submitting representation under section 140(4)(iii) of the Companies Act, 2013 and indication of
willingness to continue as an auditor if reappointed by shareholders, does not leads to solicitation.
(iv) Conclusion:-Therefore, CA. Smart will not be held guilty for professional misconduct under Clause
(6) of Part I of First Schedule to the Chartered Accountants Act, 1949.

CA Final Audit - By CA. Sarthak Niraj Jain 265


Professional Ethics

13. Soliciting the Professional Work from another CA


CA. Nikhil, in practice, started project consultancy work as a part of his practice and to advance the same,
sent mail to all the CAs in the country informing them of his services and for securing professional work.
Comment with reference to the Chartered Accountants Act, 1949, and Schedules thereto. (MTP-May-2020)
Ans. (i) As per Clause:- (6) of Part I of First Schedule to the Chartered Accountants Act, 1949
A Chartered Accountant in practice is deemed to be guilty of professional misconduct if he solicits
clients or professional work
• either directly or indirectly by circular, advertisement, personal communication or
• interview or by any other means
(ii) Exception:- nothing herein contained shall be construed as preventing or prohibiting, any chartered
accountant from applying or requesting for or inviting or securing professional work from
another chartered accountants in practice.
(iii) In the instant case:- CA. Nikhil has written email to all the CA for securing professional work from
them and has not approached any other person or professional or communicated with any client,
(iv) Conclusion:- Thus as per exception to the Clause (6), CA. Nikhil is well within the regulation of the act
and has not committed any professional misconduct.
14. Soliciting Professional Work
Mr. S, a Chartered Accountant published a book and gave his personal details as the author. These details also
Amendment
mentioned his professional experience and his present association as partner with M/s RST, a firm.
(Study Material)
Ans. (i) As per Clause:- (6) of Part I of First Schedule to the Chartered Accountants Act, 1949
A Chartered Accountant in practice is deemed to be guilty of professional misconduct if he solicits


FAST
clients or professional work
• either directly or indirectly by circular, advertisement, personal communication or
interview or by any other means

CA.Sarthak Niraj Jain


(ii) Interpretation of Clause:-
• Members should not adopt any indirect methods to advertise their professional practice
with a view to gain publicity and thereby solicit clients or professional work.
• Such a restraint must be practiced so that members may maintain their independence of
judgment and may be able to command the respect of their prospective clients.
(ii) Council General Guidelines :-
While elaborating forms of soliciting work, the Council has specified that a member is not permitted
to indicate in a book or an article, published by him, the association with any firm of chartered
accountants.
(iv) In the given case:- Mr. S a Chartered Accountant published the book and mentioned his
professional experience and his association as a partner with M/s RST, a firm of chartered
accountants.
(v) Conclusion:- Mr. S being a chartered accountant in practice has committed the professional
misconduct by mentioning that at present he is a partner in M/s. RST, a chartered accountants
firm.
SJ Note -Revised Ethics, permit mentioning the firm name in the book, but ICAI module concludes this to be a
misconduct, may be because of showing association as a partner.
15. Responding to Tenders
M/s LMN, a firm of Chartered Accountants responded to a tender from a State Government for
computerization of land revenue records. For this purpose, the firm also paid ` 50,000 as earnest deposit as
part of the terms of the tender. (Study Material, MTP-Nov-2018)
OR

266 Ab Audit Hoga Sabse Scoring - By CA. Sarthak Niraj Jain


Professional Ethics
M/s PQR, a firm of Chartered Accountants responded to a tender from a State Government for Government
for computerization of land revenue records. For this purpose, the firm also paid ` 50,000 as earnest deposit
as part of the terms of the tender. Comment with reference to the Chartered Accountants Act, 1949, code of
Ethics and schedules to the Act. (Jan-2021-Old)
OR
M/S ABC a firm of Chartered Accountants responded to a tender from a State Government department for
switching over to double entry system of accounting. For the purposes the firm also paid ` 25,000 as earnest
money deposits as per the terms of the tender.
Ans. (i) As per Clause:- Clause (6) of Part I of the First Schedule to the Chartered Accountants Act, 1949 a
CA is permitted to respond to tenders. Further chapter XIII to Council General Guidelines, 2008 lays
down guidelines for responding g to tenders, etc.
(ii) As per the guidelines
A member of the institute is practice shall not respond to any tender issued by an organization or
user of professional services in areas of services which are exclusively reserved for chartered
accounts such as audit and attestation services. However, such restriction shall not be applicable
where minimum fee of the assignment is prescribed in the tender document itself or where the areas
are open to other professionals along with the Chartered Accountants.
(iii) Conclusion:- Since computerization of land revenue records does not fall within exclusive areas
for chartered accountants, M/s LMN can respond to tender as well as deposit ` 50,000 as
earnest deposit and shall not have committed any professional misconduct.
16. Publishing an Advertisement Containing Photograph
An advertisement was published in a Newspaper containing the photograph of Mr. X, a member of the

Ans. (i)
FAST
institute wherein he was congratulated on the occasion of the opening ceremony of his office.
(Study Material)
As per Clause:- (6) of Part I of First Schedule to the Chartered Accountants Act, 1949

CA.Sarthak Niraj Jain


A Chartered Accountant in practice is deemed to be guilty of professional misconduct if he solicits
clients or professional work
• either directly or indirectly by circular, advertisement, personal communication or
• interview or by any other means
(ii) In the given case:- Mr. X published an advertisement in a Newspaper containing his photograph on
the occasion of the opening ceremony of his office.
(iii) Analysis of the case:- On this context, it may be noted that the advertisement which had been put in
by the member is quite prominent. If soliciting of work is allowed, the independence and
forthrightness of a Chartered Accountant in the discharge of duties cannot be maintained. The
above therefore amounts to soliciting professional work by advertisement directly or indirectly.
(iv) Conclusion:- Mr. X would be therefore held guilty under Clause (6) of Part I of the First Schedule to
the Chartered Accountants Act, 1949.
17. Printing of Designation “Chartered Accountant” on Religious Ceremony Invitation Cards
CA. Shrishti and CA. Mishti are two partners of the CA firm ‘Shrishti Mishti & Associates’. Being very pious,
CA. Srishti organised a religious ceremony at her home for which she instructed her printing agent to add her
designation “Chartered Accountant” with her name in the invitation cards. Later on, the invitations were
distributed to all the relatives, close friends and clients of both the partners. (RTP-Nov-2020)
OR
CA. Suresh and CA. Mahesh are two partners of the CA firm ‘SM & Co. Being very pious, CA. Suresh organised
a religious ceremony at his home for which he instructed his printing agent to add his designation “Chartered
Accountant” with his name in the invitation cards. Later on, the invitations were distributed to all the
relatives, close friends and clients of both the partners.

CA Final Audit - By CA. Sarthak Niraj Jain 267


Professional Ethics

Ans. (i) As per Clause:- (6) of Part I of First Schedule to the Chartered Accountants Act, 1949
A Chartered Accountant in practice is deemed to be guilty of professional misconduct if he solicits
clients or professional work:
• either directly or indirectly by circular, advertisement, personal communication or
• interview or by any other means
(ii) As per guidelines:-
• However, the Council of the ICAI is of the view that the designation “Chartered Accountant” as
well as the name of the firm may be used in greeting cards, invitations for marriages and
religious ceremonies and any other specified matters,
• Provided that such greeting cards or invitations etc. are sent only to clients, relatives and
close friends of the members concerned.
(iii) In the given case:- CA. Shrishti is adding the designation Chartered Accountant with her name on
invitation cards and these cards were distributed to her and her partner’s relatives, close friends
and clients.
(iv) Conclusion:- CA. Shrishti shall be deemed to be guilty of professional misconduct for sending the
invitation cards using designation chartered accountant to partner’s relatives, close friends and
clients.
18. Responding to Tenders
OPAQ & Associates, a firm of Chartered Accountants responded to a tender issued exclusively for Chartered
Accountants by an organisation in the area of tax audit. However no minimum fee was prescribed in the
tender document. (RTP-May-2020), (May-2017)
OR
M/s ABC & Co., a firm of Chartered Accountants responded to a 5 tender from a State Government for

FAST
appointment of Tax Auditor wherein no minimum fee was prescribed. For this purpose, the firm also paid
50,000 as earnest deposit as part of the terms of the tender. Comment on the above with reference to the
Chartered Accountants Act, 1949, Code of Ethics and Schedules to the Act. (Nov-2020-Old)
Ans. CA.Sarthak Niraj Jain
(i) As per Clause:- Clause (6) of Part I of the First Schedule to the Chartered Accountants Act, 1949 a
CA is permitted to respond to tenders. Further chapter XIII to Council General Guidelines, 2008 lays
down guidelines for responding g to tenders, etc.
(ii) As per Council General Guidelines:- (Chapter XII of Council General Guidelines, 2008)
A member of the Institute in practice shall not respond to any tender issued by a organization or
user of professional services in areas of services which are exclusively reserved for chartered
accounts such as audit and attestation services. However, such restriction shall not be applicable
where minimum fee of the assignment is prescribed in the tender document itself or where the areas
are open to other professionals along with the Chartered Accounts.
(iii) In the instant case:- M/s OPAQ & Associates responded to a tender of tax audit which is exclusively
reserved for Chartered Accountants even though no minimum fee was prescribed in the tender
document.
(iv) Conclusion:- Therefore, M/s OPAQ & Associates shall be held guilty of professional conduct for
responding to such tender in view of abovementioned guideline.
19. Responding to Tenders
PK Foundation decided to review its historical financial statements. For this, it proposed a tender exclusively
Amendment for Chartered Accountants to obtain assurance, primarily by performing inquiry and analytical procedures,
about whether the financial statements as a whole are free from material misstatement. However, the
foundation did not prescribe the minimum fee in the tender document. M/s Sodhi & Co., a Chartered
Accountant firm, responded to such tender. (Nov-2017)
OR

268 Ab Audit Hoga Sabse Scoring - By CA. Sarthak Niraj Jain


Professional Ethics
Sapna Foundation decided to review its historical financial statements. For this, it proposed a tender
exclusively for Chartered Accountants to obtain assurance, primarily by performing inquiry, and analytical
procedures about whether the financial statements as a whole are free from material misstatement.
However, the foundation did not prescribe the minimum fee in the tender document. M/s Joshi & Sons, a
Chartered Accountant firm, responded to such tender
Ans. (i) As per Clause:- (6) of Part I of First Schedule to the Chartered Accountants Act, 1949
A Chartered Accountant in practice is deemed to be guilty of professional misconduct if he solicits
clients or professional work:
• either directly or indirectly by circular, advertisement, personal communication or
• interview or by any other means
(ii) As per guidelines:- The relevant guidelines in the context of the tender are:
1. A member of the Institute in practice shall not respond to any tender in areas of services which are
exclusively reserved for Chartered Accountants by statute viz. Audit and Attestation Services
such as Audit under Companies Act 2013, Income Tax Act 1961, etc.;
2. However, a member may respond to tenders as mentioned above wherever the minimum fee of the
assignment is prescribed in the tender document itself.
(iii) In the instant case:- M/s Sodhi & Co., responded to a tender relating to review of historical
financial statements of PK Foundation which is exclusively reserved for Chartered Accountants even
where no minimum fee was prescribed in the tender document.
(iv) Conclusion:- Therefore, M/s Sodhi & Co. shall be held guilty of professional misconduct for
responding to such tender in view of above mentioned guideline.
SJ Note -Answer given assuming that such review is statutorily required to be done by CA only and not just that

20. FAST
the client intends to conduct it from a CA
Acceptance of Original Professional Work - Client Introduced to Him by Another Member
During the opening ceremony of a new branch office of CA. Young, his friend CA. Old introduced to CA. Young,

CA.Sarthak Niraj Jain


his friend and client Mr. Rich, the owner of an Export House whose accounts had been audited by CA. Old for
more than 15 Years. After few days, Mr. Rich approached CA. Young and offered a certification work which
earlier to have been done by CA. Old. CA. Young undertook the work for a fee which was not less than fee
charged by CA. Old in earlier period. Comment whether CA. Young had done any professional misconduct.
(Nov-2018-New)
Ans. (i) As per Clause:- (6) of Part I of First Schedule to the Chartered Accountants Act, 1949
A Chartered Accountant in practice is deemed to be guilty of professional misconduct if he solicits
clients or professional work:
• either directly or indirectly by circular, advertisement, personal communication or
• interview or by any other means
(ii) As per Council Guidelines:-
• The Council has prohibited include that a member should not accept the original
professional work emanating from a client introduced to him by another member.
• If any professional work of such client comes to him directly, it should be his duty to ask the
client that he should come through the other member dealing generally with his original
work.
(iii) In the given case:- CA Old introduced his friend CA. Young to his friend and client Mr. Rich, the
owner of an Export House whose accounts has been audited by CA. Old for more than 15 years.
After a few days Mr. Rich approached CA. Young and offered a certification work which hitherto had
been done by CA. Old. Fees charged by CA. Young is also not less than fee charged by CA. Old.
(iv) Conclusion:- In view of above decision CA Young should ask the client to come through CA Old.
However, CA Young undertook the work without informing CA. Old. Thus, CA. Young is held guilty
under Clause (6) of Part I of the First Schedule to the Chartered Accountants Act, 1949.

CA Final Audit - By CA. Sarthak Niraj Jain 269


Professional Ethics
21. Soliciting Clients
M/s. SR & Associates is one of the three firms shortlisted by ARG Cooperative Bank for assignment of
Statutory Audit for the F.Y 2021-2022. Bank mailed the list of branches to the audit firms along with the
maximum fee per branch and asked them to submit the quotations. SR & Associates responded to the bank
and submitted their quotation. Comment with reference to the provisions of the Chartered Accountants Act,
1949 and schedules thereto. (July-2021-New)
Ans. (i) As per Clause:- (6) of Part I of First Schedule to the Chartered Accountants Act, 1949
A Chartered Accountant in practice is deemed to be guilty of professional misconduct if he solicits
clients or professional work:
 either directly or indirectly by circular, advertisement, personal communication or
 interview or by any other means
Provided that nothing herein contained shall be construed as preventing or prohibiting –
a) Any Chartered Accountant from applying or requesting for or inviting or securing professional
work from another chartered accountant in practice; or
b) A member from responding to tenders or enquiries issued by various users of professional
services or organizations from time to time and securing professional work as a consequence.
(ii) As per Council Guideline
 However, as per the guideline issued by the Council of the Institute of Chartered Accountants of
India, a member of the Institute in practice shall not respond to any tender issued by an
organization or user of professional services in areas of services which are exclusively reserved
for chartered accountants, such as audit and attestation services.
 However, such restriction shall not be applicable:
(a) where minimum fee of the assignment is prescribed in the tender document itself or

FAST
(b) where the areas are open to other professionals along with the Chartered Accountants.
(iii) In the given case of ARG Cooperative Bank: - Bank mailed the list of branches to the audit firms
along with maximum fees per branch, in response to which SR & Associates responded and

CA.Sarthak Niraj Jain


submitted their quotation.
(iv) Conclusion: -Keeping in view the facts, clause 6 and guideline issued by the council, it can be
concluded that SR & Associates is guilty of Professional misconduct.
Council Guidelines for Advertisement, 2008: Website
22. Soliciting Clients
Mr. Avi, a newly qualified Chartered Accountant, started his practice and sought clients through telephone
calls from his family and friends, almost all of them employed in one or the other retail trade business. One of
his friends Mr. Ravi gave him an idea to start online services and give stock certifications to traders with Cash
Credit Limits in Banks. Mr. Avi started a website with colorful catchy designs and shared the website address
on his all social media posts and stories and tagged 30 traders of his local community with the caption “Easy
Online Stock Certification Services”. Besides, Mr. Avi entered in an agreement with a Digital Marketer to give
him 5% commission on each service procured through him. Discuss if the actions of Mr. Avi are valid in the
light of the Professional Ethics and various pronouncements and guidelines issued by ICAI.
(RTP-May-2022)(Study Material)
Ans. (i) As per Clause:- (6) of Part I of First Schedule to the Chartered Accountants Act, 1949
A Chartered Accountant in practice is deemed to be guilty of professional misconduct if he solicits
clients or professional work
 either directly or indirectly by circular, advertisement, personal communication or
 interview or by any other means
(ii) As per Council Guidelines:- Mr. Avi is wrong in seeking clients through family and friends.
Creating a website is not a non - compliance provided it is in line with the guidelines issued by the
Institute in this regard. One of the guidelines is that the website should not be in push mode.
Further, mentioning of clients’ names is also prohibited as per the guidelines.

270 Ab Audit Hoga Sabse Scoring - By CA. Sarthak Niraj Jain


Professional Ethics
(iii) In the given situation:- Mr. Avi shared the website address on his all social media posts and stories
and tagged 30 traders of his local community with the caption “Easy Online Stock Certification
Services” mentioning his current clients as well. Further sharing of fees with Digital marketer would
also be in voiolation of Clause 2, Part 1, Schedule 1 to the CA Act, 1949.
(iv) Conclusion:- This is in complete contravention of the guidelines on website issued by the ICAI.
Thus, CA, Avi would be held guilty of professional misconduct under clause 6 of Part 1 of First
Schedule of the Chartered Accountants Act, 1949.
23. Posting of Particulars on Website
PQR and Associates, Chartered Accountants have their website and on the letterhead of the firm it is
mentioned that "Visit our website: PQR com". In the website the nature of assignments handled, names of
prominent clients and fees charged is also displayed.
Ans. (i) As per Clause :-The Council of the Institute had approved posting of particulars on website by
Chartered Accountants in practice under Clause (6) of Part I of First Schedule to the Chartered
Accountants Act, 1949 subject to the prescribed guidelines.
(ii) Guidelines:- The relevant guidelines in the context of the website hosted by M/s XYZ are:
• No restriction on the colors used in the website;
• The websites are run on a “pull” technology and not a “push” technology
• Names of clients and fees charged not to be given.
(iii) Conclusion:- In view of the above, M/s XYZ would have no restriction on the colors used in the website
but failed to satisfy the other two guidelines. Thus, the firm would be liable for professional
misconduct since it would amount to soliciting work by advertisement.
24. Posting Photograph on Website

FAST
CA. Intelligent, a Chartered Accountant in practice, provides part -time tutorship under the coaching
organization of the Institute. On 30th June, 2022, he was awarded ‘Best Faculty of the year’ as gratitude from
the Institute. Later on, CA. Intelligent posted his framed photograph on his website wherein he was receiving

Ans. CA.Sarthak Niraj Jain


the said award from the Institute.
(i) As per Clause:- (6) of Part I of First Schedule to the Chartered Accountants Act, 1949
A Chartered Accountant in practice is deemed to be guilty of professional misconduct if he solicits
clients or professional work:
• either directly or indirectly by circular, advertisement, personal communication or
• interview or by any other means
(ii) In the given case:- CA. Intelligent shared his framed photograph on website wherein he was
receiving ‘Best Faculty of the year’ award from the Institute.
(iii) As per guidelines:- In this context, it may be noted that according to the guidelines approved by the
Council of the Institute of Chartered Accountants of India,
• No photographs of any sort are permitted.
• Only display of passport size photograph is permitted.
(iv) Conclusion:- Therefore, CA. Intelligent is guilty of professional misconduct under Clause (6) of
Part I of the First Schedule to the Chartered Accountants Act, 1949.
Schedule – 1, Part – 1, Clause – 7
25. Advertisement of Professional Attainment
B, a Chartered Accountant in practice is a partner in 3 firms. While printing his personal letter heads, B gave
the names of all the firms in which he is a partner. (Study Material)
OR
Mr. Adnan, a Chartered Accountant in practice, is a partner of 4 firms. While printing his personal letter
heads, Mr. Adnan gave the names of all the firms in which he is a partner. Comment. (Nov-2015)

CA Final Audit - By CA. Sarthak Niraj Jain 271


Professional Ethics

Ans. (i) As per Clause:- clause (7) of Part I of First Schedule of the Chartered Accountant Act, 1949,
A CA in practice is deemed to be guilty of professional misconduct if he
(i) advertises his professional attainments or services or
(ii) uses any designation or expressions other than ‘Chartered Accountant” on professional
documents, visiting cards, letter heads or sign boards
(iii) Unless it be a degree of a university
• established by law in India or
• recognized by the Central Government or
• a title indicating membership of the ICAI or
• of any other institution that has been recognized by the Central Government or
• may be recognized by the council.
(ii) In the instant case:- Mr. Adnan, a Chartered Accountant in practice, is a partner of 4 firms and while
printing his personal letter heads, he gave the names of all the firms in which he is partner.
(iii) Conclusion:- Mr. Adnan is not guilty of any professional misconduct in the above case.
26. Use of Designation other than Chartered Accountant
Mr. SP, a Chartered Accountant, obtains registration as category IV Merchant Banker under the SEBI’s Rules
and Regulations and act as Advisor to a capital issue of MB Co. Ltd. He designates himself under the caption
“Merchant Banker” in client offer documents and ‘Advisor to issue’ in his own letterheads, visiting cards and
professional documents. (May-2015)
Ans. (i) As per Clause:- clause (7) of Part I of First Schedule of the Chartered Accountant Act, 1949,
A CA in practice is deemed to be guilty of professional misconduct if he
(i) advertises his professional attainments or services or

FAST
(ii) uses any designation or expressions other than ‘Chartered Accountant” on professional
documents, visiting cards, letter heads or sign boards
(iii) Unless it be a degree of a university

CA.Sarthak Niraj Jain




established by law in India or
recognized by the Central Government or
• a title indicating membership of the ICAI or
• of any other institution that has been recognized by the Central Government or
• may be recognized by the council.
(ii) As per Guidelines:-
• It may be noted that, in Client Companies’ offer documents and advertisements regarding
capital issue, name and address of the Chartered Accountant acting as Advisor or Consultant
to the Issue could be indicated under the caption “Advisor/ Consultant to the Issue”.
• Further, such members should not use the designation of either ‘Merchant Banker’ or
‘Advisor/Consultant to Issue’ in their own letterheads, visiting cards, professional
documents, etc.
(iii) In the given case:- Mr. SP, a Chartered Accountant, has obtained registration as category IV Merchant
banker and acted as advisor to a capital issue of MB Co. Ltd. He has designated himself under the
caption “Merchant Banker” in client offer documents and “advisor to issue’ in his own letterheads,
visiting cards and professional documents.
(iv) Conclusion:- Mr. SP shall be held guilty of professional misconduct as per Clause (7) of Part I of
First Schedule to the Chartered Accountants Act, 1949.
27. Using Designation of CA and Advocate Simultaneously
Mr. B, a practicing Chartered Accountant as well as a qualified lawyer, was permitted by the bar council to
practice as a lawyer also. He printed his visiting card where he mentioned his designation as Chartered
Accountant and Advocate.

272 Ab Audit Hoga Sabse Scoring - By CA. Sarthak Niraj Jain


Professional Ethics

Ans. (i) As per Clause:- clause (7) of Part I of First Schedule of the Chartered Accountant Act, 1949,
A CA in practice is deemed to be guilty of professional misconduct if he
(i) advertises his professional attainments or services or
(ii) uses any designation or expressions other than ‘Chartered Accountant” on professional
documents, visiting cards, letter heads or sign boards
(iii) Unless it be a degree of a university
• established by law in India or
• recognized by the Central Government or
• a title indicating membership of the ICAI or
• of any other institution that has been recognized by the Central Government or
• may be recognized by the council.
(ii) Interpretation of clause:-
• This clause prohibits advertising of professional attainments or services of a member.
• It also restrains a member from using any designation or expression other than that of a
Chartered Accountant in documents through which the professional attainments of the
member would come to the notice of the public.
(iii) Designation other than CA:-
• Members of the Institute in practice who are otherwise eligible may practice as advocates
subject to the permission of the Bar Council but in such case, they should not use
designation ‘chartered accountant in respect of the matters involving the practice as an
advocate.
• In respect of other matters they should use the designation ‘chartered accountant’ but they
should not use the designation ‘chartered accountant’ and ‘advocate’ simultaneously.

FAST
(iv) In the present case:-Mr. B has printed his visiting card where he mentioned his designation as
Chartered Accountant and Advocate which is prohibited under the above clause
(v) Conclusion:- Mr. B is guilty of professional misconduct.
28.
CA.Sarthak Niraj Jain
Mentioning 'Insolvency Professional' on Visiting Cards
A Chartered Accountant in practice, empanelled as an Insolvency Professional (IP) has mentioned the same
on his visiting cards, letter heads and other communications also. A person residing in his neighbor-hood has
filed a complaint for professional misconduct against the said member for such mention of IP. You are
required to examine the same with reference to The Chartered Accountants Act 1949.
(May-2019-Old) (Nov-2018)
Ans. (i) As per Clause:- Clause (7) of Part I of First Schedule of the Chartered Accountant Act, 1949,
A CA in practice is deemed to be guilty of professional misconduct if he
 advertises his professional attainments or services or
 uses any designation or expressions other than ‘Chartered Accountant” on professional
documents, visiting cards, letter heads or sign boards
 Unless it be a degree of a university
 established by law in India or
 recognized by the Central Government or
 a title indicating membership of the ICAI or
 of any other institution that has been recognized by the Central Government or
 May be recognized by the council.
(ii) In the present case:- A Chartered Accountant empanelled as IP (Insolvency Professional) can
mention 'Insolvency Professional' on his visiting cards, Letter heads and other communication,
as this is a title recognised by the Central Government in terms of Clause-7 of Part-1 of First
Schedule to the Chartered Accountants Act, 1949.
(iii) Conclusion:- Thus, complaint of neighbor is not enforceable/valid.

CA Final Audit - By CA. Sarthak Niraj Jain 273


Professional Ethics

29. Printing of Visiting Cards and using QR Code


Mr. M, a chartered accountant in practice, has printed visiting cards which besides other details also carries a
Quick Response (QR) code. The visiting card as well the QR code contains his name, office and residential
address, contact details, e-mail id and name of the firm’s website. Comment with reference to the Chartered
Accountants Act, 1949 and schedules thereto. (May-2017, MTP-Nov-2018, RTP-May-2019)
Ans. (i) As per Clause:- Clause (7) of Part I of First Schedule of the Chartered Accountant Act, 1949,
A CA in practice is deemed to be guilty of professional misconduct if he
 advertises his professional attainments or services or
 uses any designation or expressions other than ‘Chartered Accountant” on professional
documents, visiting cards, letter heads or sign boards
 Unless it be a degree of a university
 established by law in India or
 recognized by the Central Government or
 a title indicating membership of the ICAI or
 of any other institution that has been recognized by the Central Government or
 may be recognized by the council.
(ii) As per guidelines:- Issued under Clause 7, printing photograph on the visiting cards is not
permissible. However Quick Response Code may be printed on visiting card provided that the Code
does not contain information that is not otherwise permissible to be printed in their visiting card.
(iii) In the present case:- Mr. M, a chartered accountant in practice, has printed visiting cards which
besides other details also carries a Quick Response (QR) code. The visiting card as well the QR code
contains his name, office and residential address, contact details, e-mail id and name of the

FAST
firm’s website.
(iv) Conclusion:- No misconduct arises on part of Mr. M as information printed on visiting card and
information contained in QR Code are permissible.
30.
CA.Sarthak Niraj Jain
Printing of Visiting Cards
Comment on the following with reference to CA Act, 1949 and schedules thereto: A is a partner of 2 firms,
M/s ABC & Company and M/s A & Associates. For most of the audits handled by him, the appointment is in
the name of M/s ABC & Company. The visiting cards of A carries the names of both the firms.
Ans. (i) As per Clause:- Clause (7) of Part I of First Schedule of the Chartered Accountant Act, 1949,
A CA in practice is deemed to be guilty of professional misconduct if he
(i) advertises his professional attainments or services or
(ii) uses any designation or expressions other than ‘Chartered Accountant” on professional
documents, visiting cards, letter heads or sign boards
(iii) Unless it be a degree of a university
• established by law in India or
• recognized by the Central Government or
• a title indicating membership of the ICAI or
• of any other institution that has been recognized by the Central Government or
• may be recognized by the council.
(ii) As per guidelines:- There is no prohibition for printing names of all firms on the personal letter
heads in which a member holding certificate of practice is a partner.
(iii) In the present case:- A is a partner of 2 firms, M/s ABC & Company and M/s A & Associates. For
most of the audits handled by him, the appointment is in the name of M/s ABC & Company. The
visiting cards of A carries the names of both the firms.
(iv) Conclusion:- Mr. A is not guilty of any professional misconduct in the above case.

274 Ab Audit Hoga Sabse Scoring - By CA. Sarthak Niraj Jain


Professional Ethics

31. Using Designation Other Than a CA and Providing Details of Services Offered
Mr. Nigal, a Chartered Accountant in practice, delivered a speech in the national conference organized by the
Ministry of Textiles. While delivering the speech, he told to the audience that he is a management expert and
his firm provides services of taxation and audit at reasonable rates. He also requested the audience to
approach his firm of chartered accountants for these services and at the request of audience he also
distributed his business cards and telephone number of his firm to those in the audience. Comment.
(Study Material, MTP-May-2021, MTP-Nov-2019)
Ans. (i) As per Clause:- (6) of Part I of First Schedule to the Chartered Accountants Act, 1949
A Chartered Accountant in practice is deemed to be guilty of professional misconduct if he solicits
clients or professional work:
• either directly or indirectly by circular, advertisement, personal communication or
• interview or by any other means
Such a restraint has been put so that the members maintain their independence of judgment and
may be able to command respect from their prospective clients.
(ii) As per Provision:-Section 7 of the Chartered Accountants Act, 1949 read with Clause (7) of Part I
of the First Schedule to the said Act prohibits advertising of professional attainments or services
of a member. It also restrains a member from using any designation or expression other than that of a
chartered accountant in documents through which the professional attainments of the member
would come to the notice of the public.
(iii) As per Guidelines:- Issued under clauses 6 & 7 permits a practicing member to give lectures at
forums and may give their names and describe themselves as Chartered Accountants, but no
reference should be made, to the name and address or services of his firm.

FAST
(iv) In the present case:- Mr. Nigal uses the designation of "Management Expert" and also made
reference to the services provided by his firm of Chartered Accountants at reasonable rates and
distribute business cards to audience.

CA.Sarthak Niraj Jain


(v) Conclusion:- Mr. Nigal will be held guilty of professional misconduct under clauses 6 & 7 of Part I
of First Schedule to the CA Act 1949 due to solicitation of professional work and advertisement of
services rendered by his firm.
32. Communication with the Previous Auditor
Vineet & Associates have been offered Statutory Audit of TLP Ltd. As a part of ethical requirements of the
Institute of Chartered Accountants, CA V, partner of the firm, communicated with the previous auditor
enquiring as to whether any professional reason exists for which he should not accept the audit assignment.
Previous auditor informed that he issued a qualified report, so management is changing the auditor. Comment
with reference to the provisions of the Chartered Accountants Act, 1949 and schedules thereto as to whether
Vineet & Associates can accept the audit. (July-2021-New)
Ans. (i) As per Clause:- Clause 8 of Part I of First Schedule to the CA Act 1949
A chartered accountant in practice is deemed to be guilty of professional misconduct:
• If he accepts a position as auditor previously held by another chartered accountant or a certified
auditor who has been issued certificate under the Restricted Certificates Rules 1932
• Without first communicating with him in writing.
(ii) The professional reasons for not accepting an audit would be:
a. Non-compliance of the provisions of Sections 139 and 140 of the Companies Act, 2013 as
mentioned in Clause (9) of the Part - I of First Schedule to The Chartered Accountants Act, 1949; and
b. Non-payment of undisputed Audit Fees by auditees other than in case of Sick Units for carrying out
the Statutory Audit under the Companies Act, 2013 or various other statutes; and
c. Issuance of a qualified report.
There is no rule, written or unwritten, which would prevent an auditor from accepting the
appointment offered to him under the circumstance of Issuance of qualified report. However, before

CA Final Audit - By CA. Sarthak Niraj Jain 275


Professional Ethics
accepting the audit, he should ascertain the full facts of the case. For nothing will bring the profession
to disrepute so much as the knowledge amongst the public that if an auditor is found to be
“inconvenient” by the client, he could readily be replaced by another who would not displease the
client and this point cannot be too over-emphasised.
(iii) Conclusion
 From the above it can be concluded that Vineet & Associates may accept the audit of TLP Ltd if CA V
is satisfied that the attitude of the retiring auditor was not proper and justified.
 If, on the other hand, CA V feels that the retiring auditor had qualified the report for good and valid
reasons, he should refuse to accept the audit of TLP Ltd.
Schedule – 1, Part – 1, Clause – 9
33. Compliance of Statutory Requirements Before Accepting Appointment
CA Raja was appointed as the Auditor of Castle Ltd. for the year 2021-22. Since he declined to accept the
appointment, the Board of Directors appointed CA Rani as the auditor in the place of CA Raja, which was also
accepted by CA Rani. (Study Material) (May-2015)
OR
CA X was appointed as the Auditor of ABC Ltd. for 2021-22. Since he declined to accept the appointment, the
Board of Directors appointed CA Y as the auditor in the place of CA X, which was also accepted by CA Y.
Ans. (i) As per Clause:- Clause (9) of Part I of the First Schedule to the Chartered Accountants Act, 1949
Provides that a member in practice shall be deemed to be guilty of professional misconduct:
• If he accepts an appointment as auditor of a company
• Without first ascertaining from it whether the requirements of Section 139 and 140 read
with Section 141 of the Companies Act, 2013,

FASTIn respect of such appointment have been duly complied with.
(ii) As per Companies Act, 2013:-
• Board can appoint the auditor in the case of casual vacancy under Sections 139(8)(i) and

CA.Sarthak Niraj Jain


Section 139(6) of the Companies Act, 2013. The non -acceptance of appointment by CA. X does
not constitute a casual vacancy to be filled by the Board. In this case, it will be deemed that
no auditor was appointed in the AGM.
• As per Section 139(10) of the Companies Act, 2013:- When at any annual general meeting, no
auditor is appointed or re-appointed, the existing auditor shall continue to be the auditor
of the company. The appointment of the auditor by the Board is defective in law.
(iii) Conclusion:- Hence CA. Y is guilty of professional misconduct as per Clause (9) of the First
Schedule as he accepted the appointment without verification of statutory requirements.
34. Charging of Fees based on Percentage
Agarwal Pvt. Ltd. approached CA. Prem, a Chartered Accountant in practice, for debt recovery services. CA
Prem accepted the work and insisted for fees to be based on 2% of the debt recovered. (RTP-May-2020)
OR
PQR Pvt. Ltd. approached CA. Whai, a Chartered Accountant in Practice, for debt recovery services. CA Whai
accepted the work and insisted for fees to be based on 2% of the debt recovered. (May-2017)
OR
Ajanta Pvt. Ltd. approached CA. Jasmeet, a practicing Chartered Accountant since 1996, for recovery of debts
amounting ` 40 crores. CA Jasmeet accepted the work and requested to charge fees @ 2.5% of the debt
recovered. Later on, she raised a bill for debts recovered and charged ` 90 lacs for recovering 90% of the
debts
Ans. (i) As per Clause:- Clause (10) of Part I of First Schedule to the Chartered Accountants Act, 1949, A
Chartered Accountant in practice shall be deemed to be guilty of professional misconduct
• If he charges or offers to charge, accepts or offers to accept

276 Ab Audit Hoga Sabse Scoring - By CA. Sarthak Niraj Jain


Professional Ethics
• In respect of any professional employment fees
• Which are based on a percentage of profits or
• Which are contingent upon the findings, or results of such employment,
• Except as permitted under any regulations made under this Act.
(ii) Exemption:- The Council of the Institute has framed Regulation 192 which exempts debt recovery
services where fees may be based on a percentage of the debt recovered.
(iii) In the given case:- CA. Prem has insisted for fees to be based on percentage of the debt recovered
(which is exempted under Regulation 192).
(iv) Conclusion:- Hence, CA. Prem will not be held guilty for professional misconduct.
Schedule – 1, Part – 1, Clause – 11
35. Permission for Providing Private Tutorship
CA Ram is practicing in the field of financial management planning for over 12 years. He has gained expertise
in this domain over others. Mr. Ratan, a student of Chartered Accountancy course, is very much impressed
with the knowledge of CA. Ram. He approached CA. Ram to take guidance on some topics of financial
management subject related to his course. CA. Ram, on request, decided to spare some time and started
providing private tutorship to Mr. Ratan along with some other aspirants. However, he forgot to take specific
permission for such private tutorship from the Council. (Nov-2018)
OR
CA Raghu is practicing in the field of Income–Tax over a period of 12 years. He has gained experience in this
domain over others.Sam, a student of Chartered Accountancy Course is very much impressed with the
knowledge of CA Raghu. He approached CA Raghu to take guidance on some topics of Income–Tax related to
his course. CA Raghu, on request decided to spare time and started providing Private Tutorship to Sam and

Ans. (i)
FAST
some of his friends along with. However, he forgot to take specific permission from the ICAI, for such Private
Tutorship. Is CA Raghu, professionally liable for misconduct? (May-2016, RTP-May-2018)
As per Clause:- Clause (11) of Part I of the First Schedule to the CA Act, 1949

CA.Sarthak Niraj Jain





A CA in practice cannot engage in any business or occupation
Other than the profession of chartered accountants
Unless permitted by the Council so to engage.
(ii) As per regulation 190A of the Chartered Accountants Regulation, 1988
Provides that a Chartered Accountant in practice shall not engage in any other business or
occupation other than the profession of accountancy except with the permission granted in
accordance with a resolution of the Council. According to the same there is no specific permission
from the council would be necessary in the case of private tutorship.
(iii) In the given case:- CA. Ram has started providing private tutorship to Mr. Ratan along with some
other aspirants, without obtaining specific or prior approval of the Council.
(iv) Conclusion:- CA. Ram would not be held guilty of professional misconduct under Clause (11) of
Part I of the First Schedule to the Chartered Accountants Act, 1949.
36. Engaging into Business/Profession Other Than the Profession of CA
CA. Raj is a leading income tax practitioner and consultant for derivative products. He resides in Bangalore
Amendment near to the XYZ commodity stock exchange and does trading in commodity derivatives. Every day, he invests
nearly 50% of his time to settle the commodity transactions, though he has not taken any permission for this.
Is CA. Raj liable for professional misconduct? (May-2018)
OR
P, a Chartered Accountant holding Certificate of Practice, is a leading Income Tax Practitioner in Gurugram.
He is also trading in derivatives. Comment with reference to the Chartered Accountant Act, 1949 and
schedules thereto. (May-2017)
OR
CA. Z who is a leading Income Tax Practitioner and consultant in Jaipur is also trading in derivatives.

CA Final Audit - By CA. Sarthak Niraj Jain 277


Professional Ethics

Ans. (i) As per Clause:- Clause (11) of Part I of the First Schedule to the CA Act, 1949
• A CA in practice cannot engage in any business or occupation
• Other than the profession of chartered accountants
• Unless permitted by the Council so to engage.
(ii) Council General Guidelines:- The Council has granted general permission to the members to engage
in 12 specific occupations. In respect of all other occupations specific permission of the Institute is
necessary.
(iii) In the given case:- CA. Raj is engaged in the occupation of trading in derivatives which is not covered
under the general permission.
(iv) Conclusion:- Hence specific permission of the Institute has to be obtained otherwise he will be
deemed to be guilty of professional misconduct under Clause (11) of Part I of First Schedule of the Act.
[From Nov 2021 attempt Answer to change, if CA is trading in equity derivatives as ICAI has now generally
permitted CAiP to trade in derivatives hence CA.Z will not be guilty.]
37. Engaging into Business/Profession Other Than the Profession of CA
Mr. Kartik Aryan, a Chartered Accountant in practice has been appointed editor of a monthly journal which
analyses performance of the Stock Market and Mutual Fund Schemes.
Ans. (i) As per Clause:- Clause (11) of Part I of the First Schedule to the CA Act, 1949
• A CA in practice cannot engage in any business or occupation
• Other than the profession of chartered accountants
• Unless permitted by the Council so to engage.
(ii) Council General Guidelines:- General permission is granted under Regulation 190A for being
appointed as editor of professional journal.

FAST
(iii) In the given case:- Mr. Kartik Aryan has accepted the appointment as editor of a monthly journal
which analyses the stock performance.
(iv) Conclusion:- Clause 11 permits editorship of professional journals, but in the instant case journal

CA.Sarthak Niraj Jain


related to performance analysis of stock market and mutual fund schemes cannot be treated as
professional journal hence, Mr. Kartik Aryan would be held guilty of professional misconduct by
virtue of Clause 11 of Part I of First Schedule.
38. Engaging into Business/Profession Other Than the Profession of CA
CA. Moni is practicing since 2021 in the field of company audit. Due to her good practical knowledge, she was
offered editorship of a ‘Company Audit’ Journal which she accepted. However, she did not take any
permission from the Council regarding such editorship. (RTP-Nov-2020, MTP-May-2021)
Ans. (i) As per Clause:- Clause (11) of Part I of the First Schedule to the CA Act, 1949
• A CA in practice cannot engage in any business or occupation
• Other than the profession of chartered accountants
• Unless permitted by the Council so to engage.
(ii) Council General Guidelines:- General permission is granted under Regulation 190A for being
appointed as editor of professional journal.
(iii) In the given case:- CA Manav has accepted the appointment as editor of a “Company Audit”, which
is a professional journal.
(iv) Conclusion:- Clause 11 permits editorship of professional journals, hence no misconduct arises on
part of Mr. Manav.
39. CA R, a Chartered Accountant in practice is specializing in the field of Information Systems Audit. He is
considered to be one of the experts of this field because of his command over the subject. HKC Limited, a
Company engaged in rendering management consultancy offered him to appoint as its managing director. CA
R accepted the position of managing director without obtaining prior permission from the Institute. One of
his friends , CA S informed him that now he cannot retain full time certificate of practice, thus cannot do
attest function and train articled assistants. Comment with reference to the provisions of the Chartered
Accountants Act, 1949 and schedules thereto. (July-2021-New)

278 Ab Audit Hoga Sabse Scoring - By CA. Sarthak Niraj Jain


Professional Ethics

Ans. Business or occupation other than the profession


(i) Clause (11) of Part I of First Schedule:
• A Chartered Accountant in practice shall be deemed to be guilty of professional misconduct if he
engages in any business or occupation other than the profession of chartered accountant unless
permitted by the Council so to engage.
(ii) Council Guidelines & Decision:
 The Council decided to allow members in practice to hold the office of Managing Director,
Whole-time Director or Manager of a body corporate within the meaning of the Companies Act,
2013 provided that the body corporate is engaged exclusively in rendering Management
Consultancy and Other Services permitted by the Council in pursuant to Section 2(2)(iv) of the
Chartered Accountants Act, 1949 and complies with the conditions(s) as specified by the Council
from time to time in this regard.
 The name of the Management Consultancy Company is required to be approved by the
Institute and such Company has to be registered with the Institute. The members can retain full
time Certificate of Practice besides being the Managing Director, Whole-time Director or Manager
of such Management Consultancy Company.
 There will be no restriction on the quantum of the equity holding of the members, either
individually and/ or along with the relatives, in such Company.
 Such members shall be regarded as being in full- time practice and therefore can continue to do
attest function either in individual capacity or in Proprietorship/Partnership firm in which
capacity they practice and wherein they are also entitled to train articled/audit assistants.
(iii) Conclusion: - Thus, the action of CA R is valid.
40.
FAST
Engaging into a Business
C.A. Bahubali is Special Executive Magistrate. He also took over as the executive chairman of Software
Company on 1.4.2021. He is also a leading income tax practitioner and consultant for derivative products. He

CA.Sarthak Niraj Jain


resides in Chennai near to the ION commodity stock exchange and does trading in commodity derivatives.
Every day, he invests nearly 38% of his time to settle the commodity transactions. He has not taken any
permission for becoming Special Executive Magistrate. However, he has got special permission of Council of
ICAI for becoming executive chairman and for trading in commodity derivatives. Is C.A. Bahubali liable for
professional misconduct? Comment with reference to the Chartered Accountants Act, 1949, and Schedules
thereto. (MTP-Dec-2021)
Ans. (i) Clause (11) of Part I of First Schedule:
 A Chartered Accountant in practice shall be deemed to be guilty of professional misconduct if he
engages in any business or occupation other than the profession of chartered accountant unless
permitted by the Council so to engage.
 However, the Council has granted general permission to the members to engage in certain
specific occupation. In respect of all other occupations specific permission of the Institute is
necessary
(ii) Facts of the case :
 In this case, C.A. Bahubali is Special Executive Magistrate, engaged in the occupation of trading
in commodity derivatives and also took over as the Executive Chairman on 01.04.2021.
 In this context, it may be noted that the Special Executive Magistrate which is generally
permitted for Members of the Institute in practice,
 Further specific permission is required for holding the position of Executive Chairman and
getting engaged in the occupation of trading in commodity derivatives.
(iii) In the given situation: - C.A. Bahubali is acting as Special Executive Magistrate which is generally
permitted for Members of the Institute in practice. Further, He is engaged in the occupation of
trading in commodity derivatives which is not covered under the general permission. He also took

CA Final Audit - By CA. Sarthak Niraj Jain 279


Professional Ethics
over as the Executive Chairman for which specific permission is required. CA. Bahubali got the
permission for the same from the Council of ICAI.
(iv) Conclusion:
 Hence, CA. Bahubali is not guilty for acting as Special Executive Magistrate as it is covered
under the general permission.
 He is also not guilty for holding the position of Executive Chairman after getting specific
permission of the Institute.
 However, he is guilty of professional misconduct under Clause (11) of Part I of First Schedule
of Chartered Accountants Act, 1949 for getting engaged in the occupation of trading in
commodity derivatives which is not covered under the general permission.
Ideally, the solution should be CA is not guilty as he has also taken permission for trading in
commodities whereas the answer given is as if no permission is taken for trading in commodity
derivatives
41. Independence of Auditor & Acting as a Recovery Consultant
M/s SS limited is a partly owned subsidiary of M/s HH limited. For the upcoming financial year, M/s DD &
Co., Chartered Accountants, were appointed as the statutory auditors of SS limited. The CEO of the holding
company was impressed with the knowledge and experience of Mr. D, one of the partners of the firm and
hence, he offered Mr. D to take up the position of Director (not MD/ wholetime director) of HH limited. At the
same time, Mr. D’s friend approaches him with an assignment to act as a Recovery Consultant for a bank. Mr.
D is now confused whether to accept or reject the offers. He approaches you and seeks your advice on the
same. Advise what Mr. D about what he can do with the offers with reference to the Chartered Accountants
Act, 1949 and Schedules thereto. (MTP-May-2021)
Ans. (i)
FAST
As per Clause: - Clause (11) of Part I of the First Schedule to the CA Act, 1949
• A CA in practice cannot engage in any business or occupation
• Other than the profession of chartered accountants

CA.Sarthak Niraj Jain


• Unless permitted by the Council so to engage.
Exception: -Provided nothing contained herein shall disentitle a chartered accountant from being a
director of a company (not being MD or whole-time director) unless he or his partners is
interested in such company as auditor.
(ii) ESB clarification: - The Ethical Standards Board (ESB) noted that Public conscience is expected to be
ahead of law. Members, therefore, are expected to interpret the requirement as regards
independence much more strictly than what the law requires and should not place themselves in
positions which would either compromise or jeopardise their independence.
In the view of the above, the Board, via a clarification, decided that the auditor of a Subsidiary
company cannot be a Director of its Holding company, as it will affect the independence of the
auditor.
(iii) Council General Guidelines: - Council has granted general permission to the members to engage in
certain specific occupation. In respect of all other occupations specific permission of the Institute is
necessary. ‘acting as Recovery Consultant in the banking sector’ is covered under general
permission.
(iv) In the given situation: -M/s SS limited is a partly owned subsidiary of M/s HH limited. For the
upcoming financial year, M/s DD & Co., Chartered Accountants, were appointed as the statutory auditors
of SS limited. The CEO of the holding company was impressed with the knowledge and experience of Mr.
D, one of the partners of the firm and hence, he offered Mr. D to take up the position of Director (not
MD/ whole-time director) of HH limited. Further, Mr. D’s friend approached him for an assignment for
acting as a Recovery Consultant for a bank.
(v) Conclusion: - Therefore, in view of above in the given case, Mr. D should not accept the offer to be
appointed as director of HH Limited.However, he can accept the assignment offered by his friend
and can act as a recovery consultant for a bank

280 Ab Audit Hoga Sabse Scoring - By CA. Sarthak Niraj Jain


Professional Ethics

Schedule – 1, Part – 1, Clause – 12


42. Allowing a Member not being a Partner to Sign Certificate
CA. Smart, a practicing Chartered Accountant was on Europe tour between 15-9-21 and 25-9-21. On 18-9-21
a message was received from one of his clients requesting for a stock certificate to be produced to the bank
on or before 20-9-21. Due to urgency, CA. Smart directed his assistant, who is also a Chartered Accountant, to
sign and issue the stock certificate after due verification, on his behalf. (MTP-Dec-2021, Study Material)
Ans. (i) As per Clause:- Clause (12) of Part I of the First Schedule to the Chartered Accountants Act, 1949
A Chartered Accountant in practice is deemed to be guilty of professional misconduct
• If he allows a person not being a member of the Institute in practice or
• A member not being his partner to sign on his behalf or on behalf of his firm, any balance sheet,
profit and loss account, report or financial statements”.
(ii) In the given case:- CA. Smart allowed his assistant who is not a partner but a member of the
Institute of Chartered Accountants of India to sign stock certificate on his behalf
(iii) Conclusion:- CA. Smart is guilty of professional misconduct under Clause (12) of Part I of First
Schedule to the Chartered Accountants Act, 1949.
43. Delegation of Authority to the Employee
Mr. 'A' is a practicing Chartered Accountant working as proprietor of M/s A & Co. He went abroad for 3
months. He delegated the authority to Mr. 'Y' a Chartered Accountant his employee for taking care of routine
matters of his office. During his absence Mr. 'Y' has conducted the under mentioned jobs in the name of M/s A
& Co.
(i) He issued the audit queries to client which was raised during the course of audit.
(ii) He issued production certificate to a client under Central Excise Act, 1944.
(iii)
FAST
He attended the Income Tax proceedings for a client as authorized representative before Income Tax
Authorities.
Please comment on eligibility of Mr. 'Y' for conducting such jobs in name of M/s A & Co. and liability of Mr. 'A'

Ans.
CA.Sarthak Niraj Jain
under the Chartered Accountants Act, 1949.
(i)
(Study Material, MTP-May-2018, RTP-Nov-2019)
As per Clause:- Clause (12) of Part I of the First Schedule to the Chartered Accountants Act, 1949
A Chartered Accountant in practice is deemed to be guilty of professional misconduct
• If he allows a person not being a member of the Institute in practice or
• A member not being his partner to sign on his behalf or on behalf of his firm, any balance sheet,
profit and loss account, report or financial statements”.
(ii) In the given case:- CA. ‘A’ proprietor of M/s A & Co., went to abroad and delegated the authority to
another Chartered Accountant Mr. Y, his employee, for taking care of routine matters of his office who
is not a partner but a member of the Institute of Chartered Accountants.
(iii) As per Council General Guidelines:- The Council has clarified that the power to sign routine
documents on which a professional opinion or authentication is not required to be expressed may
be delegated in the following instances and such delegation will not attract provisions of this
clause like issue of audit queries during the course of audit, asking for information or issue of
questionnaire, attending to routing matters in tax practice, subject to provisions of Section 288 of
Income Tax Act etc.
 In the given case, Mr. ‘Y’, a chartered accountant being employee of M/s A & Co. has issued audit
queries which were raised during the course of audit. Here “Y” is right in issuing the query, since
the same falls under routine work which can be delegated by the auditor. Therefore, there is no
misconduct in this case as per Clause (12) of Part I of First schedule to the Act.
 Further, issuance of production certificate to a client under Central Excise Act, 1944 by Mr. “Y”
being an employee of M/s A & Co. (an audit firm), is not a routine work and it is outside his
authorities. Thus, CA. ‘A’ is guilty of professional misconduct under Clause (12) of Part I of First
Schedule of the Chartered Accountants Act, 1949.

CA Final Audit - By CA. Sarthak Niraj Jain 281


Professional Ethics
 In this instance, Mr. “Y”, CA employee of the audit firm M/s A & Co. has attended the Income tax
proceedings for a client as authorized representative before Income Tax Authorities. Since the
council has allowed the delegation of such work, the chartered accountant employee can attend
to routine matter in tax practice as decided by the council, subject to provisions of Section 288
of the Income Tax Act. Therefore, there is no misconduct in this case as per Clause (12) of Part I
of First schedule to the Act.
44. Delegation of Authority to the Employee
Mr. ‘K’, a practicing Chartered Accountant is the proprietor of M/s K & Co. since 1995. He went abroad in the
month of December 2021. He delegated the authority to Mr. ‘Y’ a Chartered Accountant, his employee for
taking care of the important matters of his office. During his absence Mr. ‘Y’ has conducted the under
mentioned jobs in the name of M/s K & Co.
(a) He issued Net worth certificate to a client for furnishing to a Bank.
(b) He attended the GST proceedings for a client as authorized representative before GST Authorities.
Please comment on eligibility of Mr. ‘Y’ for conducting such jobs in name of M/s K & Co. and liability of
Mr. ‘K’ under the Chartered Accountants Act, 1949. (Nov-2019-New)
Ans. (i) As per Clause:- Clause (12) of Part I of the First Schedule to the Chartered Accountants Act, 1949
A Chartered Accountant in practice is deemed to be guilty of professional misconduct
• If he allows a person not being a member of the Institute in practice or
• A member not being his partner to sign on his behalf or on behalf of his firm, any balance sheet,
profit and loss account, report or financial statements”.
(ii) In the given case:- CA ‘K’ proprietor of M/s K & Co., went abroad and delegated the authority to
another Chartered Accountant Mr. Y, his employee, for taking care of routine matters of his office.

FAST
(iii) As per Council General Guidelines:- The Council has clarified that the power to sign routine
documents on which a professional opinion or authentication is not required to be expressed may
be delegated in the following instances and such delegation will not attract provisions of this

CA.Sarthak Niraj Jain


clause like issue of audit queries during the course of audit, asking for information or issue of
questionnaire, attending to routing matters in tax practice, subject to provisions of Section 288 of
Income Tax Act etc.
(iv) Conclusion:-
(a) Issuance of Net worth Certificate to a client for furnishing to Bank by Mr. “Y” is not a routine
work and it is outside his authorities. Thus, CA ‘K’ is guilty of professional misconduct under
clause 12 of Part I of First Schedule of the Chartered Accountants Act, 1949.
(b) Attending GST proceedings for a client as authorized representative before GST Authorities falls
under routine work, hence Mr. Y, the employee of M/s K & Co. can attend to routine matter in tax
practice. Therefore, there is no misconduct in this case.
First Schedule, Part II–Professional Misconduct in relation to Members in Service
Schedule – 1, Part – 2, Clause – 2
45. Referral Fee from Lawyer
Mr. 'C', a Chartered Accountant holds a certificate of practice while in employment also, recommends a
particular lawyer to his employer in respect of a case. The lawyer, out of the professional fee received from
employer paid a particular sum as referral fee to Mr. 'C'. (Study Material)
Ans. (i) As per Clause:- Clause (2) of Part II of First Schedule of the Chartered Accountant Act, 1949
A member of the Institute(other than a member in practice) shall be guilty of professional misconduct,
• If he being an employee of any company, firm or person accepts or agrees to accept
• Any part of fee, profits or gains from a lawyer, a chartered accountant or broker
• Engaged by such company, firm or person or agent or customer of such company, firm or
person by way of commission or gratification.

282 Ab Audit Hoga Sabse Scoring - By CA. Sarthak Niraj Jain


Professional Ethics
(ii) In the present case:- Mr. C who beside holding a certificate of practice, is also an employee and by
referring a lawyer to the company in respect o f a case, he receives a particular sum as referral fee from
the lawyer out of his professional fee.
(iii) Conclusion:- Mr. C is guilty of professional misconduct by virtue of Clause (2) of Part II of First
schedule.
First Schedule, Part III – Professional Misconduct in Relation to Members Generally
Schedule – 1, Part – 3, Clause – 2
46. Failed to Supply Information
XYZ Associates, a Chartered Accountants Firm is having a relationship with a multi- national accounting firm
in India. The ICAI required that all firms having networking relationship with any other entity need to
furnish information online within the stipulated time. XYZ Associates failed to respond. Comment on this
with reference to Professional misconduct, if any. (Nov-2018)
Ans. Failed to Supply Information Called For:
(i) As per Clause:- Clause (2) of Part III of First Schedule to the Chartered Accountants Act, 1949
• A member shall be held guilty if a Chartered Accountant, in practice or not,
• Does not supply the information called for, or
• Does not comply with the requirements asked for, by the Institute, Council or any of its
Committees, Director (Discipline), Board of Discipline, Disciplinary Committee, Quality Review
Board or the Appellate Authority;
(ii) In the given case:- Mr. XYZ Associates, a chartered accountant firm is failed to furnish the
information of its relationship with multi-national accounting firm in India. The ICAI required this
information to be submitted online within the stipulated time. XYZ Associates failed to respond and

FAST
submit the required information.
(iii) Conclusion:- Therefore, XYZ Associates is held guilty of professional misconduct as per Clause (2)
of Part III of the First Schedule to the Chartered Accountants Act, 1949.

47.
CA.Sarthak Niraj Jain
First Schedule, Part IV - Other Misconduct in relation to Members Generally
Schedule – 1, Part – 4, Clause – 1 & 2
Engaging Articled Assistance for Personal Work
CA Kumar who is contesting Central Council Elections of Institute, engages his Articled Assistant for his
election campaigning promising him that he will come in contact with influential people which will help to
enhance his career after completion of his training period. (RTP-May-2022)(RTP-Nov-2018, Nov-2019)
Ans. (i) As per Part:- Part IV of the First Schedule to the Chartered Accountants Act,
A member of the Institute, whether in practice or not, shall be deemed to be guilty of other
misconduct, if he
(1) is held guilty by any civil or criminal court for an offence which is punishable with
imprisonment for a term not exceeding six months;
(2) In the opinion of the Council, brings disrepute to the profession or the Institute as a result of
his action whether or not related to his professional work.
• A Chartered Accountant is expected to maintain the highest standards of integrity even
in his personal affairs and any deviation from these standards, even in his non-
professional work, would expose him to disciplinary action.
• A member may be found guilty of “Other Misconduct”, as per clause 2, if he uses the
services of articles for personal work.
(ii) In the given case:- CA Kumar has engaged his Articled Assistant for his own election campaigning for
the central Council elections of ICAI.
(iii) Conclusion:-CA. Kumar would be guilty of Other Misconduct under Part IV of First Schedule and
liable to disciplinary action under Section 21.

CA Final Audit - By CA. Sarthak Niraj Jain 283


Professional Ethics

48. Bringing Disrepute to Profession


YKS & Co., a proprietary firm of Chartered Accountants was appointed as concurrent auditor of a bank. YKS
used his influence for getting some cheques purchased and thereafter failed to repay the loan/overdraft.
(Study Material, MTP-May-2019)
Ans. (i) As per Clause:- Clause (2)of Part IV of First Schedule to the Chartered Accountants Act, 1949,
A member of the Institute, whether in practice or not, shall be deemed to be guilty of other misconduct,
• If he in the opinion of the Council, brings disrepute
• To the profession or the Institute as a result of his action
• Whether or not related to his professional work.
• Here the Chartered Accountant is expected to maintain the highest standards and integrity
even in his personal affairs and any deviation from these standards calls for disciplinary
action.
(ii) In the present case:- YKS & Co, being a concurrent auditor used his position to obtain the funds and
failed to repay the same to the bank. This brings disrepute to the profession of a Chartered
Accountant. This act of YKS & Co is not pardonable.
(iii) Conclusion:- YKS & Co will be held guilty of other misconduct under Clause (2) of Part IV of First
Schedule to the Chartered Accountants Act, 1949.
49. Bringing Disrepute to the Profession
Mr. R, a Chartered Accountant in practice approached Manager of a Nationalised Bank for a loan of `25 lakhs.
He has also informed the Manager that if the loan is sanctioned, the Income Tax return of the Manager and
staff will be filed without charging any fees, as quid Pro quo for the loan sanctioned.
Ans. (i) As per Clause:- Clause (2)of Part IV of First Schedule to the Chartered Accountants Act, 1949,


FAST
A member of the Institute, whether in practice or not, shall be deemed to be guilty of other misconduct,
• If he in the opinion of the Council, brings disrepute
To the profession or the Institute as a result of his action

CA.Sarthak Niraj Jain


• Whether or not related to his professional work.
• Here the Chartered Accountant is expected to maintain the highest standards and integrity even
in his personal affairs and any deviation from these standards calls for disciplinary action.
(ii) In the present case:- The action of Mr. R, a Chartered Accountant in practice offering free service in
return to sanction of loan brings disrepute to the profession of a Chartered Accountant.
(iii) Conclusion:- Mr. R will be held guilty of other misconduct under Clause (2) of Part IV of the First
Schedule to the Chartered Accountants Act, 1949.
50. Disrepute to the Profession
Ms. Preeto, a CA, had an account with a bank. The normal balance in this account remained at a level below
`5,000. The bank inadvertently credited this account with a cheque of `2,70,000 belonging to another
account holder. When CA. Preeto came to know about this she withdrew the amount of `2,75,000 and closed
the bank account. After 1 year the bank noticed the mistake and claimed `2,75,000 with interest. CA. Preeto
contested this claim. Can the bank approach the Institute of Chartered Accountants of India for disciplinary
action against CA. Preeto? (RTP-Nov-2020)
Ans. (i) As per Clause:- Clause (2)of Part IV of First Schedule to the Chartered Accountants Act, 1949,
A member of the Institute, whether in practice or not, shall be deemed to be guilty of other misconduct,
• If he in the opinion of the Council, brings disrepute
• To the profession or the Institute as a result of his action
• Whether or not related to his professional work.
• Here the Chartered Accountant is expected to maintain the highest standards and integrity even
in his personal affairs and any deviation from these standards calls for disciplinary action.

284 Ab Audit Hoga Sabse Scoring - By CA. Sarthak Niraj Jain


Professional Ethics
(ii) In the instant case: - CA. Preeto, a CA, had an account with a bank from which she withdrew the
amount of ` 2,75,000 and closed the account. This amount of ` 2,75,000 was pertaining to ` 5,000
minimum balance and ` 2,70,000 belonging to other account holder and inadvertently credited to
his account by the bank.
(iii) Conclusion : -The said act of CA. Preeto to withdraw the money which does not belongs to her will
bring disrepute to the profession. Hence under this clause the bank can file a suitable complaint
under Clause 2 of Part IV of First Schedule of the Chartered Accountant Act, 1949 with the Institute
of Chartered Accountants of India.
51. Grossly Negligent and Bringing Disrepute to the Institute
Mr. Yuvi, a Chartered Accountant in practice, is the auditor of Prime Ltd. He advised the Managing Director of
the company to include ‘orders under negotiation’ in sales, to reflect higher profit and better financial
position for obtaining bank loans in future. Mr. Yuvi, thereafter, gave clean reports on the balance sheet
prepared accordingly without examining the accounts. Comment with reference to the Chartered
Accountants Act, 1949, and Schedules thereto. (MTP-May-2020)
OR
Mr. Brainy, a Chartered Accountant in practice, is the auditor of Fair Ltd. He advised the Managing Director of
the company to include ‘orders under negotiation’ in sales, to reflect higher profit and better financial
position for obtaining bank loans in future. Mr. Brainy, thereafter, gave clean reports on the balance sheet
prepared accordingly without examining the accounts.
Ans. (i) As per Clause:- Clause (7) of Part I of the Second Schedule to the Chartered Accountants Act, 1949
states that A Chartered Accountant in practice shall be deemed to be guilty of professional misconduct
• If he does not exercise due diligence, or

FAST is grossly negligent in the conduct of his professional duties.
(ii) As per Clause:- Clause (2)of Part IV of First Schedule to the Chartered Accountants Act, 1949,
A member of the Institute, whether in practice or not, shall be deemed to be guilty of other misconduct,

CA.Sarthak Niraj Jain





If he in the opinion of the Council, brings disrepute
To the profession or the Institute as a result of his action
Whether or not related to his professional work.
(iii) In the given case:- Mr. Yuvi, a Chartered Accountant in practice, is grossly negligence in conduct
of his professional duties by issuing clean reports on the balance sheet without examining the
accounts. Further, he has also brought disrepute to the profession by advising unethical practice
to the managing director of the company.
(iv) Conclusion:- Mr. Yuvi will be held guilty for professional and other misconduct under
abovementioned Clauses to the Chartered Accountants Act, 1949.
52. Bringing Disrepute to Profession
Mr. P, a practicing Chartered Accountant did not reply within a reasonable time and without any cause to the
letter received from the local police station, a public authority, soliciting his suggestions as regards some
non-professional work. Comment with reference to Chartered Accountant Act, 1949. (Jan-2021-Old)
Ans. (i) As per Clause:- Clause (2)of Part IV of First Schedule to the Chartered Accountants Act, 1949,
A member of the Institute, whether in practice or not, shall be deemed to be guilty of other misconduct,
• If he in the opinion of the Council, brings disrepute
• To the profession or the Institute as a result of his action
• Whether or not related to his professional work.
• Here the Chartered Accountant is expected to maintain the highest standards and integrity
even in his personal affairs and any deviation from these standards calls for disciplinary
action.
(ii) “Other Misconduct”Includes Non-replying within a reasonable time and without a good cause to
the letter of the public authorities.

CA Final Audit - By CA. Sarthak Niraj Jain 285


Professional Ethics
(iii) In the present case:- Mr. P, a practicing Chartered Accountant did not reply within a reasonable time
and without any cause to the letter received from the local Police Station, a public authority, soliciting
his suggestions as regards some non-professional work.
(iv) Conclusion:- Mr. P will be held guilty of other misconduct under Clause (2) of Part IV of First
Schedule to the Chartered Accountants Act, 1949.
53. Bringing Disrepute to Profession
P, a Chartered Accountant availed a loan against his securities held as investments from a nationalised bank.
He issued 2 cheques towards repayment of the said loan. Both she cheques were returned unpaid by the
bank with the remark “Refer to Drawer”. Comment with reference to the Chartered Accountants Act, 1949 as
amended by the Chartered Accountants (Amendment) Act, 2006 and schedules thereto. (Jan-2021-Old)
Ans. (i) As per Clause:- Clause (2) of Part IV of First Schedule to the Chartered Accountants Act, 1949,
A member of the Institute, whether in practice or not, shall be deemed to be guilty of other misconduct,
• If he in the opinion of the Council, brings disrepute
• To the profession or the Institute as a result of his action
• Whether or not related to his professional work.
• Here the Chartered Accountant is expected to maintain the highest standards and integrity
even in his personal affairs and any deviation from these standards calls for disciplinary
action.

(ii) As per Provision:- A member is liable to disciplinary action under Section 21 of the Chartered
Accountants Act, if he is found guilty of any professional or “Other Misconduct”.

(iii) In the present case- The cheque was dishonoured with the remark “refer to drawer”. However, such

FAST
dishonour need not necessarily be only due to insufficiency of funds.

(iv) Conclusion:- If it is proved that the cheques were dishonoured due to insufficiency of funds, the

CA.Sarthak Niraj Jain


CA P would be held guilty of “other misconduct”.
Second Schedule, Part I “Professional Misconduct in relation to Members in Practice
Schedule – 2, Part – 1, Clause – 1
54. Disclosure of Confidential Information to Tax Authorities
Give your comments with reference to Chartered Accountants Act, 1949 and schedules thereto: Mr. C, a
practicing Chartered Accountant, in the course of the audit of a listed company discovered serious violations
of the provisions of the companies Act, 2013, informed the registrar of companies out of public interest.
Ans. (i) As per Clause:- Clause (1) of Part I of the Second Schedule to the Chartered Accountants Act, 1949
deals with the professional misconduct relating to:
• The disclosure of information by a chartered accountant in practice
• Relating to the business of his clients to any person other than his client
• Without the consent of his client or
• Otherwise than as required by any law for the time being in force would amount to breach of
confidence.
(ii) Provision of Company Law:- Section 143(12) of Companies Act 2013 requires that if an auditor of
a company, in the course of the performance of his duties as auditor, has reason to believe that an
offence involving fraud is being or has been committed against the company by officers or
employees of the company, he shall immediately report the matter to the Central Government within
60 days of his knowledge and after following the prescribed procedure.
(iii) In the given case:- Mr. C has disclosed serious violations of the provisions of the Companies Act,
2013 to Registrar of Companies without the consent of the client under the impression that it
would be in public interest. It is not clear from the questions whether the violations of provisions
involve fraud being committed by officers or employees of the company.
(iv) Conclusion:- Mr. C will be guilty of professional misconduct covered by clause 1 of Part I of
Second Schedule to the Chartered Accountants Act, 1949.

286 Ab Audit Hoga Sabse Scoring - By CA. Sarthak Niraj Jain


Professional Ethics

Schedule – 2, Part – 1, Clause – 3


55. Certification of Financial Forecast
Mr. 'E', a practicing Chartered Accountant, was requested by one of his client to prepare a projection for next
five years and also a report on the same. Mr. 'E' after h prepared the same stated in his report ‘The sources of
information, the basis of forecasts and also the major assumptions made in arriving at the forecasts. He also
stated that he does not vouch for the accuracy of the forecasts.
OR
CA. Kalpana, a practicing Chartered Accountant, certifies a financial forecast of Surya Ltd. one of her clients,
which was forwarded to the company's bank based on which the bank sanctioned a loan amounting ` 3.5
crore to the company. CA. Kalpana, however, mentioned in the report that her responsibility is to examine
the evidence supporting the assumptions and other information in the Prospective financial information, her
responsibility does not include verification of the accuracy of the projections, therefore, she does not vouch
for the accuracy of the same.
Ans. (i) As per Clause: Clause (3) Part I of Second Schedule to the Chartered Accountants Act, 1949
A Chartered Accountant in practice shall be deemed to be guilty of professional misconduct,
• If he permits his name or the name of his firm
• To be used in connection with an estimate of earnings
• Contingent upon future transactions
• In a manner which may lead to the belief that he vouches for the accuracy of the forecast.
(ii) As per Standards:- SAE 3400 “The Examination of Prospective Financial Information”,
Provides that the management is responsible for the preparation and presentation of the
prospective financial information, including the identification and disclosure of the sources of

FAST
information, the basis of forecasts and the underlying assumptions.
(iii) In the instant case:- Mr. E after having prepared the projections for next five years stated in his
report, “the sources of information, the basis of forecasts and also the major assumptions made in

CA.Sarthak Niraj Jain


arriving at the forecasts.” He also stated that he does not vouch for the accuracy of the forecasts
(iv) Conclusion:- There is no violation of the Chartered Accountants Act, 1949 and its Regulations.
Schedule – 2, Part – 1, Clause – 4
56. Evaluation of Cost of Products
A firm of Chartered Accountants was appointed by a company to evaluate the costs of the various products
manufactured by it for its information system. One of the partners of the firm was a Non-Executive Director
of the company. (Nov-2020-Old)
OR
AP & Co., a firm of Chartered Accountants, was appointed by D Ltd., to evaluate the cost of a new product
manufactured by it for their information system and fixation of fair market price. Partner 'P' of the CA firm is
a non-executive director of the Company. Comment with reference to Chartered Accountants Act, 1949 and
Regulations there to. (May-2018-Old)
Ans. (i) As per Clause: Clause (4) of Part I of the Second Schedule to Chartered Accountants Act, 1949,
states that:-
• Expressing an opinion on financial statements of any business or enterprise
• In which he, his firm or a partner in his firm
• Has a substantial interest would constitute misconduct.
(ii) As per Council General Guidelines
The Council of the Institute of Chartered Accountants of India has stated that in cases where a
member of the Institute is a director of a company, or the firm in which the said member is a
partner, should not express any opinion on its financial statements.
(iii) Facts of the case: - The firm has been retained to evaluate the cost of products manufactured by it
for its information system. It is a part of management consultancy service of the firm and moreover
its partner was on the Board.
CA Final Audit - By CA. Sarthak Niraj Jain 287
Professional Ethics
(v) Conclusion: - The firm can perform this assignment and it will not constitute misconduct.
However, the firm while accepting the position as auditor in future would have to consider whether it
would be possible to act in independent manner and express opinion on financial statements.
57. Appointment of Auditor in case of Relative Holding Substantial Interest
Mr. A has been appointed statutory auditor of a private limited company where his spouse’s sister's husband
is having 75% ownership.
Ans. (i) As per Clause: Clause (4) of Part I of the Second Schedule to Chartered Accountants Act, 1949,
states that:-
• Expressing an opinion on financial statements of any business or enterprise
• In which he, his firm or a partner in his firm
• Has a substantial interest would constitute misconduct.
(ii) As per Council General Guidelines:-
A member of the Institute shall not express his opinion on financial statements of any business or
enterprise in which one or more persons, who are his “relatives”
Definition: - The expression “relative”, in relation to a member, means the husband, wife, brother or sister
or any lineal ascendant or descendant of that member;
(iii) As per Standards:- Spouses’ sisters’ husband is not covered in the definition of the term relative.
(iv) In the given case:- Mr. A, has been appointed as statutory auditor of a private limited company where
his spouses’ sisters’ husband is having 75 % ownership i.e. substantial interest.
(v) Conclusion:- Appointment of Mr. A as statutory auditor in such company would not amount to
professional misconduct as per Clause (4) of Part I of Second Schedule to the Chartered
Accountants Act, 1949.
58.
FAST
Appointment of Auditor in case of Director Simplicitor
Mr. D, a practicing CA, is appointed as a Director Simplicitor in XYZ Pvt. Ltd. After one year of appointment,
Mr. D resigned as the Director and accepted the Statutory Auditor position of the company. Is Mr. D right in

Ans.
CA.Sarthak Niraj Jain
accepting the auditor position?
(i)
(Study Material)(MTP-May-2021)
As per Clause: Clause (4) of Part I of the Second Schedule to Chartered Accountants Act, 1949,
states that:-
• Expressing an opinion on financial statements of any business or enterprise
• In which he, his firm or a partner in his firm
• Has a substantial interest would constitute misconduct.
(ii) As per Companies Act 2013:-
Section 141 of the Companies Act, 2013 specifically prohibits a member from auditing the
accounts of a company in which he is an officer or employee. Although the provisions of the
aforesaid section are not specifically applicable in the context of audits performed under other
statutes, e.g. tax audit, yet the underlying principle of independence of mind is equally applicable in
those situations also.
(iii) Council’s view:- The Council’s views are clarified in the following situations. As per the clarifications
issued by the Council, a member shall not accept the assignment of audit of a Company for a period
of two years from the date of completion of his tenure as Director, or resignation as Director of the
said Company.
(iv) In the given case:- Mr. D, a practicing CA, is appointed as a Director Simplicitor in XYZ Pvt. Ltd.
After one year of appointment, Mr. D resigned as the Director and accepted the Statutory Auditor
position of the company.
(v) Conclusion:- In view of above provisions Mr. D cannot accept the Directorship of the company until
the completion of two years after his resignation. Thus, CA, D would be held guilty of professional
misconduct under clause 4 of Part 1 of Second Schedule of the Chartered Accountants Act, 1949.

288 Ab Audit Hoga Sabse Scoring - By CA. Sarthak Niraj Jain


Professional Ethics

59. Violation of Provision & Punishment


The Director (Discipline) of the ICAI received information of alleged misconduct against Mr. Jayprakash, the
proprietor of JP & Associates, as follows:-
(i) Audit of a college was accepted by JP & Associates in which Mr. Jayprakash is working as a part-time
lecturer and also, he had not taken permission of the ICAI for working as a part-time lecturer in the
college.
(ii) An event relating to Corporate Social Responsibility was sponsored by JP & Associates, whereby in the
sponsorship banner, name of Mr. Jayprakash as ‘CA Jayprakash, Proprietor, JP & Associates’ was
mentioned.
On the basis of above information and along with certain evidence against Mr. Jayprakash, he was found
guilty and so he was reprimanded and a fine of ` 1 lakh was imposed by an order passed against him dated
12th July, 2020. Against the said order, Mr. Jayprakash preferred an appeal with the Appellate Authority on
17th August, 2020 by submitting a statement of appeal along with the application form of appeal. During such
appellate proceedings, it was discovered that the said statement of appeal contained some facts which were
false to which Mr. Jayprakash admitted it to be false and apologized for it.
(a) Mr. Jayprakash has violated which of the provisions of the Chartered Accountants Act, 1949?
(b) Before which authority, the matter of Mr. Jayprakash would have been placed and what maximum
punishment could have been imposed on him by the said authority in accordance with the Chartered
Accountant Act, 1949? (RTP-Dec-2021)
Ans. Professional misconducts in various cases
(a) Mr. Jayprakash has violated following provisions of the Chartered Accountants Act, 1949:
(i) Clause: - As per Clause (4) of Part I of the Second Schedule to the Chartered Accountants Act,

FAST1949, a Chartered Accountant in practice shall be deemed to be guilty of professional misconduct,


if he expresses his opinion on financial statements of any business or enterprise in which he,
his firm, or a partner in his firm has a substantial interest.

CA.Sarthak Niraj Jain


Council guidelines : -In this connection, as per the decision of the Council of the ICAI, a Chartered
Accountant should not by himself or in his firm name accept the audit of a college, if he is
working as a part-time lecturer in the college.
Conclusion: -Thus, by accepting audit of a college in which he is working as a part-time lecturer,
Mr. Jayprakash has violated the restriction imposed under Clause (4) of Part I of the Second
Schedule to the Chartered Accountants Act, 1949.
(ii) Clause: - As per Clause (11) of Part I of the First Schedule to the Chartered Accountants Act,
1949, a Chartered Accountant in practice shall be deemed to be guilty of professional
misconduct, if he engages in any business or occupation other than the profession of chartered
accountant unless permitted by the Council so to engage.
Council guidelines : -Members of the Institute in practice may engage in a part-time or full-
time tutorship under any educational institution other than the coaching organization of the
Institute, after obtaining the specific and prior approval of the Council in each case.
Conclusion: -Mr. Jayprakash had not taken permission of the ICAI for working as a part-time
lecturer in the college and so has violated the restriction imposed under Clause (11) of Part I of the
First Schedule to the Chartered Accountants Act, 1949.
(iii) Clause: - As per Clause (6) of Part I of the First Schedule to the Chartered Accountants Act,
1949, a Chartered Accountant in practice shall be deemed to be guilty of professional misconduct,
if he solicits clients or professional work either directly or indirectly by circular,
advertisement, personal communication or interview or by any other means.
In this connection, members sponsoring activities relating to Corporate Social Responsibility
may mention their individual name with the prefix “CA”. However, mentioning a firm’s name
or CA Logo is not permitted.
In the given case: -An event relating to Corporate Social Responsibility was sponsored by JP &

CA Final Audit - By CA. Sarthak Niraj Jain 289


Professional Ethics
Associates, whereby in the sponsorship banner, name of Mr. Jayprakash as ‘CA Jayprakash,
Proprietor, JP & Associates’ was mentioned.
Conclusion: -Thus, firm’s name was mentioned which is not allowed and thus, Mr. Jayprakash
has violated the restriction imposed under Clause (6) of Part I of the First Schedule to the
Chartered Accountants Act, 1949.
(iv) Clause: -As per Clause (3) of Part II of the Second Schedule to the Chartered Accountants Act,
1949, a member of the ICAI shall be deemed to be guilty of professional misconduct, if he includes
in any information, statement, return or form to be submitted to the Institute, Council or any of
its Committees, Director (Discipline), Board of Discipline, Disciplinary Committee, Quality Review
Board or the Appellate Authority, any particulars knowing them to be false.
Conclusion: -Mr. Jayprakash in the statement of appeal submitted with the Appellate Authority
mentioned some facts knowing them to be false and thus, he has violated the restriction
imposed under Clause (3) of Part II of the Second Schedule to the Chartered Accountants Act,
1949.
(b) As Mr. Jayprakash has been alleged of misconduct falling in First as well as Second Schedule, so
the matter would be placed before the Disciplinary Committee.
The maximum punishment which could have been imposed on him by the said authority would be:-
(i) Reprimanding the member.
(ii) Removing name of the member permanently or for any duration, it thinks fit.
(iii) Imposing fine upto ` 5,00,000.
Schedule – 2, Part – 1, Clause – 5
60. Disclosure of Material Facts

FAST
The superannuation-cum-pension fund for the employees of a company was under a separate ‘trust’. Both
the company and the trust were under the same management. The auditor, who was auditing the accounts of
the company as well as the trust noted some irregularities in the operation of the trust and commented upon

Ans.
CA.Sarthak Niraj Jain
these irregularities in the confidential report given to the trustees, but did not mention about these
irregularities in his report on the Annual accounts of the Trust.
(i) As per Clause:- Clause (5) of Part I of the Second Schedule
A Chartered Accountant in practice is deemed to be guilty of professional misconduct If he:
• Fails to disclose a material fact known to him
• Which is not disclosed in a financial statement but
• Disclosure of which is necessary to make the financial statement not misleading
(ii) In the given case:-
• The Chartered Accountant was aware of the contraventions and irregularities committed by
the trust as these were referred to in the confidential report given by the Chartered
Accountant to the trustees of the company.
• However, he had issued the annual accounts without any qualification.
(iii) Case study:- It was held by the Supreme Court in Kishorilal Dutta vs. P. K. Mukherjee that it was
the duty of the Chartered Accountant to have disclosed the irregularities and contravention to the
beneficiaries of the fund in the statement of accounts signed by him.
(v) Conclusion:- Chartered Accountant is guilty of professional misconduct if the amount of
irregularities is proved material.
61. Failure to Disclose Material Facts
Mr. Joe, a Chartered Accountant during the course of audit of M/s XYZ Ltd. came to know that the company
has taken a loan of ` 10 lakhs from Employees Provident Fund. The said loan was not reflected in the books
of account. However, the auditor ignored this information in his report. (MTP-Dec-2021, Study Material)
Ans. (i) As per Clause: Clause (5) of Part I of the Second Schedule
A Chartered Accountant in practice is deemed to be guilty of professional misconduct If he:

290 Ab Audit Hoga Sabse Scoring - By CA. Sarthak Niraj Jain


Professional Ethics
• Fails to disclose a material fact known to him
• Which is not disclosed in a financial statement but
• Disclosure of which is necessary to make the financial statement not misleading
(ii) In the given case:- Mr. Joe has come across information that a loan of ` 10 lakhs has been taken by the
company from Employees Provident Fund. This is contravention of Rules and the said loan has not
been reflected in the books of accounts. Further, this material fact has also to be disclosed in the
financial statements.
(iii) Conclusion:- The very fact that Mr. Joe has failed to disclose this fact in his report, he is attracted by
the provisions of professional misconduct under Clause (5) of Part I of Second Schedule to the
Chartered Accountants Act, 1949.
Schedule – 2, Part – 1, Clause – 6
62. Submitting Information as Authorised Representative
A practicing Chartered Accountant was appointed to represent a company before the tax authorities. He
submitted on behalf of his clients certain information and explanations to the authorities, which were found
to be false and misleading. (Study Material)
Ans. (i) As per Clause: Clause (5) of Part I of the Second Schedule
A Chartered Accountant in practice is deemed to be guilty of professional misconduct If:
• He Fails to disclose a material fact known to him
• Which is not disclosed in a financial statement but
• Disclosure of which is necessary to make the financial statement not misleading
(ii) As per Clause: Clause (6) of Part I of Second Schedule
A Chartered Accountant in practice is deemed to be guilty of professional misconduct If
• He fails to report a material misstatement known to him


FAST To appear in a financial statement
With which he is concerned in a professional capacity

CA.Sarthak Niraj Jain


(iii) In the given case:- The Chartered Accountant had submitted the statements before the taxation
authorities. These statements are based on the data provided by the management of the company.
Although the statements prepared were based on incorrect facts and misleading, the Chartered
Accountant had only submitted them acting on the instructions of his client as his authorized
representative.
(iv) Conclusion:- Hence the Chartered Accountant would not be held liable for professional
misconduct. [Assumed he had no responsibility to verify or vouch the information before submission]
63. Disclosure of Material Facts
Mr. X partner of X & co. Chartered Accountants, has compiled and signed the balance sheet of False Limited.,
for submission to the bankers of the said company. Mr. X has also compiled and signed at the request of the
company another balance Sheet inflating the value of assets by 20%, for submission to a term lending
institution. Both the Balance Sheets were not in conformity with the books of accounts maintained by the
company as they were not up-to-date. Comment on Mr. X liability.
Ans. (i) As per Clause:- Clause (5) of Part I of the Second Schedule
A Chartered Accountant in practice is deemed to be guilty of professional misconduct If he:
• Fails to disclose a material fact known to him
• Which is not disclosed in a financial statement but
• Disclosure of which is necessary to make the financial statement not misleading
• Where he is concerned with that financial statement in a professional capacity.
(ii) As per Clause:- Clause (6) of Part I of Second Schedule
A Chartered Accountant in practice is deemed to be guilty of professional misconduct If
• He fails to report a material misstatement known to him
• To appear in a financial statement
• With which he is concerned in a professional capacity
CA Final Audit - By CA. Sarthak Niraj Jain 291
Professional Ethics
(iii) In the given case
Mr. X partner of X & co. Chartered Accountants, has compiled and signed the balance sheet of False
Limited., for submission to the bankers of the said company. Mr. X has also compiled and signed at
the request of the company another balance Sheet inflating the value of assets by 20%, for
submission to a term lending institution. Both the Balance Sheets were not in conformity with the
books of accounts maintained by the company as they were not up-to-date.
(v) Conclusion:- Mr. X would be held guilty under clauses 5 and 6 of Part I of Second Schedule to the
CA Act, 1949 as Mr. X had compiled the two different balance sheets for the same date without
reference to the actual books of accounts, but on instructions of the client. As per clause 5 he has
failed to disclose material fact known to him and further, as per clause 6 he has also failed to report a
material misstatement known to him.
Schedule – 2, Part – 1, Clause – 7
64. Auditor’s Negligence in Performance of Duties
A search under section 132 of Income Tax Act in the premises of a leading merchant led to the discovery of
two sets of account books – one set to record all the income correctly and second to record only limited
income. A CA has issued audit report on the basis of second set of books. What would be your answer if the
first set of account books carried evidence of checking by the CA?
Ans. (i) As per Clause:- Clause (7) of Part I of the Second Schedule to the Chartered Accountants Act, 1949
states that A Chartered Accountant in practice shall be deemed to be guilty of professional misconduct
• If he does not exercise due diligence, or
• is grossly negligent in the conduct of his professional duties.
(ii) In the present case:- It has been found that client has maintained two sets of account books – one

FAST
set to record all the income correctly and second to record only limited income. CA has issued audit
report on the basis of second set of books. In this case, there will not be any misconduct on part of CA if
the auditor is not aware of the first set of books. However, if there exist evidences that auditor is

CA.Sarthak Niraj Jain


aware of first set of books also, then he will be deemed to be guilty of professional misconduct being
grossly negligent in performance of his duties.
(iii) Conclusion:- In the first case, he would not be deemed to be guilty of misconduct. But in the
second case, he would be deemed to be guilty of misconduct.
65. Grossly Negligent in Conduct of Duties
Mr. A, a Chartered Accountant was the auditor of 'A Limited'. During the financial year 2021-22, the
investment appeared in the Balance Sheet of the company of ` 10 lakhs and was the same amount as in the
last year. Later on, it was found that the company's investments were only ` 25,000, but the value of
investments was inflated for the purpose of obtaining higher amount of Bank loan.
(Study Material, MTP-May-2018)
OR
Mr. Aniket, a Chartered Accountant was the auditor of 'Alpha Limited' for the year 2020-21 and 2021-22.
During the financial year, the investment appeared in the Balance Sheet of the company amounting ` 11 lac
and was the same amount as in the last year 2020-21. Later on, it was found that the company's investments
were only for ` 45,000, however, the value of investments was inflated for the purpose of obtaining higher
amount of Bank loan. Comment with reference to the Chartered Accountants Act, 1949, and Schedules
thereto. (MTP-Nov-2020)
Ans. (i) As per Clause: Clause (2) Part I of Second Schedule to the Chartered Accountants Act, 1949
A Chartered Accountant in practice shall be deemed to be guilty of professional misconduct,
• If he, certifies or submits in his name or in the name of his firm,
• A report of an examination of financial statements
• Unless the examination of such statements and the related records has been
• Made by him or by a partner or an employee in his firm or by another CA in practice or

292 Ab Audit Hoga Sabse Scoring - By CA. Sarthak Niraj Jain


Professional Ethics
(ii) As per Clause 7 of Part I of Second Schedule to the Chartered Accountants Act, 1949
A CA in practice shall be guilty of professional misconduct
• If does not exercise due diligence, or
• Is grossly negligent in the conduct of his professional duties. or
As per Clause 8 of Part I of Second Schedule to the Chartered Accountants Act, 1949
A CA in practice shall be guilty of professional misconduct
• If he fails to obtain sufficient information which is necessary for expression of an opinion or
• Its exceptions are sufficiently material to negate the expression of an opinion
(iii) Auditor’s responsibility:-
• The primary duty of physical verification and valuation of investments is of the management.
• However, the auditor’s duty is also to verify the physical existence and valuation of
investments placed, at least on the last day of the accounting year.
• The auditor should verify the documentary evidence for the cost/value and physical existence
of the investments at the end of the year.
• He should not blindly rely upon the Management’s representation.
(iv) In the instant case:-Such non-verification happened for two years. It also appears that auditors
failed to confirm the value of investments from any proper source. In case auditor has simply
relied on the management’s representation, the auditor has failed to perform his duty.
(v) Conclusion:- Mr. A, will be held liable for professional misconduct under Clauses (2), (7) and (8)
of Part I of the Second Schedule to the Chartered Accountants Act, 1949.
66. Failure to Exercise Due Diligence & Obtain Necessary Information
Nam & Co, conducted Stock Audit of DEF Ltd. as per instructions issued by HEG Bank. However instead of

FAST
visiting the site where the stock was lying, the firm relied on the Management Information Systems report
along with inspections reports and photographs of Stock taken by the employees of DEF Ltd. The
photographs were also carrying the date and time printed on them. Comment with reference to the

Ans. CA.Sarthak Niraj Jain


Chartered Accountants Act, 1949 and its schedules thereto.
(i)
(Jan-2021-New)
As per Clause: Clause (2) Part I of Second Schedule to the Chartered Accountants Act, 1949
A Chartered Accountant in practice shall be deemed to be guilty of professional misconduct,
• If he, certifies or submits in his name or in the name of his firm,
• A report of an examination of financial statements
• Unless the examination of such statements and the related records has been
• Made by him or by a partner or an employee in his firm or by another CA in practice or
(ii) As per Clause 7 of Part I of Second Schedule to the Chartered Accountants Act, 1949
A CA in practice shall be guilty of professional misconduct
• If does not exercise due diligence, or
• Is grossly negligent in the conduct of his professional duties. or
As per Clause 8 of Part I of Second Schedule to the Chartered Accountants Act, 1949
A CA in practice shall be guilty of professional misconduct
• If he fails to obtain sufficient information which is necessary for expression of an opinion or
• Its exceptions are sufficiently material to negate the expression of an opinion
(iii) As per Clause:- Clause (8) of Part I of the Second Schedule to the Chartered Accountants Act, 1949
states that A Chartered Accountant in practice shall be deemed to be guilty of professional misconduct
if he:
 Fails to obtain sufficient information to warrant the expression of an opinion or
 His exceptions are sufficient material to negate the expression of an opinion
 This indicates a Chartered Accountant must determine the extent of information, which, should be
obtained by him before he expresses an opinion on the financial statements.

CA Final Audit - By CA. Sarthak Niraj Jain 293


Professional Ethics
(iv) Facts of the case:- Nam & Co, conducted Stock Audit of DEF Ltd. as per instructions issued by HEG
Bank. However instead of visiting the site where the stock was lying, the firm relied on the
Management Information Systems report along with inspections reports and photographs of Stock
taken by the employees of DEF Ltd.
(v) Conclusion:- As per the provision & facts given above Nam & Co, will be held guilty of Professional
Misconduct under Clause (2), (7) & (8) of Part I of Second Schedule to the Chartered Accountants
Act, 1949.
67. Failure to Disclose Material Facts
Comment with reference to the Chartered Accountants Act, 1949 and schedules thereto: CA. Dice had signed
the Balance sheet of QR Ltd. for the year ended 31 st March, 2022 which failed to give disclosure of the charge
created for ` 4.35 Crores against the Corporate Guarantee given in favour of a Group Company. The Balance
Sheet size of the company filed with the Registrar of Companies was ` 26.12 Crores. (Nov-2020-New)
Ans. (i) As per Clause: As per Clause (5) of Part I of Second Schedule to the Chartered Accountants Act, 1949, a
chartered Accountant in practice will be held liable for misconduct
 If he fails to disclose a material fact known to him which is not disclosed in a financial
statement,
 But disclosure of which is necessary in making such financial statement not misleading
 Where he is concerned with that financial statement in a professional capacity.
(ii) Analysis of the Clause & Materiality as per SA-320:
 It may be observed that this clause refers to failure to disclose a material fact, which is known
to him, in a financial statement reported on by the auditor.
 It is obvious, that before a member could be held guilty of misconduct, materiality has to be

FAST established.
The determination of materiality has been provided in SA 320, “Materiality in Planning and
Performing an Audit”.

CA.Sarthak Niraj Jain



Financial reporting frameworks often discuss the concept of materiality in the context of the
preparation and presentation of financial statements.
Although financial reporting frameworks may discuss materiality in different terms, they
generally explain, among other points, that Judgments about materiality are made in the light
of surrounding circumstances, and are affected by the size or nature of a misstatement, or a
combination of both.
(iii) In the present case: - CA Dice has signed a Balance Sheet which failed to give disclosure of ` 4.35
crores (considered material fact applying above SA 320 principle) against the corporate guarantee
given in favour of a Group Company. Size of Balance Sheet of QR Ltd is ` 26.12 crore. This material
fact has to be disclosed in the financial statements.
(iv) Conclusion:- Keeping in view the above, he is attracted by the provisions of professional
misconduct under Clause (5) of Part I of Second Schedule to the Chartered Accountants Act, 1949.
(Note: Answer has been updated)
Schedule – 2, Part – 1, Clause – 8
68. Failure to Obtain Information
Z, a practicing Chartered Accountant issued a certificate of circulation of a periodical without going into the
most elementary details of how the circulation of a periodical was being maintained i.e. by not looking into
the financial records, bank statements or bank pass books, by not examining evidence of actual payment of
printers bills and by not caring to ascertain how many copies were sold and paid for. (Study Material)
Ans. (i) As per Clause:- Clause(8) of Part I of the Second Schedule to the Chartered Accountants Act, 1949 states
that A Chartered Accountant in practice shall be deemed to be guilty of professional misconduct if he:
• Fails to obtain sufficient information to warrant the expression of an opinion or
• His exceptions are sufficient material to negate the expression of an opinion

294 Ab Audit Hoga Sabse Scoring - By CA. Sarthak Niraj Jain


Professional Ethics
(ii) In the present case:- Mr. Z, a practicing Chartered Accountant issued a certificate of circulation of a
periodical without going into the most elementary details of how the circulation of a periodical was
being maintained i.e., by not looking into the financial records, bank statements or bank pass books, by
not examining evidence of actual payment of printers bills and by not caring to ascertain how many
copies were sold and paid for.
• The chartered accountant should not express his opinion before obtaining the required
data and information. As an auditor, Mr. Z ought to have verified the basic records to ensure
the correctness of circulation figures.
(iii) Conclusion:- Mr. Z will be held guilty of professional misconduct as per Clause (8) of Part I of
Second Schedule to the Chartered Accountants Act, 1949.
Alternative Solution is possible on the basis of Clause (7) of Part I of Second Schedule to the
Chartered Accountants Act, 1949.
69. Failure to Keep Money in Separate Bank Account
CA N was appointed as an auditor of JAL Ltd. The company has branches all over the state of Haryana. CA N,
in consultation with management, decided to Visit 6 out of 10 branches. Management decided to pay him
advance of `2.00 Lacs against the estimated expenses of 2.50 Lacs on visits to be conducted as a part of
services rendered. As agreed, `2.00 Lacs was transferred in his bank account from which he met all the
expenses. Comment with reference to Chartered Accountants Act, 1949 whether the action of CA N of
receiving the advance money in his saving accounts and not keeping it in separate bank account is valid.
(Jan-2021-New)
Ans. (i) As per Clause:- Clause (10) of Part I of the Second Schedule to the Chartered Accountants Act, 1949 states that a
Chartered Accountant in practice shall be deemed to be guilty of professional misconduct if he:

FAST
 Fails to keep moneys of his clients in a separate bank account other than the fees or remuneration or
 Fails to use such moneys for purposes for which they are intended within a reasonable time
(ii) Interpretation of Clause :

CA.Sarthak Niraj Jain


 In the course of his engagement as a professional accountant, a member may be entrusted with
moneys belonging to his client.
 If he should receive such funds, it would be his duty to deposit them in a separate banking
account, and to utilize such funds only in accordance with the instructions of the client or for the
purposes intended by the client.
(iii) Council Guidlines :
 In this connection the Council has considered some practical difficulties of the members and the
following suggestion, among other suggestions, has been made to remove these difficulties:
 “An advance received by a Chartered Accountant against services to be rendered does not fall
under Clause (10) of Part I of the Second Schedule”
(iv) In the given case: -CA N was given an advance of ` 2 Lakhs against the estimated expenses of ` 2.50
Lakhs on visits to be conducted as a part of services rendered.
(v) Conclusion: - Applying the above, it can be concluded that CA N is not guilty of professional
misconduct under Chartered Accountants Act, 1949. (Note: Answer has been updated)
Second Schedule, Part II “Professional Misconduct in relation to the Members of the Institute Generally”
Schedule– 2, Part – 2, Clause- 1
70. Contravening Provisions of the Act
Mr. Hopeful, an aspiring student of ICAI, approached Mr. Witty, a practicing Chartered Accountant, for the
purpose of articleship. Mr. Witty, the principal, offered him stipend at the rate of ` 2,000 per month to be paid
every sixth month along with interest at the rate of 10% per annum compounded monthly to compensate
such late payment on plea that cycle of professional receipts from clients is six months. Mr. Hopeful agreed
for such late payment in the hope of getting extra stipend in the form of interest. Mr. Witty, however, used to
disburse salary to all of his employees on time.

CA Final Audit - By CA. Sarthak Niraj Jain 295


Professional Ethics

Ans. (i) As per Clause:- Clause (1) of Part II of the Second Schedule to the Chartered Accountants Act, 1949
• A member of the Institute, whether in practice or not, shall be deemed to be guilty of
professional misconduct :
• If he contravenes any of the provisions of this Act or
• The regulations made there under or
• Any guidelines issued by the Council.
(ii) In the given case:- Mr. Witty has failed to make the payments of stipend to articled assistant every
month in accordance with Regulation 48. The fact that the articled assistant will be compensated
with extra sum in the form of interest on late payment is not relevant and the plea that cycle of
professional receipts from clients is six months is not acceptable as Mr. Witty has disbursed salary to
all of his employees on time.
(iii) Conclusion:- Therefore, Mr. Witty is guilty of professional misconduct under Clause (1) of Part II of
the Second Schedule to the Chartered Accountants Act, 1949 as he has contravened Regulation 48 by
not making the payment every month.
71. Failure to Observe the Regulations
A Chartered Accountant in practice had confirmed in the application made by his articled clerk to the Council
for permission to study that the normal working hours of his office were 11 a.m. to 6 p.m. and the hours
during which the articled clerk was required to attend college classes were 7 a.m. to 9.30 a.m. On inquiry
from Principal of College, it was ascertained that the articled clerk used to attend classes from 10 a.m. to 1.55
p.m. The Chartered Accountant pleaded ignorance about the articled clerk attending the college classes
during office hours. Will the Chartered Accountant be held guilty of professional misconduct?
Ans. (i) As per Clause:- Clause (1) of Part II of the Second Schedule to the Chartered Accountants Act, 1949


FAST A member of the Institute, whether in practice or not, shall be deemed to be guilty of
professional misconduct :
If he contravenes any of the provisions of this Act or

CA.Sarthak Niraj Jain




The regulations made there under or
Any guidelines issued by the Council.
And
(ii) As per Clause (3) of Part II of Second Schedule to the CA Act, 1949,
A member is deemed to be guilty of professional misconduct:
• If he includes in any information, statement, return or form
• To be submitted to the Institute, Council or any of its Committees, Director (Discipline),
Board of Discipline, Disciplinary Committee, Quality Review Board or the Appellate
Authority
• Any particulars knowing them to be false.
(iii) As per regulation:-The chartered accountant, as per Regulations also, is expected to impart proper
practical training.
(iv) In the present case:- The articled clerk must have not been attending office on a regular basis and the
explanation of the Chartered Accountant cannot be accepted particularly in view of the fact that the
chartered accountant did not obtain certificate from the Principal to confirm the timings. It is also quite
likely that the articled clerk would be availing leave quite often and coming late to the office.
(v) Conclusion:- He will be deemed to be guilty of professional misconduct under Clause 1 of Part II of
Second Schedule for contravention of regulations of the Institute and under Clause 3 of Part II of
Second Schedule for submission of false information to the ICAI.
Schedule – 2, Part – 2, Clause- 3
72. Submitting Wrong Information to the Institute
Mr. P and Mr. Q are running a firm of Chartered Accountants in the name of M/s PQ & Co. On 23.05.2022,
they included the name of Mr. R, a practicing Chartered Accountant, without his knowledge, as a partner

296 Ab Audit Hoga Sabse Scoring - By CA. Sarthak Niraj Jain


Professional Ethics
while submitting an application for empanelment as auditor for Public Sector Bank branches to the Institute.
However, they added Mr. R as a partner to their firm offering a share of 25% of the profits, on 25.05.2022.
(RTP-May-2020, May-2016)
OR
AB & Co., a firm of Chartered Accountants, included the name of P as a partner while filing an application for
empanelment as auditor for Public Sector bank branches. It was subsequently noticed that on the date of
application, P was not a partner with AB & Co.
Ans. (i) As per Clause:- As per Clause (3) of Part II of Second Schedule to the CA Act, 1949,
A member is deemed to be guilty of professional misconduct:
• If he includes in any information, statement, return or form
• To be submitted to the Institute, Council or any of its Committees, Director (Discipline),
Board of Discipline, Disciplinary Committee, Quality Review Board or the Appellate
Authority
• Any particulars knowing them to be false.
(ii) In the instant case:- Mr. P and Mr. Q, partners of M/s PQ & Co., included the name of Mr. R, another
Chartered Accountant, as partner in their firm, without his knowledge, in their application for
empanelment as auditor of branches of Public Sector Banks submitted to the Institute. However, such a
member was not a partner of the said firm as on the date of application submitted. Here, Mr. P and Mr.
Q have submitted wrong information to the Institute.
(iii) Conclusion: - Mr. P and Mr. Q, both, would be held guilty of professional misconduct under
Clause (3) of Part II of the Second Schedule to the Chartered Accountants Act, 1949.
73. KYC Norms for CA in Practice

FAST
Mr. F, a Chartered Accountant, gave advisory services to PQR Pvt. Ltd. Further, he gave them GST
consultancy, compilation engagement for historical financial information and helped in ERP set up. Later, the
company turned out to be a part of a group of companies involved in money laundering. Mr. F was asked to

CA.Sarthak Niraj Jain


provide details of the companies. Mr. F refused on the grounds that he gave only consultancy services to the
company and wasn’t supposed to keep any information about the company. Is Mr. F right as per the
guidelines issued by the ICAI? (Study Material)
Ans. (i) Council Guidelines:- The financial services industry globally is required to obtain information of
their clients and comply with Know Your Client Norms (KYC norms). Keeping in mind the highest
standards of Chartered Accountancy profession in India, the Council of ICAI issued such norms to be
observed by the members of the profession who are in practice.
(ii) In the given situation:- CA. F, gave GST consultancy, compilation engagement for historical financial
information and helped in ERP set up along with advisory services to PQR Pvt. Ltd. Mr. F was asked to
provide details of the companies as the company, turned out to be a part of a group of companies,
involved in money laundering. Contention of Mr. F that he gave only consultancy services to the
company and wasn’t supposed to keep any information about the company is not valid as Mr. F should
have kept following information in compliance with KYC Norms which are mandatory in nature and
shall apply in all assignments pertaining to attestation functions.
(iii) Conclusion:- In the given case of PQR Pvt. Ltd., a Corporate Entity, Mr. F should have kept following
information:
A. General Information
 Name and Address of the Entity
 Business Description Name of the Parent Company in case of Subsidiary
 Copy of last Audited Financial Statement
B. Engagement Information
 Type of Engagement

CA Final Audit - By CA. Sarthak Niraj Jain 297


Professional Ethics
C. Regulatory Information
 Company PAN No.
 Company Identification No.
 Directors’ Names & Addresses
 Directors’ Identification No.
74. Entire Audit Fees Received in Advance
D, who conducts the tax audit u/s 44AB of the Income Tax Act, 1961 of M/s ABC, a partnership firm, has
received the audit fees of ` 2,50,000 on progressive basis in respect of the tax audit for the year ended
31.3.2022. The audit report was, however, signed on 25.5.2022. What is your answer if fees is recovered on
progressive basis? (Study Material)
Ans. (i) As per Clause:- Clause (1) of Part II of the Second Schedule to the Chartered Accountants Act, 1949
• A member of the Institute, whether in practice or not, shall be deemed to be guilty of
professional misconduct :
• If he contravenes any of the provisions of this Act or
• The regulations made there under or
• Any guidelines issued by the Council.
(ii) As per Council General Guidelines, 2008 (Chapter X)
A member of the Institute in practice or a partner of a firm in practice or a firm shall not accept
appointment as auditor of a concern while indebted to the concern or given any guarantee or
provided any security in connection with the indebtedness of any third person to the concern, for
limits fixed in the statute and in other cases for amount exceeding ` 1,00,000/-.
(iii) Provision of Income tax Act:-


FAST As per explanation to Section 288 to Income Tax Act, 1961, an individual who, or his relative
or partner is indebted to the assessee cannot conduct tax audit
Provided that the relative may be indebted to the assessee for an amount not exceeding `

CA.Sarthak Niraj Jain


1,00,000.
(iv) The Research Committee of the ICAI:- Has expressed the opinion that where in accordance with the
terms of engagement of auditor by a client, the auditor recovers his fees on a progressive basis as
and when a part of the work is done without waiting for the completion of the whole job, he cannot be
said to be indebted to the company at any stage.
(v) In the instant case:- Mr. D is appointed to conduct a tax audit u/s 44AB of the Income Tax Act, 1961.
He has received the audit fees of ` 2,50,000 in respect of the tax audit for the year ended 31.3.2020
which is on progressive basis.
(vi) Conclusion:- Therefore, Mr. D will not be held guilty for misconduct.
75. Contravening Provisions of the Act/Regulations/Guidelines
Mr. Kushal, a practicing Chartered Accountant has signed the Tax Audit Reports u/s 44AB of the Income tax
Act, 1961 for the financial year 2021-22 that are filed online using Digital Signature and without generating
UDIN on the ground that there is no field for mentioning UDIN on digitally signed online reports. Is the
contention of Mr. Kushal valid? Give your comments with reference to the Chartered Accountants Act, 1949
and schedules thereto. (Nov-2020-New)(MTP-Dec-2021)
Ans. (i) As per Clause:- Clause (1) of Part II of the Second Schedule to the Chartered Accountants Act, 1949
• A member of the Institute, whether in practice or not, shall be deemed to be guilty of
professional misconduct :
• If he contravenes any of the provisions of this Act or
• The regulations made there under or
• Any guidelines issued by the Council.

298 Ab Audit Hoga Sabse Scoring - By CA. Sarthak Niraj Jain


Professional Ethics
(ii) As per Council Guidelines (Chapter XV) :- In exercise of the powers conferred on it Part- II of the
Second Schedule to the Chartered Accountants Act, 1949, the Council of the Institute of Chartered
Accountants of India made it mandatory for the members of the Institute in practice to generate
Unique Document Identification Number (UDIN) for all Tax Audit Reports w.e.f. 1st April, 2019.
(iii) In the instant case:- Mr. Kushal, a practicing Chartered Accountant has signed the Tax Audit
Reports u/s 44AB of the Income tax Act, 1961 for the financial year 2019-20 that are filed online
using Digital Signature and without generating UDIN on the ground that there is no field for
mentioning UDIN on digitally signed online reports.
(iv) Conclusion:- Contention of Mr. Kushal is not valid as non-generating of UDIN amounts to
professional misconduct under Clause 1, Part II of Second Schedule.
SJ Note: - Alternatively CA. may be held guilty under Clause 9 Part 1, Schedule 2
76. Undercutting of Fees
Mr. Rahul, a locally based Chartered Accountant, accepted an audit assignment at a fee lower than that
charged by the previous auditor, who was stationed in another town and had to spend a lot of money on
travel for which he did not charge separately.
Ans. (i) As per Council General Guidelines:-
• Council of ICAI has prescribed minimum audit fees for the practicing members. However,
prescribed minimum audit fee is recommendatory, not mandatory in nature.
• Fees in any professional assignment depend upon the mutual agreement and understanding
between the member and the client.
(ii) In the given case:- Mr. Rahul is a locally based Chartered Accountant, accepted an audit assignment
at a fee lower than that charged by the previous auditor, who was outstation based Chartered

FAST
Accountant and had to spend a lot of money on travel which was included in his audit fee and was
not charged by him separately.
(iii) Interpretation of guidelines with case:-

CA.Sarthak Niraj Jain


Since the previous auditor was stationed in another town and therefore, had to incur higher cost on
account of conveyance, and the previously the fee was decided on a composite basis inclusive of
travelling expenses of the auditor, it cannot be said that Mr. Rahul has accepted an audit assignment
based on under cutting of fees.
(iv) Conclusion:- Mr. Rahul will not be held guilty for misconduct.
77. Signing of Tax Audit Report
A member of the institute shall not accept in a year more than the specified number of tax audits under
section 44AB of the Income Tax Act. Mr. Gaurav is a partner in M/s XYZ & Co., a firm of Chartered
Accountants with 6 partners. During the assessment year 2021-22, Mr. Gaurav alone had signed 290 tax
audit reports consisting of both corporate and non-corporate assesses. (Study Material)
Ans. (i) As per Chapter VI of Council General Guidelines 2008:- A member of the Institute in practice
shall not accept, in a financial year, more than the “specified number of tax audit assignments” u/s
44AB of the Income-tax Act, 1961.
(ii) In the case of a firm of CAs in practice:-
• The “specified number of tax audit assignments” shall be construed as the specified number
of tax audit assignments for every partner of the firm.
• The specified number of tax audit assignments in the case of firm of CAs in practice, 60 tax audit
assignments per partner in the firm, in a financial year, whether in respect of corporate or
non-corporate assesses.
• It is further clarified by the Council of ICAI that tax audit report accepted by the firm of
Chartered Accountants can be signed by any partner on the behalf of the firm.
(iii) In the present case:- There are six partners in the firm and hence the firm can accept 360 tax
audit assignment and any partner can sign the tax audit report on the behalf of the firm.
(iv) Conclusion:- Mr. Gaurav can sign the 290 tax audit reports on behalf of the firm.

CA Final Audit - By CA. Sarthak Niraj Jain 299


Professional Ethics

78. Signing of Tax Audit Report


M/S PQR & Co. is a partnership firm of 3 partners P, Q and R. All partners are exclusively associated with the
firm in practice and are not doing practice in individual capacity. For the year ended 31st March, 2022, the
partners have undertaken audits and signed audit reports under section 44AB/ 44AD of the Income Tax Act,
1961 as under:
Under Section 44AB Under Section 44AD
P 10 15
Q 60 5
R 100 5
Discuss whether there is any professional misconduct by the firm in regard to the aforesaid audits.
(Nov-2019-Old)
Ans. (i) As per Chapter VI of Council General Guidelines 2008:- A member of the Institute in practice
shall not accept, in a financial year, more than the “specified number of tax audit assignments” u/s
44AB of the Income-tax Act, 1961.
(ii) In the case of a firm of CAs in practice:-
• The “specified number of tax audit assignments” shall be construed as the specified number
of tax audit assignments for every partner of the firm.
• The specified number of tax audit assignments in the case of firm of CAs in practice, 60 tax audit
assignments per partner in the firm, in a financial year, whether in respect of corporate or
non-corporate assesses.
• It is further clarified by the Council of ICAI that tax audit report accepted by the firm of
Chartered Accountants can be signed by any partner on the behalf of the firm.

FASTThe audits conducted under Section 44AD, 44AE, 44AF of the Income Tax Act, 1961 shall not
be taken into account for the purpose of reckoning the “specified number of tax audit
assignments”.
CA.Sarthak Niraj Jain
(iii) In the present case:- There are three partners in the firm and hence the firm can accept 180 tax
audit assignment and any partner can sign the tax audit report on the behalf of the firm. Firm has
accepted 170 tax audit assignments other than the audit under section 44AD.
(iv) Conclusion: - No Misconduct arises on part of Firm or partners.
79. Charging Excess Fees for a Professional Assignment
MNC Pvt. Ltd. appointed CA. Posh for some professional assignments like company’s ROC work, preparation of
minutes, statutory register etc. For this, CA. Posh charged his fees depending on the complexity and the time
spent by him on each assignment. Later on, MNC Pvt. Ltd. filed a complaint against CA. Posh to the Institute of
Chartered Accountants of India (ICAI) that he has charged excessive fees for the assignments comparative to
the scale of fees recommended by the Committee as well as duly considered by the Council of ICAI.
OR
M/s ABC, a partnership firm carrying on business has complained to the Institute of Chartered Accountants
of India (ICAI) that Mr. M, a Chartered Accountant has charged the firm excessive fees for a professional
assignment.
Ans. (i) The prescribed scale of fees for the professional assignments done by the chartered accountants is
recommendatory in nature.
(ii) Charging an excessive fee for a professional assignment does not constitute any misconduct in the
context of the provisions of the Chartered Accountants Act, 1949 and regulation made thereunder
(iii) In the given case:- CA. Posh has charged excess fees comparative to the scale of fees recommended by
the Committee as well as duly considered by the Council of ICAI. In this context, it may be noted that
the scale of fees is the minimum prescribed scale of fees.
(iv) Conclusion:- From the above facts and provisions, it may be concluded that CA. Posh is not liable for
any misconduct under the Chartered Accountants Act, 1949. Therefore, the contention of MNC Pvt.
Ltd. is not tenable.

300 Ab Audit Hoga Sabse Scoring - By CA. Sarthak Niraj Jain


CA Final Audit DEC- 2021 New Course Paper

CA FINAL AUDIT DEC- 2021 NEW COURSE PAPER


1. M/s Kumar & Co., Chartered Accountants were appointed as statutory auditors of PC limited for the financial
year 2020-21. During the course of audit, one of the partners CA. Kumar observed that there is
misappropriation of assets in the form of theft of entity’s inventory and is perpetrated by employees in
relatively small and immaterial amounts. CA. Kumar is concerned with the existence of certain circumstances
for increasing the susceptibility of assets to misappropriation.
Guide CA. Kumar with respect to Risk factors related to misstatements arising from misappropriation of assets
with reference to relevant Standard on Auditing.
2. In the course of audit of Tech limited you observed that processing of accounting data was given to a third party
on account of certain considerations like cost reduction, own computer working to full capacity. Tech Limited
used a service organisation to record transactions and process related data. As an auditor, what would be your
considerations regarding the nature and extent of activities undertaken by service organisation so as to
determine whether those activities are relevant to the audit and, if so, to assess their effect on audit risk.
Discuss with reference to relevant Standard on Auditing.
3. CA Madhu is the statutory auditor of Lakshmi Ltd. for the Financial year 2020-21. In respect of loans and
advances of ` 75 Lakh given to Sriman Pvt. Ltd., the Company has not furnished any agreement to CA Madhu
and in absence of the same, he is unable to verify the terms of repayment, chargeability of interest and other
terms.
Justify the type of opinion which CA Madhu should give in such situation. Also, Draft an appropriate Opinion
paragraph and Basis of opinion paragraph.
4. During the course of audit of Fortune Ltd., CA Prasad is concerned with the quality and effectiveness of internal
control. Towards achieving his objective he wants to assess and evaluate the control environment. Guide CA
Prasad with well-defined set of the Standard Operating Procedures in the assessment and evaluation of control

5.
FAST
Mr. Khanna has been appointed as statutory auditor of RST Ltd. For the financial year ended 31 st March, 2021.

CA.Sarthak Niraj Jain


The financial statements of RST Ltd. are to be drawn up in compliance of the Companies (Indian Accounting
Standards) Rules, 2015. The Chief financial officer is of the view that the disclosure requirements specified
under Division II of Schedule III of the Companies Act, 2013 are complete and no other additional disclosures
shall be made in the Notes or by way of additional statements. Advise on the General instructions to be
considered by RST Ltd. while preparing its financial statements.
6. The Cashier of a company committed a fraud and absconded with the proceeds thereof. The Chief Accountant of
the company also did not know when the fraud had occurred. In the course of the audit, the auditor failed to
discover the fraud. After the audit was completed, however, the fraud was discovered by the Chief Accountant.
Investigation made at that time indicates that the auditor did not exercise proper skill and care and performed
his work in a desultory and haphazard manner. With this background, the Directors of the company intend to
file disciplinary proceedings against the auditor. Comment with reference to the Chartered Accountants Act,
1949 and schedules thereto.
7. The report of the Comptroller and Auditor General of India on the audit of PSUS are presented to
the parliament and to various state legislatures to facilitate a proper consideration. Enumerate
the contents of Audit Report presented by C & AG.

8. You have been appointed as Concurrent auditor of one of the branches of Coin Bank Ltd. This
branch is dealing mainly in foreign exchange. State the suggested audit procedures to be covered
by you to check the foreign exchange transactions of this branch while doing Concurrent audit.

9. CA Mehta was appointed as the Auditor of CS Ltd. for the year 2020-21 in the place of retiring auditor CA Gupta.
CA Mehta accepted the appointment after obtaining a certificate from the management that the provisions of
the Sections 139 and 140 of the Companies Act, 2013 have been complied with.
Comment with reference to the Chartered Accountants Act, 1949 and schedules thereto.

CA Final Audit - By CA. Sarthak Niraj Jain 301


CA Final Audit DEC- 2021 New Course Paper
10. M & B investments Ltd. is a company having paid up share capital of ` 1 Crore. It has a subsidiary, Investors
Fund Management Ltd. Major business of M & B Investments Ltd. is to pool money from investors on a
collective basis and invest this money in various funds. This company pooled ` 10 Crore from a number of
clients, which represent the Company’s shareholders.
While auditing books of accounts of M & B Investments Ltd. CA. X observed that whole amount of ` 10 crore
pooled has been invested in shares and debentures of various companies and profit earned due to appreciation
of the prices of these shares has been distributed to various shareholders of the company.
Now, CA X raised an issue while auditing financial statements of M & B Investments Ltd. whether the
consolidated financial statements are required as per Section 129(3) of the Companies Act, 2013?
Analyse the above issue and give your opinion.
11. ACT Silk Industries is a leading textile manufacturing listed company. In the course of evidence collection and
analysis, it was observed that the company is involved in siphoning of funds through payments to shell
companies. Hence, SEBI appointed B & S Associates, Chartered Accountants, as forensic auditors of the
company.
Enumerate in brief the steps to be taken by B & S Associates in forensic audit process.
12. ABC Ltd. is a company registered under the Companies Act, 2013. The company is engaged in the business of
loans and advances, acquisition of shares / stocks / bonds / debentures / securities issued by Government or
local authorities. For the year ended 31st March, 2021 following are some extracts from the financial
statements:
(i) Paid-up share capital ` 40.53 Cr.
(ii) Non-Current Assets - Loans & Advances ` 55.90 Cr.
(iii) Current Assets - Loans and advances ` 344.47 Cr.
(iv) Total assets of the company ` 530 Cr.
(v) Intangible assets ` 3 Cr.
(vi)
(vii)
(viii)
FAST
Profit for the Year
Income from interest and dividends
Gross income
` 7.25 Cr.
` 52 Cr.
` 102.57 Cr.
Directors intend to apply for registration as Non-Banking Financial Company (NBFC) under Section 45-IA of

13. CA.Sarthak Niraj Jain


the Reserve Bank of India (Amendment) Act, 1997. Advise.
Mr. Agarwal, in the course of audit of PQ Limited, wants to perform external confirmation procedures to obtain
audit evidence. Guide Mr. Agarwal, listing out the factors that may assist him in determining whether external
confirmation procedures are to be performed as substantive audit procedures.

14. Long Age Foundations Ltd. (LAF), a pharmaceutical company, collected the data from some hospitals and their
experts tried to understand medical needs of elderly people. After complete study, their experts developed an
application where LAF will provide complete health care after charging a nominal amount from the customers
if customers download this application in their mobile phones. CA P in his audit has used data analytics method
also known as Computer Assisted audit techniques
Give illustrations of suggested approach to get the benefit from the use of CAATs.
15. CA. M has been appointed as an auditor of Life Secure Insurance Ltd. He observed that few insurance policies
have been sold by the company in the last month of the financial year ending 31 st March, 2021. While
recognizing income in the income statement of the company, it is the responsibility of CA. M to make an
assessment of the reasonability of the risk pattern managed by the management. Also, it is to be ensured by him
that Life Secure Insurance Ltd. should not issue policies, if the risk is not established before the closure of the
F.Y. 2020-21.
Indicate the circumstances when the company should not issue the policy documents.
16. CA. Pradeep is appointed auditor of Delicious Foods Ltd. (DFL) a listed company to audit the financial
statements for the year ended 31st March 2021. Paid-up share capital of DFL is ` 5.97 Cr. While auditing
director’s remuneration CA. Pradeep observed that Mr. Shrinivas Gupta has been appointed as an independent
director. Mr. Srinivas Gupta is holding shares of ` 8,95,500 in DFL and his wife is holding shares of ` 2,98,500
in the same company.

302 Ab Audit Hoga Sabse Scoring - By CA. Sarthak Niraj Jain


CA Final Audit DEC- 2021 New Course Paper
CA. Pradeep raised an objection on the appointment of independent director, but other directors explained that
holding of shares by Mr. Srinivas is less than the prescribed limit hence, he is eligible to be appointed as
independent director.
How will CA. Pradeep deal with this situation and how will he report this issue ?
17. CA Nitesh, while carrying out the Tax audit of PQR Ltd. observed that PQR Ltd. has entered into specified
financial transactions covered under Section 285BA of the Income tax Act, 1961. PQR Ltd. Has furnished
statement of the specified financial transaction in Form No. 61 & Form No. 61 A.
Guide CA. Nitesh with reporting requirements under clause 42 of Form 3CD?
The management contends that tax auditor need not report, if the transactions are not covered in the ambit of
Section 269ST. Comment.
18. Comment with reference to the Chartered Accountants Act, 1949 and schedules thereto:
CA. D, a Practicing Chartered Accountant, was appointed as a simplicitor Director in a Pvt. Ltd. company on 01-
012020. After serving 18 months, Mr. D resigned as the Director He accepted the appointment as the Statutory
Auditor of the company with effect from 01-10-2021
Is CA. D right in accepting the audit?
19. CA. Manoj has been appointed as Peer Reviewer of M/s UV Associates, a Chartered Accountant firm consisting
of 18 partners. As a Practicing unit what are the obligations that are to be complied by M/s UV associates in
addition to furnishing the questionnaire, statements and such other Particulars as the Board may deem fit?

Notes

FAST
CA.Sarthak Niraj Jain

CA Final Audit - By CA. Sarthak Niraj Jain 303


CA Final Audit DEC- 2021 Old Course Paper

CA FINAL AUDIT DEC-2021 OLD COURSE PAPER


1. Ace Limited (manufacturer of textile goods) got an order of manufacturing of PPE kits in December 2020. But
there was shortage of machinery and manpower to accomplish the ordered requirement of PPE kits. Ace Ltd.
approached another manufacturing unit Jack Limited for purchase of the unit. Jack Limited was interested in
the sale of unit, so the deal went through and Ace Limited acquired ninety five percent shares of Jack Limited.
The new management of Jack Limited proposed and appointed NKB Associates, Chartered Accountants,
(already auditors of Ace Limited) as new auditors of Jack Limited. NKB Associates accepted the assignment
without considering information whether the conclusions reached regarding the acceptance and continuance of
client relationships and audit engagements are appropriate. Comment with respect to appropriate Standard of
Auditing what type of information assists the engagement partner in determining whether the conclusions
reached regarding the acceptance and continuance of client relationships and audit engagements are
appropriate or not?
2. You as a statutory auditor had audited the financial statements of A Ltd., a listed company, for F.Y. 2019-2020.
The company has included the comparative financial information in the financial statements prepared for the
current F. Y. 2020-2021. You as an auditor want to obtain sufficient appropriate audit evidence that
comparative financial information has been presented, in all material aspects, in accordance with the
requirements in the applicable financial reporting framework. List out audit procedures, as specified in
relevant SA, which you are required to follow for the purpose.
3. You are an auditor of KAL Ltd., a multinational company dealing in auto parts. The company is operating in a
CIS environment comprising of hardware, software, people and transmission media and has implemented an
internal control. Different types of controls are used to enhance the reliability of basic components with respect
to each type of error. List down major classes of controls that you must evaluate as an auditor.
4. FAST
While auditing the complete set of consolidated financial statements of J Ltd., a listed company, using a fair
presentation framework, PQR & Co., a Chartered Accountant firm, discovered that the consolidated financial

CA.Sarthak Niraj Jain


statements are materially misstated due to the non-consolidation of one of the subsidiary. The material
misstatement is deemed to be pervasive to the consolidated financial statements. The effects of the
misstatement on the consolidated financial statements could not be determined because it was not practicable
to do so. Thus, PQR & Co. decided to provide an adverse opinion for the same and further determined that,
there are no key audit matters other than the matter to be described in the Basis for Adverse Opinion section.
Comment whether PQR & Co. need to report under SA 701 ‘Communicating Key Audit Matters in the
Independent Auditor's Report'?
5. While conducting the tax audit of PJP Ltd. you observed that company has timely filed ETDS return for TDS
deducted on salary u/s 192 of The Income Tax Act, 1961 in form 24Q in respect of fourth quarter period from
1st January 2021 to 31st March 2021. The company has not furnished list of details which are not reported in the
statement of tax deducted at source under the pretext that TDS statements are furnished within the prescribed
time. As a Tax Auditor of PJP Ltd. how you would deal and report?
6. CA S has been appointed as peer reviewer of JS & Associates, a Level-I CA Firm. CA S wants to
appoint CA K as his assistant, as he has working experience in a Level-I CA Firm. What aspects CA S
should take care of before appointing CA K as Assistant, for the peer review of JS & Associates?
7. AB & Associates, a chartered accountant firm, was appointed auditors of KEY Company Ltd. for the financial
year ended 31st March 2021. Being the first year of audit, the audit firm AB & Associates, as per its system of
quality control, involve senior partner of the firm to review the overall audit strategy prepared by the team
members. What additional matters would be considered in initial audit engagement by the senior partner in
establishing the overall audit strategy and audit plan of KEY Company Ltd.?
8. HG & Co. is the statutory auditor of KFN NBFC Ltd. While planning the audit procedures to be done during the
audit of entity, there was a difference of opinion between Mr. H and his partner Mr. G. Mr. G is of the opinion

304 Ab Audit Hoga Sabse Scoring - By CA. Sarthak Niraj Jain


CA Final Audit DEC- 2021 Old Course Paper
that evaluation of Internal control system and verification of registration with RBI should not be the part of
audit procedure, as it is the part of internal audits only. Is the contention of Mr. G correct? Also state what broad
areas should mandatorily become part of the audit procedure of HG & Co. for conducting the audit of KFN NBFC
Ltd.?
9. Mr. Joe, a practicing chartered accountant, has accepted an appointment as auditor of cooperative
society and agreed to charge fees @ 7% of the profits of the society during the financial year
2020-21. Comment on action of Mr. Joe with reference to the Chartered Accountants Act, 1949
and Schedules thereto.
10. You are an auditor of XYZ Insurance Company Ltd. which offers variety of risk management products to
business entities wishing to protect their business activities against losses due to various probable risks. XYZ
Insurance Company Ltd. is in the process of offering to ABC Ltd., a multinational group having worldwide
market, "Trade Credit Insurance Policy" to cover domestic risk, export risk and political risk. You as an auditor
of Insurance Company have been requested to ensure that all the requirements have been met by XYZ
Insurance Company Ltd. before Trade Credit Insurance Product is offered to ABC Ltd. List down those
requirements.
11. "What constitutes a true and fair view' is the matter of an auditor's judgement in particular circumstances of a
case." Do you agree ? Enlist the requirements you as an auditor will observe to ensure true and fair view.

12. AJ & Associates and PK & Co., Chartered Accountant firms have joined the Network firm A to Z & Affiliates
registered with Institute. AJ & Associates was statutory auditor of B Ltd. for last 10 years. Due to rotation of
auditor as per section 139(2) of Companies Act, 2013, B Ltd. retires AJ & Associates and appoints PK & Co. as

13.
FAST
auditor for the year 2020-21. Comment as per Chartered Accountant Act, 1949-Guidelines for Networking.

TEA Ltd, FMCG Company having its tea gardens in north eastern states of the country is exclusively dealing in

CA.Sarthak Niraj Jain


blending, processing, packing and selling of various brands of Tea. During the year under audit, the company
entered into joint venture for purchasing Tea Gardens in Sri Lanka and Kenya. M/s GN & Associates are the
statutory auditors of the company for the financial year 2020-21. During the course of audit, the audit team was
unable to obtain sufficient appropriate evidence about a single element of the consolidated financial statements
being Joint venture investment in Kali Ltd. representing over 90% of the group's net assets having both
material and pervasive possible effect to the consolidated financial statements. The group's investment in Kali
Ltd. is carried at ` 100 crores in the group's consolidated balance sheet.
Draft the opinion paragraph and basis of opinion paragraph to be included in the Independent Auditor's report.
14. What will be the liability of Mr. X, an auditor in the following situations? Support your views :
(i) Mr. X has appeared before the Income Tax Authorities as an authorized representative of his Auditee
and submitted to the Income Tax Authorities a false declaration
(ii) As an auditor, not appropriately dealing with a misstatement appearing in financial statements
amounting to ` 9.84 lakhs. Misstatement does not involve public interest.
15. "In reviewing any System of Procedure, the management auditor must concern himself with its purpose as well
as its design." Elucidate how you as a management auditor will study system and procedural functions.

16. C&AG appointed a chartered accountant firm to conduct Performance audit of COP Ltd., a PSU of Govt. of India.
The firm conducted the audit with a view to check all the expenses of the unit are in conformity to the public
interest and publicly accepted customs. The audit report submitted by audit firm was rejected by C&AG. Give
your opinion on the action of C&AG.

CA Final Audit - By CA. Sarthak Niraj Jain 305


CA Final Audit DEC- 2021 Old Course Paper
17. Your firm has been appointed as a statutory auditor of a nationalized bank which has multiple banking
accounts and consortium accounts of corporate borrowers and of which many accounts are falling within the
purview of CDR (Corporate Debt Restructuring) mechanism. Enumerate the audit procedure to be carried out
for assessing such borrowers' accounts. (Audit of Banks) – Content deleted from SM hence not including in DC
18. You are appointed as an auditor of BHK Ltd., a company engaged in export of agricultural equipment. During
the course of audit, your audit team informed you regarding non-deduction of TDS on huge payments made to
legal counsel of BHK Ltd. You want to alert your team on the possibility of non-compliance with Laws and
Regulations by BHK Ltd. Help your audit team in identifying any other indications of non-compliance with Laws
and Regulations particularly related to payments made by the company.
19. As a tax auditor of RM & Co. for the assessment year 2021-22, how would you deal and report the following:
(i) RM & Co. has borrowed ` 75 lakhs from a bank @10% p.a. for purchase of machinery. The machinery was
purchased on 30.09.2020, but was put to use on 10.04.2021. The interest paid to bank for the year 2020-
21 was charged to Profit and Loss Account.
(ii) RM & Co. is a trading concern and maintains the quantitative details of items traded. The said details
have been provided to you by the company for meeting reporting requirements, if any.

Notes

FAST
CA.Sarthak Niraj Jain

306 Ab Audit Hoga Sabse Scoring - By CA. Sarthak Niraj Jain

You might also like